피드백 제공자로서 환자: 의과대학생의 신뢰 판단 탐색(Perspect Med Educ. 2023)
Patients as Feedback Providers: Exploring Medical Students’ Credibility Judgments

M. C. L. EIJKELBOOM R. A. M. DE KLEIJN W. J. M. VAN DIEMEN C. D. N. MALJAARS M. F. VAN DER SCHAAF J. FRENKEL 

소개
Introduction

환자가 학생들의 학습에 기여할 수 있다는 사실이 널리 인식되면서 의학교육에 환자를 참여시키는 이니셔티브가 증가하고 있습니다[1, 2]. 환자로부터 배우는 한 가지 방법은 환자의 피드백입니다[3]. 의료 서비스의 사용자로서 환자는 의료 성과에 대한 독특한 관점을 제공할 수 있습니다. 따라서 환자의 피드백은 교수진이나 동료의 피드백을 보완할 수 있습니다. 이는 환자들이 동일한 상담에 대해 학생이나 레지던트를 교수진과 다르게 평가한다는 연구 결과에서 잘 드러납니다 [4]. 환자 피드백이 학생의 학습에 기여하기 위해서는 학생이 피드백에 참여해야 하며, 이는 학생이 피드백을 찾고, 이해하고, 행동으로 옮겨야 한다는 것을 의미합니다[5]. 학생이 피드백에 참여할지 여부는 피드백 제공자의 신뢰성에 대한 판단에 따라 부분적으로 결정됩니다[5, 6, 7, 8, 9, 10, 11]. 이에 따라 Baines 등의 연구에 따르면 환자의 피드백이 의사의 성과에 미치는 영향은 환자의 신뢰성에 대한 의사의 판단에 영향을 받는다고 보고했습니다[12]. 그러나 피드백 제공자로서 환자의 신뢰성에 대한 이러한 판단이 어떻게 이루어지고 어떤 논거에 근거하여 이루어지는지에 대해서는 알려진 바가 거의 없습니다. 
It is becoming widely recognized that patients can contribute to students’ learning, and the number of initiatives that involve patients in medical education is growing [1, 2]. One way to learn from patients is through their feedback [3]. As the users of healthcare, patients can provide unique perspectives on medical performance. Therefore, their feedback can be complementary to faculty or peer feedback. This is illustrated by studies in which patients rate students or residents differently on the same consultation compared to faculty [4]. In order for patient feedback to contribute to student learning, students need to engage with this feedback, meaning they should seek it, make sense of it, and act upon it [5]. Whether students engage with feedback is partly determined by their judgment of the feedback provider’s credibility [5, 6, 7, 8, 9, 10, 11]. In accordance with this, a review by Baines et al. reports that the impact of patient feedback on doctors’ performance was influenced by doctors’ judgments of patients’ credibility [12]. However, little is known about how these judgments regarding patients’ credibility as feedback providers are made and on the basis of which arguments.

Molloy와 Bearman에 따라 우리는 신뢰성을 타인의 판단에 의해 정의되는 개인의 속성으로 간주합니다[13]. 이와 관련하여 신뢰성은 개인이 소유하는 절대적인 특성이 아니라 타인의 인식에 따라 달라집니다. 신뢰는 타인과의 관계 속에서 형성되며 관계에 따라 달라집니다 [7, 14, 15]. 따라서 한 학생은 어떤 환자를 매우 신뢰할 수 있다고 생각하지만, 다른 학생은 같은 환자를 신뢰할 수 없다고 판단할 수 있습니다. 신뢰성 판단은 판단이 이루어지는 맥락, 학습 문화 및 이러한 맥락 내에서 형성되는 관계에 의해 형성됩니다 [7, 16, 17]. 따라서 이론적으로 한 학생이 교실 환경에서는 환자를 매우 신뢰할 수 있다고 판단할 수 있지만, 병원 환경에서는 같은 환자를 신뢰할 수 없다고 판단할 수도 있습니다. 따라서 신뢰성은 역동적이고 대인관계에 따라 달라집니다. 환자가 의대생에게 피드백을 제공할 때, 그 맥락에서 그 환자의 신뢰도는 그 학생의 환자에 대한 판단에 의해 정의됩니다.
In line with Molloy and Bearman, we view credibility as an attribute of an individual, which is defined by judgments of others [13]. In this regard, credibility is not an absolute quality someone possesses, rather it depends on the perceptions of others. It is established in relationships with others and varies between these relationships [7, 14, 15]. So, whereas a patient can be seen as highly credible by one student, another student can judge the same patient as not credible. Credibility judgments are shaped by the context in which the judgment takes place, by its learning culture and the relationships that are formed within these contexts [7, 16, 17]. So, in theory, a student could judge a patient as highly credible in a classroom setting, but this same student might judge the same patient as not so credible in a hospital setting. Thus, credibility is dynamic and interpersonal. When a patient provides feedback to a medical student, this patient’s credibility is defined by that student’s judgment of that patient, in that context.

신뢰도를 더 개념화하기 위해 우리는 교사 신뢰도 연구에 일반적으로 적용되는 맥크로스키의 연구를 기반으로 합니다[18, 19]. 맥크로스키는 신뢰성을 역량, 신뢰성, 선의로 구성된 3차원적 구조로 정의하고 운영했습니다[18]. 유능성은 관심 있는 주제에 대한 지식과 전문성을 갖춘 사람을 의미합니다[20]. 신뢰성은 좋은 성품과 정직성을 가진 사람을 의미합니다. 선의는 상대방을 배려하고 좋은 의도를 가지고 있는지를 의미합니다[18]. 따라서 환자-학생 피드백 상호작용에서 이러한 신뢰성의 개념화를 적용하면, 환자의 신뢰성에 대한 학생의 판단은 환자의 유능성, 신뢰성, 선의에 대한 판단으로 구성되어야 합니다.
To further conceptualize credibility we build on the work of McCroskey, which is commonly applied in research on teacher credibility [18, 19]. McCroskey defined and operationalized credibility as a three-dimensional construct consisting of Competence, Trustworthiness, and Goodwill [18]. Competence refers to someone having knowledge of, and expertise in, the subject of interest [20]. Trustworthiness refers to someone being of good character and being honest. Goodwill refers to whether someone cares about the receiver and has good intentions [18]. Thus, using this conceptualization of credibility in patient-student feedback interactions, students’ judgments of patients’ credibility should comprise judgments of patients’ Competence, Trustworthiness and Goodwill.

교육 분야, 특히 최근에는 의학교육 분야의 연구에서 학습자의 교사 또는 감독자에 대한 신뢰도 판단이 여러 요소로 구성된다는 사실이 밝혀졌지만, 환자와 (미래의) 의료인 간의 피드백 상호작용에 대한 경험적 연구는 주로 단일 요소의 신뢰도 판단을 보고하고 있습니다 [7, 19]. 일반적으로 학생들은 (잠재적) 피드백 제공자로서 환자에 대해 긍정적인 판단을 내리는 반면[3, 21, 22, 23, 24], 의사는 피드백 제공자로서 환자에 대해 긍정적인 판단과 부정적인 판단을 모두 내리는 것으로 보고됩니다[25, 26, 27].

  • 성과를 관찰할 수 있는 능력은 환자의 신뢰도에 긍정적으로 기여하는 요인으로 설명됩니다[26, 28, 29].
  • 의사나 학생이 언급하는 신뢰도를 낮추는 요인으로는 의학 지식 부족, 환자의 진정성에 대한 우려, 피드백이 환자의 진단에 영향을 받을 수 있다는 우려 등이 있습니다[8, 24, 27, 30, 31, 32, 33].

Although studies in education, and more recently in medical education, have shown that learners’ credibility judgments of teachers or supervisors are comprised of multiple elements, empirical studies on feedback interactions between patients and (future) healthcare providers mainly report single elements of credibility judgments [7, 19]. In general, students hold positive judgments about patients as (potential) feedback providers [3, 21, 22, 23, 24], whereas doctors report both positive and negative judgments of patients as feedback providers [25, 26, 27].

  • The ability to observe performance is described as an argument positively contributing to patients’ credibility [262829].
  • Arguments for reduced credibility, as mentioned by doctors or students, are: lack of medical knowledge, concerns about the sincerity of patients, and concerns about feedback being influenced by a patient’s diagnosis [8242730313233].

의대생은 강의실, 병원, 환자의 집, 지역사회 등 다양한 환경에서 환자와 함께 그리고 환자로부터 학습합니다. 앞서 언급했듯이 신뢰성 판단은 맥락에 의해 형성됩니다[7, 16, 17]. 의대생들이 환자의 피드백에 참여하여 환자로부터 배우기를 원한다면, 학생들이 환자에 대한 신뢰성 판단을 내리는 과정, 이러한 판단의 근거가 되는 논거, 그리고 이러한 판단이 맥락에 의해 어떻게 형성되는지에 대한 더 나은 이해가 필요합니다. 이를 통해 학생들의 환자 피드백에 대한 참여를 개선할 수 있는 기회를 발견할 수 있습니다. 따라서 이 연구는 다음과 같은 연구 질문에 답하는 것을 목표로 합니다: 의대생들은 임상 및 비임상 맥락에서 피드백 제공자로서 환자의 신뢰도를 어떻게 판단할까요?
Medical students learn with and from patients in various contexts, such as the classroom, the hospital, a patient’s home or the community. As previously mentioned, credibility judgments are shaped by the context [7, 16, 17]. If we want medical students to engage with patient feedback, and thereby learn from patients, we need a better understanding of the process by which students make credibility judgments regarding patients, on which arguments they base these judgments, and how this is shaped by the context. This might reveal opportunities to improve students’ engagement with patient feedback. Therefore, this study aims to answer the following research question: How do medical students judge patients’ credibility as feedback providers in a clinical and in a non-clinical context?

방법
Methods

연구 디자인
Study design

피드백 상호작용에서 환자의 신뢰도에 대한 학생들의 판단을 이해하기 위해 후향적 질적 인터뷰 연구를 수행했습니다. 이 연구에 대한 윤리적 승인은 네덜란드 의학교육협회에서 제공했습니다(NVMO, NERB 파일 번호: 2020.5.8 및 NERB 파일 번호 2021.8.8).  
We undertook a retrospective qualitative interview study to understand students’ credibility judgments of patients in feedback interactions. Ethical approval of this study was provided by the Dutch Association for Medical Education (NVMO, NERB file number: 2020.5.8 and NERB file number 2021.8.8).

연구 배경 및 참여자
Context and participants

우리는 임상적 맥락과 비임상적 맥락에서 환자에 대한 학생들의 신뢰성 판단을 조사했습니다. 두 상황 모두에서 학생들은 (온라인) 일대일 대화를 통해 환자로부터 피드백을 수집했습니다. 
We explored students’ credibility judgments regarding patients in a clinical and a non-clinical context. In both contexts students collected feedback from patients through (online) one-on-one dialogues.

비임상적 맥락에서는 6주간의 선택 과목으로, 6학년 의대생들이 한 조를 이루어 시청각 환자 정보인 지식 클립을 개발했습니다. 지식 클립은 선천성 심장병에 대해 설명하는 짧은 동영상(주로 애니메이션)이었습니다. 학생들은 환자, 그리고 커뮤니케이션 및 정보 과학(CIS) 학생과 협력하여 이러한 지식 클립을 개발했습니다[34]. 의대생의 목표는 환자의 정보 요구를 파악하고, 이해하기 쉬운 정보를 만들고, 환자 및 CIS 학생과 협업하는 방법을 배우는 것이었습니다. 이 과정을 진행하는 동안 두 의대생은 환자와 세 차례 만났습니다. 첫 번째 미팅에서는 의대생과 환자가 서로 친해지고 지식 클립의 주제를 결정했습니다. 2번과 3번 미팅에서 의대생은 지식 클립과 협력 기술에 대한 환자의 피드백을 받았습니다. 의대생 쌍은 시청각 정보 개발 방법에 대한 조언을 제공하고 지식 클립에 대한 피드백을 제공한 CIS 학생과도 세 차례 만났습니다[34]. 2020년 8월, 네덜란드 중심부의 한 대학병원에서 12명의 의대생이 이 선택 과목에 등록했습니다. 이 과정은 코로나19 팬데믹으로 인해 전적으로 온라인으로 진행되었습니다. 이 과정에 등록한 의대생 12명 중 11명이 이 연구에 참여하기 위해 사전 동의를 제공했습니다. 
The non-clinical context was a six-week elective course, in which pairs of sixth-year medical students developed audiovisual patient information, a knowledge clip. The knowledge clips were short (often animated) videos, where for instance a congenital heart disease is explained. Students developed these knowledge clips in collaboration with a patient and a Communication and Information Sciences (CIS) student [34]. The goal for medical students was learn how to identify a patient’s information need, create understandable information, and collaborate with a patient and a CIS student. During the course, the pair of medical students met three times with the patient. In meeting 1, the medical students and the patient got acquainted and determined the subject of the knowledge clip. In meetings 2 and 3, medical students received patient feedback on their knowledge clip and cooperation skills. The medical student pair also met three times with the CIS student, who provided advice on how to develop audiovisual information and gave feedback on the knowledge clip [34]. In August 2020, twelve medical students from an academic hospital in the center of the Netherlands enrolled in this elective course. The course was entirely online, due to the COVID pandemic. Eleven out of twelve medical students who signed up for the course enrollees provided informed consent to participate in this study.

임상 상황은 소아과와 산부인과를 결합한 12주간의 임상 실습이었으며, 4학년 의대생들은 의학 커리큘럼의 필수 과정으로 이 실습에 참여했습니다. 실습 기간 동안 학생들은 두 명의 환자에게 피드백을 요청했습니다. 이에 대비하여 학생들은 대화를 통해 적절한 시기에 적절한 피드백을 요청하는 자기 주도적 피드백 과정을 이수했습니다[35]. 임상 실습이 끝날 무렵, 학생들은 의미 파악과 행동 계획에 초점을 맞춘 촉진된 성찰 세션에 참여했습니다[35]. 2022년 6월부터 8월까지 사무원 과정을 마친 학생들은 이 연구에 참여하도록 요청받았습니다. 54명 중 10명의 학생이 사전 동의를 통해 참여에 동의했습니다. 
The clinical context was a twelve-week clerkship combining Pediatrics and Gynecology, in which fourth-year medical students participated as a mandatory part of their medical curriculum. During the clerkship, students asked two patients for feedback. In preparation, students completed a self-directed feedback course on asking for relevant feedback, at the right time, through dialogue [35]. At the end of the clerkship, students participated in a facilitated reflection session, which focused on sense-making and action-planning [35]. Students who completed their clerkship between June – August 2022, were asked to participate in this study. Ten students out of 54 provided informed consent to participate.

익명성을 보장하고 명확성을 기하기 위해 이 논문에서 모든 학생은 '그녀'로, 모든 환자는 '그'로 지칭합니다. 간결성을 위해 의대생은 '학생'으로 지칭합니다.
To ensure anonymity, and for the sake of clarity, all students in this paper will be referred to as ‘she’, and all patients will be referred to as ‘he’. For the sake of brevity, medical students will be referred to as students.

데이터 수집
Data collection

비임상 분야의 경우, 신뢰성 문헌을 기반으로 반구조화된 인터뷰 가이드를 개발했습니다. 두 명의 학생을 대상으로 인터뷰 가이드를 시범 운영하여 약간의 조정을 거쳤습니다. 그런 다음, 후속 인터뷰가 유사하고 비교 가능한 방식으로 진행될 수 있도록 CE와 NM이 함께 처음 두 번의 인터뷰를 진행했습니다. 나머지 9번의 면접은 NM과 CE가 나누어 진행했습니다. 
For the non-clinical context, we developed a semi-structured interview guide based on credibility literature. We piloted the interview guide with two students, which resulted in minor adjustments. Then, CE and NM conducted the first two interviews together to ensure subsequent interviews were conducted similarly and comparably. The remaining nine interviews were divided between NM and CE.

인터뷰 가이드를 임상 상황에 맞게 조정하고 비임상 상황의 결과를 기반으로 했습니다. 세 번째 연구자(WD)가 임상 맥락에서 대부분의 인터뷰를 수행했습니다. 먼저, CE와 WD가 함께 두 번의 인터뷰를 진행하여 WD가 비임상 상황과 유사한 인터뷰를 수행할 수 있도록 훈련시켰습니다. 그런 다음 나머지 인터뷰는 WD가 진행했습니다. 
We adjusted the interview guide to fit the clinical context and build on the results from the non-clinical context. A third researcher (WD) performed most of the interviews from the clinical context. First, CE and WD conducted two interviews together to train WD in performing the interviews comparably to the non-clinical context. Then, WD conducted the remaining interviews.

인터뷰 가이드에는 환자의 신뢰도, 학생과 환자의 관계, 피드백 메시지에 대한 질문이 포함되어 있었습니다(부록 A 참조). 학생들은 선택 과목 또는 임상실습을 마친 후 개별적으로 인터뷰를 진행했습니다. 화상 통화 또는 대면으로 진행된 인터뷰는 음성으로 녹음하고 그대로 필사했습니다. 인터뷰는 약 60분간 진행되었습니다. 
The interview guide included questions on patients’ credibility, students’ relationship with the patients and the feedback messages (See Appendix A). Students were interviewed individually after completion of the elective course or clerkship. The interviews, which were conducted via video-call or face-to-face, were audio recorded and transcribed verbatim. Interviews lasted approximately 60 minutes.

데이터 분석
Data analysis

학생들이 환자의 신뢰도를 판단하는 방법을 이해하기 위해 신뢰도에 대한 학생들의 추론을 분석했습니다. 우리는 주제별 분석의 한 형태인 템플릿 분석을 수행하여 신뢰도 증가 또는 감소에 대한 학생들의 주장을 파악했습니다[36]. 이 방법을 선택한 이유는 이전의 피드백 신뢰도 문헌을 기반으로 구축할 수 있고, 귀납적으로 환자 신뢰도에 관한 새로운 코드를 개발할 수 있는 여지를 남겨두었기 때문입니다. 학생들이 어떻게 판단을 내리는지, 그리고 이것이 맥락적 요소의 영향을 받는지 이해하기 위해 인과적 네트워크 분석을 수행했습니다. 이 방법을 선택한 이유는 프로세스의 변수 간 일관성을 매핑하기 때문인데, 이는 신뢰성 판단과 같은 인지 프로세스에도 적용될 수 있습니다[37]. 학생들이 환자의 신뢰도를 판단하는 추론의 요소를 이 연구에서는 '논증'이라고 부릅니다. 
To understand how students make credibility judgments of patients, we analyzed their reasoning about credibility. We performed template analysis, which is form of thematic analysis, to identify students’ arguments for increased or reduced credibility [36]. We chose this method because it allowed for building on previous feedback credibility literature, and left room for inductively developing new codes regarding patient credibility. To understand how students built their judgment and how this was impacted by contextual elements, we performed a causal network analysis. We chose this method because it maps the coherence between variables of a process, which can also be applied to cognitive processes such as making a credibility judgment [37]. The elements in students’ reasoning for determining a patient’s credibility are called ‘arguments’ in this study.

템플릿 분석
Template analysis

코딩 템플릿은 귀납적 접근법과 연역적 접근법을 모두 사용하여 세 단계로 개발되었습니다.

  • 첫째, 신뢰성 판단의 근거가 될 수 있는 문헌에 기술된 논거를 구성하는 선험적 코드를 정의했습니다(부록 B)[6, 10, 11, 14, 15, 18, 19, 38].
  • 둘째, 초기 코딩 템플릿을 구성하기 위해 비임상 맥락의 인터뷰를 사용했습니다. CE와 NM은 코딩 템플릿을 인터뷰에 반복적으로 적용하면서 새로운 코드를 추가하고 기존 코드를 개선했습니다. 코드를 더욱 세분화하고 구조화하기 위해 이들은 1-3차, 4-5차 인터뷰 코딩 후, 그리고 5-12차 인터뷰 후 코딩 템플릿에 대해 RK와 논의했습니다. 코드는 세 가지 신뢰성 차원에 따라 구조화되었습니다: 역량, 신뢰성, 선의의 세 가지 차원에 따라 코드를 구조화했습니다[18]. 몇몇 코드는 신뢰성 차원에 맞지 않았습니다. 여기에는 맥락의 요소와 두 가지 새로운 주제, 즉 피드백 메시지와 환자 피드백에 대한 이전 경험이 포함되었습니다.
  • 셋째, CE와 WD는 초기 코딩 템플릿을 임상적 맥락에서 처음 6개의 인터뷰에 적용하고 새로운 코드를 추가하고 기존 코드를 재정의하여 템플릿을 수정했습니다. 이러한 수정 사항은 최종 코딩 템플릿을 정의하기 위해 RK와 논의했습니다.
  • 마지막으로 CE는 최종 코딩 템플릿을 전체 데이터 세트에 적용하고 최종 해석에 대한 합의에 도달할 때까지 전체 연구팀과 결과를 논의했습니다.

The coding template was developed in three steps, using both an inductive and deductive approach.

  • First, a priori codes were defined constituting arguments described in literature on which credibility judgments can be based (Appendix B) [610111415181938].
  • Second, interviews from the non-clinical context were used to construct the initial coding template. CE and NM iteratively applied the coding template to the interviews, whilst adding new codes and refining existing codes. To further refine and structure the codes, they discussed the coding template with RK after coding interviews 1–3, 4–5, and after interviews 5–12. Codes were structured according to the three credibility dimensions: Competence, Trustworthiness, and Goodwill [18]. Several codes did not fit the credibility dimensions. These included elements of the context and two emergent themes: the feedback message and previous experiences with patient feedback.
  • Third, CE and WD applied the initial coding template to the first six interviews from the clinical context and modified the template by adding new codes and redefining existing codes. The modifications were discussed with RK to define the final coding template.
  • Lastly, CE applied the final coding template to the full dataset and findings were discussed with the entire research team until consensus was reached about the final interpretation.

인과 관계 네트워크 분석
Causal network analysis

판단이 어떻게 만들어지고 주장이 어떻게 연관되어 있는지 살펴보기 위해 사례 간 인과 네트워크 분석을 수행했습니다[37]. 

  • 먼저, 학생들이 신뢰성 판단의 근거로 삼은 모든 주장의 목록을 작성했습니다. 이 목록은 템플릿 분석에서 도출된 것으로, 이 코드북의 각 코드는 하나의 논증과 유사했습니다.
  • 둘째, 학생 한 명당 하나씩 21개의 인과 네트워크를 구성하여 이러한 주장과 그 관계를 표시했습니다. 관계를 다음을 통해 식별했습니다.
    • 학생들이 서로 조합하여 언급한 논증(텍스트 조각이 서로 겹치거나 이어지는 것을 의미),
    • 신호어 분석(예: 그렇다면, 그러므로, 그래서, 왜냐하면),
    • 시간성 분석(한 논증이 다른 논증에 영향을 미치는 경우 이 논증이 학생들의 추론에서 먼저 발생해야 함)
  • 셋째, 21개의 개별 인과 네트워크는 개별 논증 간의 모든 상호작용을 포함하는 전체 네트워크로 결합되었습니다. 이 네트워크를 이해하기 쉽게 하기 위해 신뢰도 차원과 맥락, 피드백 메시지, 이전 경험의 요소 간의 상호작용만 설명하여 네트워크를 단순화했으며, 그 결과 그림 1과 같이 나타났습니다.

To explore how judgments were built, and how arguments were related, we performed cross-case causal network analyses [37].

  • First, we constructed a list of all arguments on which students based their credibility judgments. This list was derived from the template analysis, each code of this codebook resembled an argument.
  • Second, we constructed 21 causal networks, one for each individual student, in which we displayed these arguments and their relations. Relations were identified
    • by selecting arguments that students mentioned in combination with each other (meaning text-fragments overlapped or followed-up on each other),
    • by analyzing signal words (for instance: then, therefore, so, because), and
    • by analyzing temporality (if one argument affects the other, this argument must happen first in students’ reasoning).
  • Third, the 21 individual causal networks were combined in an overarching network, that comprised all interactions between individual arguments. To make this network comprehensible, we simplified the network by only illustrating the interactions between dimensions of credibility and the elements of the context, the feedback message, and previous experiences, which resulted in Figure 1.

이 그림은 피드백 제공자인 환자에 대한 학생들의 신뢰도 판단을 시각화한 것입니다. 학생들의 신뢰도 판단은 환자의 선의, 신뢰성 및 역량에 관한 여러 가지 논증으로 구성되었습니다. 각 차원 내 및 차원 간의 논증은 서로 상호작용했습니다. 또한 학생들의 신뢰도 판단은 회색 원으로 표시된 것처럼 맥락, 피드백 메시지, 이전 경험 등의 요소에 의해 영향을 받았습니다. 이러한 요소는 환자의 선의, 신뢰성 및 역량에 관한 논쟁에 영향을 미칠 수 있습니다.
This figure visualizes students’ credibility judgments of patients as feedback providers. Students’ credibility judgments consisted of multiple arguments regarding a patient’s Goodwill, Trustworthiness and Competence. Arguments within and between the dimensions interacted with each other. Moreover, students’ credibility judgments were influenced by elements of the context, the feedback message and previous experiences, which is depicted by the gray circle. These elements could affect arguments regarding a patient’s Goodwill, Trustworthiness, and Competence.

반사성
Reflexivity

연구팀은 의사(CE, JF), 교육 과학자(RK, MS), 학생(WD, NM)으로 구성되었습니다. 연구팀의 다양한 배경과 역할은 데이터에 대한 다양한 관점을 제공했습니다. 우리 팀은 환자를 학습의 중요한 파트너이자 정당한 피드백 제공자라는 관점을 가지고 있습니다. 우리의 관점이 연구 참여자에게 전달되는 것을 제한하고 학생들이 자신의 관점을 공유할 수 있도록 인터뷰 질문을 신중하게 구성했습니다. 일부 저자는 비임상 과정(CE, JF)과 임상 맥락에서의 피드백 교육(CE, JF, RK) 개발에 참여했습니다. 연구 기간 동안 이들은 이러한 교육에 강사로 참여하지 않았습니다. 
The research team consisted of medical doctors (CE, JF), educational scientists (RK, MS), and students (WD, NM). The varied backgrounds and roles of our team provided multiple perspectives on our data. Our team has the viewpoint of patients being important partners in learning, and being legitimate feedback providers. To limit transferring our viewpoint to the study participants, and facilitate students in sharing their own viewpoints, we carefully formulated the interview questions. Some authors were involved in the development of the non-clinical course (CE, JF) and the feedback training in the clinical context (CE, JF, RK). During the study period they did not participate as teachers in these trainings.

또한 데이터 수집 및 분석 기간 동안 반사성을 높이기 위해 격주로 연구 회의를 개최하여 데이터 해석과 기본 가정을 논의했습니다. 
Furthermore, to enhance reflexivity biweekly research meetings were held during data collection and analysis, in which interpretation of the data and underlying assumptions were discussed.

결과
Results

21명의 학생을 인터뷰했으며, 이 중 비임상 환경의 학생은 11명, 임상 환경의 학생은 10명이었습니다. 학생들은 신뢰성의 세 가지 차원을 모두 포함하는 여러 논거를 바탕으로 환자의 신뢰성을 판단했습니다[18]. 학생들은 환자의 신뢰도를 평가할 때 환자의 능력, 신뢰성, 선의에 대해 추론했습니다. 학생들의 추론은 맥락의 지각된 요소에 의해 판단이 형성된다는 것을 보여주었습니다. 맥락적 요소 외에도 지각된 피드백 메시지와 학생의 이전 환자 피드백 경험도 신뢰성 판단에 영향을 미쳤습니다. 
Twenty-one students were interviewed, out of which 11 students in the non-clinical and 10 in the clinical setting. Students based their credibility judgments of patients on multiple arguments, comprising all three dimensions of credibility [18]. In estimating a patient’s credibility, students reasoned about his Competence, Trustworthiness, and Goodwill. Students’ reasoning showed that their judgments were shaped by perceived elements of the context. Besides contextual elements, the perceived feedback message and student’s previous experiences with patient feedback also impacted their credibility judgments.

학생들이 신뢰성 판단을 위해 제공한 논거와 이러한 논거가 맥락에 의해 어떻게 형성되었는지, 그리고 이러한 논거가 피드백 메시지와 이전 경험에 의해 어떻게 영향을 받았는지에 대해 자세히 설명함으로써 학생들이 신뢰성 판단을 어떻게 구축했는지에 대해 논의할 것입니다.
We will discuss how students built their credibility judgments by elaborating on the arguments that students provided for their credibility judgments, how these were shaped by the context, and how these were affected by the feedback message and their previous experiences.

학생이 신뢰도 판단의 근거로 삼은 논거
Arguments on which students based their credibility judgments

환자의 신뢰성에 대한 학생들의 판단에는 환자의 역량, 신뢰성, 선의에 관한 여러 논거가 포함되었습니다(표 1 참조). 학생들은 네 가지 논거를 바탕으로 환자의 유능성을 판단했습니다.

  • 첫째, 학생들은 특정 과제에 대한 지식을 평가했습니다. 비임상적 맥락에서는 시청각적 의사소통에 대한 지식이 될 수 있습니다. 임상적 맥락에서는 의학 지식이 될 수 있습니다. 예를 들어, 학생들은 환자가 의료계에 종사할 때 더 유능하다고 판단했습니다.
  • 둘째, 학생들은 예를 들어 환자가 직접 관찰을 통해 얻을 수 있는 학생의 수행 방식에 대한 지식을 추정했습니다.
  • 셋째, 학생들은 환자가 피드백을 제공하는 데 경험이 많고 숙련되어 있는지를 평가했습니다.
  • 마지막으로 학생들은 환자의 실제 경험을 추정했습니다.
    • 대부분의 학생들은 환자가 경험적 지식이 많을수록 더 유능하다고 생각했습니다.
    • 반대로, 일부 학생들은 '새로운' 환자가 새로운 관점을 가지고 있기 때문에 더 신뢰할 수 있다고 생각했습니다.

Students’ judgments about a patient’s credibility contained multiple arguments regarding a patient’s Competence, Trustworthiness, and Goodwill, see Table 1. Students judged patients’ Competence based on four arguments.

  • First, they estimated their knowledge of the specific task. In the non-clinical context, this could be knowledge about audiovisual communication. In the clinical context, this could regard medical knowledge. For instance, students judged patients as more competent when they were in the medical profession.
  • Second, students estimated patients’ knowledge of how the student performed, which patients for instance could have gained through direct observation.
  • Third, students estimated whether patients were experienced and skilled in providing feedback.
  • Lastly, students estimated patients’ lived experience.
    • Most students regarded patients as more competent when they possessed more experiential knowledge.
    • Conversely, some students reasoned that their ‘new’ patient was actually very credible because he had a fresh perspective.

"그는 병원에 대해 잘 몰랐고 어떻게 운영되는지 등에 대해 잘 몰랐습니다. 그래서 오히려 그의 말이 더 진솔하게 들렸던 것 같아요. 또한 무지에서 비롯된 무지에서 그는 '이건 이해가 안 된다, 저건 이해가 안 된다'는 식의 질문을 많이 했어요. 그렇지만 그는 자신이 느끼는 것을 그대로 말한다는 것을 의미했고, 그래서 더 신뢰가 갔습니다." - St4clinical(실제 경험)
“He was not familiar with the hospital and didn’t know about how things worked and so on. And I think that actually ensured that what he said came out really sincere. Also from this ignorance, he asked many questions like we don’t understand this, or we don’t understand that. That did mean that he really just says what he feels, … I found him more credible because of it.” – St4clinical (Lived experience)

학생들은 두 가지 논거를 바탕으로 환자의 신뢰도를 판단했습니다. 

  • 첫째, 학생들은 환자가 피드백에 성실하고 정직한지 여부를 평가했습니다. 
  • 둘째, 학생들은 환자의 개방성과 개선할 점을 과감하게 언급하는지 여부를 평가했습니다.

Students judged patients’ Trustworthiness on two arguments.

  • First, students estimated whether a patient was sincere and honest in his feedback.
  • Second, students estimated a patient’s openness and whether he dared to mention points for improvement.

"[협업에 대한 피드백]의 경우, 환자의 피드백이 대부분 긍정적이었고 100% 정직했는지는 잘 모르겠기 때문에 조금 낮게 평가했습니다. 추정하기 어렵습니다. 그래서 저는 그가 저에게 피드백을 주는 것에 대해 개인적으로 특별히 신뢰할 수 없다고 생각합니다. 그가 피드백에 완전히 정직하게 임하고 있는지 잘 모르겠어요." -St6비임상(성실성)
“In terms of [feedback on] our collaboration, it’s a bit lower, because his feedback was mostly positive and I am not sure if he was being 100% honest. I find that hard to estimate. And that’s why I don’t find him particularly credible, as a person, to give me feedback. I can’t quite sense whether he is being completely honest in his feedback.” –St6nonclinical (Sincerity)

학생들은 두 가지 논거를 바탕으로 환자의 선의를 판단했습니다. 

  • 첫째, 학생들은 환자가 시간을 내어 피드백을 제공하고 교육 프로젝트에 참여하는 등 학생의 학습을 돕고자 하는 의지가 있는지를 평가했습니다. 
  • 둘째, 학생들은 환자가 학생의 사생활에 관심을 보이고 학생의 존재에 감사를 표하는 등 학생을 인격적으로 돌보는지 여부를 평가했습니다. 후자는 신뢰도를 높이거나 낮출 수 있습니다. 어떤 학생들은 환자가 학생의 감정을 상하게 하지 않기 위해 비판을 피할 것이라고 추론했습니다.

Students judged patients’ Goodwill on two arguments.

  • First, they estimated whether the patient had the intention to help the student learn, for instance by taking time to provide feedback and by being engaged in the educational project.
  • Second, students estimated whether the patient cared for the student as a person, for instance by showing interest in the student’s personal life and by expressing gratefulness for the student’s presence. The latter could both increase and decrease credibility. Some students reasoned that a patient would avoid criticism to avoid hurting the student’s feelings.

"글쎄요, '그냥 학생일 뿐인데 상처 주고 싶지 않아요'라고 생각할 수도 있죠. 네, 그리고 그 학생도 최선을 다하고 있을 뿐이죠. 부정적인 말을 해서 저에게 상처를 주고 싶지 않을 수도 있죠." -St4clinical(돌보는 사람)
“Well, he might think ‘she’s just a student, I don’t really want to hurt her. Yes, and she’s also just trying to do the best she can.’ He may not want to hurt me by saying anything negative.” –St4clinical (Caring for person)

인과 관계 네트워크 분석 결과, 학생들은 여러 주장에 무게를 두어 판단을 내리는 것으로 나타났습니다. 서로 다른 차원의 주장이 서로 상호작용했습니다(그림 1 참조). 예를 들어, 한 학생은 환자가 자신의 학습에 관심을 갖고 있다고 가정했기 때문에(선의) 환자가 성실하고 개방적이라고 추론했습니다(신뢰성). 연구 결과에 따르면 학생들의 환자의 신뢰도에 대한 판단은 '전부 아니면 전무'의 판단이 아니었습니다. 오히려 학생들은 신뢰도에 기여하는 논거와 신뢰도를 떨어뜨리는 논거를 고려하여 환자에게 어느 정도의 신뢰도를 부여했습니다.
Causal network analysis showed that students built their judgments by weighing multiple arguments. Arguments of different dimensions interacted with each other (See Figure 1). For instance, a student reasoned that the patient was sincere and open (Trustworthiness), since she assumed that the patient cared for her learning (Goodwill). Our results suggest that students’ judgments of a patient’s credibility were not ‘all or nothing’ judgments. Rather, students assigned a certain degree of credibility to a patient, taking into consideration arguments that contributed to credibility, and arguments that reduced credibility.

맥락의 요소가 신뢰도 판단에 미치는 영향
How elements of the context affect credibility judgments

학생들은 신뢰도 판단에 영향을 미치는 네 가지 맥락적 요소를 언급했습니다(표 2 참조).
Students mentioned four contextual elements that affected their credibility judgments (See Table 2).


세 가지 맥락 요소는 임상적 맥락에서 학생들이 언급했거나 비임상적 맥락에서 학생들이 언급했습니다.
Three contextual elements were either mentioned by students in the clinical context, or by students in the non-clinical context.

첫째, 임상적 맥락에서 학생들은 자신과 환자 사이에 치료적 관계가 있다고 언급했는데, 환자는 치료가 필요한 상태이고 학생은 치료를 제공하는 팀의 일원이었습니다. 학생들은 환자가 힘의 불균형을 인식하고 자신에게 의존한다고 느낄 수 있기 때문에 피드백 과정에 방해가 될 수 있다고 추론했습니다. 따라서 환자들이 감히 개방적이고 솔직하게 말하지 못해(신뢰성) 신뢰도가 낮아질 수 있다는 것이었습니다.
First, in the clinical context, students mentioned that there was a therapeutic relationship between them and the patient, with the patient being in need of care and the student being part of the team that provides care. Students reasoned that this could have hindered the feedback process, since patients could perceive a power imbalance and feel dependent on them. Therefore, patients might not dare to be open and honest (Trustworthiness), which lowered their credibility.

"나중에 그 환자도 우리에게 의존하고 있기 때문에 치료 관계를 유지하고 싶다고 말했습니다. 그리고 그도 마음 한구석에 우리와 우호적인 관계를 유지해야 한다는 생각을 가지고 있었기 때문에 너무 비판적으로 대하고 싶지 않았던 것 같습니다."-St4clinical
“He said that later on too, that he wants to maintain the treatment relationship, because he does depend on us as well. And I think that’s why he also has in the back of his mind that he has to remain friendly with us, which is perhaps why he didn’t want to be too critical.”-St4clinical

흥미롭게도 한 학생은 시간이 지남에 따라 환자가 학습자로서의 자신의 역할을 더 잘 인식하게 되었고, 이로 인해 치료 관계는 더 뒷전으로 밀려났으며 학습을 돕고자 하는 환자의 의지가 높아졌다고 말했습니다(Goodwill).
Interestingly, one student mentioned that over time the patient became more aware of her role as a learner, which had put the therapeutic relationship more to the background, and increased the patient’s willingness to help the student learn (Goodwill).

"그 덕분에 그는 인턴이 무엇을 수반하는지 더 잘 알았던 것 같아요. 그리고 실제로 더 나은 피드백을 줄 수 있었던 것 같아요. '당신은 정말 배우러 왔고 내 의사가 아니기 때문에 내 치료 계획을 세우지 않을 거야'라고 생각했기 때문이죠. 그래서 저는 그 부분(즉, 학습)을 도와드리고 싶습니다."-St7clinical
“because of that, I think he knew more what an intern entailed. And I think that actually allowed him to give better feedback, because he was more like okay, so you’re really here to learn and you’re not my doctor, so you’re not going to set my treatment plan. So I want to help you with that [ i.e. learning].”-St7clinical

둘째, 임상 맥락에서 학생들은 환자의 감정 상태가 신뢰도에 영향을 미친다고 언급했습니다. 학생들은 피드백 메시지가 환자의 감정에 의해 형성된다고 추론했습니다. 예를 들어 환자가 좋은 소식을 듣고 행복하거나 안심한 상태라면 피드백이 더 긍정적일 것이고, 그 반대의 경우도 마찬가지입니다. 또한, 학생들은 환자가 낯선 환경에 처해 있거나 매우 아프다는 등의 이유로 취약하다고 느끼면 감히 비판적인 피드백(신뢰성)을 제공하지 않을 것이라고 추론했습니다. 또한 학생들은 스트레스를 받거나 긴장한 환자에 비해 차분하고 편안해 보이는 환자를 더 신뢰할 수 있다고 답했는데, 그 이유는 후자의 경우 학생의 학습에 관심을 갖거나(선의) 학생을 관찰하고 수행 능력을 판단할 수 있는 적절한 마음 상태가 아닐 수 있기 때문입니다(유능성).
Second, in the clinical context, students mentioned that the patient’s emotional state impacted his credibility. Students reasoned that feedback messages were shaped by patients’ emotions. For example, if a patient was happy or relieved due to receiving good news, then his feedback would be more positive, and vice versa. Moreover, students reasoned that patients would not dare to provide critical feedback (Trustworthiness) if they felt vulnerable, for instance because they were in an unknown environment or were very sick. Besides, students mentioned that patients who appeared calm and at ease where more credible compared to patients who were stressed or nervous, because the latter might not be in the right state of mind to care about students’ learning (Goodwill) or observe the student and make judgments about her performance (Competence).

"네, [이 환자는] 따라서 덜 긴장하고 덜 감정적이었으며, 이 업무 환경에서 [내가] 어떻게 하고 있는지 생각할 수 있는 여유가 더 많았습니다. 따라서 자신에게 너무 집중하는 대신 상대방을 바라볼 수 있는 여유도 생겼습니다. 제 생각에 그는 더 나은 피드백을 줄 수 있었고 더 신뢰할 수 있었습니다. 그리고 그는 좀 더 객관적이었던 것 같아요." - St6clinical
“Yes [this patient was] therefore less nervous, and therefore less emotional, with more space to think about how [I am] doing in this work environment. There is therefore also more space to look at the other person instead of focusing too much on yourself. In my opinion, he could therefore give better feedback and was more credible. And he was a bit more objective as well, I think.” – St6clinical

셋째, 비임상 환경의 학생들은 시청각 환자 정보(지식 클립)에 대한 다른 제공자의 피드백 메시지가 환자의 신뢰도에 영향을 미친다고 언급했습니다. 교육 과정에서도 학생들은 의료 전문가와 동료로부터 지식 클립에 대한 피드백을 받았습니다. 환자의 피드백이 의료 전문가나 동료의 피드백에 추가되거나 그에 부합하는 경우, 환자는 더 신뢰할 수 있는 것으로 간주되었습니다.
Third, students in the non-clinical context mentioned that feedback messages of other providers on their audiovisual patient information (the knowledge clip) affected the patient’s credibility. In the educational course, students also received feedback on their knowledge clip from healthcare professionals and peers. When the patient’s feedback added to, or was in accordance with, feedback from healthcare professionals or peers, the patient was seen as more credible.

"실제로 [심리학자]가 제공한 조언은 [환자]의 조언과 완전히 같거나 거의 똑같았습니다. 그래서 항상 매우 비슷했습니다. ... 그래서 [그의] 신뢰성이 강화되었습니다." -St9비임상
“Actually, the tips given by [a psychologist] were exactly the same, or almost exactly the same as [the patient’s]. So that was always very similar. … which did reinforce [his credibility].” -St9nonclinical

맥락의 한 가지 요소인 '신뢰 관계'는 두 맥락 모두에서 학생들이 언급했습니다. 상호 존중과 신뢰로 특징지어지는 유대감은 신뢰도를 높이거나 낮출 수 있습니다. 

  • 일부 학생들은 긍정적인 유대감으로 인해 환자가 학습에 더 많은 관심을 갖게 되고(선의), 더 개방적으로 행동하게 된다고 추론했습니다(신뢰성).
  • 그러나 다른 학생들은 긍정적인 유대감 때문에 환자들이 관계의 손상을 두려워하여 개선해야 할 점에 대해 덜 솔직해졌다고 추론했습니다(신뢰성). 

One element of the context, ‘trusting bond’, was mentioned by students in both contexts. A bond, characterized by mutual respect and trust, could both increase or decrease credibility.

  • Some students reasoned that their positive bond made patients care more about their learning (Goodwill) and made them dare to be more open (Trustworthiness).
  • However, other students reasoned that their positive bond made patients less open about points for improvement (Trustworthiness), because they feared damaging the relationship.

"아마도 그 때문에 왠지 신뢰도가 조금 떨어졌을 것 같아요... 우리는 아주 좋은 관계를 유지하고 있는데, 그가 여전히 감히 엄격할 수 있을까, 말하자면 여전히 감히 팁을 제시할 수 있을까 하는 생각이 들었어요." -St3비임상 
“Maybe that somehow made him a little less credible, that I thought … we have a very good relationship, does he still dares to be strict, so to speak, does he still dare to come up with tips.” –St3nonclinical

피드백 메시지와 이전 경험이 신뢰도 판단에 미치는 영향
How the feedback message and previous experiences affect credibility judgments

맥락의 요소 외에도 피드백 메시지와 환자의 피드백에 대한 이전 경험도 학생들의 신뢰성 판단에 영향을 미치는 것으로 나타났습니다. 피드백 메시지와 관련하여, 학생들은 피드백 메시지가 구체적이고 광범위하며 긍정적인 피드백과 개선점을 모두 포함하고 있다고 인식할 때 환자가 자신의 학습에 관심을 갖고(선의), 개방적이고 정직하며(신뢰성), 피드백 제공에 능숙하다고(유능함) 추론했습니다. 이는 그 반대의 경우도 마찬가지였습니다: 
Besides elements of the context, students reasoning also showed that the feedback message, and previous experiences with patient feedback, affected students’ credibility judgments. With regard to the feedback message, when students perceived the feedback message as specific, extensive, and containing both positive feedback and points for improvement, they reasoned that the patient cared for their learning (Goodwill), was open and honest (Trustworthiness), and that he was skilled in providing feedback (Competence). This also worked vice versa:

"그의 피드백은 대부분 긍정적이었지만 그가 100% 정직했는지는 잘 모르겠습니다. 평가하기 어렵습니다. 그래서 저는 그가 저에게 피드백을 주는 것에 대해 인간으로서 특별히 신뢰할 수 없다고 생각합니다. 그가 피드백에 대해 완전히 정직한지 잘 모르겠습니다."-St6nonclinical(성실성) 
“his feedback was mostly positive and I am not sure if he was being 100% honest. I find that hard to estimate. And that’s why I don’t find him particularly credible, as a person, to give me feedback. I can’t quite sense whether he is being completely honest in his feedback.”–St6nonclinical (Sincerity)

개인적 특성과 관련하여 비임상 환경의 일부 학생들은 과거에 환자 피드백에 대한 긍정적인 경험을 가지고 있었습니다. 이 때문에 환자 피드백의 가치를 더 잘 인식하게 되었고, 따라서 환자를 더 신뢰할 수 있다고 판단했습니다. 
With regard to personal characteristics, some students in the non-clinical context had positive past experiences with patient feedback. This made them more aware of the value of patient feedback, and therefore judge patients as more credible.

"환자의 피드백은 궁극적으로 메시지를 전달하고 싶거나 안심시키고 싶은 사람이기 때문에 환자의 피드백에 많은 가치가 있다는 것을 이전의 경험을 통해 알게 된 것 같습니다. 목표를 달성하고자 하는 사람이 바로 환자이기 때문에 목표를 달성했는지 여부와 그 이유 또는 그렇지 않은 이유에 대해 가장 좋은 피드백을 줄 수 있는 사람이기도 합니다." -St1비임상 
“I do think I have learnt through that earlier experience that there is a lot of value in the patient’s feedback, because the patient is ultimately the person you either do want to convey a message to or you want to reassure. That’s the person you want to achieve your objective with, so that’s also the person who can actually give you the best feedback on whether you achieved that goal and why or why not.” –St1nonclinical

신뢰성에 대한 학생들의 추론을 분석한 결과, 환자에 대한 신뢰성 판단은 여러 가지 상호 작용하는 논거를 종합적으로 고려한다는 사실이 명확해졌습니다. 이러한 주장은 맥락, 피드백 메시지 및 이전 경험에 의해 형성됩니다(그림 1 참조). 부록 C에는 한 학생의 인과 네트워크가 표시되어 있는데, 이 학생은 환자에 대한 신뢰성 판단을 어떻게 구축했는지 보여줍니다. 
Analysis of students’ reasoning about credibility clarified that their credibility judgments regarding patients is a weighing of multiple interacting arguments. These arguments are shaped by the context, the feedback message and previous experiences (See Figure 1). Appendix C displays the causal network of a student, which shows how she built her credibility judgment of a patient.

토론
Discussion

이 인터뷰 연구에서는 임상 및 비임상 맥락에서 피드백 제공자로서 환자에 대한 의대생의 신뢰성 판단을 살펴보았습니다. 학생들은 맥락, 피드백 메시지 및 개인적 특성의 영향을 받는 여러 가지 상호 작용하는 논거를 바탕으로 환자에 대한 신뢰도 판단을 내렸습니다. 연구 결과가 문헌에 어떻게 기여하는지, 그리고 그 교육적 함의에 대해 논의합니다.
In this interview study, we explored medical students’ credibility judgments of patients as feedback providers in a clinical and non-clinical context. Students based their credibility judgments of patients on multiple interacting arguments, which were affected by elements of the context, the feedback message and personal characteristics. We will discuss how our results contribute to the literature and we discuss their educational implications

연구 결과에 따르면 학생들의 환자 신뢰도 판단은 맥크로스키의 3차원 신뢰도 모델[18]과 일치하는 것으로 나타났습니다. 학생들은 신뢰도를 판단할 때 환자의 역량, 신뢰성, 학생에 대한 선의 등 다양한 측면을 평가하는 것으로 나타났습니다. 흥미롭게도 '생생한 경험'과 관련하여 학생들이 원하는 피드백이 무엇인지에 따라 생생한 경험의 유무가 신뢰도에 영향을 미칠 수 있었습니다. 많은 학생들은 오랜 병력이 신뢰도에 기여한다고 추론했는데, 그 이유는 경험적 지식을 통해 환자들이 다른 병원 방문과 비교하여 학생의 성과를 판단할 수 있기 때문입니다. 그러나 다른 학생들은 새로운 환자가 치료에 대한 새로운 관점을 제공할 수 있기 때문에 실제로 더 신뢰할 수 있다고 추론했습니다. 따라서 학생과 레지던트 간의 상호작용과 마찬가지로, 연구 결과는 환자의 신뢰도에 대한 결정이 내용에 따라, 즉 학생들이 원하는 피드백 메시지에 따라 달라진다는 것을 시사합니다[7].
Our findings indicate that students’ credibility judgments of patients align with McCroskey’s three-dimensional credibility model [18]. We found that in determining credibility, students estimated various aspects of a patient’s Competence, Trustworthiness, and Goodwill towards the student. Interestingly, regarding the argument ‘lived experience’, depending on what feedback students were looking for, presence or absence of lived experience could both contribute to credibility. Many students reasoned that a prolonged medical history contributed to credibility, because experiential knowledge allowed patients to judge a student’s performance in comparison to other hospital visits. However, other students reasoned that new patients were actually more credible, because they could provide a fresh perspective on care. Similar to student-resident interactions, our results therefore suggest that decisions about patient credibility were content specific, meaning dependent on the feedback message that students were seeking [7].

교육 실습의 경우, 맥크로스키의 3차원 신뢰도 모델을 사용하여 학생들과 신뢰도 판단에 대해 토론하고 학생들이 자신의 주장을 정리하고 이해하는 데 도움을 줄 수 있습니다. 신뢰성에 대한 학생들의 추론에 대한 통찰력은 학생들이 어떤 종류의 학생이 어떤 종류의 피드백을 제공할 수 있는지에 대해 더 많은 피드백을 이해하는 데 도움이 될 수 있습니다. 향후 연구에서는 학생들과 신뢰도 판단에 대해 토론하면 학생들이  환자로부터 피드백 정보를 더 신중하게 찾고 사용할 수 있는지 살펴볼 것을 권장합니다.
For educational practice, McCroskey’s three-dimensional credibility model can be used to discuss credibility judgments with students, and to help them organize and understand their arguments. Insight into their reasoning about credibility might help students to become more feedback literate in terms of which kinds of patients can provide which kinds of feedback. We encourage future research to explore whether discussing credibility judgments with students will enable them to seek and use feedback information from patients more deliberately.

이번 연구 결과는 맥락이 신뢰성 판단을 어떻게 형성하는지 보여줍니다. 학생과 환자의 역할이 다른 임상적 맥락과 비임상적 맥락을 모두 살펴본 결과, 두 관계의 측면이 신뢰도에 어떤 영향을 미치는지 강조되었습니다. 흥미롭게도, 우리의 결과는 슈퍼바이저와 수련의 관계를 설명하고 피드백 과정이 그들의 관계에 의해 어떻게 영향을 받는지 이해하는 데 사용되는 교육적 제휴 프레임워크[39]와 공명하는 것 같습니다[7, 40]. 연구 결과에 따르면 이 프레임워크는 학생-환자 상호작용에도 적용되는데, 두 맥락 모두에서 학생들은 자신과 환자 사이의 교육적 동맹의 요소를 설명했으며, 이는 신뢰성 판단에 영향을 미쳤습니다[39]. 수퍼바이저-레지던트 상호작용에서 볼 수 있듯이, 피드백 제공자(즉, 환자)가 학생에게 인격체로서 관심을 보이고 학생의 학습 과정에 기여하고 싶다는 것을 보여줄 때 신뢰도가 증가했습니다[7]. 그러나 감독자와 레지던트 간의 상호작용과는 달리, 신뢰 관계는 신뢰도를 높이거나 낮출 수 있습니다[7]. 일부 학생들은 환자가 자신의 마음을 자유롭게 말할 것이라고 추론한 반면, 다른 학생들은 환자가 학생의 감정을 상하게 하지 않고 긍정적인 관계를 유지하기 위해 비판적인 피드백을 보류할 수 있다고 추론했습니다. 아마도 이 학생들은 동료 피드백에서 볼 수 있는 현상인 우정 점수 매기기(친구나 동료를 너무 가혹하게 평가하는 것을 불편해하여 과도한 점수 매기기로 이어지는 현상)를 두려워했을 수 있습니다[41]. 
Our results illustrate how context shapes credibility judgments. Exploring both a clinical and non-clinical context, with different roles for students and patients, highlighted how aspects of their relationship affected credibility. Interestingly, our results seem to resonate with the Educational Alliance Framework [39], which is used to describe the supervisor and trainee relationship and to understand how the feedback process is affected by their relationship [7, 40]. Our results suggest that this Framework also applies to student-patient interactions: in both contexts students described elements of an educational alliance between them and the patient, which affected their credibility judgment [39]. As seen in supervisor-resident interactions, credibility increased when the feedback provider (i.e. the patient) showed interest in the student as a person and demonstrated that they wanted to contribute to the student’s learning process [7]. However, in contrast to supervisor-resident interactions, a bond of trust could both increase and decrease credibility [7]. Whereas some students reasoned that patients would feel free to speak their mind, other students reasoned that patients might withhold critical feedback in order to avoid hurting the student’s feelings, and preserve the positive relationship. Possibly, these students feared so called friendship-marking, a phenomenon seen in peer feedback, in which students find it uncomfortable to grade friends or peers too harshly, which leads to over-marking [41].

임상 상황은 비임상 상황과 달랐는데, 학생은 학습자일 뿐만 아니라 의료 제공자이기도 하고 환자가 직접 치료를 필요로 하는 상황이었기 때문입니다. 따라서 학생들은 교육적 동맹 외에도 치료적 동맹도 인정했습니다[42]. 결과적으로 학생과 환자 관계의 주요 목표가 학생의 학습 과정에서 환자의 치료로 바뀌면서 권력의 이동이 감지되었습니다. 이러한 목표와 권력의 변화는 종종 환자의 신뢰도 저하로 이어졌습니다. 학생들은 환자가 자신의 학습을 돕고자 하는 선의를 가지고 있다고 믿었지만, 환자가 향후 의료 서비스에 부정적인 영향을 미칠까 봐 개선할 점에 대해 감히 솔직하게 말하지 못할 것이라고 생각하여 환자의 신뢰성에 의문을 제기했습니다[31]. 
The clinical context differed from the non-clinical context, since as well as being learners, students were also healthcare providers and patients were in the direct need of care. Thus, in addition to an educational alliance, students also acknowledged a therapeutic alliance [42]. Consequently, the primary goal of the student-patient relationship shifted from the student’s learning process to the patient’s care, which created a perceived shift in power. This perceived shift of goals and power often led to reduced patient credibility: although students believed patients were of goodwill since they wanted to help them learn, students questioned patients’ trustworthiness since they reasoned that patients might not dare to be open about points for improvement, out of fear of negatively impacting their future healthcare [31].

연구 결과는 신뢰성 판단이 단순한 것이 아니라 관계와 관련 목표의 맥락에서 여러 요소를 복합적으로 고려한 결과라는 것을 보여줍니다. 임상 맥락에서 학생들은 치료 동맹의 목표(돌봄)와 교육 동맹의 목표(학습) 사이의 상충을 언급했습니다. 한 학생의 사례에서 알 수 있듯이, 학생과 환자 간의 역할 설명과 목표 대화는 열린 피드백 대화의 장을 마련할 수 있습니다[40]. 향후 연구에서는 피드백 대화에서 환자의 역할에 대한 다양한 환자의 관점을 존중하면서 학생-환자 관계에서 목표와 역할을 논의하는 방법과 이것이 신뢰성 판단에 어떤 영향을 미치는지 탐구해야 합니다[43].
Our results illustrate that credibility judgments are not straightforward, but are the result of a complex weighing of multiple factors, within the context of relationships and their associated goals. In the clinical context, students mentioned a conflict between goals of the therapeutical alliance (caring) and goals of the educational alliance (learning). As shown by a student, role clarifications and goal conversations between students and patients can set the stage for open feedback conversations [40]. Future research should explore how goals and roles can be discussed in student-patient relationships, while respecting different patient perspectives on their role in feedback conversations and how this affects credibility judgments [43].

마지막으로, 피드백 메시지에 대한 인식이 신뢰성 판단에 영향을 미친다는 사실을 발견했습니다. 학생들은 피드백 메시지가 정보가 없거나, 구체적이지 않거나, 주로 긍정적인 피드백을 담고 있다고 생각하면 환자를 덜 신뢰할 수 있다고 판단했고, 그 반대의 경우도 마찬가지였습니다. 피드백 메시지와 피드백 제공자의 신뢰도 사이의 연관성은 일반적으로 피드백 제공자의 신뢰도가 메시지의 신뢰도에 영향을 미치는 것으로 설명됩니다[7, 44, 45]. 그러나 우리의 결과는 이것이 고리loop 형태임을 시사합니다. 따라서 우리는 제공자의 신뢰도를 피드백 메시지 참여의 전제 조건으로만 볼 것이 아니라, 피드백 메시지 참여의 결과로도 간주해야 합니다. 
Lastly, we found that perceptions of feedback messages impacted credibility judgments. Students judged patients as less credible when they considered the feedback message uninformative, nonspecific, or containing mainly positive feedback, and vice versa. Associations between the feedback message and the feedback provider’s credibility are usually described as the credibility of the provider influencing the credibility of the message [7, 44, 45]. Our results, however, suggest that it is a loop. Consequently, we should not only view provider credibility as a prerequisite for engagement with feedback messages, but also as a consequence of engagement with feedback messages.

실제로 환자 피드백에 대한 학생의 참여를 높이기 위해서는 환자로부터 유익한 피드백을 수집하는 방법을 교육하여 피드백 메시지의 품질을 높이기 위해 노력해야 합니다. 환자로부터 비특이적이고 주로 긍정적인 피드백을 받는 경우가 종종 보고되므로[23, 46, 47, 48], 구체적인 질문을 구성하고 대화에 참여하여 유익한 피드백을 요청하는 데 이 교육의 초점을 맞출 것을 제안합니다[35, 49]. 또한 환자를 위한 자발적인 피드백 교육과 설문지와 같은 피드백 도구가 이 과정을 지원할 수 있습니다[46, 50, 51]. 향후 연구에서는 이러한 교육 활동이 신뢰성 판단에 어떤 영향을 미치는지 살펴볼 수 있습니다.
In practice, to enhance student engagement with patient feedback, we should strive to increase the quality of patients’ feedback messages by training students to collect informative feedback from patients. Since receiving non-specific and mainly positive feedback from patients is often reported [23, 46, 47, 48], we suggest focusing this training on asking for informative feedback, by formulating specific questions and engaging in a dialogue [35, 49]. Moreover, voluntary feedback training for patients, and feedback tools like questionnaires, could support the process [46, 50, 51]. Future research could explore how these training activities affect credibility judgments.

향후 연구를 위한 제한점 및 제안
Limitations and suggestions for future research

이 연구에서는 학생들의 신뢰도 판단에 대한 인식에 초점을 맞추었기 때문에 필연적으로 한계가 있습니다. 연구 데이터는 회상에 기반한 것이기 때문에 신뢰성에 대한 논거가 숨겨져 있을 가능성이 남아있습니다. 또한 지각을 연구했기 때문에 제도적 문화와 같이 신뢰성 판단에 영향을 줄 수 있는 외적, 무의식적 측면도 숨겨져 있었을 수 있습니다[52, 53]. 이러한 측면을 수정하고 통제하는 실험 설계를 통해 신뢰도 판단에 미치는 영향을 밝힐 수 있습니다.
In this study we focused on students’ perceptions of their credibility judgments, which inevitably brings limitations. The study data is based on recall, which leaves the possibility that arguments for credibility remained hidden. Moreover, since we studied perceptions, external and subconscious aspects that can influence credibility judgments, like institutional culture, might also have remained hidden [52, 53]. Experimental designs that modify and control these aspects could reveal their impact on credibility judgments.

또한, 연구 대상 집단이 숙련된 의료 전문가로 제한되어 있어 데이터의 전달성이 제한될 수 있습니다. 임상 현장에 있는 대부분의 학생들에게는 환자에게 피드백을 요청하는 것이 처음이었기 때문입니다. 연구 결과에 따르면 신뢰성 판단은 피드백을 받는 사람의 과거 경험에 의해 부분적으로 형성됩니다. 따라서 숙련된 의료진은 환자의 신뢰도를 다르게 판단할 수 있습니다. 그러나 우리의 목적은 학생의 환자에 대한 신뢰도 판단에 대해 심층적으로 탐구하는 것이었습니다. 따라서 우리의 연구 결과는 (미래의) 의료 전문가가 환자에 대한 신뢰성 판단을 내리는 복잡한 과정을 밝히기 위한 향후 연구의 출발점이 될 수 있습니다. 
Moreover, our study population might limit the transferability of our data to experienced healthcare professionals. For most students in the clinical context, it was their first time asking patients for feedback. Our results show that credibility judgments are partly shaped by past experiences of the feedback receiver. Therefore, experienced healthcare professionals might judge a patient’s credibility differently. However, our purpose was to conduct an in-depth exploration of students’ credibility judgments regarding patients. Our results therefore serve as a starting point on which future studies can build, to further unravel the complex process by which credibility judgments regarding patients are made by (future) healthcare professionals.

결론적으로, 의대생들은 환자의 역량, 신뢰성, 선의의 측면을 판단하여 환자가 신뢰할 수 있는 피드백 제공자인지 여부를 결정합니다[18]. 학생들의 판단은 관계와 관련 목표라는 맥락에서 여러 가지, 때로는 상충되는 요소들을 종합적으로 고려하여 이루어집니다. 임상적 맥락에서 학생-환자 관계의 치료 목표와 교육 목표 사이에 인식되는 긴장은 신뢰성을 떨어뜨릴 수 있습니다. 향후 연구에서는 학생과 환자 간에 목표와 역할을 논의하여 열린 피드백 대화의 장을 마련할 수 있는 방법을 모색해야 합니다. 
In conclusion, medical students judge aspects of patients’ Competence, Trustworthiness and Goodwill to determine whether they are credible feedback providers [18]. Students’ judgments are a weighing of multiple and sometimes conflicting factors, within the context of relationships and their associated goals. In the clinical context, perceived tensions between therapeutic goals and educational goals of the student-patient relationship can diminish credibility. Future research should explore how goals and roles can be discussed between students and patients to set the stage for open feedback conversations.

 


Perspect Med Educ. 2023 Apr 12;12(1):129-140. doi: 10.5334/pme.842. eCollection 2023.

Patients as Feedback Providers: Exploring Medical Students' Credibility Judgments

Affiliations collapse

1Faculty of Medicine, Utrecht University, the Netherlands.

2Department of Pediatrics, University Medical Center Utrecht, the Netherlands.

3Utrecht Center for Research and Development of Health Professions Education, University Medical Center Utrecht, the Netherlands.

4Freudenthal Institute, Utrecht University, the Netherlands.

PMID: 37064270

PMCID: PMC10103723

DOI: 10.5334/pme.842

Free PMC article

Abstract

Introduction: Patient feedback is becoming ever more important in medical education. Whether students engage with feedback is partly determined by how credible they think the feedback provider is. Despite its importance for feedback engagement, little is known about how medical students judge the credibility of patients. The purpose of this study was therefore to explore how medical students make credibility judgments regarding patients as feedback providers.

Methods: This qualitative study builds upon McCroskey's conceptualization of credibility as a three-dimensional construct comprising: competence, trustworthiness, and goodwill. Since credibility judgments are shaped by the context, we studied students' credibility judgments in both a clinical and non-clinical context. Medical students were interviewed after receiving feedback from patients. Interviews were analyzed through template and causal network analysis.

Results: Students based their credibility judgments of patients on multiple interacting arguments comprising all three dimensions of credibility. In estimating a patient's credibility, students reasoned about aspects of the patient's competence, trustworthiness, and goodwill. In both contexts students perceived elements of an educational alliance between themselves and patients, which could increase credibility. Yet, in the clinical context students reasoned that therapeutic goals of the relationship with patients might impede educational goals of the feedback interaction, which lowered credibility.

Discussion: Students' credibility judgments of patients were a weighing of multiple sometimes conflicting factors, within the context of relationships and their associated goals. Future research should explore how goals and roles can be discussed between students and patients to set the stage for open feedback conversations.

 

새로운 의료인문학(들)과 한국 의료인문학의 자리* (Philosophy of Medicine 2023)
최은경(경북대학교 의과대학 의료인문·의학교육학교실)

 

1. 서론

의료인문학은 아한국에서 생소한 학문이다. 한국에서 의료인문학을 설명할 때 대부분은 이 학문이 설고 지의 학문에 가깝다는 반. 하지만 한국에서 의료인문학 중 중요한 분과학문으로 여겨지는 의사학(醫史學, history of medicine)이 의과대학 내에 자리 잡은 것은 만 75년 전인 1947년이다.1) 방 후 를 정비하는 의 상황에서도 의학의 역사를 설명하고 탐색하는 것이 의학교육에서 설다고 여겨지지 않았다는 것이다

의학이 한국에서 새로운 형태로 형성되어야 할 이 시기에 의사학 (醫史學)학문이 의학 아카데입된 식은 여러 가지로 미롭. 아마도 대다수 의사에게도 의학의 역사를 배워야 하는 물음에 대해 명이 나지는 을 것이다. 의과대학에서 소화해야 한다고 여겨지는 은 과과 비해서는 더욱더 그러하다. 그럼에도 의사학 과목이 오늘날까지 한국 의과대학에서 많이 가르쳐지고, 일종의 중요한 교양과목과 같이 남겨져 있게 된 까닭은 무엇일까.2) 그것은 인문학이 의학이학문에 영향바와 여러모로 관지어 생각하지 을 수 없다. 서양에서는 중세 대학 때부고전과 문구하는 기이 모학문 연구에서 필수였다. 의학 역시 갈스를 비롯한 위대한 이론가들의 원전을 우는 것이. 과거에는 역사학적 문을 고하는 것과 의학은 개가 아니. 역사적 리는 다르나 한의과대학에서 고전 원전을 는 것이 으로 아 있는 것 한 이러한 관적으로 이해수 있다.

의학에서 과학적 법론과 임상 증례 방법론이 부상하기 시작한 19세기 이에도 문화와 문의 비구는 의학의 한 법론으로 남았. 르나르를 중심으로 실험의학 법론이 등장하고 실험실 내 구가 지식 생산 법론이 되으나 질병의 과거 모이 어하였는 지는 실험실에서 재현되기어려. 헌의 비판적 검토를 통해서 질병의 역사 지리적 분포를 탐구하는 것은 초기 의학사의 방법론이었고, 이후 학제로서의 의학사가 되었다.3) 한국에서 의학사는 제국대학 의 영향을 받은 식민지 의학 엘리트들의 교양으로 처음 시작되었다고 볼 수 있다. 일제강점기 제국대학에서 학부와 사를 마친 윤은 해방 후 두종함께 대학의과대학에 의사학실을 설립 하고 대한의사학회 창립을 이하면서 의사학이 의학교육의 분과로 는 데 기여하였다. 여기에는 그가 토제국대학에서 형성한 양주의적 관심이 영향을 것으로 작된다.4)

그럼에도 불구하고 한국 의료인문학의 역사에서 오래된 의사학의 역사는 곧잘 누락된다. 이는 학문으로서의 의료인문학이 의과대학 내에서 등장한 이와 이전의 역사가 절된 . 그러나 으로 한국 의학의 역사에서 의료인문학을 소거시으로써 한국 의학의 성장과 의료인문학을 개로 간주하는 경과도 관없지 . 이러한 경은 한국 의학의 맥락에서 한국 의료인문학의 위치를 사유하기 어게 만. 나아가 의학의 주요한 요소로서 의료인문학의 위상과 역할을 고려하기 난망하게 만.

본 글은 다양한 의료인문학의 맥락과 갈래 속에서 한국 의료인문학의 역할을 탐색하는 것을 적으로 한다. 이를 위해 우‘인문학으로서의 의학’, 의료인문학이 의과대학에서 하나의 학제로 형성되어 온 과정을 추적한다. 특히 전인적 인간 형성을 표로 하는 인문학 부 가 타인에 대한 연공감 등을 요청하는 의과학 반성의 요구와 으면서 의학교육에 강조되는지를 살펴본다. 1990대 이국내에 부상한 의료인문학 도입 경으로서 의학교육의 글로라이제이을 살, 이를 의학을 인문학적 요청이라는 측면에서 비적으로 검토한다. 마지으로 한국 의료인문학의 위상과 역할을 ‘전문성’과 ‘’이라는 역을 로 들며 제하고자 한다.

1) 1947년 서울대학교 의과대학에서 수립된 의사학교실이 수립된 해방 후 최초로 설립된 의사학 전담 부서이다.
2) 2014년 예방의학회에서 전국 의과대학 의료법규와 인문사회의학 교육현황을 조사한 데 에 따르면 60% 이상의 대학에서 해당 명칭으로 개설되어 있다. 의료법규 및 인문사회 의학 교육개선 특별위원회(2014).
3) 최초의 근대적 의미로 의학의 역사에 관한 서적을 남긴 사람은 커트 스프랭글(Kurt Polycarp Joachim Sprengel, 1766-1833)이었다. 그는 실용적 문헌 고찰을 통해 문 화의 역사로부터 의학의 이론을 추론해 낼 수 있다고 믿었다.
4) 홍종욱(2018), 185-204. 홍종욱은 윤일선이 교육을 받은 구제 고등학교와 제국대학 이 ‘교양주의’의 산실이었고, 그 ‘교양주의’는 ‘학력엘리트의 신분문화’를 낳았다고 설명 한다. 윤일선은 해방 후 서울대학교 의과대학 교수이자 초대 대학원장을 역임하며 서울대학교 의과대학에 의사학교실이 설치·운영하는 데에 기여하였다.

2. 의학교육에서의 의료인문학

“인문학으로서의 의학”은 달리 말하면 ‘의생명과학으로서의 의학’이 급부상한 후 만들어진 거울상이다. 과거 의학은 이론과 의학 간의 연결고리가 강하였다. 하지만 과학적 법론이 부상하고 주화되면서 문헌 탐구와 은 의학의 래된 인문학적 법론은 일의 그, 여물로 남았. 우가 '문화'에서 지한 상호 이해 불능한 문화처럼 의과학과 인문학은 상호 다른 두 문화의 지칭처럼 여겨지기도 하였다.5) 그러나 의학에서 인문학적 요소가 완전히 배제된 적은 없다. 도리어, 인문학적 법론이 과학적 법론으로 대체되면 록 오히려 인문학과 의과학은 어더욱 더 긴장된 관계에 여 있다고 수 있다. 1919년 오슬러가 마지대중 강연에서 의과학과 인문학의 ''을 비을 때 그는 과거를 히포크스와 갈스를 아는 것만으로 진료하기에 했던 시간으로 상한다.6) 주의 시대에 접어 과학 법론이 더욱 많성하면서 의과학 법론은 인문학적 위와 무게가 받아들일 만한지, 과학 이론에 비어 효과적인지 물. 반면 인문학은 도리어 의과학의 법론에 비어 인문학의 효용을 고 되리고자(revival) 하였다. 의학 지식이 점차 세분화되고 교육제도가 격하고 과학적 법론이 기를 잡았던 20세기 전반, 도리어 오슬러와 같은 의사들은 의과학에 대비되는 의학의 오랜 이상은 인문학에 있으며, 의학교육의 핵심에 자리를 잡아야 한다고 보았.7) 그것은 질병과 병리로 환원되지 는 인간에 대한 은 이해를 바으로 한 동정과 공감이 인간을 치유할 수 있다는 이상을 전제로 한 것이. 오슬러가 보기에 과학은 상대적으로 냉혹하고, 인간을 전체적인 존재로 간주하지 않으며, 비인간화할 가능성이 높아서 인문학을 갖춘 의학은 이러한 과학의 독성을 치유할 수 있는 해독제와 같은 것이었다.

오슬러 등의 우려 이'의료인문학'이 하나의 별도 영역이자 학제 로서 등장한 것은 1990년대로 접어들고 나서이다. 의료를 형용사로 조합어인 '의료인문학(medical humanities)'이 하나의 간-학문적인 학제로서 와 유을 중심으로 의학교육의 새로운 물결로 도입되 기 시작하였다. 의학의 인문학은 의학의 '비인간화'에 대한 우려, 고령화로 인한 만성 질병 중심의 의료 이용의 시대의 도래 등에 힘입어 의과학에 대한 환희가 잦아든 이후에 본격적으로 중래하기 시작하였다. 물론 1960-70대에도 인문학에 가운 학문으로 생명리학이학제가 새게 등장하였다. 이 학제는 주로 기존에는 생각하지 못했 던 생과 음의 리적 문제를 다루으며, 의료 자원 분와 인간 대상 연구의 리적 이등 주제를 . 한 에드문드 그리나 에은 명망 높은 의사-학자들은 오슬러의 이상을 이어받아 생명리학 학제 내외에서 교류하며 '인문적 의사' 에 대한 이상을 논의해 나. 문학 전공자들을 중심으로 개별적인 의료인문학 교육들이 조금씩 이루어지고 서사의학에 대한 관심이 생 겨난 것도 이때라 할 수 있다.

다면, 의료인문학이학제가 본격 등장하기 이전인 이 시기에서 다의학 내 휴머니즘, 또는 인간학이란 무엇일까. 그리는 르상스와 계주의에서 비롯된 휴머니즘의 이상이 의학의 인간적 가치에서도 숨쉬어야 한다고 보고 합리적이고 과학적인 요소로 자하면 이를 러서는 안 된다고 보.8) 그가 보기에 은 문학적 교육학적 이상이, 교육적 인식론적 이상(그리스 어원으로 paideia)과에 대한 공감''(그리스 어원으 로 philantropia) 가지 요소를 것이. 의학 내 휴머니즘 (humanism in medicine)은 의사가 인식론적으로 언어와 기예를 갖춤과 동시에, 인간으로서의 환자에 대해 의사가 정념적인 감정을 가지는 것 둘 다를 의미하였다. 전자는 자유게 계된 인간이라면 갖추어야 할 기다면 자는 관적 치료라는 위를 어서 인격으로서의 환자가 가지는 유일을 마주하는 것이. 그리전자의 표를 위해 고전 언어의 습득과 문헌의 독해 등 전통적인 인문학 훈련에 반드시 기댈 필요는 없다고 보았. 하지만 구에게도 속되지 교육받은 자유인으로서의 의사는 비판적으로 현상을 바라보고 사고하는 훈련을 거쳐야 했고, 단지 기술자인 동시대인으로 부터 자신을 구분할 수 있어야 하였다. 

5) Snow(1959)/ 오영환 역(2001).
6) Osler(1919), pp.1-7.
7) Osler et al.(2002).
8) Pellegrino(1974), pp.12881294.

그리가 의학교육의 인문학 필요성을 강조한 데에서 수 있 , 의학 내 인문학 전통의 재소환은 20세기 들어서 과학적 기술의 독점과 방법론의 강화에 대한 반발로 제시되었. 의과학 법론의 대는 주로 가지 차원에서 기존의 의사상에 대한 위협처럼 여겨. 과학기술에 대한 으로 인해 한기능인으로 전할 것에 대한 우려의 한으로 환자를 지 기술의 대상으로 소외시킬 것에 대한 우려가 존재. 기간 의학 내 '인문학'용어가 지하는 바는 때로는 교양인으로서 의사가 지녀야 할 품성이었고, 의사가 가져야 할 인간에 대한 이해였다. 의 비적 해을 통한 인 문학적 법론은 전자를 위한 방편자는 고통을 이해하고 동정과 공감을 나는 인문학적 치유 이상이. 그러나 두 가지가 꼭 '인문학'이란 용어로 통합되어야 하는 것인지에 대해서는 불분명한 채로 남아 있었. 왜냐하면 인문학적 법론과 인문학적 치유 이상이 일치하는 것이 아니며, 때로는 갈등적일 수 있기 때문이다. 그리역시 인문학의 특성에서 'paideia'의 요소와 'philantropia' 의 요소를 구분하여 제시으로써 다 의학에서 필요을 설하였 으나, 적은 똑같않았. 하나는 의사가 교육받은 젠틀맨이자 사회의 주요 구성원으로서 지녀야 할 역량과 품성을 위해 필요한 것이었고, 다른 하나는 의사가 환자를 대할 때 발휘해야 할 덕목에 가까웠다.

의사가 기능인 이상의 직업인이 되기 위해서 인문학은 우초 교교육으로서 의사로서의 성장에 기여해야 . 의학교육 영향보고서를 작성하면서 과학적 구 능력을 양하는 것을 강조한 한 인문학은 기초 교교육으로 필수적으로 전제되어야 한다고 보.9) 

9) Doukas(2010), pp. 318–323. 플렉스너는 자신의 보고서를 통해 과학 커리큘럼 중심 으로 미국 의학교육을 재편하는 데에 큰 영향을 끼쳤으나 15년 후 '의학은 오늘날 문화적 철학적 배경을 잃어 가고 있다'라며 후회하였다고 한다. 플렉스너는 의사는 환자 돌봄의 영역에서 특히 인문학적 역량을 갖추어야 한다고 보았다.

그러나 인문학은 양인으로서의 의사를 비하는 기반에만 국한된 것이 아니. 1984년 펠레그리노는 의학 내에서 인문학을 가르쳐야 하는 이유에 대해 더욱 강조하면서 다음과 같은 이유를 들었다. 10) 의사는 인문학을 동반자적 구로 면서

  • 1) 환자와의 관계에서 그리고 과학자 사회에서 언어를 읽고 이해하고 판단하는 사고 능력을 교육하는 교양을 얻고 마음을 자유롭게 할 수 있고
  • 2) 과학적으로 설명 하거나 측정하기 어려운 현상의 심상(imagination)을 자유롭게 하며
  • 3) 인간으로서의 '영혼의 즐거움'을 함양할 수 있었다.

, 초교양인 뿐만 아니라 임상의로 성장하는 데에도 인문학은 임상의로 성장하는 데에 없어서는 안 되는 지속적인 자양분이 될 수 있었, 그 성장 과정에서 더욱 성해질 수 있. 세 가지 모환자의 고통이라는 미망 한가운데에서 의사가 한 명의 계된 자유로운 으로 살아갈 수 있는 해결.

인문학이 의사의 성장에만 도거라 믿은 것은 아니. 초기 의료인문학자들은 실천적 영역에서 인문학은 환자를 단지 기술의 대상으로 축소하지 않고 환자의 고통의 의미를 해석하고 포용하는 것을 도울 거라 보았. 인문학적 양에 대한 강조가 환자에 대한 이해를 이리라는 것에 대해 의문을 제기할 수 있다. 특히 인문학적 양에 대한 강조가 지 자들만의 리트주의적 취향과 이해로만 결된다면 공감 등의 성을 강조하는 것이 무의할 것이다. 이것은 '인문학(humanities)' '인문주의(humanism)'란 단어에 내장이기도 하다. 래전 로마의 시세로(Cicero)가 인문과 부라 는 용어를 결합한 소위 liberal arts, "studia humanities"라는 인문학 을 설한 이래 인문학은 교육의 이상으로 자리 잡았. 하지만 paideia와 philantropia가 같은 인문학(humanities)에 묶일 수 있을지에 관해서는 고대부터 의문의 여지가 있었.

  • 이를테면 로마 저술가 아 울루스 겔리우스(Aulus Gellius)는 humanitas와 philanthropia의 결합이 잘못된 해석이라고 주장했다.11)
  • 독일 교육철학자 프리드리히 니데 머(Friedrich Niethammer) 역시 시세로의 인문학 개념을 재발견하면 서 제2의 인문주의(humanism)운동을 불러일으켰으나 동료 시민의 교양을 부흥시키는 박애주의 교육(philanthropic education)에는 반대했다.12)
  • 그럼에도 불구하고 교양으로서의 인문학의 특징이 희석되면서 오늘날 인문주의는 로크와 루소의 교육의 이상, '인간에 대한 사랑', 즉 박애주의적 교육의 의미에 더 무게를 두는 것으로 이해된다.

서구의 역사에서 교육과 의술은 박애주의라는 전통을 공유한다. 특히 의료에서 각인간에 대한 아가페적 헌신, 주의적 전통에 대 한 강조는 의사-환자 관계치유(healing)라는 이상을 소환한다. 인으로서의 의사가 이러한 치유의 이상과 마나 밀하게 결합할 수 있을지는 운 물음은 아니다. 궁극적으로 신으로부터만 가능하다고 여긴 치유의 이상을 세속 인간의 몫으로 넘기는 순간 긴장은 피할 수 없을지도 모른. 그리를 비롯한 의학-인문학의 구자들은 이 사라진 시대 의술의 모은 의사-환자 관계 속에 있다고 보. 은 치유하는 관계로서 환자-의사 관계가 내하는 본질적 구속에 가, 의과학만으로는 히기 어려운 성질을 고 있.13) 의과학 방법론에 대한 지나친 강조는 치유하는 관계로서의 환자-의사 관계가 소외될 가능성이 있는데, 특히 고통에 대한 적절한 이해력이 훼손될 수 있었. 고통은 지 생물학적 지에서 한 사람의 유기체로서의 조화가 실하는 것만이 아니. 유기체로서의 조화에 별 이상이 없더라도 고통은 겪을 수 있었다. 체적 고통을 어 고통은 한 사람의 자의 인생에서 상실과 훼손의 위협에 을 의하기 때문이. 은 의학교육에서 인문학의 장소를 물으면서 과학으로 국소적이고 제한된 이해만으로는 환자가 가져오는 기대, 이해, 판타지, 의미, 공포, 두려움을 직면할 수 없고 온전히 이해하기 어렵다고 보았다.14) 의료인들은 계속 환자의 고통과 음을 마주할 수에 없다. 이를 받아들이는 법은 인간의 어를 다루고 소화하고 어 이면을 다수 있는 능력을 통해서이다. , 인문학적 이해와 능력은 고통에 관한 과학적 구와 개념화를 지하고 고하 는 데에 도것으로 기대되. 

10) Pellegrino(1984), pp. 253-266
11) Adler(2020).
12) Becker and Becker(1992). p.801.
13) 펠리그리노는 임상의학의 내재적 모럴(internal morality)는 아리스토텔레스식의 ‘좋음’의 목적 속에 내포되어야 하며, 결과주의적 목적과는 구분된다고 본다. 그리고 이것이 의료전문직 윤리의 근간이라고 주장한다. Pellegrino(2011a), p.63 참조.
14) Cassell(1984).

3. 서구의 의료인문학 교육 등장과 글로벌 의료인문학, 한국 의료인문학

국과 등지에서 1990대 들어 의료인문학이 일의 새로운 물결처럼 등장하였을 때 에서 언급한 의료인문학 교육제 모를 다루고 있다고 보.

  • 1972년 문학 전공자가 펜실베이니아 의과대학 교수로 처음 임명된 이후 북미에서는 문학 전공자들 이 주도하여 <문학과 의학> 붐을 주도하였고 의료인문학을 대표하는 흐름으로 자리 잡았. 이 당시 의료인문학은 의사들의 감성적 성장과 환자의 질환과 고통을 하나의 스토리로서, 서사로서 이해하는 것을 목적으로 하였고, 사회적 변화와 그 속의 의사의 위상 변화는 좀 더 다른 이야기였다.
  • 1990년대 들어 환자 중심 권리의 등장과 의료의 시장화, 관료화 등의 변화 속에서 의료계에서도 기존의 견고했던 교육받은 젠틀맨이라는 의사의 위상이나 그의 가부장적인 권위가 쇠퇴하기 시작하였다. 의사라는 전문직업성 교육이 새롭게 등장하였고, 변화하는 사회에 능동적으로 대응하는 전문가 형성을 목표로 하였다. 펠레그리노 등이 덕 공동체로서의 의사 직업공동체를 강조하고 전문직 관계의 덕목을 촉구하였을 때 이는 의료인문학 교육의 얘기에만 국한된 것이 아니었다.15) 미국의 전문직업성 교육 주창은 전미 의과대학 학장협의회, 졸업 후 의학교육 인증위원회, 미국 내과의사회 등 에서 이루어졌으며, 근본적으로 이들 의료계가 사회와 맺고 있는 책무와 깊게 관련지었다.

국내의 맥락좀 더 다르다고 보아야 할 것이다. 한국 의과대학에 서 의료인문학 교육이 어떻게 서구와 다른 영역들을 포괄하게 되는 지에 관해서는 복규, 중의 글에서 잘 다루고 있다.16) 의사학이란 오래된 학제가 있었지만, 한국에서 의료인문학 교과목이 본격 소 개된 것은 2000년 의사 파업 이후였다. 처음으로 단체 행동에 나서면서 의사들은 소위 '사회화'의 과정을 겪었고, 전문직으로서의 의사의 힘을 각성하였다. 인문학 교육은 사회와의 '소통'의 측면에서 강조되었. 이는 의료계가 사회로부터 고립된 측면을 탈피하고 사에서 재차 리을 발하는 데에 도수 있을 것으로 기대하였다. 과거 국에서 교육득한 학생들이 의과대학에 진입으로써 전문으로서 지위를 고히 하는 데에 도수 있을 것이라는 기대가 있다면, 다시 인문학 교육을 통해 사회에 대한 이해와 의사 소통 능력이 향상될 수 있을 것으로 여겼다.

그러나 인문학적 이해와 교육의 토대 위에 분히 이지 한국 의학교육에 도입된 의료인문학 교육, 그동안 생소했던 윤리와 인문학에 관한 갖가지 교육과정이 의과대학에 난무하는 결과로 이어졌. 의료인문학은 기존의 의료와 사에 관한 교육당해 왔예방의학, 의료관리학 등의 학제와의 구분이 불분명한 채로 '인문사회의학'이란 용어로 뭉뚱그린 채 회자되었. 그리고 20년 후 또 의 거대한 의사 이 일어나고 의료시스과학이새로운 교 육과정들을 도입하려는 움직임이 지면서 인문학 교육다시 자의 존재 의를 물어야 할 필요에 . , 주지하다시서구와 한국의 의료인문학교육은 의과학에 대한 반성과 그걸로 소급되지 않은 의학의 가치를 강조하는 일환으로 도입된 것이 아니었. 인문학적 요소가 개화되지 않았던 과거 의학교육에 대한 래된 기을 가지고 도입된 것도 아니. 그것을 도입한 이들은 '' 인재이자 전문인으로서의 의료인이 수 있기를 하였다. 한국 의사들은 노동자성, 즉 일종의 노동자 정체성으로서 파업을 수행하였으나 그 결과로 노동자성과 다른 전문직업인 교육으로서 의료인문학 교육을 도입하였다. 그래서 한국의 의료인문학 교육은 의과학 방법론 강화에 따른 인간 가치의 소외라는 서구의 테제보다는 주로는 의료계에 대한 정부의 인정과 리더십에 대한 불만에 초점을 둔. 그리고 과학 이후 또는 과학 외부에서 학문으로서의 의료인문학이 지니는 특수성-즉, 환자의 관심사나 환자의 이해를 고려하며 의사-환자 관계의 가치를 다루는 특수성이 그다지 많지 않. 한국 의학과 의료계에서는 소위 의과학 중심성에 대한 반성과 한계라는 서구 의료인문학의 주요한 테제가 주류가 되지 못한다.

하지만 이를 한국 의료계의 협소한 이해 때문이라고 국한하는 것은 구조적 측면을 소거한 타자화의 식에 지나지 을 것이다. 해야 할 점은 의생명과학이 글로벌가 된 한으로 의료 인문학 역시 글로가 되다는 점이다. 글로화된 의생명과학의 지식 생산과 환시스에서 의료인문학은 게 논외가 아니며, 전세계적 의학교육인증 시스에서 의료인문학은 "관련 행동 및 사회과학 영역(relevant behavioural and social sciences)"으로 다소 애매하나 주요한 인증 기준으로 자리잡고 있다.17) 여기서 인문학이라고 명시되어 있지 는 데에는 의료인문학의 가치에 대해서는 인정되나 그것이 글로의학교육의 표이자 역으로 정의되기에는 모호과 어려이 있기 때문이 아닌지 의심된다. 의학교육의 글로화의 움직임은 의생명과학 기술이 글로화되고 그에 따른 보건·의료인력 환자, 자원의 이동이 심화하면서 글로벌 보건(global health)이 중요한 화두로 제시되는 데에 비롯된다.

  • 다양한 자원의 환경 속에서 보건인력들은 기존의 의사, 간호사, 기타 치료사 등의 인력 양성 구분을 넘어선 역량 중심 양성에 기반을 둔 협력을 요구받으며 의사소통 및 문화적 역량, 윤리적 행동능력 등을 키울 것을 요구받는다.18)
  • 그리고 글로벌 보건 인력들이 개발되어야 할 역량으로 "유연성, 적응성, 문화적 감수성, 간-문화적 소통 역량" 등이 제시되면서 글로벌 보건 인문학 영역 또한 탐색에 들어서기 시작한다.19)
  • 최근에 한센(Narin Hassan) 등은 COVID-19를 계기로 글로벌 보건 인문학이 전환기에 들어섰다고 주장하며 20) 글로벌 보건의 하위 개념으로 글로벌 보건 인문학이 위치하는 것이 아니라 보건 인문학의 하위 개념으로 글로벌 보건 인 문학이 위치해야 한다고 주장한다.

그러나 개념적인 바람직함개로 오늘날 의료인문학 또는 보건인문학(health humanities)이 글로벌 보건만큼 영향력 있는 개념이 되었다고 보기는 어렵. 의료인문학의 글로화는 필연적으로 의과학기술을 수하고 적용되는 식과 지식 생산식이 이상 하나의 전통과 양태에 국한되지 음을 보여. 전세계적으로 기능인으로서 인으로서 글로보건을 하는 의사 "행동 과학 지식" 역시 득하며 글로세계시으로서 일차의료 역을 수행할 수 있어야 한다. 글로벌화의 지평에서 대두된 의료인문학의 가치는 오늘날 모두에게 건강을 추구하며 초국가적 협력을 강조하는 글로벌 건강 레짐과 별개로 사고하기는 어렵. 그것은 각국의 의학교육의 전통과 인문학교육의 전통이 어떠했는지, 전문직업성의 전통이 어떠했는지와는 큰 관련이 없다. 의 특수성은 자원의 격차와 문화적 차이 속에서 주로 거론되며, 의료인문학 교육의 비전은 글로벌 건강 세계 속 시민으로서의 비전과 좀 더 관련이 있다.

15) Pellegrino(2011b).
16) 권복규(2022); 김택중(2022).
17) World Federation for Medical Education(2020).
18) Frenk(2010), pp.19231958.
19) Stewart & Swain(2016), pp.25862587.
20) Hassan & Howell(2022), pp.133137.

그러므로 국내 의료인문학 교육의 현황을 논하면서 서구의 의료인 문학의 전통에 가음을 논하는 것은 일할 만하나 맥락바르게 이해한 것이라고 보기는 어. 주지하다시, 한국 의과대학이 공고한 기존의 교실 기반 의학교육을 넘어서 의료인문학 교육을 대대적으로 도입한 데에는 1990년대 중반부터 본격화된 한국 의학교육인증원의 의과대학 인정 평가, CanMEDs, 세계의학교육연합회 (World Federation of Medical Education, WFME)를 위시한 글로벌 보건의료교육기관 인증 체계의 도입에 있다.21) 의학교육이 글로스 체계에 입되면서 의료인문학의 도입 역시 강화되. 그러므로, 오늘날 건강 레짐 속에서 로컬의 의료인문학은 글로벌 의료인 문학의 일부로 기능할 수 있기를 기대받는다. 글로건강 속에서 오늘날의 의사는 로컬의 '현장 속에서(in)' 근무할 수 있는 인력으로서 로컬의 '옆에서(by) 로컬의 요구를 다룰 수 있는' '글로벌 관점' 으로 훈련받기를 요구받는다. 글로건강 속 의학교육은 경제성과 이타성을 재할 것을 요구받는다. 22) 그러므로, 한국 사회를 주도하는 인문학적 소양을 갖춘 고급의 의사 인력을 양성할 필요가 있 다는 엘리트주의적인 기대는 글로벌화된 의학교육의 맥락에서는 실현되기 어렵. 한국 의료계가 응당 한국의 의료 전통을 이해해야 하고 새로운 의료인문학 전통, 어쩌면 새로운 의학교육 및 교육의 전통을 쌓아가야 한다는 부름은 존재할 수 있으나, 적어도 최근 한국 의과대학에 도입된 의료인문학은 그런 맥락에서 도입된 것은 아니다. 그것은 한국 의사가 한국의 현장이란 전통 속에서, 그리고 전통을 비판적으로 사유하며 전통 옆에서 일할 수 있기를 요구하나 한국의 전통을 새롭게 창의적으로 건설할 것을 요구하지는 않는다. 글로세계화의 요구가 그러하, '의적으로' 되는 바는 글로의료 시이며, 한국의 의료 전통이 아니다.

21) 2000년대 이후 의과대학 의학교육 평가인증을 주도해 온 한국의학교육평가원에서는 세계의학교육연합회의 2015년 기준을 참고하여 2017년 ASK2019(Accreditation Standards of KIMEE 2019)를 개발, 평가 인증에 활용하기 시작하였다. 이 때 세계 의학교육연합회의 2015년 기준에 걸맞게 평가항목에 의료인문학 항목을 별도 삽입하 여 ‘의과대학은 의료인문학 교육과정을 운영한다’를 평가하고 있다. 다만 의료인문학 은 “행동과학, 사회과학, 의료윤리, 의료법규 등을 포함”한다고 정의하여 의료인문학 의 본래적 의미와 맞지 않는다는 비판이 존재한다.
22) Prideaux(2019), pp.2531.

 

4. 글로벌/로컬의 긴장과 비판적 의료인문학의 가능성

되어 의료인문학이 꼭 글로벌화의 요구에 복무해야 할 것인지에 대해 물음을 질 수 있을 것이다. 를 중심으로 시작된 CanMEDs 이나 World Federation of Medical Education (WFME)의 의학교육 인증 흐름은 정확하게는 의학교육의 경제적 비용 절과 분산적 , 그리고 각국의 의사 지망 엘리트들의 국적 을 가능하게 한다.23) 보건의료의 세계화라는 시대적 흐름은 거스를 수 없다 하더라도 그것이 초래할 결과가 글로벌 복지의 증진일지는 물음을 던질 수 있다. 그것은 로의 의료 수요를 논하지만 그를 통해 받은 로리트가 그에 무하지 게 만들 수 있으며, 과거의 서구 의학교육 전파의 예처럼 광범위한 두뇌 유출로 이어질 수 있다. 무성 교육이 글로의학교육 인증의 주요 기이나 그 결과 양성된 의사는 컬의 의료 수요에 복무하는 책무성 있는 의사가 되기보다는 해외의 의료 수요의 흐름에 따르기 쉽다. 더 나아가 이들은 더 의료 취약 지역의 수요에 따른 인력 배치에 저항한다. 이는 공공의대 설립 움직임에 격히 저항했던 한국의 의사 에서도, 스트일리아의 의료인력 치 문제에서도 전세계가 반적으로 면하고 있는 문제이다.

의학교육의 글로벌화에 직면해서 그것이 과연 현재 의료가 봉착한 문제를 해결하고 로컬의 의료 수요를 충족할 수 있을지에 관해서는 다양한 비판을 할 수 있을 것이다. 한 가지는 현재 의학교육을 주도 하는 역기반 교육이나 표화된 글로교육 방식이 과연 의료가 한 문제의 해결에 바람한지 비하는 것이다. 마가이 가 잘 지적하, 역량 기반 교육(competency-based learning)은 의과학 기술에 대한 단순한 기술 전달(technical knowledge) 에서 벗어나 있다는 점에서 더 우월한 것처럼 여겨지나, 사실상은 지식 습득의 한 가지 방법만을 고수하는 방법이다.24)

  • 역량 기반 교육은 역량을 결과 이자 성과로 객관적으로 측정하고, 계량화하고, 표준화하는 것을 목표로 한다. 이 교육은 전문직의 이해와 행동의 복잡성과 미묘함, 개개인 의 고유함과 독특함을 무시하고 그 결과를 수치로 환원한다고 비판받는다.
  • 그뿐 아니라 의료, 그리고 의료인이 사회와 맺어 온 관계를 알고, 이해하고, 관계를 새로이 맺고자 할 때 필수적인 개별 개체들과의 무수한 상호작용의 맥락과 과정들을 소거한다. 단순한 상처의 봉합이나 술기의 재현은 객관적으로 측정 가능하다. 환자를 대할 때 공감의 표현을 하고 위기 상황을 해결할 수 있는 능력 역시 가능할 수 있다.25)
  • 그러나 복잡한 사회적 환경 속에서 조건을 숙고하고 자신의 가치를 실천적으로 지켜나가는 방법, 한 사람의 전문가이자 시민으로서 의 삶을 영위하고 관계를 맺는 방법을 글로벌의 관점에서 객관적으로 측정하는 것은 어렵거나 불가능하다.
  • 쿠마가이는 역량 기반 학습이 의료인이 될 수 있는 다양한 앎의 방법(ways of knowing)들을 담아 내지 못한다고 주장한다. 26) 이는 오늘날 의학교육이 자율적으로 스스로를 성찰하고 규제하며 가치를 적용할 수 있도록 전문직업성과 윤 리를 함양하는 것을 목표로 함에 있어 치명적이다.
23) Prideaux(2019). pp.25–31.
24) Kumagai(2014), pp.97883.

하나는 글로벌화된 시선이 로컬의 의료 수요를 충족하고 글로벌 건강 평등에 기여할 수 있을지를 질문하는 것이다. 리가 잘 지적하, 글로의학교육에서 가하는 문제 중심 득법, 성과 중심 교육 등의 교육방법론은 자적 개인을 고로 하는 서구의 개인주의를 한다. 그리고 그것이 컬의 의료 문제를 해결할 수 있을지 거의 질문하지 않는다.27) 때때로 이러한 서구식 교육 문화는 이러한 교육 방법을 체내화한 컬의 상층 계급 시민들에게 더 부합하며의 의료 갈등을 더욱 심각하게 착 계급-빈민들의 문화를 타자화한다. 로컬의 의료 질 문제 이면의 의료 자원 교환의 불평등과 의료 붕괴의 양상을 질 좋은 인력 개발의 문제로 가린. 그리고 의학교육의 질이 은 것으로 받는 국의 의료 질이 그다고 치부되는 일본, 한국 등의 나라의 의료보다 효과적이고 효적이지 은지 질문하지 는다. 그 결과 신자유주의 이후 남미, 중앙 아시아 국가들에서는 다양한 의료 붕괴의 현장과 높아진 의료 질 인증 기준이 공존하는 모순을 경험한다. WHO-의 의학교육 인증 기구 등 다양한 국제 체들이 의료 붕괴 과정에 관여 하며, 의 의료와 의학교육이 글로인력 기맞추어 재편될 수 있기를 기대한다.

한국의 의료인문학의 도입은 의학교육의 글로맥락 도입과 무관하지 .28) 그러나 의료인문학 자체가 이들 문제를 무비판적으로 수용할 수 있을? 은 의생명과학이 환자의 증상과 고통을 관화되고 표화된 과학기술의 어로 치환으로써 의사가 환자의 고통 문제를 수 없게 만다고 비하였다. 유사하게, 로컬의 의료 부족과 고통을 글로벌 기준에 따라 평가하고 재단하는 것은 이들 문제 이면의 사회 문화적 맥락들을 소거하고 이해할 수 없게 만들 수 있다. 의학교육의 글로벌화 요구는 로컬의 의료 문화를 외부의 기준에 맞추고 순응하게 만든. 그것은 로의 의료들이 마나 스스로의 요구와 고통에 민감하게 반하고 해결하고자 하였는지를 아내지 으며 인식론적으로 소거한다. 의료 속에 존해 왔으되 지표화 되기 어려운 더더욱 포착하지 는다.

25) 하지만 『병든 의료』의 저자 오마호니는 의료인문학의 공감력 강조를 비판하며 의료인문학이 의사-환자 관계 사이의 소통 기술 등을 평가하고 가르칠 수 있다고 주장하나 실제로는 사적이고 신비스러워서 가르칠 수 없다고 주장한다. 그의 공감 교육 비판 은 경청할 만한 부분이 많다. O'Mahony(2020)/ 권호장 역(2022), 281-299쪽.
26) Kumagai(2014), pp.978983.
27) Bleakley(2008), pp. 266
270.
28) 물론, 글로벌화 흐름이 거세지기 이전에 한국 의료의 모순 속에서 의료인문학의 움직 임을 미리 찾고 희구했던 한국 의료계 주체들을 삭제해서는 안 된다. 그들의 고민은 소중한 씨앗이 되었다.

의료 속의 의료인과 환자들의 주체성을 탐색하고 ''를 지하는 의료인문학은 이러한 경을 비적으로 는다. 물론 의료 인문학이 내재적으로 적 요청에 부한다고 기는 어. 전술하였이 의료인문학은 의료가 보다 환자 중심적으로 환자의 요구에 민감하게 반할 수 있어야 한다는 요청에 라 도입되. 그러나 한국 사회에서 어떠한 행위가 환자 중심적으로, 민감하게 반응하는 행위인지에 대한 내용적 이해는 아직 불충분하다.

  • 현재 한국 에서 의과대학 의료인문학 교육 내용 대부분을 채우는 의료윤리는 서구의 기준인 4원칙-자율성 존중의 원칙, 선행과 악행 금지의 원칙, 정 의의 원칙 등을 제시하며 이들 원칙에 부응하는 것이 보다 윤리적이거나 환자 중심적이라고 믿는다.29) 그러나 이는 환자 중심성을 환자/ 가족의 의사를 형식적으로 확인하는 절차로 치환하는 경향으로 나타난다.30)
  • 또한 초기에 서구의 의료인문학 학자들이 이상으로 생각했던 인간적인 의사상이 서구의 교육받은 중산층 엘리트에 가깝다는 점 역시 무시할 수 없다. 고전적 의료인문학은 인간 중심적 전통에 기대면서 전인성(全人性)을 희구하는 근대 인문학을 유산으로 삼는 경향이 있다.31) 이는 오늘날 인류세 시대를 맞이하여 동물, 무생물 등 비인간 의 존재를 적극 포용하려는 최근의 인문학의 반성(탈식민주의 이론, ANT, 신유물론 등)을 도외시하기 쉽.
  • 이러한 양상을 보면 의료인 문학의 도입은 서구에서 출발한 이상적인 인간상-인문학적 질문과 규 범을 추종하는 것처럼 보인다.
29) 정의의 원칙은 환자중심성 보다는 의료자원 분배 원칙과 관련이 있어 결이 다르다.
30) 강지연(2021), 144-169.
31) 국내에 의료인문학을 종합적으로 소개한 황임경은 현재 의료인문학이 의학의 호르몬 역할을 하며, 그것은 의학의 휴머니즘 전통에 기댄다고 설명한다. 황임경(2021), 482 . 이 경우 의학의 본래성이란 무엇인가에 대해 고민하지 않을 수 없을 것이다.

과연 의료인문학은 외부의 규범에 대한 부응 이상의 것이 될 수 있을? 서구에서는 그간 의과대학 내에 성적으로 안한 의료인문학을 새로운 의료인문학의 흐름이 제기되고 있다.32) 현재 제기 되고 있는 판적 의료인문학(critical medical humanities)/건강 인문학(health humanities) 등의 새로운 의료인문학들은 이러한 의료인문학 내부의 장을 인식한다.

  • 비판적 의료인문학은 주로 의료의 현장 을 다양한 주체들의 얽힘으로 확장시킨다. 의료인문학이 과거 인간중 심적으로 정립해 온 전통적인 구분에 대한 비판적 탐구가 비판적 의료인문학의 주요한 테마가 된다.33) 비판적 의료인문학이 최근의 비판 이론, 퀴어 이론, 장애학 연구, 신유물론 등의 흐름에 기대어 전통적인 의료/비의료/의생명과학의 구분과 규범성에 좀 더 집중한다면,
  • 건강인문학의과대학의 의료인문학을 넘어선 확장을 시도하는 학문이다. 그것은 그간 의사 양성에 초점을 둔 의과대학 의료인문학이 다양한 영역의 건강 관련 주제와 관련된 인문학으로 확장되길 시도한다. 건강과 보건의료를 둘러싼 다양한 학제들의 시도들을 민주적으로 담아내고자 한다.34) 건강인문학의 초점은 그간 의료인문학의 테마였던 인문학적 의사-됨을 넘어서 건강/의료의 인문학적 의제를 확장하고자 하는 데에 있다.
32) 물론, 이들 '새로운' 의료인문학(들)은 주로 영국, 유럽을 중심으로 의과대학 밖에서 진행 중이란 점은 언급해야겠다. 의료인문학을 삽입한 커리큘럼의 전범이 된 미국에서도 의료인문학은 의과대학 커리큘럼을 둘러싼 거대한 전쟁 가운데에 자신들의 효용성을 입증하기 위해 분투 중이다.

33) 비판적 의료인문학을 적극 제기한 비니(Viney) 등은 학문의 특징을 다음과 같이 정의 한다.
(i) '
의료'의 현장과 범위를 임상적 만남 이상의 것으로 넓힌다.
(ii) 
단순히 건강과 질병의 경험과 맥락에만 집중하는 것이 아니라 다층적 차원의 구성을 인지한다.
(iii) 
비판 이론, 퀴어 이론, 장애학 연구, 액티비스트 및 연관 분야와 밀접하게 연관을 맺는다.
(iv) 
예술, 인문학, 그리고 사회과학이 임상 그리고 생명과학의 대척점이 아니라 '의생명과학 문화'와 생산적으로 얽혀 있는 지점을 인식한다 
(v) 
새로운 형태의 간학문적, 부문교차적 협력에 기여한다. Viney(2015), p. 2-7.

34) 건강인문학의 흐름을 주도하는 크로포드(Crawford) 등은 건강인문학이 예술과 인문 학에 대한 간학문적, 포괄적, 응용적, 민주주의적, 그리고 액티비스트적 접근을 채 택하는 학문이며 ‘의료인문학’에서 주변화된 기여들을 적극 수렴한다고 설명한다. Crawford(2010), pp.410.

이들 '새로운' 의료인문학이 제기된 경과 지하는 바는 조다르다. 하지만 이들 의료인문학이 그간 의과대학 내에서 좁은 의미의 규범화된 의사 양성과 그들의 역량 상승에 목표를 두었던 기존의 한계를 넘어서고자 함은 분명하다. 전지구적인 의과학기술 전영 향력 강화, 그리고 건강 의 형성 속에서 건강과 의료는 모, 그리고 인간-의 문제에 접적 관는다. 건강과 의료는 각 개인들이 세계와 관계 구이며 인식론적 토대이다. 지능기술과 과학기술의 발전에서 보이 전통적으로 인간의 것으로 여겨진 역 역시 의과학기술의 대상이 되고 있어 이를 계기로 몰락, 네오, 스트의 부상이 점지기도 한다. 이에 대해 새로운 인문학은 '인간'을 위기로부구원하고 재설 정하는 것을 표로 는다. 건강과 의료를 둘러싼 의사/일반인, 건강/비건강, 인간/기술 등의 전통적인 위계화된 구분이 흔들리 는 시점에 서서 그간의 인식론적 위계에 저항한다. 그것은 의과학기술이 비인간적이라는 그간의 인문학자들의 투덜거림과는 거리를 둔. 그동안 글로건강 에 관계, 협상하며 저하는 소리들을 고 사유의 자원으로 는다. 의사 중심의 의료와 임상 중심 적용을 선 민주적 관계 기의 탐색한다.

를 들어 건강 문해력(health literacy)의 문제를 생각해 보자. 건강 문해력은 날 공중보건과 건강증진정에서 핵심적인 결정 요인으로 이해된다. 히들 건강 정책 담당자들은 건강 문해력의 증진이 어 능력과 의사소통 능력, '무지(illiterate)'로 인한 장고 대중들을 건강 증진을 위한 로 인도할 것을 기대한다. 그러나 건강 문해력은 의사가 생각하기에 바람직한 방향으로 정보를 이해할 능력으로 환원될 수 없다. 혹은 기적인 의료 서비스를 이용하는지 여부로 단될 수 없다. 무지한 대중이 극복해야 할 으로 치부수 없다. 건강 문해력은 의학 관련 정보들을 자신의 상황을 고려하여 맥락화하고 자신의 결정으로 만드는, 자기 성찰력을 갖추어야 한다. 정보가 콘텍스트를 고려하고 수은 정보의 력을 검증하며 그것을 의사, 그리고 건강 정보의 세계와 소통할 수 있어야 한다. 그리고 자의 역한계 내에서 선택할 수 있어야 한다. 루비 (Rubinelli)는 건강 문해력이 아리스토스의 지식 분따른 실용적 지(practical wisdom), 즉 실(, phronesis)여야 한다고 주장한다.35) 그에 르면, 건강 문해력은 외부의 평가에 따라 건강 목 표-준수 여부로 평가되지 않는, 주체의 정보에 관련된 태도로 이해될 필요가 있다. 이러한 건강 문해력의 문제는 의료에 관한 지식들을 이해하고 역량을 갖추고 결정, 행동하는 문제가 더 이상 의사만의 것이 아님을 보여준. 건강의 자기 관리, 자기실현이 더욱 중요해진 의료는 일반인들에게도 하나의 기술이자 실이 된다. 인문학적 능력 함양과 주체적 해방은 일반인에게서도 중요한 과제가 된다. 의료인문학이 좋은 의사-되기에만 국한시키지 않아야 한다는 새로운 의 료인문학(들)의 요청이 설득력을 얻는다. 

물론 은 의사-되기에 의료인문학이 마나 기여할 수 있는지도 분히 검토되지 은 한국 사에서 이들 새로운 의료인문학()마나 유의할 수 있을지 의적일 수 있다. , 한국에서 의료 인문학을 도적으로 탐색한 황임경 등은 이들 건강인문학 적 의료인문학이 질문하는 문제들의 유의미함정하지만 해외 의료인 문학 소개의 일부로 국한시.36) 오랫동안 한국 사회는 이공계의 공부는 단지 지식-기술의 것으로, 인문계의 공부와는 별개로 여겨져 온 탓에, 의료인문학은 여전히 낯선 조합이다. 인성 교육 등 태도에 대한 교육양은 동안 교육이 도구적 단편적 지식의 득에 치중한 현실에서 부차화되어 왔다.37) 의대생 부에 대한 의적인 시각 한 의과대학 내에서 여전히 어야 할 제이다. 지식적 요소만 아니라 실적 요소가 의학교육에서 중요하게 자리매해야 한다는 요청도 아상황에서 태도를 교육을 통해 양한다는 것은 교육에 대한 기존의 위의 붕괴물려 더욱 진다.

35) Rubinelli(2009), p.307-311.
36) 황임경(2021), 30-32.
37) 정창우(2010), 1-33.

그럼에도 한국 사에서 기존의 의료인문학 만 아니라 새로운 의료인문학()이 기여할 수 있는 바는 존재한다. 새로운 의료인문학 (들)은 한국의 식민주의적∙후기 식민주의적 발전국가 경험을 직시한다. 그것은 한국 사회의 의료가 놓인 맥락들-예를 들어, 과도하게 기 중심적이고 침습지향적이며 단기 효과 중심적인 맥락들을 인식하게 해준. 한국 사의 의료가 서구로부이식되다는 단순한 이해, 그리고 서구와 비서구를 단순히 이분화하는 이분법을 지양 할 수 있게 도와. 그리고 한국의 의료가 한국의 물질 문화를 배경으로 하는 부분을 살피며, 한국의 의료 문화를 발전주의적 성공 신 화에서 벗어나 낯설게 보도록 촉구한다. 이를 통해 환자와 의사 등 다양한 의료 주체들이 의생명과학 문화와 비적으로 관계 을 수 있도는다. 은 의사-되기 규범콘텍스트를 통해 비적으로 적용할 수 있게 만. 그것은 동시에 지한국 의료가 여 있는 자리를 는 것이다.

한국 의료에서는 소아청소과의 전의 지원제로 사건 등 에서 수 있위기 론이 발 중이다. 소아청소붕괴에 대한 우려와 함께 필수 의료가 무질 것이라는 소리가 . 의과학 기술의 격한 발전과 성장한 의료 시장 , 은 수의 치료 성등의 화려한 한국 의료의 성공 신화 이면을 지하는 것은 무? 소아청소년과의 붕괴는 한국 사회의 의료적 필요와 의료 산업의 성공과 똑같지는 않을 수 있다는 점이 적나라하게 드러난 것에 가깝다. 마치 과거에 작동해 왔던 (좋은) 의사 되기를 위한 여러 시스템은 더 이상 작동하기를 거부하는 것 같다. 이것은 정로 한국 의료의 위기일? 혹은 이러한 위기에서 의료인문학은 도수 있을? 위기는 일반적으로 정상 상태를 전제로 한다. 그러나 한국의 의료 문화에서 정상 상태(혹은 시스)는 과도한 병원의 전동 의존도에서 수 있, 래를 보로 한 력에의 의존에 가깝다. 위기 론은 한국 의료가 것을 반성하라는 구로 이어질 수 있 으나, 반성은 한국 의료의 만이 되. 여기에는 저출생과 한국 사에서 ''를 유하는 , 숙련 구조의 붕괴, 그리고 과학기술적 성동 가능한 상태로 회복 외의 의료 가치에 대한 경시 등이 겹겹이고 얽혀 있다. 이러한 위기를 견디어 나가는 법은 서구의 인문학 독본이나 교양주의적 이상에 있지는 않다. 그것은 지금 이 자리에서 현재의 위기들을 서로에 대한 이해와 상상의 힘에 빗대 정직하게 보고 살펴나가는 데에 있다. 

5. 한국에서 의료인문학(들)의 질문과 자리들

다시, 한국에서 의료인문학(들)의 자리는 어디일까? 이 글의 서에 한국의 의료인문학은 의사학의 형태로 일본의 제국대학과 일 의학의 경을 경험으로 한 의학 리트들의 양주의적 관심에서 도입되 다고 밝혔. 그것은 어면 문해를 위주로 했던 과거 의학의 전통이 의학 리트들의 관심을 끌었던 것인지도 모. 이러한 전통이 1990대 의학에 대한 역사학적 관심을 지닌 이들에 의해 발 되기 전지 여전히 의학 리트들의 양주의적 관심에 . 1990년대는 본격적으로 의료인문학이 발돋움한 시기이며, 한국 의료가 발전 신화를 넘어 직종 간 갈등에 부딪히기 시작한 시기이다. 한국 의료의 정체성에 대한 탐색이 시작된 이때 의료리 등의 서구 의료인문학 론도 도입되기 시작하였다. 의과대학 인증평가제도의 도입과 2000년 의사 파업은 본격적으로 한국 의학교육이 '사회지향적' 으로 전환해야 한다는 요청에 부응하도록 만들었. 한국 의료 위기의 론들은 의료 외부에서 중요한 해법들을 , 임없이 도입하도록 견인차가 되으며 나름의 으로 이끌었. 이러한 위기 론과 , 그리고 기의 상이 해왔역할을 부정할 수 없을 것이다. 교육이 결국 의과대학 내 의료인문학의 양적 을 가져 왔기 때문이다. 그러나 한국 의료인문학이 제대로 생각하고 질 문할 수 있는 자리를 제대로 찾고 있는지, 비판, 창조로 나아가고 있는지에 대해서는 긍정적으로 응답하기 망설여지는 것도 사실이다.

전술했듯이 한국 의료인문학의 도입 배경이 서구의 의료인문학 도입 배경과 같지 않기에, 의료인문학이 응답할 수 있는 몫 또한 같지 않다. 그럼에도 불구하고 의료인문학을 다에 있어 가장 이 다루 어지는 가지 주제를 한국 의료인문학의 과 관지어 보고자 한다.

  • 하나는 의학교육 개혁과 의료인문학 도입의 직간접적 계기가 된 한국 의사들의 전문직업성 문제이다.
  • 두 번째는 의료인문학 본연의 몫으로 보다 논해지는 환자의 삶과 고통을 이해함으로써 이루어지는 치유이다.

이들 모환자, 의사, 그리고 자를 비롯한 한국 의료와 관계는 다양한 주체들이 얽혀 있는 주제이다. 한 한국 사의 법 제도 역사와 물질 문화적 토대를 함께 살펴보지 을 수 없 다. 그것은 의료인문학의 기원과 역할을 분명하게 구분해내는 것은 아니다. 그것은 한국 의료에서 '인문학적 필요'를 일푸코 식으로 계보학적으로 살펴보는 작이다. , 의료인문학 도입을 의학교육 따른 결과로 역사화하는 것, 는 한국 의료인문학을 서양 고전의 전통으로부기원화하는 것과 다르게 한국 의료에서 인문학적 필요에 대한 무수한 요청들을 어내는 것이다.

1) 전문직업성

한국 의사는 어떻게 정체화하는가? 집단으로서의 의사는 과연 존재 하는가? 이는 2000의사 (혹자에 르면, 권 쟁)에서 경험한 이후 줄곧 제기되는 질문이기도 하다. 발표 원고에서 필자는 의사들 스스로가 의료 시장과 보건의료체계를 스스로 결정 하지 못하고 있다는 의 정동이 '수호'라는 구호 속에서 출로를 정체성이 되, 으로 나원동력이다고 적.38) 잘 알려져 있듯이, 서구의 전문직업성은 중세부터 비롯된 직업 길드의 역사, 질 관리와 훈련의 역사에 기대어 있다. 지식의 습득과 전달이 스스로 외에는 허용되지 않기 때문에 부여된 일종의 독점권이라 할 수 있다. 이는 대에 들어와서 국가와 시장의 력으로부스스로의 한을 보호하기 위한 토대가 되으며, 전문리의 간이 되.

그러나 한국 의사들은 길드에 근간을 둔 자율적 통제의 역사를 갖고 있지 않. 서구 전문역사에서 리 강리트로서 스스로 를 다치유자들과 구분하며 도적 우위를 강제하는 역할을 수행. 그러나 한국 의사들은 식민지 시기 형성된 엘리트이자 기술 테크니션이며, 개항 이후 근대화 이데올로기에 따라 위로부터 형성된 직업군에 가깝다. 기술로부터 분리된 별개의 아카데미즘을 갖지 못하고 국가의 이해에 따라 균점되어왔고, 의사를 위한 자율적 공간이 많지 않. 서구의 의사들이 전통적으로 아카데미즘 정체성에 가까운 내과의테크니션에 가까운 외과의들이 존하다가 일정 시점에 타협한 것과 한국의 의사들에게는 아카데미즘과 테크니션이 혼종의 정체성으로 공존하며, 별개의 것이 아니다. 그리고 병원의 이해에 스스 로를 속시키기 때문에 한국 의사의 정체성은 국가의 이해에 라 계도화된 유사-조합주의 정체성에 가운 측면이 . 한국 의사들은 기술자본이 적된 병원의 이익과 한 보호를 일의 자으로 이해하며, 서구의 의사들처럼 은 분을 통한 조합의 한 행세를 자으로 이해하지 못한다. 이는 1995대한의학협가 대한 의사협로 명이 전환된 시점부강화된 현상이라 할 수 있다. 1990년대 중반부터 한국 의사들은 의료보험제도 도입 이후 보장된 의료시장과 확대된 병원의 이익 속에서 소수의 엘리트 정체성에서 조합주의적 정체성으로 전환하기 시작하였다.39)

38) 이러한 집단의 정동이 집단의 이익에 도움이 되었을지, 유리한 결과를 낳았을지는 또 다른 평가를 필요로 하는 문제이지만 말이다. 캐나다의 역량 기반 교육 CanMED framework 역시 1986년 온타리오 의사 파업을 기점으로 한 의사들에 대한 신뢰 하락에 대한 대응으로 도입되었다. Butt and Duffin(2018), pp.196–98. 2000년대 한국의 의료인문학 도입이나 의학교육개혁도 의사 파업의 영향력을 제외하고는 설명하기 어렵다.

39) 파슨스의 설명에 따르면 노동조합이나 전문가 조직 모두 구성원의 이해를 추구하나 전문가 조직은 한 사회에 규칙을 가져오기 때문에 행동 강령, 자율 분업화 등의 특징을 지니며 노동조합과 구분된다. Parsons(1969). 한국의 경우 길드의 역사 없이 행동 강령, 자율 분업화 등의 요소가 발생하기 전에 조합주의적 정체성을 먼저 갖춘 경우라 할 것이다.

이러한 한국 의료전문의 정체성을 서구 전문의 자적 통제와 질 관리의 역사와 등치시키는 것은 콘텍스트를 무시하는 것에 가깝다. 우리는 한국 의사의 전문직-됨에 대한 이해를 넓힐 것을 요청받는다. 율적 통제의 불/가능성은 한국 의사가 처한 독특한 딜레마이다. 고도로 발달한 의생명과학 지식과 기술을 습득하고 적용할 권한은 갖추었으되 그것을 통제하지 못함에서 오는 이상과 현실의 괴리이다. 시장 경이 격화되면서 과거와 만기술에 대한 을 유지하지 못하는 상황에 여 있음에 라 위기이 자연스라 온다. 그것은 의료를 운용하는 인력보다 관기술 도입에 중해 온 한국 의료의 현주소이기도 하다.

한국 의사들은 외형적으로는 기술을 독점하는 전문직이나 실제로는 내외적으로 그렇게 존중받지 못한다. 여기서 한국 의사들의 전문- 존재론을 다시 구할 필요가 있다. 의 기대와 개인의 도적 정체성이 일치되지 못에서 는 도적 아역시 살펴보아야 한다.40) 산재된 도적 위기은 필수의료 위기에서 수 있한국 사곳곳에서 '의료의 붕괴', '의료의 모라토리'은 재으로 이어지고 있다. 그에 비해 의사 개인을 도적 개인으로 보호할 수 있는 제도는 현재로서는 전무하다. 의사의 윤리를 단지 개인의 윤리가 아닌 집단 도덕의 윤리 가능성, 의협으로 대표되는 조합의 윤리 이상의 가능성을 텍스트와 콘텍스트를 통해 탐색해야 한다.

40) 여기서 의사의 도덕적 위기감, 아노미는 기타 직역들의 생존의 위기감과 구분한다. 뒤르켐은 『직업윤리와 시민도덕』에서 길드가 쇠퇴하면서 자유시장경제 하에서는 과거 길드와 같은 연대와 협동 없이는 직업적 결속감 없이는 도덕적 아노미 상태에 빠지고 만다고 설명한다. 권오상(2021), 1-22쪽. 의사 파업은 강력한 직업적 결속감을 보이는 것 같지만, 이면에는 그러한 연대가 없음이 오히려 드러난 현상에 가깝다.

 

2) 치유와 돌봄

전술하였서구의 의료인문학에서는 테크놀로지에서 소외된 환자-의사 관계라는 테제를 공유한다. 그것은 의과학기술이 개입되기 이전 환자에 대한 치유를 전인적으로 담당했던(또는 한다고 여겼던) 과거의 기억과 무관하지 않. 그리고 의과학기술이 의사들을 지 기능인으로 전시킬 것에 대한 우려와도 관있다. 기에 서사의학처럼 환자에 대한 콘텍스트를 으면서 환자에 대한 해력을 이고 시각을 시키는 법이 은 각을 받을 수 있.

그러나 근대화 과정에서 과학기술의 독점을 통해 스스로를 한의사들과 구분지었던 한국의 의사들에게 있어 의과학기술에 따른 소외라는 주제는 크게 와닿지 않는다. 적어도 이 주제는 통상의 '환자-의사 관계'에는 영향치지 는다. 이것은 의과학기술을 대하는 다수의 환자들에게도 마찬가지이다. 과도한 의과학기술이 스스로의 존엄함을 침해할 것이라는 서구 환자들의 우려에 비해 다수의 한국 환자들은 의과학기술에 열려 있다.41) 사체의 부검에는 부정적이나 체에 을 대는 성형수술 등의 기술에 규범적으로 려 있는 모에서 확인된다. 적어도 고통이 없는 일반인이 의사/의과학기술을 대면하거나 선망하는 간에는 소위 치유보다 자본주의 시장 경제 속 소비의 양상에 실하다. '3분 진료'로 대표되는 공장식 진료 시스템 역시 치유나 공감에 대한 공감대를 넓히기 어렵게 만든. 진료 결과에 대한 환자의 불만은 나은 의료를 위한 인차가 되거나 의과학기술의 불확실성에 대한 인정보다는 의사들에게 공포어 진료로 이어지는 결과를 .

지만 이것을 한국 의료에서 치유나 공감제가 공감받기 어다는 것으로 이해해서는 하다. 한국 의과학기술의 권위는 그것이 더 화려한 시술로 회복가능한 영역에서는 유지된다. 그러나 기술이 회복을 약속할 수 없거나 그 정도가 제한적인, 일례로 장애와 노년의 경우에는 이미 치명적으로 '치유'나 공감이 부재하다. 한국 사에서 장회복을 통해 환할 수 있는 장밋빛 래 자본이 부재하기에, 상상할 수 있는 사역의 여 있다. 그 리고 그 고통은 로지 개인과 가, 그리고 일부 의료진이 당하도 . 한국 병원의 3분 진료를 질타하는 담론은 많으나 3 분 진료에도 오지 못하는 사람들은 담론에서 거론되지 않는다. 이것 은 모사람들이 3분 진료에 접가능해야 한다는 기가 아니다. 의료의 공급이 지처럼 수도의 대형합병원 중심의 일루전에만 치중치는 부분이 훨씬 다는 기이다.

한국에서는 치유라는 제에 대해 논할 때 해당 어에 대한 그간 의 모호한 의들도 고려할 필요가 있다. 영어권에서 "heal"은 오래된 영어 단어 haelen으로부터 왔는데, "전체(wholeness)"라는 의미를 갖고 있으며 상처를 딛고 다시 하나가 된다는 의미를 갖고 있으며, 오늘날에는 심리적 정신적 영적 차원의 통합하는 회복을 의미한다. 반면 "cure" care은 어원인 라어 어원 curare로부온 것으로 주로는 체적 차원의 의학적 치료를 지한다. 일반적으로 영어권에 서 cure는 신체적 질병을 뿌리 뽑는 "근치"의 의미를 가지며 오늘날에는 heal과 대비되는 경우가 많. 그러므로 질병의 cure는 불가능하라도 더 높은 차원의 heal이 가능하다. 그러나 한국에서는 cure나 heal이나 둘 다 완치, 치유로 번역된다. 완치나 치유의 치()가 다스다는 의로서 단순한 질병이 아닌 질서로의 회복을 의하는 이다. 로 한국 사회 회복의 서사가 그사이에 여 있는 장의 고통을 마나 대속해야 할 로 환원하는지를 잘 기술하고 있는 은정(2019)의 원제는 Curative Violence이나 한국어로는 『치유라는 이름의 폭력』이라고 번역하고 있다. 여기서 다시 하나로 회복되는 것은 개인의 이나 전체성이 아니라, 라는 상처와 고통이 가의 생으로 대속된 하나의 가이다. 이 경우 "치유"는 그것에 도하지 못하는 이들에게 력이 된다.42)

한국의 대형종합병원들은 하나의 테크노 산업체로서 보험과 의과학 기술의 독점을 통해 발전해 왔다. 병원과 환자, 기술의 연결체의생명과학기술의 개발과 도입, 적용으로 지속가능하다고 여기는 상상체에 가까웠. 의생명과학기술의 치유 가능성을 하나의 화적 정점으로 은 일의 동맹이라 할 수 있다.43)

  • 여기에는 현재보다 더 저렴한 비용을 들여 영위할 만한 삶을 지탱하는 방법을 묻지 않는다.
  • 기술 이후의 공감, 돌봄 관계의 틈들을 상상에서 배제한다.
  • 기술을 적용한 후 살아가는 신체들을 돌보는 힘에 대해서는 고려하지 않는다.
  • 원하는 방식으로 삶을 마무리하는 방법은 의료 외부의 과제가 된다.

테크놀로지의 적용 자체가 환자를 소외시다는 것이 아니다. 테크놀로지의 적용과 살만한 위하는 것 사이에는 은 연결과 의 과정이 필요하며, 간 다양한 고려가 필요하다. 의사가 병원 밖 환자의 삶에 대한 상상과 이해를 넓혀내지 않는 한, 이러한 과정을 고려하며 의술을 제공하지 않는다. 중요한 의학적 의사결정을 환자의 의 이해를 합적으로 고려하며 할 수도 없다.

41) 이것에 대한 역사적 분석은 DiMoia(2013) 참조.

42) Kim(2017)/ 
강진경, 강진영 역(2022). 김은정은 심청전, 영자의 전성시대 등 근대 한국의 장애 관련 텍스트를 독해하며 이들 서사에서 가족이 젠더 구조 속에서 어떻 게 희생을 통해 장애인의 상처를 치유하는 것으로 나타나는지를 그려낸다. 여성 가 족은 희생을 감당하며 장애 남성을 치유하게 되는데, 이때 회복의 지향은 정상가족 이라 할 수 있다.

43) 김상현과 자사노프는 사회기술적 상상체(sociotechnical imaginaries)를 “과학 기술 의 진전을 통해, 사회적 삶과 질서를 달성할 수 있다는 공유된 믿음으로 움직이는 집 단적으로 가지는, 제도적으로 안정된 바람직한 미래 수행 비젼”으로 정의한다. 김상 현과 자사노프는 이 개념을 한국에만 국한시키지 않으며, 한국의 경우에는 황우석 의 줄기세포 연구가 주로 다루어진다. 연구-기술-병원의 복합체에서 회복을 정점으 로 추구된다는 점에서 한국 상급종합병원 일반으로도 확장 가능할 것이다. Jasanoff & Kim(2015).  

한국 의사들에게 의과학기술 외의 '전체로의 회복'이라는 치유의 이상을 설하거나 공감 커교육리는 것으로 이 문제를 해결할 수 있을? 적어도 한국 사회가 의과학기술을 이용한 질병 치유 이상의 것을 상상하지 못한다면 무의미한 일일 수밖에 없다. 에 질병을 비롯한 다양한 기가 자되면서 질병 치유 이상의 서사를 만들어내는 것은 고무적이다. 히려 환자를 온전한 주체로 만드는 치료의 전인성(全人性)을 논하는 것은 치유보다 논의에서 이루어지는 하다. 에 관한 이야기 역시 새게 한국 사가 경청해야 할 이야기가 되고 있다. 의과대학에서는 이들 서사를 적극 끌어들이지 못하고 있지 못한 이다. 하지만 의료전문가가 질병의 소리를 경청하고 기 위해 필요한 도 있을 것으로 생각된다.

6. 나가며

한국 사에서 의료인문학은 무, 이어야 할? 혹은 그것이 한국 의료에 필요한 실(phorenesis)수 있을 ? 한국의 의료인문학은 한국 의료가 새롭게 갱신되어야 한다는 부름에 따라 과거의 것이 복원되고 유입되었다. 그것은 크게는 전세계적으로 질병 패턴의 변화, 새로운 질병의 등장, 의료 서비스 주체의 다변화, 유전자 시대의 돌입 등에 따라 의생명과학 패러다임이 전환 되기 시작한 것과 흐름을 같이 한다. 의학이 과학기술학, 인문학, 술 등 새로운 학문들과 관계 으면서 간학문적으로 치유와 건강에 기여할 수 있을 것이기대가 존재한다. 물론 전세계적 의생명과학 의 전환과 한국 의료의 위기가 기에, 한국 의료인문학 에는 도의 과제가 있다. 해외의 의료인문학 흐름을 익히고 배우는 것만으로는 충분하지 않다. 한국 의료의 주체와 맥락들을 두껍게 읽어내 야 할 몫이 있다면, 그건 한국 의료인문학의 몫이다.

본 글에서는 전세계적 차원의 의료인문학의 제가 한국의 그것과 을 수 있음을 설명하고자 하였다. 의 이해를 부하게 다는 역할 속에서 한국 의료인문학의 제 역시 다을 필요가 있다. 지정학적 차원의 이해를 사리 증발시키하는 한국 의학의 발전 동력을 생각을 때 한국 차원의 도 과제를 상정하는 것은 운 일이 아니다. 빠르게 리는 기차 안에서는 경을 제대로 관하기 들 수 있다. 위기와 제의 진에 동의하지 을 수도 있으며, 한 가지 차원의 해만 있을 필요도 없다. 다만 격한 의 흐름 속에서도 다양한 소리들에 려 있기를 실적으로 지해야 할 것이 다. 

 

 


【주요어】의료인문학, 로컬의 관점, 글로벌 의학교육, 전문직업성, 치유와 돌봄
【요약문】한국 사회에서 의료인문학은 무엇이었고, 무엇이어야 할까? 본 글은 전 세계적 의료인문학의 역사와 부흥의 맥락 속에서 한국의 의료인문학의 역할과 위상을 점검한다. 한국의 의료인문학은 한국 의료가 새롭게 갱신되어야 한다는 부름에 따라 과거의 것이 복원되고 유입되었다. 그것은 크게는 전세계적으로 질병 패턴의 변화, 새로운 질병의 등장, 의료 서비스 주체의 다변화, 유전자 시대의 돌입 등에 따라 의생명과학 패러다임이 전환되기 시작한 것과 흐름을 같이 한다. 그러나 전세계적 의생명과학의 전환과 한국 의료의 위기가 같지 않으며, 전세계적 차원의 의료인문학 테제가 한국의 그것과 꼭 같지 않을 수 있다. 이를 위해 다양한 의료인문학의 맥락과 갈래 속에서 한국 의료인문학의 역할을 탐구할 필요가 있다. 또한 지정학적 차원의 이해와 로컬의 이해를 풍부하게 다듬는 속에서 한국 의료인문학의 테제 역시 다듬을 필요가 있다. 본 글은 우선 ‘인문학으로서의 의학’, 의료인문학이 의과대학에서 하나의 학제로 형성되어 온 과정을 추적한다. 특히 전인적 인간 형성을 목표로 하는 인문학 공부가 타인에 대한 연민과 공감 등을 요청하는 의과학 반성의 요구와 맞닿으면서 의학교육에 강조되었는지를 살펴 본다. 이후 1990년대 이후 국내에 급부상한 의료인문학 도입 배경으로서 의학교육의 글로벌라이제이션을 살피고, 이를 의학을 둘러싼 인문학적 요청이라는 측면에서 비판적으로 검토한다. 마지막으로 한국 의료인문학의 위상과 역할을 ‘전문직업성’과 ‘돌봄’이라는 영역을 예로 들며 제언하고자 한다.

CanMEDS 2025의 지구보건(Can Med Educ J. 2023)
Planetary Health in CanMEDS 2025

 

소개
Introduction

기후 비상사태가 건강에 미치는 악영향이 점점 더 분명해지고 있습니다. 최근의 문헌 검토에 따르면 지구 보건은 CanMEDS 의사 역량 프레임워크와 관련된 진화하는 개념으로 확인되었습니다.1 의사 역량의 맥락에서 지구 보건은 기후 비상사태에 대한 의료의 영향과 기후 비상사태가 환자 치료에 미치는 영향을 최소화하는 데 관심을 두고 있습니다. 이 원고는 CanMEDS에 지구 보건 관련 내용을 포함하는 것이 얼마나 중요한지 밝히고, 지구 보건 개념을 예시하기 위해 향후 CanMEDS를 반복할 수 있는 기회를 제안하는 것을 목표로 합니다.  
The worsening impact of the climate emergency on health is increasingly apparent. A recent literature review identified Planetary Health as an evolving concept that is relevant to the CanMEDS physician competency framework.1 In the context of physician competencies, planetary health is concerned with minimizing the impact of healthcare on the climate emergency and the impact of the climate emergency on patient care. This manuscript aims to establish the critical importance of including planetary health links in CanMEDS and to propose opportunities for future iterations of CanMEDS to exemplify planetary health concepts.

행성 건강이란 무엇이며 의사의 역량에 중요한 이유는 무엇인가요?
What is planetary health and why is it important to physician competency?

지구 보건 연합은 지구 보건을 "지구의 자연 시스템에 대한 인간의 교란이 인간 건강과 지구상의 모든 생명체에 미치는 영향을 분석하고 해결하는 데 초점을 맞춘 솔루션 중심의 초학제적 분야이자 사회 운동"이라고 정의합니다.2 지구 보건은 기후 비상사태와 토지 파괴, 생물 다양성 손실, 오염 등 기타 생태 위기를 모두 다룹니다.3 기후 비상사태는 금세기 인류 생명에 가장 큰 건강 위협으로 여겨지며 2030년부터 매년 25만 명이 추가로 사망할 것으로 예측됩니다.4
The Planetary Health Alliance defines planetary health as “a solutions-oriented, transdisciplinary field and social movement focused on analyzing and addressing the impacts of human disruptions to Earth’s natural systems on human health and all life on Earth.”2 Planetary health addresses both the climate emergency and other ecological crises, including land destruction, biodiversity loss, and pollution.3 The climate emergency is considered the greatest health threat to human life this century, with predictions of 250,000 additional deaths per year from 2030 onward.4

캐나다 사람들의 건강은 이미 기후 비상사태의 영향을 받고 있습니다.5

  • 2021년에는 섭씨 1.2도의 온난화로 브리티시컬럼비아주(B.C.주)의 열돔 현상으로 700명 이상이 사망했고,6
  • 매니토바와 서스캐처원주의 가뭄으로 농작물이 실패하여 식량 가격이 상승했으며,7
  • 홍수로 B.C.와 뉴펀들랜드의 지역사회 전체가 사라졌고,8
  • 산불이 캐나다 서부9와 온타리오 북부 대부분을 휩쓸고 지나갔습니다.10
  • 극심한 날씨로 인해 주요 인프라가 손상되고 공급망이 중단되면서 의료 서비스 제공에도 차질이 빚어지고 있습니다.11
  • 기후 비상사태는 캐나다 전역의 원주민 건강에 불균형적인 영향을 미치고 있으며,12
  • 환경 파괴는 전통적인 생활 방식을 위협하고 지속적인 식민지화 및 토지 박탈 과정을 악화시키고 있습니다.13

The health of people in Canada is already affected by the climate emergency5:

  • in 2021, at just 1.2 degrees Celsius of warming, over 700 people died in British Columbia's (B.C.) heat dome event,6 
  • crops failed in Manitoba and Saskatchewan droughts contributing to rising food prices,7 
  • flooding took out entire communities in B.C. and Newfoundland,8 and
  • wildfires engulfed large parts of western Canada9 and northern Ontario.10 
  • Healthcare service delivery is also being increasingly disrupted as extreme weather damages critical infrastructure and interrupts supply chains.11 
  • The climate emergency has a disproportionate impact on the health of Indigenous populations across Canada,12 
  • as environmental destruction threatens traditional lifestyles and exacerbates ongoing processes of colonization and land dispossession.13

보건 부문은 이 문제에 크게 기여하고 있습니다. 캐나다 보건 부문은 지구상에서 두 번째로 탄소 집약적인 의료 시스템으로,4 캐나다 온실가스 배출량의 4.6%를 차지하며 이는 캐나다의 항공 산업과 동등한 수준입니다.14 영국 국민보건서비스의 모델링에 따르면 배출량의 거의 70%가 의약품, 장비 및 공급망의 기타 측면에서, 10%가 여행에서, 10%가 건물 에너지에서, 5%가 물과 폐기물에서, 5% 전체가 마취 가스 및 용량 측정 흡입기(15)에서 발생한다고 합니다. 연방 정부는 다른 국가들과 함께 기후에 탄력적인 저탄소 의료 시스템을 제공하겠다고 약속했으며, 이러한 전환이 이루어지려면 의사들의 상당한 지지가 필요합니다.16 지속 가능한 의료 모델이 개발되었으며, 이를 실행해야 할 것입니다. 지속 가능한 의료 시스템을 위한 이러한 프레임워크 중 하나는 세 가지 범주의 개입을 제안합니다.17

  • 건강의 사회적 결정요인을 해결하고 건강 증진 및 질병 예방에 참여함으로써 의료 서비스 수요를 줄인다.
  • 의료 서비스 공급을 수요에 맞춰 적절한 치료를 보장하고 불필요한 검사 및 치료를 피한다.
  • 의료 서비스 공급으로 인한 탄소 배출을 줄인다.

The health sector is a significant contributor to the problem. The Canadian health sector is the second most carbon-intensive health care system on the planet,4 contributing 4.6% of Canada’s greenhouse gas emissions—on par with the country’s aviation industry.14 Modelling from the National Health Service in the United Kingdom suggests that nearly 70% of emissions come from medicines, equipment, and other aspects of the supply chain; 10% comes from travel; 10% comes from building energy; 5% from water and waste; and a full 5% from anesthetic gases and Metered Dose Inhalers (15). The federal government, together with other countries, has pledged to deliver a climate-resilient, low-carbon health care system, and significant advocacy from physicians will be required to ensure this transformation occurs.16 Models of sustainable healthcare have been developed and will need to be implemented. One such framework for sustainable health systems proposes three categories of intervention:

  • reduce demand for health services by addressing the Social Determinants of Health and engaging in health promotion and disease prevention;
  • match the supply of health services to demand, ensuring appropriate care and avoiding unnecessary tests and treatment; and
  • reduce emissions from the supply of health services.17

존경받는 목소리로서 의사는 지구의 건강과 기후 비상사태에서 고유한 역할을 수행해야 합니다.18 의사는 지구 건강을 위한 옹호자가 될 준비가 되어 있어야 합니다. 의사는 미시적(환자), 중시적(진료소, 병원, 지역사회), 거시적(정책) 수준에서 행동할 수 있습니다.19 의사는 기후 행동 및 완화를 옹호하고 의료에 기후 회복력을 구축할 수 있습니다.11,17 의과대학부터 평생 의학 교육을 통해 지구 건강 및 옹호에 대한 교육을 개선하여 의사가 기후 비상사태에서 진료할 수 있도록 준비시켜야 할 것입니다. 
As respected voices, physicians have a unique role to play in planetary health and the climate emergency.18 They must be prepared to be advocates for planetary health. Physicians can act at the micro (patient) level, meso (clinic, hospital, community) level, and macro (policy) level.19 Physicians can advocate for climate action and mitigation and build climate resilience into healthcare.11,17 Improved training in planetary health and advocacy from medical school through Continuing Medical Education will be necessary to prepare physicians to practice in a climate emergency.

2015 CanMEDS 역량 프레임워크에서 지구 건강은 어떻게 표현되어 있나요?
How is planetary health represented in the 2015 CanMEDS competency framework?

2015 CanMEDS 의사 역량 프레임워크에는 지구 건강에 대한 명시적인 언급이 없습니다. 하지만 건강 옹호자 역할에는 건강의 사회적 결정 요인, 질병 예방, 건강 증진, 건강 감시, 지역사회 건강 개선과 관련하여 지구 건강과 관련될 수 있는 몇 가지 역량이 포함되어 있습니다(표 1B). 기후 과학자들은 수십 년 동안 기후 비상사태에 대한 경종을 울려왔지만, 대부분의 보건 전문가들은 최근에야 기후 비상사태를 악화시키는 데 있어 보건 시스템의 역할과 적응 조치를 채택하여 기후 비상사태로 인한 이환율과 사망률을 줄여야 할 필요성을 인식하기 시작했습니다. 
There are no explicit references to planetary health in the 2015 CanMEDS physician competency framework. The Health Advocate role does contain some enabling competencies that could be related to planetary health as it relates to the social determinants of health, disease prevention, health promotion, health surveillance, and improving community health (Table 1B). While climate scientists have been sounding the alarm on the climate emergency for decades, most health professionals have only recently begun to acknowledge the health system’s role in causing the worsening planetary emergency as well as the need to reduce the climate emergency’s associated morbidity and mortality by adopting adaptive measures.

2025년 캔메즈 역량 프레임워크 내에서 지구 건강을 어떻게 더 잘 표현할 수 있을까요?
How can planetary health be better represented within the 2025 CanMEDS competency framework?

표 1C에서는 지구 건강에 대한 우리의 진화하는 이해를 CanMEDS 의사 역량 프레임워크에 통합하는 데 도움이 될 것으로 생각되는 역량을 포함하거나 수정할 것을 제안합니다. 이러한 제안에는 네 가지 주제가 포함됩니다: 
In Table 1C, we propose the inclusion or modification of competencies that we believe would help to integrate our evolving understanding of planetary health within the CanMEDS physician competency framework. These suggestions incorporate four themes:

  • 의료 시스템의 지속가능성 개선. 이 주제는 의료 서비스 제공이 환경에 미치는 막대한 영향과 이를 해결함으로써 의사가 "해를 끼치지 않는다"는 직업적 의무를 다해야 할 필요성을 인식하고 있습니다.17
    Improving the sustainability of our health system. This theme recognizes the enormous environmental impact of healthcare delivery and the need for physicians to meet our professional obligation to “do no harm” by addressing this.17
  • 기후 비상사태로 인한 혼란에 대한 의료 시스템의 회복력 향상. 캐나다의 의료 시스템은 홍수, 산불, 폭염, 폭풍과 같은 극심한 기상 이변으로 인한 혼란에 취약하며, 이는 기후 회복력 있는 의료 시스템의 구현을 가속화하여 중환자 치료 제공을 유지함으로써 줄일 수 있습니다.4 의사는 의료 분야의 환경적 취약성을 인식하고 기후 회복력을 내재화하기 위한 품질 개선 과정에 기여해야 합니다.20
    Improving the resilience of our health system to disruption from the climate emergency. Canadian healthcare systems are vulnerable to disruption by extreme weather events such as floods, wildfires, heatwaves, and storms that could be reduced by accelerating the implementation of climate-resilient healthcare systems to maintain critical care delivery.4 Physicians must recognize environmental vulnerabilities in healthcare and contribute to quality improvement processes to embed climate-resilience.20
  • 환경이 환자 건강에 미치는 영향에 대처하기. 의사가 환자의 사회적 맥락을 고려하는 것처럼, 우리도 환자의 환경적 맥락을 고려해야 합니다. 토착민과 인종차별 인구, 빈곤층, 정치적으로 소외된 사람들은 환경 파괴로 인해 불균형적인 영향을 받습니다. 의사는 환자와 그 가족에게 미치는 환경의 광범위하고 다양한 영향을 이해해야 합니다.
    Addressing the impact of the environment on patient health. Just as physicians consider a patient’s social context, we must also consider their environmental context. Indigenous and racialized populations, those living in poverty, and others who are politically marginalized are disproportionately affected by environmental degradation. Physicians must understand the breadth and variety of environmental impacts on patients and their families.
  • 사회적 책임에서 지구 건강 관점의 중요성사회적 책임이란 "의학이 사회와 맺는 사회적 계약"으로 정의됩니다.21 사회적 책임 의료는 환자, 지역사회, 인구의 건강 요구에 대응합니다.19 사회적 책임을 위하여 의사는 질병, 고통, 사망에 기여하는 사회적, 환경적 조건에 대해 목소리를 내야 합니다.
    The importance of a planetary health lens in social accountability. Social accountability is defined as “the social contract that medicine has with society.”21 Socially accountable healthcare is responsive to patient, community, and population health needs.19 Socially accountable physicians must speak out about the social—and environmental—conditions that contribute to disease, suffering, and death.

 


Can Med Educ J. 2023 Mar 21;14(1):46-49. doi: 10.36834/cmej.75438. eCollection 2023 Mar.

Planetary Health in CanMEDS 2025

Affiliations

1University of Toronto, Ontario, Canada.

2University of Saskatchewan, Saskatchewan, Canada.

3McGill University, Quebec, Canada.

4University of Calgary, Alberta, Canada.

5University of Ottawa, Ontario, Canada.

PMID: 36998490

PMCID: PMC10042797

DOI: 10.36834/cmej.75438

CanMEDS 2025의 적응적 전문성(Can Med Educ J. 2023)
Adaptive Expertise in CanMEDS 2025

소개
Introduction

적응형 전문성은 급변하는 세계에서 효과적이고 혁신적으로 진료하고 학습하는 데 필요한 지식과 기술의 진화, 개선 및 개발과 관련된 역량을 통합합니다. 이 개념은 최근의 문헌 검토에서 CanMEDS 의사 역량 프레임워크에서 잘 드러나지 않는 필수 개념으로 확인되었습니다.1 이 원고는 이 개념과 이전 및 향후 CanMEDS의 반복에 대한 연관성을 요약하는 것을 목표로 합니다. 
Adaptive expertise incorporates competencies related to the evolution, refinement, and development of the knowledge and skills required to practice and learn effectively and innovatively in a rapidly changing world. It was identified as an essential concept that was underrepresented in the CanMEDS physician competency framework in a recent literature review.1 This manuscript aims to summarize the concept and its links to former and future iterations of CanMEDS.

적응형 전문성이란 무엇이며 의사 역량에 중요한 이유는 무엇인가요?
What is adaptive expertise and why is it important to physician competency?

적응적 전문성잘 알려진 문제에 직면했을 때 이전 지식을 효율적이고 효과적으로 적용하는 것이전 지식이 불충분할 때 새로운 지식을 창출하여 해결책을 마련하는 것 사이의 균형을 강조하는 전문가 개발 및 성과 모델입니다. 이와는 대조적으로 일상적 전문성은 적응적 전문성의 효율성 차원만을 반영하는 것으로 이해됩니다. 적응적 전문성의 '효율성'과 '혁신' 사이의 균형 일상적인 문제 해결을 학습과 개선의 기회로 인식하는 접근법으로 보완됩니다.2,3 따라서 이전 지식의 적용이 불충분한 문제에 직면했을 때

  • 적응적 전문성지식을 유연하게 사용하고 새로운 솔루션을 생성하는 능력을 가능하게 하는 반면,
  • 일상적 전문성'알려진 솔루션에 미지의 문제를 맞추려는 시도'를 하다가 실패로 이끌게 됩니다."4

Adaptive expertise is a model of expert development and performance that emphasizes a balance between the efficient and effective application of previous knowledge when facing well-known problems and the creation of new knowledge to generate solutions when previous knowledge is insufficient. In contrast, routine expertise is understood to reflect only the efficiency dimension of adaptive expertise. This balance between ‘efficiency’ and ‘innovation’ in adaptive expertise is complemented by an approach to practice that recognizes daily problem solving as an opportunity to learn and improve.2,3 Thus, when facing problems for which the application of previous knowledge is insufficient,

  • adaptive expertise enables the flexible use of knowledge and the ability to generate new solutions while
  • routine expertise will result in failed attempts to ‘fit unknown problems to known solutions.’4

적응적 전문성은 무엇을 해야 하는지(절차적 유창성)와 왜 해야 하는지(개념적 이해)를 모두 아는 것이 특징입니다.5 이러한 개념적 이해는 새롭고 불확실한 임상 상황의 가변성에 적응할 수 있게 합니다. 즉, 알려진 해결책이 충분하지 않은 경우 적응적 전문가는 근본적인 '이유'를 다루는 새로운 해결책을 생성할 수 있습니다. 적응형 전문성을 키우려면 교육이 효과적인 지식 보유와 적용에만 중점을 두는 것에서 벗어나 학생들이 새로운 지식을 지속적으로 학습하고 생성할 수 있도록 준비시키는 방향으로 전환되어야 합니다. 이러한 역량을 '미래 학습을 위한 준비(PFL)'로 정의할 수 있습니다.6 
Adaptive expertise is characterized by both knowing what to do (procedural fluency) as well as knowing why you’re doing it (conceptual understanding).5 It is this conceptual understanding that permits adaptation to variability in novel and uncertain clinical situations i.e. if known solutions are insufficient, adaptive experts can generate new solutions that still address the underlying ‘why’. To foster the development of adaptive expertise, education must shift beyond a sole emphasis on effective knowledge retention and application towards preparing students to continue to learn and generate new knowledge. This capacity has been defined as ‘preparation for future learning (PFL).6

보건 전문직 교육에서 PFL을 지원하는 교육적 접근법에는 다음이 포함됩니다.7,8

  • a) 임상 징후 및 증상을 근본적인 메커니즘과 명시적으로 통합(예: "왜"라고 묻거나 말하기),
  • b) 학생들에게 의미 있는 변형에 노출(예: "만약에 이렇다면.." 질문하기),
  • c) 학습에서 투쟁과 발견을 활용한 다음, 즉각적인 피드백과 통합이 뒤따르는 것

불가피하게 보건 전문가는 정식 교육을 받지 않은 질환을 가진 환자를 마주하게 될 것입니다. 따라서 새로운 문제에 직면하여 새로운 지식을 창출하고 학습하는 능력은 의사가 환자의 고유한 요구에 대응하고 끊임없이 변화하는 역동적인 의료 종사자 간 네트워크 내에서 치료를 제공할 수 있도록 보장합니다.9
In health professions education, pedagogical approaches that support PFL include

  • a) explicit integration of clinical signs and symptoms with underlying mechanisms (e.g asking or telling “why”),
  • b) exposing students to meaningful variation (e.g asking “what if” questions) and
  • c) leveraging struggle and discovery in learning followed by immediate feedback and consolidation.7,8 

Inevitably, health professionals will face patients presenting with conditions for which they have not received formal training. Thus, the ability to create new knowledge and learn in the face of novel problems ensures that physicians are responsive to the unique needs of their patients, as well as able to provide care within ever-changing, dynamic, interprofessional networks of healthcare workers.9

적응형 전문성은 2015 CanMEDS 역량 프레임워크에서 어떻게 표현되나요?
How is adaptive expertise represented in the 2015 CanMEDS competency framework?

의료 전문가 역할은 환자 중심의 임상 평가 및 관리, 임상 의사결정의 복잡성, 모호성, 불확실성, 의사결정 시 진화하는 지식의 활용 등 적응적 전문성의 여러 측면을 반영합니다. 그러나 2015년 프레임워크에는 이러한 역할과 관련하여 적응적 전문성에 대한 구체적인 언급이 부족하여(표 1A 및 및1B),1B) 교육자가 효과적이고 혁신적으로 실습하고 학습하는 데 필요한 지식과 기술을 의도적으로 배양할 수 있는 환경이 조성되지 않을 수 있습니다. 적응형 전문지식을 CanMEDS의 핵심 의료 전문가 역량으로 명시적으로 통합하면 의료 전문지식과의 관계를 강조하고, 일상적인 진료에서 7가지 CanMEDS 역할을 모두 통합하는 데 핵심적인 기능을 강조하며, 전문 역량 및 지속적인 전문성 개발의 지표로서 진료에서 새로운 지식을 혁신하고 창출하는 역량을 장려할 수 있습니다.
The Medical Expert role reflects many aspects of adaptive expertise: patient-centered clinical assessment and management; complexity, ambiguity, and uncertainty in clinical decision-making; and drawing on an evolving body of knowledge when making decisions. However, the 2015 framework lacks specific reference to adaptive expertise in relation to this role (Table 1A and and1B),1B), which may create an environment where educators are not primed to deliberately cultivate the necessary knowledge and skills required to practice and learn effectively and innovatively. Explicitly incorporating adaptive expertise as a key Medical Expert competency in CanMEDS would highlight its relationship with medical expertise, underscore its central function to the integration of all seven CanMEDS roles in daily practice, and promote the capacity to innovate and create new knowledge in practice as a marker of professional competence and continuing professional development.

2025년 CanMEDS 역량 프레임워크 내에서 적응형 전문성을 어떻게 더 잘 표현할 수 있을까요?
How can adaptive expertise be better represented within the 2025 CanMEDS competency framework?

임상 의사 결정에서 적응형 전문 지식의 중요성을 인식하여 의료 전문가 역할에 새로운 핵심 역량을 통합할 것을 제안합니다(표 1C). 이 역량에는 효율성과 혁신 사이의 균형을 맞추고, 다양한 관점을 수용하며, 임상 진료 전반에 걸쳐 학습할 수 있는 역량이 포함됩니다. 적응형 전문성을 의료 전문가 역할의 핵심 역량으로 설정함으로써, 우리는 CanMEDS 프레임워크에서 의료 전문성의 중심적인 역할과 내재적인 역할을 자신의 업무 범위에 통합하는 방법을 인식하고 있습니다. 또한 커뮤니케이터, 협력자, 리더, 학자 역할에 속하는 역량들이 이러한 영역에서 적응형 전문성의 중요한 요소를 더 잘 통합할 수 있도록 약간의 제안을 했습니다(표 1C). 발달적 관점에서 볼 때, 분야별 교육에 적응적 전문성을 조기에 도입하는 것은 PFL을 포함하는 교육이 지속적인 전문성 개발에서 임상적 의사 결정의 지속적인 역량과 성장을 보장하는 데 도움이 될 수 있음을 인식하는 것입니다. 초심자에서 숙련자로 넘어가는 CanMEDS 마일스톤의 구성은 적응형 전문성과 더욱 연계되어 전문적 역량의 지표로서 실무에서 새로운 지식을 혁신하고 창조하는 역량을 촉진할 수 있습니다. 
Recognizing the importance of adaptive expertise in clinical decision-making, we propose incorporating a new key competency into the Medical Expert role (Table 1C). Its enabling competencies incorporate the capacity to balance between efficiency and innovation, to embrace multiple perspectives, and to learn throughout clinical practice. By situating adaptive expertise as a key competency within the Medical Expert role, we recognize the central role of medical expertise in the CanMEDS framework and how it incorporates the intrinsic roles into one’s scope of practice. Additionally, we have made minor suggestions to enabling competencies under the Communicator, Collaborator, Leader, and Scholar roles to better integrate important elements of adaptive expertise in these areas (Table 1C). From a developmental perspective, the introduction of adaptive expertise early within discipline-specific training recognizes that education that includes PFL can help ensure that sustained competence and growth in clinical decision-making is accounted for in continuing professional development. The progression from novice to mastery in the organization of CanMEDS milestones can further align with adaptive expertise to promote the capacity to innovate and create new knowledge in practice as a marker of professional competence.

 


Can Med Educ J. 2023 Mar 21;14(1):18-21. doi: 10.36834/cmej.75445. eCollection 2023 Mar.

Adaptive Expertise in CanMEDS 2025

Affiliations

1The Wilson Centre, Temerty Faculty of Medicine, University of Toronto, Ontario, Canada.

2University of Toronto, Ontario, Canada.

3College of Family Physicians of Canada, Ontario, Canada.

4McMaster University, Ontario, Canada.

5University of Alberta, Alberta, Canada.

6Glenrose Rehabilitation Hospital, Alberta, Canada.

7University of Ottawa, Ontario, Canada.

8Association of Faculties of Medicine of Canada, Ontario, Canada.

9Royal College of Physicians and Surgeons of Canada, Ontario, Canada.

10McGill University, Quebec, Canada.

11University of Saskatchewan, Saskatchewan, Canada.

PMID: 36998497

PMCID: PMC10042796

DOI: 10.36834/cmej.75445

CanMEDS 2025의 임상추론(Can Med Educ J. 2023)
Clinical Reasoning in CanMEDS 2025

소개
Introduction

임상 추론은 의학교육 연구의 초기 초석 중 하나였으며, 이러한 초기 연구 초점은 의학 교육에서 임상 추론의 중심적인 역할을 반영합니다.1 따라서 임상 추론이 CanMEDS 의사 역량 프레임워크에서 잘 알려지지 않은 개념으로 확인된 것은 놀라운 일이 아닐 수 없습니다.2,3 그러나 프레임워크를 자세히 살펴보면 임상 추론은 그 중심적인 중요성에도 불구하고 CanMEDS에서 명시적으로 언급되거나 통합된 적이 거의 없다는 것을 알 수 있습니다. 임상 추론의 일부 측면이 의료 전문가 역할에 역량으로 포함되어 있기는 하지만, 임상 추론에 대한 우리의 이해는 CanMEDS 2015에서 명시적으로 언급된 몇 가지를 넘어 확장되었습니다. 이 입문서에서는 임상 추론이 CanMEDS 2025의 핵심 개념인 이유, 임상 추론에 대한 우리의 이해가 어떻게 발전해 왔는지, CanMEDS 2015에서 임상 추론이 어떻게 표현되는지, 그리고 임상 추론과 관련된 역량을 보다 신중하고 포괄적으로 통합하기 위해 CanMEDS 2025를 개정할 수 있는 방법을 제안합니다. 
Clinical reasoning was one of the early cornerstones of medical education research, and this early research focus reflects its central role in medical training.1 Therefore, it may be surprising that clinical reasoning was identified as an underrepresented concept in the CanMEDS physician competency framework.2,3 However, a close examination of the framework demonstrates that clinical reasoning, despite its central importance, is rarely explicitly mentioned or integrated in CanMEDS. While some aspects of clinical reasoning are woven within the Medical Expert role as enabling competencies, our understanding of clinical reasoning has expanded beyond these few explicit mentions in CanMEDS 2015. This primer will orient readers to why clinical reasoning is a key concept for CanMEDS 2025, how our understanding of clinical reasoning has evolved, highlight how clinical reasoning is represented in CanMEDS 2015, and suggest ways that CanMEDS 2025 could be revised to more deliberately and comprehensively incorporate competencies related to clinical reasoning.

임상 추론이란 무엇이며 의사의 역량에 중요한 이유는 무엇인가요?
What is Clinical Reasoning and why is it important to physician competency?

임상적 추론은 환자 치료의 모든 측면에서 핵심적인 요소이기 때문에 의료 분야에서 전문적 진료의 핵심으로 묘사되어 왔습니다.1. 또한 추론 오류는 환자와 의료진에게 높은 비용을 초래하기 때문에 환자 안전을 위협하는 요인으로 지목되었습니다.4 이러한 핵심성과 중요성에도 불구하고,5 임상 추론을 정의하는 방법, 효과적인 추론을 개념화하는 방법, 교육, 평가 및 연구 목적으로 이를 운영하는 방법은 크게 나뉘지는 않았지만 여전히 다양합니다.5,6
Clinical reasoning has been described as the core of professional practice in healthcare,1 as it is a key component of all aspects of patient care. Additionally, errors in reasoning have been named as a threat to patient safety given the high cost of errors to patients and practitioners.4 Despite its centrality and importance,5 how we define clinical reasoning, how we conceptualize effective reasoning, and how we operationalize it for the purposes of teaching, assessment, and research remain varied if not deeply divided.5,6

효과적인 임상 추론의 개념은 의학교육 내 여러 커뮤니티에서 다양한 개념화 또는 추론의 요소를 교육 목표, 평가 대상 또는 집중 연구 영역으로 사용함에 따라 다양하게 설명되어 왔습니다. 임상 추론에 대한 이러한 각 사고 방식은 인간의 인지 구조에서 실천의 인식론에 이르기까지 서로 다른 학문, 영역 또는 이론적 기반에서 비롯되며, 이는 임상 추론 과정의 각기 다른 요소에 초점을 맞춘다는 것을 의미합니다. 임상 추론에 접근하는 이러한 다양한 방식은 중요한 면에서 차이가 있습니다.

  • 일부는 추론의 결과에 초점을 맞춥니다.
    • 의료 오류(추론의 불행한 결과7)에서 진단 정확도(바람직한 결과8)까지
  • 일부는 개별 활동으로서의 임상 추론에 초점을 맞춘다.
    • 의사 또는 학습자의 인지 과정
  • 다른 일부는 사회적으로 내재된 활동으로서의 임상 추론을 탐구합니다:
    • 팀의 진료 제공(추론을 상호작용 활동으로 배치9),
    • 상황별 의사 결정(상황 또는 구체화된 활동으로서의 추론10) 또는
    • 공유 의사 결정(파트너로서의 환자11) 등에 주목
  • 어떤 사람들은 의사의 임상 추론에서 발생가능한 한계에 관심을 집중하고 있습니다:
    • 인지적 부하 고려사항(임상 추론 및 작업 수행은 작업 기억 제약에 의해 제한됨12)
    • 편향의 위험성(예: 불평등 및 의료 서비스 격차13)
  • 마지막으로, 일부는 에러에 대한 인적 요인시스템 기여4 및 복잡한 맥락에 개인이 어떻게 적응하는지에 초점을 맞춥니다.16
    • 적응적 전문성,14 집단적 역량,15 복잡성 이론 등

The notions of effective clinical reasoning have been variously described by different communities within medical education—as different conceptualizations or elements of reasoning have been used as objectives for instruction, targets for assessment, or areas of focused research. Each of these ways of thinking about clinical reasoning draw from different disciplines, domains, or theoretical homes—from human cognitive architecture to epistemologies of practice; which means each of these ways of thinking about clinical reasoning focus on different elements of the clinical reasoning process. These numerous ways of approaching clinical reasoning vary in important ways.

  • Some focus on the outcomes of reasoning;
    • from medical error (an unfortunate outcome of reasoning7) to diagnostic accuracy (an aspired outcome8).
  • Some focus on clinical reasoning as an individual activity,
    • focused on the cognitive processes of the practitioner or learner;
  • while others explore clinical reasoning as a socially embedded activity, with attention paid to
    • team provision of care (placing reasoning as an interactional activity9),
    • decisions-in-context (reasoning as a situated or embodied activity10), or
    • shared decision-making (patient-as-partner11).
  • Still other members of the community have focused their attention on the limits of what is possible in a practitioners’ clinical reasoning—from
    • cognitive load considerations (clinical reasoning and task performance are limited by working memory constraints12), to the
    • dangers of bias (e.g. inequities and health care disparities13).
  • Finally, some focus on human factors and system contributions to error4 and how individuals adapt to complex contexts such as
    • adaptive expertise,14 
    • collective competence,15 and
    • complexity theory.16 

이러한 각 관점은 추론 과정의 서로 다른 구성 요소를 강조하고, 추론의 서로 다른 "결과"를 중요시하며, "효과적인" 추론을 만드는 요소에 대해 서로 다른 개념화를 가지고 있습니다. 이러한 가치 있는 구성 요소, 결과, 효과적인 추론에 대한 개념의 차이는 모두 매우 다른 집중 교육 영역, 다른 평가 목표, 다른 연구 접근 방식에 영향을 미칩니다.17
Each of these perspectives highlight different components of the reasoning process, value different “outcomes” of reasoning, and have different conceptualizations of what makes “effective” reasoning. These differences in valued components, outcomes, and notions of effective reasoning all feed into very different areas for focused teaching, different assessment targets, and different approaches to research.17

임상 추론과 같이 크고 복잡한 개념을 의학교육에서 가르치고, 평가하고, 연구할 수 있는 것으로 만들려면 각 상황이나 전문 분야에서 무엇이 효과적인 임상 추론을 구성하는지에 대한 결정이 내려져야 합니다. 효과적인 추론으로 판단되는 요소에 따라 임상 추론 과정의 다른 구성 요소, 다른 맥락적 변수 또는 임상 추론의 다른 결과가 우선시됩니다.18 예를 들어, 효과적인 추론은 다음으로 특징지을 수 있습니다.12

  • 속도,19
  • 정확성,20
  • 비용 영향,21
  • 환자 욕구와 임상적 권고(즉, 공유 의사 결정)의 균형,10
  • 과도한 검사와 (불)확실성에 대한 욕구의 균형,22
  • 맥락적 변수에 대한 인식 및 적응,23
  • 관리 계획의 효과적인 적응adaptation,24
  • 작업 기억 한계 내에서 인지 부하를 효과적으로 관리할 수 있는 것

이러한 효과적인 추론의 특성은 평가, 학습 및 교육의 대상이 되며, 이는 해당 집단의 전문성 수준, 전문 분야 맥락, 치료 맥락(예: 긴급 치료 대 지역사회 치료)에 따라 달라집니다. 
To render a large and complex concept such as clinical reasoning into something that can be taught, assessed, or researched in medical education, decisions about what constitutes effective clinical reasoning in each context or specialty must be made. Depending on what is determined to be effective reasoning, different components of the clinical reasoning process, different contextual variables, or different outcomes of clinical reasoning come to the forefront.18 For example, effective reasoning can be characterized by

  • speed,19 
  • accuracy,20 
  • cost implications,21 
  • balancing of patient desires with clinical recommendation (i.e. shared decision making),10 
  • balancing over-testing with a desire for certainty,22 
  • recognition of and adaptation to contextual variables,23 
  • effective adaptation of a management plan,24 and
  • effectively managing cognitive load within working memory limitations.12 

These characteristics of effective reasoning become the targets of assessment, learning, and teaching that vary depending on the expertise level of the population in question, specialty context, and care context (i.e., urgent care vs. community care).

임상 추론은 2015 CanMEDS 역량 프레임워크에서 어떻게 표현되어 있나요?
How is Clinical Reasoning represented in the 2015 CanMEDS competency framework?

CanMEDS 2015 프레임워크에서 임상 추론의 측면은 주로 의료 수출 역할(표 1A)의 역량으로 나타나며, 일부 요소는 다른 역할(표 1B)에 걸쳐 통합되어 있습니다. 임상 추론의 이론적 및 개념적 이해와 관련된 연구는 CanMEDS 2015 이후 성장해 왔습니다.3 최근의 연구는 개인 인지를 넘어 팀 기반 추론,9 진단적 의사결정에서 관리 추론에 대한 고려,23 그리고 개별 인지 과정에서 복잡한 맥락에 놓인 행동으로서의 임상 추론을 탐구하는 것으로 확장되었습니다.10 훌륭하고 건전한 임상 추론을 구성하는 요소에 대한 우리의 개념은 임상 추론에 대한 이론적 이해의 증가와 진료의 복잡성에 대한 인식의 확대에 따라 확장되어 왔습니다. 임상적 추론은 의료인이 된다는 것의 핵심으로 남아 있지만,1 임상적 추론이 어떤 모습이고 어떻게 교육, 학습 및 평가의 목표로 운영되는지는 확장되었습니다. 임상 추론의 개념은 CanMEDS 2015의 일부 필수 역량에 반영되어 있지만(전문 분야의 맥락에 따라 이러한 역량이 어떻게 구현되는지가 달라질 수 있음을 인식하고 있습니다), CanMEDS 2025에서는 임상 추론이 보다 명시적이고 포괄적으로 표현되어야 한다고 생각합니다.
In the CanMEDS 2015 framework, aspects of Clinical Reasoning are found primarily as enabling competencies within the Medical Export role (Table 1A) with some elements interwoven throughout other roles (Table 1B). Research related to the theoretical and conceptual understanding of clinical reasoning has grown since CanMEDS 2015.3 More recent work extends beyond individual cognition to include team-based reasoning;9 from diagnostic decision-making to consideration for management reasoning,23 and from individual cognitive processes to exploring clinical reasoning as a situated behaviour in complex contexts.10 Our notions of what constitutes good and sound clinical reasoning have broadened in lockstep with our growing theoretical understanding of clinical reasoning, and our growing acknowledgement of the complexity of care. While clinical reasoning remains at the core of what it means to be a healthcare provider,1 what that clinical reasoning looks like and how it is operationalized into targets of teaching, learning, and assessment have expanded. While the concept of clinical reasoning is reflected in some enabling competencies within CanMEDS 2015—recognizing that specialty context will shape how these competencies are enacted - we believe that clinical reasoning should be more explicitly and comprehensively represented within CanMEDS 2025.

 

임상 추론이 2025 CanMEDS 역량 프레임워크 내에서 어떻게 더 잘 표현될 수 있을까요?
How can Clinical Reasoning be better represented within the 2025 CanMEDS competency framework?

임상 추론에 대한 현재의 이해에 더 잘 부합하기 위해 2025 CanMEDS 역량 프레임워크는 양질의 임상 진료를 제공하는 데 기여하는 임상 추론의 여러 측면을 보다 의도적으로 통합할 수 있으며, 통합해야 합니다. 보다 세분화된 의미에서 임상 추론에는 필요한 기본 지식의 통합과 다양한 맥락에서 시기적절하고 효과적인 방식으로 치료를 제공하면서 해당 지식을 동원하는 능력이 포함됩니다. 효과적인 임상 추론의 일부 핵심 요소는 현재 의료 전문가 역할의 중요한 역량이지만, 임상 추론의 측면은 다른 CanMEDS 역할(예, 의사소통자, 협력자, 학자, 전문가; 표 1 참조).필수 지식의 범위, 진료 표준, "좋은" 추론의 기준, 임상 추론에 영향을 미치는 복잡한 맥락적 요인은 CanMEDS 2025에 더 잘 통합될 수 있고 통합되어야 합니다.3 CanMEDS 프레임워크 내에서 임상 추론을 더 잘 반영하기 위해, 우리는 기존의 몇 가지 지원enabling 역량을 조정할 것을 제안하며, CanMEDS 2025에서 중요한 고려 사항이라고 생각하는 역량을 명확히 설명합니다. 제안된 지원enabling 역량(표 1, 섹션 C)은 모호한 개념의 근거를 마련하고 임상 추론이 여러 역할의 맥락에서 관찰, 교육, 평가 및 연구될 수 있다는 개념을 반영합니다. 즉, 의료 서비스를 제공하려면 효과적인 임상 추론이 필요하고, 효과적인 임상 추론에 필요한 여러 가지 역량이 필요하며, 이러한 역량은 여러 CanMEDS 역할에 걸쳐 통합되어 있습니다. 임상적 추론은 효과적인 치료 제공을 위해 여러 CanMED 역할에 걸쳐 필요한 역량을 통합해야 하는 방법을 보여주는 훌륭한 예입니다. 
To better align with current understandings of clinical reasoning, the 2025 CanMEDS competency framework can, and should, more deliberately integrate the many aspects of clinical reasoning that contribute to providing high quality clinical care. In a more granular sense, clinical reasoning includes the integration of necessary fundamental knowledge, and the ability to mobilize that knowledge while delivering care in a variety of contexts in a timely and effective way. While some key components of effective clinical reasoning are important current enabling competencies for the Medical Expert Role, aspects of clinical reasoning are also woven throughout other CanMEDS roles (i.e., communicator, collaborator, scholar, and professional; see Table 1).The scope of requisite knowledge, standards of care, standards of “good” reasoning, and complex contextual factors that influence clinical reasoning could and should be better integrated into CanMEDS 2025.3 In order to better reflect clinical reasoning within the CanMEDS framework, we suggest adapting several existing enabling competencies, and articulate those that we believe are important considerations for CanMEDS 2025. The suggested enabling competencies (Table 1; section C) helps ground an already nebulous concept, and reflect the notion that clinical reasoning can be observed, taught, assessed, and studied in the context of several Roles. Meaning, the delivery of care necessitates effective clinical reasoning, several enabling competencies needed for effective clinical reasoning, and these enabling competencies are integrated across several CanMEDS roles. Clinical reasoning is an excellent example of how enabling competencies across multiple CanMEDs roles need to be integrated to effectively deliver care.

표 1C에서 제안하는 지원enabling 역량은 임상 환경에서 임상 추론이 어떻게 동원되는지에 대한 이해의 증가를 반영하며, 임상 추론 과정과 결과를 형성하는 치료 환경의 복잡성을 인식합니다. '좋은' 임상 추론이 여러 상황과 진료 환경에 걸쳐 어떤 모습인지에 대한 중요한 차이가 존재하기 때문에, 이러한 역량 제안은 각 의료 전문 분야 내에서 맥락화를 계속 요구할 것입니다. 임상 추론이 독립적인 역량으로 명명되어야 한다고 생각하지는 않지만,25 임상 추론의 복잡성은 CanMEDS 프레임워크 전반에 걸쳐 몇 가지 새로운 역량과 몇 가지 조정된 지원 역량에 더 잘 반영될 수 있다고 생각합니다.

The enabling competencies we propose in Table 1C reflect our growing understanding of how clinical reasoning is mobilized in the clinical environment and recognize the complexity of care environments that shape clinical reasoning processes and outcomes. These suggested enabling competencies will continue to require contextualization within each of our medical specialties, as important distinctions do exist regarding what “good” clinical reasoning looks like across contexts and care environments. While we do not believe clinical reasoning should be named as an independent competency,25 we believe the complexity of clinical reasoning can be better reflected in several new, and several adapted enabling competencies across the CanMEDS framework.


Can Med Educ J. 2023 Mar 21;14(1):58-62. doi: 10.36834/cmej.75843. eCollection 2023 Mar.

Clinical Reasoning in CanMEDS 2025

Affiliations 

1Institute of Health Sciences Education, McGill University, Quebec, Canada.

2Queen's University, Ontario, Canada.

3NOSM University, Ontario, Canada.

4University of Saskatchewan, Saskatchewan, Canada.

5McMaster University, Ontario, Canada.

PMID: 36998494

PMCID: PMC10042778

DOI: 10.36834/cmej.75843

CanMEDS 2025의 버추어 케어(Can Med Educ J. 2023)
Virtual Care in CanMEDS 2025

 

소개
Introduction

가상 진료(VC)는 잘 훈련된 의사의 손에 의해 효과적이고 안전합니다.1 그러나 대면 진료의 역량이 반드시 가상 진료의 역량과 동일하지는 않습니다.2 특정 VC 교육에 대한 여러 요구에도 불구하고,3-6 최근 문헌 검토에 따르면 이러한 개념은 2015 CanMEDS 의사 역량 프레임워크7에서 충분히 반영되지 않은 것으로 확인되었습니다.8 이러한 새로운 역량을 반영하여 오늘날의 의사가 환자와 사회의 진화하는 요구를 충족할 수 있도록 CanMEDS 2025를 업데이트해야 합니다. 이 입문서에서는 효과적인 원격 상담 및 환자 치료를 제공하기 위해 환자 및 동료와 가상으로 연결하는 데 필요한 기술에 초점을 맞춘 CanMEDS 역량에 대한 변경 사항을 제안합니다. 
Virtual care (VC) is effective and safe in the hands of well-trained physicians.1 However, competency in in-person care does not necessarily equate to competency in VC.2 Despite multiple calls for specific VC training,3-6 these concepts were identified as being underrepresented in the 2015 CanMEDS physician competency framework7 by a recent literature review.8 CanMEDS 2025 must be updated to reflect these novel competencies, ensuring that today’s physicians are able to meet their patients’ and societies’ evolving needs. For this primer, we propose changes to the CanMEDS competencies that focus on the skills necessary to connect virtually with patients and peers to deliver effective remote consultation and patient care.

가상 의료란 무엇이며 의사의 역량에 중요한 이유는 무엇인가요?
What is VC and why is it important to physician competency?

기술의 발전은 사람들이 관계를 맺고, 소통하고, 생활하는 방식을 바꾸고 있습니다. 의료 분야에서도 연구와 진료의 지형을 바꾸고 있습니다. 이 분야에는 때때로 중복되는 용어가 많이 있습니다(표 1의 정의 참조). 이 백서에서는 기술을 활용한 개별 환자 치료 제공에 초점을 맞추기 위해 가상 치료 Virtual Care 라는 용어를 사용합니다. 의료 정보학 및 데이터 분석과 같은 개념은 이 백서의 범위를 벗어나지만 실제로 중요하며 다른 곳에서 다루고 있습니다.9 
Technological advances are changing how people relate, communicate, and live. In healthcare, they are altering the landscape of research and practice. There are many, sometimes overlapping, terms in this field (see definitions in Table 1). This paper utilizes the term Virtual Care to focus on the provision of individual patient care using technology. Concepts such as health informatics and data analytics are out of the scope of this paper but are indeed important and are addressed elsewhere.9

Term 용어 정의
Definition
Chatbot 챗봇
"자연스러운 대화를 통해 의료 서비스를 제공하고...차례대로 진행되는 대화를 통해 온라인 의료 서비스를 더욱 사용자 친화적으로 만드는"10 전자 "대화형 시스템"입니다.

An electronic “conversational system” that “provides healthcare through natural conversation…making online healthcare more user-friendly…through a turn-taking dialog.”
10
Digital Health 디지털 건강
디지털 건강은 정보 기술 및 전자 통신 도구, 서비스 및 프로세스를 사용하여 의료 서비스를 제공하고 더 나은 건강을 촉진하는 것입니다.9 여기에는 "웨어러블 기기, 모바일 건강, 원격 의료, 건강 정보 기술, 원격 의료... 데이터 분석 및 인텔리전스, 예측 모델링... 의료 소셜 미디어", 전자 의료 기록 및 환자 커뮤니케이션 포털의 사용이 포함됩니다.11

Digital health is the use of information technology and electronic communication tools, services and processes to deliver health care services and facilitate better health.
9 It includes the use of, “wearable devices, mobile health, telehealth, health information technology, and telemedicine… data analytics and intelligence, predictive modelling…medical social media”, electronic medical records, and patient communication portals.11
eHealth e헬스
"의료 서비스, 건강 감시, 건강 문헌, 건강 교육, 지식 및 연구를 포함한 건강 및 건강 관련 분야를 지원하기 위해 정보통신 기술을 비용 효율적이고 안전하게 사용하는 것"12.

“The cost-effective and secure use of information and communications technologies in support of health and health-related fields, including health-care services, health surveillance, health literature, and health education, knowledge and research.”
12
Extended reality 확장 현실
컴퓨터를 사용하여 사용자 환경을 컴퓨터 생성 콘텐츠로 향상(예: 증강 현실[AR])하거나 대체(예: 가상 현실[VR])하는 것.13

Using computers to enhance (ie: augmented reality [AR]) or replace (ie: virtual reality [VR]) a user’s environment with computer generated content.
13
mHealth m헬스
원격 의료를 목적으로 스마트폰 및 웨어러블 디바이스와 같은 모바일 기술을 사용하는 것.5

The use of mobile technology such as smartphones and wearable devices for the purposes of telehealth.
5
Telehealth 원격 보건
"건강 관리 또는 의료 교육의 모든 구성 요소를 원거리에서 제공하는 것"5 여기에는 가상 진료와 의료 전문가 및 환자 교육이 모두 포함됩니다.5

“The delivery of any component of health care or medical education at a distance.”
5 It includes both virtual care and the education of both healthcare professionals and patients.5
Telemedicine 원격 의료
"정보통신 기술을 통해 원격으로 제공되는 의료 서비스"14로, 화상 또는 전화 상담 등이 이에 해당합니다.

A “medical service provided remotely via information and communication technology”
14 such as a video or telephone consultation.
Virtual Care 가상 진료
"환자 치료의 질을 촉진하거나 극대화할 목적으로 모든 형태의 통신 또는 정보 기술을 사용하여 원격으로 발생하는 환자 및/또는 환자 치료 서클 구성원 간의 모든 상호 작용"15 여기에는 원격 진료, mHealth, 원격 모니터링, 전자 의료 기록(EMR)을 통한 보안 메시징이 포함되지만 이에 국한되지는 않습니다.15

“Any interaction between patients and/or members of their circle of care, occurring remotely, using any form of communication or information technologies, with the aim of facilitating or maximizing the quality of patient care.”
15 This includes but is not limited to telemedicine, mHealth, remote monitoring, and secure messaging via electronic medical records (EMRs).15


VC는 고품질 의료 서비스에 대한 접근성을 제공하고 의료 서비스의 도달 범위, 형평성, 효율성, 잠재적 비용 효율성을 향상시킬 수 있는 기회를 제공합니다.1,3 최대 85%의 환자가 VC를 받는 데 관심이 있으며16 여러 의료 분야에서 그 효과가 입증되었습니다.1 명백한 바람직성에도 불구하고 대면 의료 역량이 가상 영역으로 자동 변환되지 않고2 의사가 현장에서 유능하게 진료하기 전에 이해해야 하는 VC의 기술적 및 사회 문화적(예: 정책 및 형평성) 한계가 있기 때문에 특정 교육이 필요합니다.17 실제로 캐나다 의사협회, 캐나다 가정의학회, 캐나다 왕립 의사 및 외과의사 대학은 "[VC] 교육을 의료 서비스 교육에 통합하여 의료 제공자와 관리자가 성숙한 [VC] 생태계에서 기능할 수 있도록 전적으로 준비되도록 해야 한다"고 권고했습니다.17
VC offers the opportunity to provide access to high-quality care and enhance the reach, equity, efficiency, and potentially cost-effectiveness of medical services.1,3 Up to 85% of patients are interested in receiving VC16 and multiple areas of medicine have demonstrated its effectiveness.1 Despite its apparent desirability, VC requires specific training because in-person medical competencies do not automatically translate to the virtual sphere2 and there are technological and sociocultural (eg: policy and equity) limitations to VC that need to be appreciated before a physician can practice competently in the field.17 Indeed, the Canadian Medical Association, College of Family Physicians of Canada, and The Royal College of Physicians and Surgeons of Canada have recommended that “[VC] training must be integrated into health service education so that providers and administrators are wholly prepared to function in a mature [VC] ecosystem.”17

2015 CanMEDS 역량 프레임워크에서 VC는 어떻게 표현되어 있나요?
How is VC represented in the 2015 CanMEDS competency framework?

레지던트3 및 의대생 수준에서 가상 의료에 대한 교육을 강화해야 한다는 요구가 오랫동안 있어 왔습니다.4 이러한 요구를 해결하기 위해 2015 CanMEDS 전자 의료 전문가 실무 그룹은 전자 의료 및 가상 의료 채택을 탐색하기 위한 특정 역량에 대한 권장 사항을 제시했지만,18 이 중 대부분은 최종 역량 프레임워크에 반영되지 않았습니다. 특히, "기술 기반 커뮤니케이션"은 2015 CanMEDS 지원 역량 중 하나에서 찾을 수 있지만(표 2 참조), 디지털 헬스, VC, eHealth 및 기타 관련 용어는 구체적으로 언급되지 않았습니다.7 
There have been long standing calls for increased training in VC at the residency3 and medical student level.4 To address these needs a CanMEDS 2015 expert working group on eHealth made recommendations on specific competencies for navigating eHealth and virtual health care adoption,18 though most of these were not reflected in the final competency framework. Notably, while “technology-enabled communication” can be found within one of the 2015 CanMEDS enabling competencies (see Table 2), digital health, VC, eHealth and other related terms are not specifically mentioned.7

그 이후로 가상 진료는 환자 치료에서 떼려야 뗄 수 없는 부분이 되었으며, 코로나19 팬데믹 기간 동안 공중 보건 조치로 인해 더욱 발전했습니다.16 의사와 의료 시스템이 이러한 전환에 충분히 준비되지 않았기 때문에 가상 진료로의 급격한 변화는 문제가 되었습니다. 곧 발표될 CanMEDS 2025 개정판은 가상 의료를 포함한 21세기 의학에서 요구되는 업무에 맞춰 의사의 역량을 조정할 수 있는 기회입니다.
Since then, VC has become an inextricable part of patient care, with advances precipitated by public health measures during COVID-19 pandemic.16 The rapid change to VC was problematic because physicians and the healthcare system were not fully prepared for this transition. The upcoming CanMEDS 2025 revision is an opportunity to align physician competencies with the tasks required in 21st century medicine including virtual care.

2025년 CanMEDS 역량 프레임워크 내에서 가상 의료가 어떻게 더 잘 표현될 수 있을까요?
How can VC be better represented within the 2025 CanMEDS competency framework?

가상 진료의 성장에 따라 의사는 가상 환경에서 진료하는 데 적응해야 합니다. 이를 위해서는 의사 역량 프레임워크의 상당한 수정이 필요합니다. 환자 중심의 팀 기반 치료를 최적화하기 위해 여러 의료 전문 분야의 질병 진단 및 치료와 P2P 커뮤니케이션에서 이러한 도구에 대한 의존도가 높아짐에 따라 이를 통합할 필요가 있습니다. 
With the growth of VC, physicians must adapt to performing in the virtual environment. This will require substantial modification of the physician competency framework. There is need to incorporate the greater dependence on these tools in the diagnosis and treatment of ailments across medical specialties and in peer-to-peer communication to optimize patient-centred team-based care.

모든 역량을 아우르는 VC를 더 잘 대표할 수 있도록 CanMEDS 의사 역량 프레임워크의 변경을 제안하며, 표 2에서 권장 사항과 그 근거를 CanMED 역할별로 세분화하여 설명합니다. 
We suggest changes to the CanMEDS Physician Competency Framework to better represent VC spanning all competencies and describe our recommendations and their rationale, broken down by CanMEDs role, in Table 2.

전반적으로 의료 전문가 역할에 대한 변화는 안전하고 포괄적인 환자 치료를 제공하기 위해 가상 도구를 사용하여 전문성을 개발하는 데 중점을 둡니다.

  • 커뮤니케이터 역할은 가상 환경에서는 의사소통 기술이 다르다는 점을 인식하고 의사가 기술 사용과 관계없이 훌륭한 커뮤니케이터가 되어야 한다는 점을 강조합니다.
  • 협력자 역할은 다학제 팀의 대면 및 가상 협업을 포함하도록 수정되었습니다.
  • 리더 역할은 의료 시스템이 가상 현실에 의해 어떻게 영향을 받는지, 그리고 가상 현실을 포함한 새로운 치료 모델이 현재의 의료 서비스 제공 방식을 어떻게 개선할 수 있는지를 강조합니다.
  • 건강 옹호자 역할은 VC에 대한 공평한 접근성, 디지털 헬스를 활용하여 지역사회의 필요를 파악하는 방법, 디지털 시대의 미디어 옹호에 대해 다룹니다.
  • 학자 역할은 기술을 최신 상태로 유지하고 사용하는 데 중점을 두며, 인터넷으로 인해 의료 문헌이 전통적인 저널 논문을 넘어 진화하고 있음을 인식합니다.
  • 마지막으로 전문가 역할은 가상 현실의 사용이 증가함에 따라 이제야 필요성이 대두된 기술과 보호 메커니즘을 포함합니다. 

Overall, changes to the Medical Expert role focus on developing expertise using virtual tools to provide safe, comprehensive patient care.

  • The Communicator role recognizes that communication skills are different in a virtual setting and emphasise that physicians need to be excellent communicators regardless of the use of technology.
  • The Collaborator role is modified to include in-person and virtual collaboration in multidisciplinary teams.
  • The Leader role highlights how the healthcare system is impacted by VC, and how new models of care (including VC) can improve upon current approaches to healthcare delivery.
  • The Health Advocate role focuses on equitable access to VC, how digital health can be leveraged to identify community needs, and touches on media advocacy in the digital age.
  • The Scholar role focuses on staying up to date with and using technology and recognizes that due to the internet, healthcare literature is evolving past the traditional journal article.
  • Finally, the Professional role includes the skills and safeguarding mechanisms that have only now become necessary, given the increase in the use of VC.

결론
Conclusion

코로나19 팬데믹으로 인해 VC는 갑작스럽게 환자 치료의 최전선에 서게 되었습니다. 이 기술은 2015년부터 사용되어 왔지만, 당시에는 이 새로운 치료 모델에 대한 역량이 상당히 부족했습니다. 의사들이 사회적 요구와 기대에 부응하는 진료에 진정으로 대비할 수 있도록 기술적으로 개선되었든 그렇지 않든 모든 환경과 관련된 역량을 반영하도록 CanMEDS 2025를 업데이트하는 것이 필수적입니다. 
The COVID-19 pandemic has abruptly forced VC to the forefront of patient care. Although it has been around since CanMEDS 2015, the competencies at the time focused substantially less on this new model of care. It is imperative that CanMEDS 2025 is updated to reflect competencies relevant to all environments, technologically enhanced or otherwise, so that physicians are truly prepared for practice relevant to societal needs and expectations.


Can Med Educ J. 2023 Mar 21;14(1):22-26. doi: 10.36834/cmej.75439. eCollection 2023 Mar.

Virtual Care in CanMEDS 2025

Affiliations collapse

1Department of Medicine, University of Toronto, Ontario, Canada.

2Department of Anesthesiology and Pain Medicine, University of Ottawa, Ontario, Canada.

3Office of Continuing Professional Development, Faculty of Health Sciences, McMaster University, Ontario, Canada.

4University of Saskatchewan, Saskatchewan, Canada.

5Department of Emergency Medicine, UBC Faculty of Medicine, British Columbia, Canada.

PMID: 36998491

PMCID: PMC10042775

DOI: 10.36834/cmej.75439

CanMEDS 2025에서 임상학습환경 (Can Med Educ J. 2023)
The Clinical Learning Environment in CanMEDS 2025

소개
Introduction

임상 학습 환경(CLE)은 현재 CanMEDS 의사 역량 프레임워크에서 잘 드러나지 않는 다면적 개념입니다.1 의료 환경은 사람 및 관계 중심 치료의 기본 원칙을 바탕으로 구축되며, 우리는 환자 치료 제공과 교육 프로그램 실행이라는 동시적 목표가 학습자와 교수진의 관심을 끌기 위해 어떻게 경쟁할 수 있는지에 대해 생각해 보아야 합니다. 이러한 긴장을 탐구하면 학습자를 포함한 환자, 가족, 지역사회, 의료진 등 학습 환경의 모든 구성원의 요구에 부응하는 개입을 설계하고 실행할 수 있습니다.
The Clinical Learning Environment (CLE) is a multi-faceted concept that is currently underrepresented in the CanMEDS physician competency framework.1 Healthcare environments are built on the foundational tenet of person and relationship-centered care, and we must reflect upon how the concurrent objectives of delivering patient care and implementing educational programs may compete for the attention of learners and faculty. Exploration of these tensions will enable the design and implementation of interventions that are responsive to the needs of all citizens in the learning environment including patients, families, communities, and the healthcare workforce, including learners.

위계와 권력 비대칭은 의료 시스템, 더 넓게는 CLE에 만연한 요소로, 더 많은 주의가 필요합니다2. 동전의 양면처럼, CLE 내에서 개입을 어떻게 설계하느냐에 따라 팀과 의료 서비스 제공이 기능적으로 또는 역기능적으로 발전할 수 있습니다. 포용적이고 심리적으로 안전한 의료 환경을 조성하면 환자 안전 및 경험 개선, 팀 역학 지원, 학습자 및 모든 팀원의 전문성 개발이 보장되며, CanMEDS는 이러한 목표를 반영해야 합니다.
Hierarchies and power asymmetries are pervasive elements of healthcare systems, and the CLE more broadly, that require further attention2. Like two sides of the same coin, how interventions are designed within the CLE, can foster functional or dysfunctional teams and healthcare delivery. Creating inclusive, psychologically safe healthcare environments ensures improved patient safety and experience, supportive team dynamics, and professional development for learners and all members of the team; CanMEDS must reflect these objectives.

임상 학습 환경이란 무엇이며 의사의 역량에 중요한 이유는 무엇인가요?
What is the Clinical Learning Environment and why is it important to physician competency?

임상 학습 환경은 "...참여자의 경험, 인식, 학습을 둘러싸고 형성하는 사회적 상호작용, 조직 문화 및 구조, 물리적 및 가상 공간"으로 정의되어 왔습니다.3 다른 정의에서는 "업무 환경과 교육적 맥락 사이의 중첩된 공간"을 언급하기도 합니다. "4

  • 이러한 정의는 학습자 연속체(학부, 대학원 및 평생 의학 교육)와 진료 맥락을 아우르며, 건축적 경계, 디지털 공간, 사회문화적 측면, 교육 커리큘럼, 다양성 및 포용성, 학습의 심리학적 이론, 실천 커뮤니티를 포함한 CLE의 모든 측면을 고려합니다.
  • 5 이러한 정의는 CLE를 복잡하고 중요한 구조, 즉 모든 CanMEDS 역량에 영향을 미치는 규범, 행동, 무언의 행동 강령에 영향을 미치는 구조로 파악합니다.
  • 대인관계는 시스템과 긴밀하게 연결되어 있습니다. 사람은 규범을 알리고 만들고, 시스템은 행동을 가능하게 하고 영향을 미칩니다.

The CLE has been defined as “…social interactions, organizational cultures and structures, and physical and virtual spaces that surround and shape participants’ experiences, perceptions, and learning.”3 Other definitions reference “the overlapping space between the work environment…and the educational context.”4 

  • These definitions span the learner continuum (undergraduate, postgraduate, and continuing medical education) as well as contexts of care, and consider all aspects of the CLE including
    • architectural boundaries, digital spaces, sociocultural aspects, educational curricula, diversity and inclusion, psychological theories of learning, and communities of practice.5 
  • These definitions cast the CLE as a complex overarching structure–one that influences the norms, behaviours, and unspoken codes of conduct that touch on all CanMEDS competencies.
  • The interpersonal is tightly linked with the systemic: people inform and create norms, while systems in turn, enable and influence behaviours.

따라서 개인이 의료 및 의료 교육 프로그램에 어떤 영향을 미칠 수 있는지, 그리고 이러한 프로그램이 의료 인력의 모든 구성원을 어떻게 형성하는지 이해하도록 하기 위해서는 모든 CanMEDS 2025 역량의 원동력으로서 CLE를 인정하는 것이 중요합니다. 
Therefore, acknowledging the CLE as an enabler of all CanMEDS 2025 competencies is critical to ensuring that individuals appreciate how they may influence healthcare and medical training programs, and how these programs, in turn, shape all members of the healthcare workforce.

환자안전, 학습자 경험, 대중의 신뢰 필요성, 의료 시스템(그리고 더 일반적으로는 사회 전반)에서의 책무성 강화 요구는 CLE에 대한 비판적이고 지속적인 검토의 원동력이 되고 있습니다. 환자안전 사고5와 그에 따른 대학원 의학 교육 환경 개선에 대한 옹호,6 지역 사회 기반의 운동은 의료 및 의학교육의 지속적인 불평등에 주목하고 있습니다.
Patient safety, learner experience, the need for public trust, and calls for increased accountability within the healthcare system (and across society, more generally) are the impetuses for a critical and ongoing examination of the CLE. Policy changes have resulted from patient safety incidents5 and subsequent advocacy for improvement in postgraduate medical training environments;6 community-based movements have put a spotlight on persisting inequities in healthcare and medical education.

여러 문헌에서는 의학 커리큘럼7,8,9을 통해 중첩된 억압 시스템이 어떻게 유지되고 있으며, 무례, 배제, 인종차별 및 학대를 허용하고 심지어 가능하게 하는 숨겨진 커리큘럼에 의해 지속되고 있는지에 대해 설명했습니다. 의학 교육의 본질적인 위계적 특성뿐만 아니라 사회적 억압의 축을 통해 학문적 의학에 존재하는 많은 권력 비대칭은 모든 학습자 상호 작용에 영향을 미칩니다. 의학 학습자와 교수진이 직면하는 협박, 차별, 괴롭힘의 높은 유병률11,12은 CLE가 대부분 사회적으로 구성되며, CLE를 형성하고 변화시키는 것은 우리의 도덕적, 신탁적 책임이라는 사실을 상기시켜 줍니다. 학습자와 교수진 모두는 긍정적인 변화의 주체가 될 수 있는 잠재력을 가지고 있으며 CLE에 의미 있는 기여를 할 수 있습니다.
The literature has described how overlapping systems of oppression are upheld directly through medical curricula7,8,9 and are perpetuated by a hidden curriculum that allows and even enables disrespect, exclusion, racism,10 and mistreatment. The many power asymmetries that exist in academic medicine, both through the inherently hierarchical nature of medical training but also through societal axes of oppression, influence every learner interaction. The high prevalence of intimidation, discrimination, and harassment faced by medical learners and faculty11,12 reminds us that the CLE is largely socially constructed, and it is our moral and fiduciary responsibility to shape and change it. Learners and faculty alike have the potential to be positive change agents and contribute meaningfully to the CLE.

CLE에 대한 다양한 증거 기반 척도가 발표되었습니다.4,13 

  • 의학전문대학원 교육 인증위원회에서는 이미 인증 절차에 CLE에 대한 공식적인 검토를 포함시켰는데, 임상 학습 환경 검토(CLER) 프로그램으로 알려진 검토 절차는 미국 인증 기관의 CLE 상태와 졸업생의 향후 진료 스타일에 대한 통찰력을 제공합니다4. 
  • 마찬가지로 캐나다 레지던트 인증 컨소시엄도 형평성, 다양성, 포용성 및 기타 학습 환경 고려 사항을 대학원 의학교육 인증 과정에 직접 인정하고 통합하는 것을 우선시하고 있습니다.14 

이는 CanMEDS 2025 내에서 CLE를 공식적으로 인정하여 의사 역량 프레임워크를 인증 표준과 일치시켜야 할 필요성을 시사하는 것입니다.
Various evidence-based measures of the CLE have been published.4,13 

  • The Accreditation Council for Graduate Medical Education has already incorporated formal review of the CLE into their accreditation processes–a review process, known as the Clinical Learning Environment Review (CLER) Program, provides insight into both the status of American accredited institutions’ CLEs and future practice styles of their graduates4.
  • Similarly, the Canadian Residency Accreditation Consortium has also prioritized acknowledging and integrating equity, diversity, inclusion, and other learning environment considerations directly into the postgraduate medical education accreditation process.14 

This speaks to the need for the CLE to be formally acknowledged within CanMEDS 2025, bringing the physician competency framework into alignment with accreditation standards.

임상 학습 환경은 2015 CanMEDS 역량 프레임워크에서 어떻게 표현됩니까?
How is the Clinical Learning Environment represented in the 2015 CanMEDS competency framework?

실무 그룹은 학자, 리더, 전문가 역할 내에서 CLE 관련 지원 역량을 확인했습니다(표 1A 및 및1B).1B). 2015년 CanMEDS 프레임워크 초안이 작성된 이후 캐나다 사회 전반에서 #미투, #흑인 생명도 소중하다, 진실과 화해 등의 지역사회 기반 운동과 관련된 책임에 대한 요구가 증가했습니다. 이러한 운동은 의료진, 특히 환자, 가족, 커뮤니티의 생생한 경험을 반영하기 때문에 의료와 직접적인 관련이 있습니다. 역사적으로 CLE의 사회적 차원에 대한 고려가 이루어졌지만, CLE를 구성하는 규범을 가능하게 하거나 방해하는 데 있어 권력, 특권, 사회적 위치가 어떤 역할을 하는지에 대한 더 많은 조사가 필요합니다. 의사와 기타 의료 서비스 제공자가 모든 행동(및 무행동)을 통해 CLE를 어떻게 형성하는지 이해하려면 여러 CanMEDS 역할에 걸쳐 역량을 업데이트해야 합니다. 
Our working group identified CLE-specific Enabling Competencies within the Scholar, Leader, and Professional Roles (Table 1A and and1B).1B). Since the 2015 CanMEDS framework was drafted, there have been increased calls for accountability across Canadian society related to community-based movements such as #MeToo, #BlackLivesMatter, and Truth and Reconciliation, among others. These movements have direct relevance to healthcare as they reflect lived experiences of healthcare teams and, importantly, patients, families, and communities. Historically, although consideration has been given to the social dimensions of the CLE, greater interrogation of the role that power, privilege, and social location play in enabling or disrupting the norms that comprise the CLE is required. Understanding the CLE, and how physicians and other health care providers shape it through their every action (and inaction), will require updates to competencies across multiple CanMEDS Roles.

 

임상 학습 환경이 2025 CanMEDS 역량 프레임워크 내에서 어떻게 더 잘 표현될 수 있을까요?
How can the Clinical Learning Environment be better represented within the 2025 CanMEDS competency framework?

다가오는 CanMEDS 2025 개정판(표 1C)에 CLE의 중심이 되는 개념을 보다 직접적으로 통합할 수 있는 여지가 있습니다. CLE는 전통적인 의미의 '기술'이 아니라는 점을 인정하지만, 심리적 안전, 문화적 겸손, 제도적 문화 변화에 대한 개념은 다른 CanMEDS 역량이 뒷받침할 수 있는 틀을 제공합니다. 
There is room to more directly incorporate concepts that are central to the CLE in the forthcoming CanMEDS 2025 revisions (Table 1C). While we acknowledge that the CLE is not a ‘skill’ in the traditional sense, it frames notions of psychological safety, cultural humility, and institutional culture change that other CanMEDS competencies would support.

심리적 안전은 "학습자가...자신의 업무 환경을 개인 내적 위험이 내재된 행동에 참여하는 데 도움이 되는 것으로 인식하는 정도"로 정의할 수 있습니다.15 심리적으로 안전한 학습 환경은 학습, 의사소통, 팀 결속, 협업, 환자 경험 및 결과에 긍정적인 영향을 미칩니다.16,17,18 심리적 안전을 증진하는 것에 대해 명시적으로 언급하는 것은 성장 마인드를 촉진하는 데 매우 중요합니다.8 역량 기반 교육에 필요한 특성이며19 의료계에 만연한 수치심 기반 교육 접근법과 대조되는 특성입니다.8
Psychological safety can be defined as “the degree to which learners…perceive their work environment as conducive to engaging in behaviours that have inherent intrapersonal risk.”15 Psychologically safe learning environments have positive impacts on learning, communication, team cohesion, collaboration, patient experiences, and outcomes.16,17,18 Being explicit about fostering psychological safety is critically important to promoting a growth mindset,8 a characteristic that is necessary in competency-based training19 and contrasts with the shame-based teaching approaches that remain pervasive in medicine.8

문화적 겸손은 자기 성찰과 비판에 대한 평생의 헌신을 반영합니다. 문화적 겸손에는 다음이 포함됩니다.20 

  • 자신의 사회적 위치에 대한 인식을 키우고, 
  • 타인의 경험을 완전히 이해하는 데 있어 자신의 한계를 인식하며, 
  • 타인을 자신의 경험에 대한 전문가로 바라보고
  • 신뢰에 기반한 관계를 조성하는 것

문화적 겸손에는 지속적인 학습과 탈학습에 대한 헌신과 함께 반인종주의 및 트라우마 정보에 기반한 접근법에 기반한 지식, 기술 및 태도를 개발하는 것이 포함됩니다.
Cultural humility reflects a lifelong commitment to self-reflection and critique. It involves

  • cultivating an awareness of one’s social location,
  • recognizing one’s limitations in fully understanding the experiences of others,
  • viewing others as experts of their own experiences, and
  • fostering trust-based relationships.20 

Cultural humility involves developing knowledge, skills, and attitudes grounded in anti-racism and trauma-informed approaches, with commitment to ongoing learning and unlearning.

마지막으로, 우리 모두는 다양한 인구의 요구를 대표하고 이에 대응할 수 있는 의료 인력을 양성하고 지원해야 할 공동의 책임이 있습니다. 이러한 목표를 달성하기 위해서는 모든 의료 서비스 제공자가 현재의 대표성 격차를 해소하고 모두가 소속감을 느낄 수 있는 환경을 조성하기 위한 제도적 문화 변화에 동참해야 합니다.
Finally, we all have a collective responsibility to be intentional about building and supporting a healthcare workforce that is representative of, and responsive to, the needs of our diverse population. This goal requires all healthcare providers to be part of creating institutional culture change to create environments in which the current representation gap is eliminated, and everyone feels like they belong.

CLE를 비정치적인 실체apolitical entity로 보는 경향은 오늘날 우리가 봉사하는 학습자, 교사, 환자, 가족 및 지역사회의 요구를 충족하는 학술 의학의 능력에 해를 끼쳤습니다. CanMEDS 2025는 모두에게 포용적이고 심리적으로 안전한 환경, 그리고 가장 중요한 것은 환자, 가족, 지역사회를 위한 최상의 경험과 결과를 보장하는 데 필요한 역량의 습득과 유지를 적극적으로 지원하는 방법과 CLE에 대해 우리가 생각하는 방식에 필요한 미묘한 차이를 가져올 수 있는 기회를 제공합니다. 광범위한 범위를 고려할 때, CLE는 자연스럽게 CanMEDS의 모든 역량을 다루게 될 것이며, 다른 실무 그룹과 권고 사항이 중복될 것으로 예상됩니다. 선택된 역량의 중요성은 해당 역량이 워킹그룹 활동 전반에서 어떻게 증폭되거나 수렴되는지 관찰함으로써 쉽게 파악할 수 있습니다.
The tendency to view the CLE as an apolitical entity has been to the detriment of academic medicine’s ability to meet the needs of the learners, teachers, patients, families, and communities that we serve today. CanMEDS 2025 offers us an opportunity to bring much needed nuance to how we think about the CLE and how we proactively support the acquisition and maintenance of the competencies needed to ensure inclusive and psychologically safe environments for all and most importantly, the best possible experiences and outcomes for our patients, families, and communities. Given its wide-reaching scope, the CLE will naturally touch on all the CanMEDS competencies, and we anticipate overlap in our recommendations with other working groups. The importance of select competencies will be readily identified by observing how they are amplified or converge across working group efforts.

자율 규제 전문직으로서, 우리는 학습자와 교수진 모두에게 CLE와 관련하여 재구상된 역량에 대한 책임을 부여해야 합니다. 이러한 책임을 다하기 위해서는 이러한 역량을 습득하고, 실행하고, 가르치는 방법을 설계, 구현 및 평가하기 위한 리소스가 필요합니다.
As a self-regulating profession, we must hold both learners and faculty accountable to these re-imagined competencies relating to the CLE. This accountability will require resources for designing, implementing, and evaluating how these competencies are acquired, practiced, and taught.

 

Can Med Educ J. 2023 Mar 21;14(1):41-45. doi: 10.36834/cmej.75537. eCollection 2023 Mar.

The Clinical Learning Environment in CanMEDS 2025

Affiliations

1University of Calgary, Alberta, Canada.

2University of Toronto, Ontario, Canada.

3University of Manitoba, Manitoba, Canada.

4McGill University, Quebec, Canada.

5University of Saskatchewan, Saskatchewan, Canada.

PMID: 36998502

PMCID: PMC10042790

DOI: 10.36834/cmej.75537

CanMEDS 2025에서 복잡-적응적 시스템(Can Med Educ J. 2023 )
Complex adaptive systems in CanMEDS 2025

소개
Introduction

복합적 적응 시스템은 CanMEDS 의사 역량 프레임워크에서 잘 알려지지 않은 개념입니다.1 복합적 적응 시스템환자와 인구를 위한 최적의 의료 결과를 제공하기 위해 복잡한 의료 시스템을 탐색하고 참여하는 것과 관련이 있으며,2 의료 팀이 복합적 적응 시스템으로 기능하는 방식부터 시작합니다.3 이 원고는 이 개념과 현재 및 향후 CanMEDS의 반복에 대한 연관성을 요약하는 것을 목표로 합니다. 
Complex Adaptive Systems are underrepresented in the CanMEDS physician competency framework.1 As a concept, it relates to navigating and engaging with our complex health system in service of optimal health care outcomes for patients and populations,2 starting with how healthcare teams function as complex adaptive systems.3 This manuscript aims to summarize the concept and its links to current and future iterations of CanMEDS.

복잡적 적응 시스템이란 무엇이며 의사의 역량에 중요한 이유는 무엇인가요?
What are Complex Adaptive Systems and why are they important to physician competency?

복잡성이란 사물이 예기치 못한 돌이킬 수 없는 방식으로 상호 작용하는 풍부한 상호 연결성을 말합니다. WHO에 따르면 "보건 시스템은 건강을 증진, 회복 및/또는 유지하는 것을 주된 목적으로 하는 조직, 사람, 행동과 같이 상호 작용하고 상호 의존하는 모든 구성 요소로 이루어져 있다."4 이는 복잡적 적응 시스템의 정의와 일치합니다:

  • 복잡적, 다양한 요소 사이에 수많은 비선형 연결이 있는 다양성을 의미하고,
  • 적응적, 변화하는 맥락 내에서 경험을 통해 학습하고 변화하는 역량을 시사하며,
  • 시스템, 상호 의존적으로 연결된 에이전트의 집합입니다.5,6

Complexity is about rich interconnectivity whereby things interact in unexpected and irreversible ways. According to the WHO, “The health system consists of all interacting and interdependent components, such as organizations, people, and actions, whose primary intent is to promote, restore, and/or maintain health.”4 That aligns with the definition of a complex adaptive system:

  • complex, implying diversity with a great number of non-linear connections between a wide variety of elements;
  • adaptive, suggesting the capacity to learn from experience and change within a changing context;
  • system, a set of interdependently connected agents.5,6

역사적으로 캐나다 의료 시스템은 병원 내 치료를 통한 급성 질환 치료에 중점을 두고 설계되었습니다.7 그 결과, 병원과 의사가 매개하는 치료가 우리 시스템의 조직과 자금에 대한 논의를 지배해 왔습니다. 레지던트 수련은 대부분 이러한 구조에 편입되어 그 틀에 맞춰져 왔습니다. 그러나 건강과 질병 관리의 결정 요인은 다층적이고 상호 의존적인 하위 시스템으로, 서로 지속적으로 상호작용하고 영향을 주고받습니다.8 이러한 시스템의 경계는 반투과적이다. 코로나19 팬데믹 기간 동안 생물학적 조건과 사회적 조건 및 상태가 상호 작용하여 개인의 건강 결과를 악화시키거나 해를 입힐 수 있는 취약성을 증가시킴으로써 이러한 시스템의 상호 연결성이 입증되었습니다.9 
As historically designed, the Canadian healthcare system focuses on curing acute disease with in-hospital care.7 As a result, hospitals and physician-mediated care have dominated the dialogue on our system’s organization and funding. Resident physician training has been largely embedded in and framed by these structures. However, the determinants of health and management of disease are multilayered and interdependent subsystems that continuously interact and influence each other.8 The boundaries of these systems are semipermeable. Their interconnectedness was demonstrated during the COVID syndemic as biologic and social conditions and states interacted to increase a person’s susceptibility to harm or worsen their health outcomes.9

각 의사가 보건 시스템의 모든 요소(예: 식량 안보, 주택, 교육, 기후 변화, 가정 간호, 아동 발달, 불평등 등)에 영향을 미칠 수는 없지만, 의사는 이 생태계를 이해하여 환자 및 이 시스템 내의 다른 에이전트와 어떻게 협력할지 선택할 수 있어야 합니다. 이를 위해서는

  • 시스템에 대한 기본적인 이해가 필요하며, 시스템이 조직 구조와 문화에 미치는 영향을 이해해야 한다
  • 관점의 다양성의 중요성을 강조하고,
  • 명확하고 공유된 목적의 범위 내에서 행동하며,
  • 지속적으로 적응할 수 있는 공간을 만들고,
  • 환경의 새로운 변화로부터 학습해야 한다.

While each physician may not be able to influence all elements of the health system (as for example, food security, housing, education, climate change, home care, child development, inequity, etc.), physicians need to understand this ecosystem in order to make choices about how they will engage with their patients and other agents within this system. This requires

  • a foundational understanding of systems and their influence on organizational structure and culture,
  • emphasizing the importance of diversity of perspective,
  • acting within the boundaries of a clear and shared purpose, and
  • creating spaces for continuous adaptation to, and
  • learning from emergent changes in the environment.10

이러한 이해를 바탕으로 의사는

  • 모든 이해관계자를 참여시키고,
  • 체계적으로 높은 레버리지(티핑 포인트)를 찾아서 트리거하며,
  • 의도하지 않은 결과 및 시스템 구조와 같은 개념을 인식할 수 있는 기술을 개발해야 합니다. 마지막으로, 의사는
  • 예측할 수 없는 급변하고 역설적이며 얽히고설킨 상황을 받아들이고 대처할 수 있는 태도와 기술을 갖추고, 자신의 행동과 건강을 신중하게 관리해야 합니다. 

Building on this understanding, physicians need to 

  • develop the skills to engage all stakeholders,
  • find and trigger systemic high leverage (tipping) points, and
  • be aware of concepts like unintended consequences and system structures. Finally, physicians need to
  • possess the attitudes and skills to accept and deal with rapid, unpredictable, paradoxical, and tangled situations and thoughtfully manage their behaviours, and own wellness.

2015 CanMEDS 역량 프레임워크에서 복합 적응 시스템은 어떻게 표현되나요?
How are Complex Adaptive Systems represented in the 2015 CanMEDS competency framework?

2015 CanMEDS 의사 역량 프레임워크의 주요 변경 사항 중 하나는 '관리자' 역할을 '리더' 역할로 변경한 것입니다.11 이러한 변경은 복잡한 의료 시스템 전반에서 질 향상과 자원 관리에 대한 의사의 역할에 더 초점을 맞추기 위해 이루어졌습니다. 리더 역할에 추가된 핵심 역량 중 두 가지(1, 3번)는 복잡한 시스템에서 기능하는 데 필요한 기술에 가장 큰 기여를 했습니다. 
One of the major changes in the 2015 CanMEDS Physician Competency Framework was the modification of the ‘Manager’ role to the ‘Leader’ role.11 This change was made to bring greater focus on physicians’ role in quality improvement and resource stewardship throughout complex health systems. Two of the key competencies added to the role of Leader (1 and 3) contributed most to the skills needed to function in complex systems.

두 번째 건강 옹호자 핵심 역량은 급성 의료 시스템의 요소를 1차 의료 및 예방 의료의 요소뿐만 아니라 건강과 웰빙에 영향을 미치고 옹호가 필요한 다른 요소와 연결하는 시스템적 사고11와도 관련이 있습니다. 마지막으로, 협력자의 역할에는 '치료의 전환' 및 '지역사회 제공자와의 협력'과 같은 개념을 포함하여 시스템적 사고와 연결되는 몇 가지 요소가 있습니다.11 
The second Health Advocate key competency also touches on systems thinking11 as it connects the elements of the acute healthcare system upstream, not only with those of primary and preventive care, but also with other factors that affect health and wellness and require advocacy. Finally, the role of Collaborator has a few elements that link with systems thinking, including concepts like “transitions of care” and “collaborations with community providers.”11

복잡한 적응형 시스템을 2025 CanMEDS 역량 프레임워크 내에서 어떻게 더 잘 표현할 수 있을까요?
How can Complex Adaptive Systems be better represented within the 2025 CanMEDS competency framework?

최근 CanMEDS 와 LEADS 리더십 프레임워크를 비교한 결과, CanMEDS 역량은 개별 환자 치료에 초점을 맞춘 일부 영역(자기 주도, 타인 참여, 결과 달성)에 상당한 관심을 기울이는 반면, 복합 적응 시스템과 관련된 광범위한 역량(연합 및 시스템 전환 개발)에는 덜 관심을 기울이는 것으로 나타났습니다.12 지난 몇 년 동안, 특히 COVID-19 팬데믹에 대응하여 발생한 빈번한 시스템 중단은 모든 의사가 복합 적응 시스템에 대한 이해를 높여야 할 필요성을 입증했습니다.13 이 분야의 역량은 의사들이 스스로를 의료 시스템 혁신의 적극적인 참여자로 인식하는 데 도움이 될 것이며, 이 개념이 CanMEDS 2025에서 더욱 구체화되어야 할 개념으로 식별되는 데 기여했을 수 있습니다.
A recent comparison of CanMEDS with the LEADS leadership framework found that CanMEDS competencies pay substantial attention to some of the domains focused on the care of individual patients (lead self, engage others, and achieve results), but less attention to broader competencies related to Complex Adaptive Systems (develop coalitions and systems transformation).12 The frequent systemic disruptions that have occurred over the past several years, particularly in response to the COVID-19 pandemic have demonstrated the need for a greater understanding of complex adaptive systems by all physicians.13 Competencies in this area will help physicians to see themselves as active participants in the transformation of the healthcare system and may have contributed to this concept being identified as one which needs to be fleshed out further in CanMEDS 2025.

CanMEDS 2025에 복합 적응형 시스템을 더 잘 통합하기 위해 제안하는 변경 사항은 표 1에 요약되어 있습니다.

  • 리더 역할에서 제안된 수정 사항은 환자별 결정과 시스템 전반의 결정 사이의 복잡한 상호작용에 대한 혁신, 출현 및 이해를 촉진하는 품질 개선과 함께 복잡성에 기반한 리더십 패러다임에 초점을 맞출 것입니다.14
  • 협력자에서는 복잡한 결정을 내릴 때 다양성과 관점의 차이에 대한 필요성을 보다 명확하게 인정하고 환자와 가족을 넘어 협력자의 개념을 확장할 것을 제안합니다.
  • '건강 옹호자'에서는 환자와 건강의 사회적 결정 요인 간의 복잡한 상호 작용을 인정하는 제안을 합니다. 마지막으로,
  • '학자'에서는 교육 활동의 설계 및 제공에 사회 학습 이론의 요소를 통합해야 하는 역량을 포함할 것을 제안합니다.

The changes that we propose to better incorporate Complex Adaptive Systems in CanMEDS 2025 are outlined in Table 1.

  • Within the Leader role, the proposed modifications would increase the focus on complexity-informed leadership paradigms alongside quality improvement that promote greater support for innovation, emergence and understanding of the complex interactions between patient-specific and system-wide decisions.14 
  • Under Collaborator, we propose more explicitly acknowledging the need for diversity and differences of perspective when making complex decisions as well as broadening the notion of who is a collaborator beyond the patient and family.
  • For Health Advocate, we make suggestions that would acknowledge the complex interactions between patients and the social determinants of health.
  • Lastly, under Scholar we suggest including competencies that require the incorporation of elements of social learning theory into the design and delivery of educational activities.

 


Can Med Educ J. 2023 Mar 21;14(1):50-53. doi: 10.36834/cmej.75538. eCollection 2023 Mar.

Complex adaptive systems in CanMEDS 2025

Affiliations

1University of Alberta, Alberta, Canada.

2Canadian Society of Physician Leaders, Ontario, Canada.

3University of Ottawa, Ontario, Canada.

4Royal College of Physicians and Surgeons of Canada, Canada.

5University of Toronto, Ontario, Canada.

6University of Saskatchewan, Saskatchewan, Canada.

PMID: 36998500

PMCID: PMC10042785

DOI: 10.36834/cmej.75538

CanMEDS 2025에서 데이터-기반 의학(Can Med Educ J. 2023 )
Data-Informed Medicine in CanMEDS 2025

 

소개
Introduction

건강 데이터 및 정보 사용에 대한 의사의 역량 강화의 필요성은 널리 인정받고 있습니다. 최근 검토1에 따르면 데이터 정보 의학은 CanMEDS 의사 역량 프레임워크2에서 충분히 반영되지 않은 개념으로 확인되었으며, 왕립대학의 "인공지능 및 신흥 디지털 기술에 관한 태스크포스 보고서"는 관련 역량을 CanMEDS에 통합할 것을 권고했습니다.3 데이터 기반 의학의사가 양질의 환자 의료 서비스를 제공하는 데 필요한 데이터 및 기타 디지털 정보의 수집, 사용, 공유에 필요한 역량과 관련이 있습니다. 이 입문서에서는 데이터 정보 의학의 개념과 이전 및 향후 버전의 CanMEDS와의 연관성을 요약합니다. 가상 진료의 관련 개념은 이번 호의 다른 기사에서 다룹니다.4
The need for enhanced physician competency in the use of health data and information is broadly acknowledged. A recent review1 Identified data-informed medicine as a concept that is underrepresented in the CanMEDS physician competency framework2 and the Royal College “Task Force Report on Artificial Intelligence and Emerging Digital Technologies” recommended incorporating related competencies into CanMEDS.3 Data-informed medicine relates to competencies required for the collection, use, and sharing of data and other digital information that is needed by physicians to deliver quality patient health service. This primer summarizes the concept of data-informed medicine and its links to former and future iterations of CanMEDS. The related concept of virtual care is addressed in another article in this issue.4

데이터 기반 의료란 무엇이며 의사의 역량에 중요한 이유는 무엇인가요?
What is Data-Informed Medicine and why is it important to physician competency?

현대 사회의 많은 부분에서 아날로그 정보 프로세스는 풍부한 데이터를 생성하는 디지털 워크플로우로 대체되었습니다. 이러한 변화는 여행, 상거래, 엔터테인먼트, 정치, 사회적 상호작용 등 대부분의 사회 분야에서 혁신을 가져왔습니다. 그러나 의료 분야에서는 이러한 가치 제안을 활용하는 속도가 느리고 디지털 의료 데이터가 기하급수적으로 증가하고 있음에도 불구하고 제대로 활용되지 않고 있습니다. 데이터 아키텍처를 의도적으로 설계하기보다는 거버넌스, 공공 정책 또는 신기술의 부산물로서 우연히 발생하는 경우가 많기 때문에 의료 데이터 구조가 무질서하게 구성되는 경우가 많습니다. 양질의 의료 서비스를 위한 증거 기반 데이터 아키텍처의 근본적인 중요성에 대한 이해가 널리 부족하다는 점이 이러한 접근 방식의 근간을 이루고 있습니다.  
In much of modern society, analogue information processes have been replaced by digital workflows that are generating an abundance of data. This has been transformative for most sectors of society including travel, commerce, entertainment, politics, and social interaction. However, the health sector has been slow to harness the value proposition, and digital health data are underutilized despite being collected at an exponentially increasing rate. Rather than being intentional in the design of data architecture, our health data construct is frequently disorganized and occurs by happenstance as a byproduct of governance, public policy, or new technologies. A widespread lack of literacy about the foundational importance of evidence-based data architecture to quality health service underlies this approach.

개인 및 인구 수준의 건강 데이터가 의학적 의사 결정 방식을 변화시키면서 의학은 진화하고 있습니다.5

  • 앞으로 의료계는 데이터의 힘을 활용하여 건강 데이터를 일상적으로 수집하고 분석하여 건강 결정 및 시스템 개선에 필요한 지식을 생성하는 학습 의료 시스템을 개발해야 합니다.6
  • 그 결과 환자 치료에 정보를 제공하고 공익을 증진하기 위해 건강 데이터의 수집, 교환, 집계 및 분석을 통합하는 데이터 기반 의학의 실천이 이루어질 것입니다.
  • 데이터 기반 의학은 인간의 인지능력을 대체하는 것이 아니라 의료 서비스 제공자가 의료 데이터를 전문적으로 활용하여 진료를 최적화하도록 요구할 것입니다. 

The practice of medicine is evolving as individual and population-level health data change how medical decisions are made.5 

  • Moving forward, the medical profession will need to harness the power of data to develop learning health systems that routinely collect and analyze health data to generate knowledge to inform health decisions and/or system improvements.6 
  • The result will be the practice of data-informed medicine that incorporates the collection, exchange, aggregation, and analysis of health data to inform patient care and promote public good.
  • Rather than replacing human cognition, data-informed medicine will require health providers to expertly leverage health data to optimize their practice.

'인간과 기계'의 데이터 협력이라는 새로운 패러다임을 뒷받침하기 위해서는 의료 데이터의 수집, 정리, 저장, 교환, 집계, 해석을 통해 양질의 환자 치료를 촉진하는 새로운 의사 역량이 필요합니다.7,8. 현재 의료 정보 시스템의 품질이 다양하고 의료 데이터 아키텍처, 의료 데이터 사용, 양질의 의료 서비스 제공 간의 관계를 보여주는 커리큘럼 콘텐츠가 부족하기 때문에 의사 수련생이 이러한 영역에서 역량을 갖추도록 하는 것은 어려운 일입니다.

  • 학부 의학교육에 보건 데이터 커리큘럼을 의미 있게 통합해야 한다는 요구에도 불구하고8 가시적인 변화는 거의 일어나지 않았으며, 기본적인 보건 데이터 역량이 부족하고 파편화된 디지털 기술 및 공공 정책 환경에서 활동할 것으로 예상되는 의료 전문가 집단이 계속 양성되고 있습니다.
  • 또한 데이터 리터러시 부족과 제대로 통합되지 않은 기술은 의사의 소진에 기여하고 있습니다.9

포괄적인 디지털 의료 데이터의 잠재력을 활용하여 환자 치료와 인구 건강을 개선하려면 의사에게 기초적인 의료 데이터 리터러시와 전문성을 갖추는 것이 필수적입니다.
To support this new paradigm of ‘human and machine’ data cooperation will require new physician competencies;7,8 the collection, organization, storage, exchange, aggregation, and interpretation of health data to facilitate high-quality patient care. Ensuring that physician trainees are competent in these areas is challenging due to current variability in quality of health information systems and a shortfall of curricular content demonstrating the relationship between health data architecture, health data use, and the provision of quality health services.

  • Despite calls for the meaningful integration of health data curriculum in undergraduate medical education8 little tangible change has occurred, and a cohort of medical professionals continues to be trained who lack basic health data competencies, and are expected to function in a fragmented digital technology and public policy environment.
  • Further, the lack of data literacy and poorly integrated technology is contributing to physician burnout.9 

Equipping physicians with foundational health data literacy and expertise is essential to harness the potential of comprehensive digital health data to improve patient care and population health.

여기에서는 의사가 데이터를 수집, 교환, 집계, 분석하여 올바른 데이터 기반 의학을 실천할 수 있도록 핵심 역량을 강화하기 위해 CanMEDS의 업데이트를 제안합니다(표 1).
Herein, we suggest updates to CanMEDS to promote core competencies that will enable physicians to collect, exchange, aggregate, and analyze data to practice sound data-informed medicine (Table 1).

 

데이터 기반 의학은 2015 CanMEDS 역량 프레임워크에서 어떻게 표현됩니까?
How is Data-Informed Medicine represented in the 2015 CanMEDS competency framework?

데이터 기반 의학은 2015 CanMEDS 역량 프레임워크에 포함된 용어가 아니었다.2 의료 정보학은 리더 역할(1.4)에 언급되어 있지만 구체적으로 정의되어 있지는 않습니다. 이러한 역량을 제안하기 위해 캐나다 의학위원회에서 채택한 정의를 활용하는데, 캐나다 의학위원회는 의료 정보학"의료 서비스에서 정보[및 데이터] 설계 및 사용에 대한 연구"로 정의합니다.10 의료 정보학에 대한 정의를 다음과 같이 제안합니다: "양질의 의료 서비스를 증진하기 위해 의사가 의료 정보학 지식을 적용하는 것." 2015년 프레임워크에서는 의료 전문가, 건강 옹호자, 학자의 역할에 따른 정보 활용에 대해 언급하고 있지만, 데이터 기반 의료를 최적화하는 데 필요한 여러 역량은 명시되어 있지 않습니다.
Data-informed medicine is not a term that is acknowledged in the 2015 CanMEDS competency framework.2 Health informatics is mentioned under the Leader role (1.4), but it is not specifically defined. For the purposes of these proposed competencies, we utilize a definition adapted from the Medical Council of Canada, who define health informatics as “the study of information [and data] design and use in health care.”10 In turn we propose the following definition of medical informatics: “the application of health informatics knowledge by physicians to promote quality health services.” While the 2015 framework does mention the use of information under the medical expert, health advocate, and scholar roles, the many competencies required to optimize data-informed medicine are not explicit.

2015 CanMEDS 의사 역량 프레임워크에 이러한 개념이 없다는 점을 고려할 때,2 최근 몇 년간 데이터 기반 의학의 중요성이 강조되면서 의료 정보 교환, 의료 데이터 형평성, 의료 데이터 상호운용성 및 데이터 분석에 대한 관심이 높아진 지난 10년 동안 이 분야에서 일어난 변화를 인식하는 것이 중요합니다.
When considering the absence of this concept within the 2015 CanMEDS physician competency framework,2 it is important to recognize the changes that have occurred in this field over the past decade, marked by an increased focus on health information exchange, health data equity, health data interoperability, and data analytics that have prompted an emphasis on the importance of data-informed medicine over recent years.

데이터 기반 의학이 2025년 CanMEDS 역량 프레임워크 내에서 어떻게 더 잘 표현될 수 있을까요?
How can Data-Informed Medicine be better represented within the 2025 CanMEDS competency framework?

왕립대학의 "인공지능 및 신흥 디지털 기술에 관한 태스크포스 보고서"는 데이터 정보 의학을 지원하기 위한 새로운 역량의 중요성을 강조하면서 '디지털 건강 리터러시'를 8번째 CanMEDS 역할로 추가할 것을 제안했습니다.3 그러나 우리는 이 개념과 관련된 역량(가상 진료에 관한 관련 기사4에서도 다루고 있음)이 기존의 CanMEDS 역할에 포함되어야 한다고 생각합니다. 따라서 고유한 역할을 신설하기보다는 데이터 기반 의료의 핵심 요소를 포함하도록 기존 지원 역량을 수정할 것을 제안합니다. 
Underscoring the importance of new competencies to support data-informed medicine, the Royal College “Task Force Report on Artificial Intelligence and Emerging Digital Technologies” suggested that ‘Digital Health Literacy’ should be added as an eighth CanMEDS role.3 However, we believe that competencies related to this concept (which are also addressed in a related article on virtual care4) should span existing CanMEDS roles. We therefore suggest modification of existing enabling competencies to include key elements of data-informed medicine, rather than the creation of a unique role.

2025 CanMEDS에 통합된 개념에는 다음 등이 포함됩니다. 

  • 의료 서비스에서 환자 데이터의 교환, 집계, 분석 및 활용을 촉진하기 위한 환자 데이터의 수집 및 저장, 
  • 개인 건강 정보에 대한 환자 접근성 개선, 
  • 개인 및 시스템 성과 개선을 위한 건강 데이터 사용 촉진 

Concepts that have been integrated include

  • the collection and storage of patient data to facilitate its exchange, aggregation, analysis, and utilization in medical service;
  • improving patient access to personal health information; and
  • promoting the use of health data for individual and system performance improvement.

이러한 역량을 효과적으로 통합하기 위해서는 의료 교육 및 의료 서비스에 대한 접근 방식에 체계적이고 문화적인 변화를 수용하는 디지털 시대의 의학교육을 체계적으로 재구상해야 합니다. 의사는 데이터 기반 의료 서비스를 제공하는 데 있어 필수적인 파트너입니다. 의사들이 진료 과정에서 생성하는 데이터는 임상 치료와 의료 시스템 계획, 혁신, 인구 건강 및 연구에 필수적입니다. 이 백서에 설명된 역량은 의료 데이터와 관련된 기회와 위험이 증가함에 따라 의사가 포괄적이고 조율된 환자 중심적인 방식으로 기여할 수 있는 역량을 보장하기 위한 것입니다. 
To effectively integrate these competencies, a systematic reimagination of digital-age medical education is required that embraces a systemic and cultural shift in our approach to medical training and health service. Physicians are essential partners in the delivery of data-informed care. The data that their practices generate are essential to clinical care and health system planning, innovation, population health, and research. The competencies described in this paper are intended to ensure the capacity of physicians to contribute in a comprehensive, coordinated and patient-centered way as the opportunities and risks associated with health data grow.


Can Med Educ J. 2023 Mar 21;14(1):54-57. doi: 10.36834/cmej.75440. eCollection 2023 Mar.

Data-Informed Medicine in CanMEDS 2025

Affiliations

1University of Saskatchewan, Saskatchewan, Canada.

2Royal College of Physicians and Surgeons of Canada, Ontario, Canada.

3University of Toronto, Ontario, Canada.

4University of Alberta, Alberta, Canada.

5Canadian VIGOUR Centre, Alberta, Canada.

6University of Ottawa, Ontario, Canada.

7College of Physicians & Surgeons of Alberta, Alberta, Canada.

PMID: 36998504

PMCID: PMC10042776

DOI: 10.36834/cmej.75440

CanMEDS 2025에서 의사 휴머니즘(Can Med Educ J. 2023)
Physician Humanism in CanMEDS 2025

 

 

소개
Introduction

캐나다 의사들은 과중한 의료 시스템에서 일하면서 취약성, 스트레스, 소진에 대한 인식이 부족합니다. 의료 인력 부족, 진료의 복잡성 증가, 코로나19 팬데믹은 이러한 문제를 더욱 심화시켜 환자 치료와 의사의 자기 관리에 영향을 미쳤습니다. 이러한 맥락에서 최근 한 검토에서 의사 휴머니즘이 CanMEDS 의사 역량 프레임워크에서 잘 알려지지 않은 개념으로 확인된 것은 놀라운 일이 아닙니다.1 의사 휴머니즘을 명시적으로 인정하고 존중하는 것은 건강하고 성실한 의사를 개발하고 유지하는 데 필요하며, 이는 환자에게 효과적인 인문학적 치료를 제공할 수 있는 의사의 능력을 뒷받침하는 것입니다.2
Canadian physicians experience underacknowledged vulnerability, stress, and burnout while working in an overburdened healthcare system. Health workforce deficits, increasing complexity of care, and the COVID-19 pandemic have magnified these challenges, impacting patient care and physician self-care. Within this context, it is no surprise that a recent review identified physician humanism as an underrepresented concept in the CanMEDS physician competency framework.1 Explicitly acknowledging and honouring physician humanity is necessary to develop and sustain healthy and wholehearted physicians, which also underpins physicians’ ability to provide effective humanistic care for patients.2

의사 인본주의란 무엇이며 의사의 역량에 중요한 이유는 무엇인가요?
What is Physician Humanism and why is it important to physician competency?

의사 휴머니즘내적 요소와 외적 요소를 모두 포함합니다.

  • 의사가 자신의 인간성을 경험하고 '자기 자신'과의 관계를 발전시키는 것
  • 환자를 인본적으로 돌보고 가족, 동료, 의료 시스템과 상호작용하는 데 기여하는 것

최근 문헌에서 주목받고 있는 주제인 의사의 웰니스는 의사 휴머니즘의 두 가지 측면에 대한 담론과 중요한 연관성을 가지고 있습니다. 역사적으로 의사들은 환자, 팀, 기관 등 타인의 필요를 자신의 필요보다 우선시하는 데 전념해 왔습니다. 의사 사회 계약에 대한 신뢰는 이러한 이타주의에 의해 촉진되며, 이타적인 의사 영웅의 패러다임은 종종 칭송받습니다. 그러나 의사도 개인적인 욕구, 한계, 취약성, 불완전성, 타락성, 사망률을 지닌 인간일 뿐입니다. 의사 휴머니즘의 개념은 의사의 신체적 필요(영양, 수면, 인지적 휴식), 사회적 필요(개인적 안전, 의미 있는 관계), 실존적 필요(가치, 의미)의 정당성과 함께 의사의 고유한 인간성과 인격성을 인정하는 것입니다. 
Physician humanism has both inward and outward facing components;

  • physician experiences of their own humanity and development of relationship with “self,” and
  • physician contributions towards humanistic care of patients and interactions with their families, colleagues, and the healthcare system.

Physician wellness, a prominent topic in the recent literature, has important links to discourse on both facets of physician humanism. Historically, physicians have committed to raise the needs of others—their patients, teams, and institutions - above their own. Trust in the physician social contract is facilitated by such altruism and the paradigm of the selfless physician hero is frequently celebrated. However, physicians are simply human with personal needs, limitations, vulnerabilities, imperfections, fallibility, and mortality. The concept of physician humanism recognizes the inherent humanity and personhood of physicians, along with the legitimacy of their physical needs (nutrition, sleep, cognitive rest), social needs (personal safety, meaningful connection), and existential needs (value, meaning).

실제로 의사가 자신의 인격과 자비로운 관계를 유지하는 것은 환자와 다른 관계에서 인문학적 치료를 확장하는 데 도움이 됩니다. 자기 돌봄은 인간성을 유지하는 데 중요한 기여를 하지만, 휴머니즘은 의사 개인의 필요를 넘어 가치와 관계, 자기와 '존재'가 세상과 타인과의 관계에서 표현되는 방식을 포함합니다.2 타인의 감정에 공감하는 능력인 공감은 의학 교육 과정에서 감소하는 것으로 나타났지만 예술과 인문학에서의 성찰과 경험을 통해 키울 수 있는 휴머니즘의 핵심 요소입니다. 의사가 자기 연민을 발휘하면서 자신의 감정을 파악하고 효과적으로 다룰 수 있는 능력을 유지하는 것은 휴머니즘적 진료의 기본입니다.

In fact, maintaining compassionate connection with their own personhood supports physicians in extending humanistic care to patients and in other relationships. While self-care is an important contributor to maintaining one’s humanity, humanism extends beyond the individual physician’s personal needs to include values and relationships, the ways in which self and “being” are expressed in the world and in relationship with others.2 Empathy, an ability to connect with the emotions of another, is a key element of humanism which has been shown to decline throughout medical training but can be nurtured through reflection and experiences in the arts and humanities. It is fundamental to humanistic care that physicians maintain the ability to identify and effectively navigate their own emotions, while employing self-compassion.

의사의 인간성에 대한 부정은 의사의 건강에 해로운 결과를 초래하며, 이는 의사의 건강 악화, 중독, 소진, 자살, 직업 이탈에 대한 통계에서 잘 드러납니다.3 의사와 환자의 인간성에 대한 무시는 현재 시스템에서 인문학적 치료가 부족하다는 피드백에 반영되어 있습니다.4 의료 및 의학교육 시스템은 과도한 업무량, 수면 부족, 부족한 자원, 경쟁 환경, 낙인, 정신건강을 위한 자원 부족 등 의사의 휴머니즘을 위협하는 수많은 요인이 존재합니다.5 의료 및 의학교육 시스템이 의사의 휴머니즘을 포용하고 육성하지 못하면 의사는 환자를 위한 양질의 인문학적 진료를 지속하는 데 어려움을 겪게 될 것입니다. 모든 리더와 이해관계자는 의사의 지속적인 자기 희생의 위험에 함께 맞서야 합니다. 막대한 개인적, 사회적 비용 없이 의사 휴머니즘을 계속 무시하는 것은 현실적이지 않습니다.4 
Denial of physician humanity has deleterious consequences for physician wellness that are underscored by statistics on physician ill-health, addiction, burnout, suicide, and exodus from the profession.3 Disregard for physician and patient humanity is reflected in feedback on the lack of humanistic care in the current system.4 Healthcare and medical education systems present countless threats to physician humanism including excessive workloads, sleep deprivation, insufficient resources, competitive environments, stigma and lack of resources for mental health.5 If healthcare and medical education systems fail to embrace and nurture physician humanism, physicians will continue to suffer as will their ability to sustain high-quality, humanistic care for patients. All leaders and stakeholders must together confront the dangers of continuous physician self-sacrifice; it is not realistic to perpetually disregard physician humanism without significant personal and societal cost.4

2015 CanMEDS 역량 프레임워크에서 의사 휴머니즘은 어떻게 표현되어 있나요?
How is Physician Humanism represented in the 2015 CanMEDS competency framework?

CanMEDS 의사 역량 프레임워크는 "역동적이고 점점 더 까다로워지는 의료 환경에서 사회적 요구"를 충족하기 위해 의사에게 요구되는 역량을 설명합니다.6 이 프레임워크는 "사람으로서의 의사" 역할에 의사 인본주의를 통합한 기초적인 온타리오 미래 의사 교육 프로젝트(EFPO)에서 파생되었습니다.7 이 역할의 일부 개념이 캔메즈 전문가, 의사소통자, 협력자, 리더 역할에 분산되어 있지만(표 1B), 의사의 사람됨은 CanMEDS 에 명확하게 포함되지 않았습니다. 의사의 인격은 인정된 현실이며 역량으로 정의할 수 있는 역할이 아니라고 주장할 수도 있지만, 우리는 의사의 인격이 의사 역량 프레임워크에서 중요한 위치를 차지할 가치가 있다고 생각합니다. 이에 비해 네덜란드에서 채택한 CanMEDS 프레임워크는 꽃의 줄기로서 ' Reflector' 역할을 우선시하며, 의사 수련과 역량에 대한 담론에서 의사 인본주의의 중요성을 강조합니다.8
The CanMEDS Physician Competency Framework describes the required competencies of physicians to meet “societal needs in a dynamic and increasingly demanding health care environment.”6 It derives from the foundational Educating Future Physicians of Ontario Project (EFPO), which notably incorporated physician humanism in a “Doctor as Person” role.7 While some concepts from this role were distributed among the CanMEDS Professional, Communicator, Collaborator, and Leader roles (Table 1B), physician personhood was not explicitly included in CanMEDS. While it could be argued that physician personhood is an acknowledged reality and not a role to be defined by competencies, we believe it warrants a prominent place in our physician competency framework. Comparatively, the Netherlands’ adaption of the CanMEDS framework prioritizes a “Reflector” role as the stem of the flower, asserting the importance of this aspect of physician humanism in discourse on physician training and competence.8

 

 

CanMEDS 2015의 의사 휴머니즘 관련 역량은 타인(환자, 동료, 기관)의 외형적 요구에만 초점을 맞추고 의사 자신의 인간성에 대한 충분한 관심을 소홀히 한다는 점에서 제한적이며 잠재적으로 해로울 수 있습니다. CanMEDS는 의사가 자신의 인간성을 효과적으로 탐구하고 육성하는 데 필요한 역량과 평생의 개인적, 직업적 정체성 형성을 통해 자아에 대한 인문학적 접근을 적절히 인정하지 않습니다.
The competencies related to physician humanism in CanMEDS 2015 are limited and potentially harmful in that they focus solely on the outward facing needs of others (patients, colleagues, and institutions), while neglecting sufficient attention to physicians’ own humanity. CanMEDS does not adequately acknowledge the competencies required by physicians to effectively explore and nurture their own humanity and a humanistic approach to self through lifelong personal and professional identity formation.

의사 인본주의가 2025 CanMEDS 역량 프레임워크 내에서 어떻게 더 잘 표현될 수 있을까요?
How can Physician Humanism be better represented within the 2025 CanMEDS competency framework?

2025 CanMEDS 프레임워크 업데이트는 의사 휴머니즘을 보다 명시적으로 인정하고 가치를 부여하여 의사, 환자, 의료 시스템의 복지에 필수적인 관련 역량을 강조할 수 있습니다.7-9 그러나 이는 쉽지 않을 것입니다. 복잡한 구조인 의사 휴머니즘은 쉽게 정의하거나 가르치거나 평가할 수 없습니다. 더 큰 문제는 이러한 구조와 관련된 역량을 정의하고 평가하는 것이 애초에 의사의 휴머니즘을 높이는 근본적인 가치와 목표를 훼손할 수 있다는 점입니다.9 예를 들어, 평가를 위해 휴머니즘적 행동을 정의하면 다음의 위험이 따릅니다.

  • 환원주의,
  • 성과 전시,
  • 교과과정에 투자할 가치가 있는 휴머니즘의 관찰 불가능한 측면(예: 감정, 가치) 무시,
  • 자기 관리 및 건강에 대한 개인적인 경험의 부적절한 평가,
  • 평생, 반성, 회복을 목적으로 하는 개발 측면에 대한 총평적 평가 스트레스

The 2025 CanMEDS framework update could more explicitly acknowledge and value physician humanism, underscoring its associated competencies as essential to the wellbeing of physicians, patients, and the healthcare system.7-9 However, this will be challenging. As a complex construct, physician humanism is not easily defined, taught, or assessed. Worse, defining and assessing competencies related to this construct may undermine the fundamental value and goals of elevating physician humanism in the first place.9 For example, defining humanistic behaviours for evaluation risks

  • reductionism,
  • performative display,
  • neglect of non-observable aspects of humanism worthy of curricular investment (i.e., emotions, values),
  • inappropriate evaluation of personal experiences of self-care and wellness, and
  • summative appraisal stress for an aspect of development intended to be lifelong, reflective, and restorative.

이러한 위험을 인식하고 14개의 새로운 핵심 역량을 제안하고 기존의 핵심 역량 1개를 조정하여 의사 휴머니즘이 나아갈 방향을 제시했습니다. 이러한 제안은 전문가 의견과 저자 그룹 간의 합의를 바탕으로 개발되었으며, 의사 휴머니즘을 CanMEDS 2025에 보다 직접적으로 통합할 수 있는 수단으로 제안되었습니다(표 1C). 이러한 역량의 평가로 인한 부정적인 영향을 방지하기 위해, 저자들은 이러한 역량을 평가의 목적으로 규정하지 않고 평생의 개인적 및 전문적 개발, 성찰적이고 협력적인 관행, 성장 마인드 증진을 위한 탐구와 육성을 목적으로 규정할 것을 제안합니다. 목표 설정, 등급별 경험, 건설적인 피드백, '실패'를 통한 학습을 통해 시간이 지남에 따라 자신과 자신의 능력을 개발할 수 있다는 믿음을 수용하는 성장 마인드를 보건 전문직 교육에 도입하면 학습 환경 개선, 학습자 복지, 어려움 속에서도 회복력 향상 등의 이점을 얻을 수 있습니다.10 
Acknowledging these risks, we have outlined a way forward for Physician Humanism by suggesting 14 new key competencies and adapting one existing key competency. These suggestions were developed based on expert opinion and consensus among our author group as a proposed means by which physician humanism could be more directly integrated into CanMEDS 2025 (Table 1C). To prevent adverse impacts from assessment of these competencies, the authors propose they are not delineated for assessment, but instead for exploration and nurturing of lifelong personal and professional development, reflective and collaborative practices, and promotion of a growth mindset. The benefits of employing a growth mindset in health professions education, whereby belief in the ability to develop oneself and one’s abilities over time through goal setting, graded experiences, constructive feedback, and learning through “failure” is embraced, include enhanced learning environments, learner well-being, and resiliency, even in the face of difficulty.10

이러한 역량을 정교화하는 '인간으로서의 의사' 역할에 우선순위를 두도록 CanMEDS를 확장하면 의사 휴머니즘의 중요성을 더욱 뚜렷하게 부각할 수 있습니다. 이 역할에는 전문성과 타인에 대한 개인적 책임을 넘어 의료 시스템의 어려운 제약 속에서 의사들이 자신에 대한 인문학적 접근 방식을 개발할 수 있도록 지원하는 역량이 포함될 것입니다. 의사 휴머니즘은 모든 역량 역할에 걸쳐 의사의 효과성에 영향을 미치므로, CanMEDS의 업데이트된 그림 표현은 '인격체로서의 의사' 역할을 꽃의 줄기, 잎, 뿌리와 같이 지지하고 양육하는 부분으로 상징할 수 있습니다.7,8
Preferably, expanding CanMEDS to prioritize a ‘Doctor as Person’ role in which these competencies are elaborated would more distinctly highlight the importance of physician humanism. This role would include competencies that extend beyond professionalism and personal responsibility to others, to support doctors in developing a humanistic approach towards themselves as well, within the challenging constraints of the healthcare system. As physician humanism influences the effectiveness of physicians across all competency roles, an updated pictorial representation of CanMEDS could symbolize the ‘Doctor as Person’ role as the supporting and nurturing parts of the flower: the stem, leaves and roots.7,8


Can Med Educ J. 2023 Mar 21;14(1):13-17. doi: 10.36834/cmej.75536. eCollection 2023 Mar.

Physician Humanism in CanMEDS 2025

Affiliations

1Department of Family Medicine, McMaster University, Ontario, Canada.

2Department of Medicine, University of Alberta, Alberta, Canada.

3Royal College of Physicians and Surgeons of Canada, Ontario, Canada.

4Department of Pediatrics, McGill University, Quebec, Canada.

5University of Saskatchewan, Saskatchewan, Canada.

6Department of Emergency Medicine, Queen's University, Ontario, Canada.

PMID: 36998505

PMCID: PMC10042789

DOI: 10.36834/cmej.75536

CanMEDS 2025에서 반인종주의 (Can Med Educ J. 2023)
Anti-racism in CanMEDS 2025

 

소개
Introduction

캐나다에서 흑인, 원주민, 유색인종은 인종차별과 억압의 빈번한 표적이 되고 있습니다. 캐나다의 흑인은 수 세기에 걸친 원주민 커뮤니티와 최근 몇 세대에 걸쳐 이민 온 커뮤니티(예: 카리브해 또는 아프리카계) 등 다양한 출신 배경을 가지고 있습니다. 이들은 모두 캐나다의 교육, 의료 및 사법 기관에 뿌리를 둔 신념, 태도, 편견, 고정관념, 차별을 강화하는 정책과 관행으로 인해 억압을 경험했다는 공통점을 가지고 있습니다. 원주민들은 다음에 의한 억압을 받아왔습니다.

  • [권력의 불균형, 제도적 장벽, 불평등한 결과]를 확립, 유지, 영속화시킨 [정착민 식민지 정책과 관행의 유산]에서 비롯된 [인종 기반 차별, 부정적인 고정관념, 불의에 의해 억압]
Black, Indigenous, and people of color are frequent targets of racism and oppression in Canada. Black people in Canada have diverse origins–centuries old communities and communities of recent immigration in the last few generations (such as with those of Caribbean or African descent). All share a common experience of oppression by policies and practices rooted in Canadian educational, healthcare, and justice organizations that reinforce beliefs, attitudes, prejudice, stereotyping, and/or discrimination. Indigenous peoples have been oppressed by
  • the race-based discrimination, negative stereotyping, and injustice stemming from the legacy of settler colonial policies and practices that have established, maintained, and perpetuated power imbalances, systemic barriers, and inequitable outcomes.

유색인종은 다양한 역사와 배경을 가진 무수히 많은 집단에 속하며, 비슷한 방식으로 억압받고 있습니다. 또한 인종, 종교, 능력, 성적 지향, 사회경제적 지위, 성별, 성 정체성 등다양한 정체성이 교차하면서 배제와 차별이 더욱 심해집니다.1
People of color span a myriad of groups with diverse histories and backgrounds that are oppressed in similar ways. Additionally, intersecting identities of race, religion, ability, sexual orientation, socioeconomic status, gender and gender identity exacerbate exclusion and discrimination.1

우리 의료기관에서 널리 인정하지는 않지만, 인종차별적 관행과 이데올로기는 서양의학의 근간을 이루고 있으며2,3 계속해서 해를 끼치고 있습니다. 우리 의료계는 아직 이 사실을 의미 있게 직시하지 못하고 있습니다. 의료 서비스 제공자들이 근무하는 환경을 조사해 보면, 직장 내 학대 및 트라우마 경험에서 인종이 다른 의료 서비스 제공자들이 과대 대표되고 있습니다. 놀랍게도 흑인 의사의 70%4와 흑인 간호사의 88%5가 캐나다에서 직장에서 인종 차별을 경험했다고 보고했습니다. 인종이 다른 환자들을 대상으로 데이터를 강력하게 수집한다면 어떤 결과가 나올지 상상만 해도 알 수 있습니다. 의식적이든 무의식적이든 인종적 편견(예: 흑인 환자는 통증을 느끼지 못한다는 믿을 수 없는 믿음6)은 현재 의사 인력의 일부에 남아 있으며 임상 학습 및 업무 환경에 스며들어 안전성을 떨어뜨리고 건강 불평등을 악화시키며 환자의 신뢰를 약화시킬 수 있습니다.7
Although not widely acknowledged by our medical institutions, racist practices and ideologies are foundational underpinnings of Western Medicine2,3 and continue to cause harm. Our profession has yet to confront this truth in a meaningful way. When we examine the environments in which care providers practice, racialized care providers are overrepresented in the experience of workplace-related mistreatment and trauma. An alarming seventy percent (70%) of Black physicians4 and eighty-eight percent (88%) of Black nurses5 report experiences of racism in workplaces in Canada. One can only imagine what the data would show if robustly collected on racialized patients. Racial biases–conscious or unconscious (i.e. the incredulous belief that Black patients do not feel pain6)–persist in aspects of the current day physician workforce and can permeate the clinical learning and work environment, making it less safe, exacerbating health inequities, and eroding patient trust.7

개인적 차원에서 인종적 편견은 인종이 다른 환자 및 동료와의 상호작용에 영향을 미칠 수 있습니다. 인종적 편견이 학습 및 임상 환경에 스며들면 환자 치료가 손상되고 결과가 나빠집니다.8-14 연구자들은 백인에 대한 암묵적 선호를 가진 백인 수련의가 심근경색 혈전용해술을 할 때 흑인 환자가 아닌 백인 환자를 치료할 가능성이 더 높다는 사실을 발견했습니다.8 이 연구는 의사의 무의식적 편견이 임상 의사 결정에서 인종/민족 격차에 기여하고 의료 시술의 사용을 예측할 수 있음을 보여주었습니다. 백인 의사는 전문 분야에 관계없이 백인을 선호하는 암묵적인 선호를 가지고 있으며 때때로 백인이 아닌 환자를 열등하게 대하는 것으로 나타난 한 체계적 문헌고찰에서 보듯이 이러한 편견을 확인하지 않고 방치하면 편견이 지속될 수 있습니다.9
At an individual level, racial bias can affect interactions with racialized patients and colleagues. When racial bias permeates the learning and clinical environment patient care is compromised and outcomes are worse.814 Researchers found that white trainees with an implicit preference for white individuals were more likely to treat white patients, and not Black patients, with thrombolysis for myocardial infarction.8 This study showed that physicians’ unconscious biases may contribute to racial/ ethnic disparities in clinical decision-making and may predict the use of medical procedures. Left unchecked, such bias persists as shown in a systematic review which found white physicians, regardless of specialty, have an implicit preference of favoring white people and will sometimes treat non-white patients in an inferior fashion.9

제도적 차원에서 인종 편견은 적극적인 인종화로 이어질 수 있으며 흑인, 원주민 및 인종 차별 의사와 환자에게 안전하지 않은 업무 및 규제 환경을 조성할 수 있습니다.15 의료 환경에서 원주민 환자는 건강 결과에 영향을 미치는 심각하고 만연한 반원주민 인종 차별을 계속 경험하고 있습니다.10 최근 증거에 따르면 원주민 신분은 백인 환자에 비해 응급실에서 더 높은 중증도 분류 점수를 받을 확률이 낮고,7 흑인, 원주민, 인종차별 환자는 백인 환자보다 치료를 받기 위해 더 오래 기다리며,11 조직적 인종차별은 사망의 원인으로 입증되었습니다.12 요컨대, 인종차별적 치료는 사망을 초래합니다. 
At the systemic level, racial bias can lead to active racialization and create unsafe work and regulatory environments for Black, Indigenous and racialized physicians and patients.15 Indigenous patients in healthcare settings continue to experience significant and pervasive anti-Indigenous racism which has an impact on their health outcomes.10 Recent evidence demonstrates: that First Nations status is associated with lower odds of receiving higher acuity triage scores in the emergency department in comparison to white patients;7 Black, Indigenous and racialized patients wait longer than white patients to receive care;11 and systemic racism is a proven cause of death.12 In short, racist care kills.

캐나다에서 인종차별이 공중보건 비상사태로 선포되었음에도 불구하고16 의료기관과 규제 기관의 전면적이고 조직적이며 체계적인 조치가 부족합니다.17 우리 의료계는 반인종주의자가 되어야 합니다. 2015년에 개정된 CanMEDS 의사 역량 프레임워크18는 의료계가 반인종주의 의료에 필요한 기술을 반영하고, 알리고, 지원할 수 있는 기회를 제공합니다. 이 원고는 의학교육에서 새롭게 떠오르는 반인종주의 개념을 요약하고 이를 2025년 CanMEDS 의사 역량 프레임워크에 통합하기 위한 권장 사항을 제공하는 것을 목표로 합니다.
Despite racism being declared a public health emergency in Canada16 full-scale, coordinated, and systemic action from our medical institutions and regulatory bodies is lacking.17 Our profession needs to become anti-racist. The revision of the 2015 CanMEDS Physician Competency Framework18 provides an opportunity for the medical community to reflect, inform the skills, and support the conditions needed for anti-racist medicine. This manuscript aims to summarize emerging anti-racism concepts in medical education and to provide recommendations for incorporating them into the 2025 CanMEDS Physician Competency Framework.

반인종주의란 무엇이며 의사의 역량에 왜 중요한가?
What is anti-racism and why is it important to physician competency?

반인종주의는 조직적 인종차별을 포함한 인종차별의 존재를 인식하는 데 뿌리를 둔 프로세스, 체계적인 분석 방법, 사전 예방적 행동 방침입니다. 인종적으로 불평등한 결과와 집단 간 권력 불균형식별, 제거, 예방, 완화하고 불평등을 지속시키는 구조를 바꾸기 위해 적극적으로 노력합니다.13 
Anti-racism is a process, a systematic method of analysis, and a proactive course of action rooted in the recognition of the existence of racism, including systemic racism. It actively seeks to identify, remove, prevent, and mitigate racially inequitable outcomes and power imbalances between groups and change the structures that sustain inequities.13

반인종주의는 여러 가지 이유로 의사의 역량에 중요합니다. 

  • 첫째, 위에서 설명한 바와 같이 인종주의는 학습자, 동료, 환자, 지역사회의 건강과 웰빙에 직접적인 영향을 미칩니다.
  • 둘째, 불균등한 건강 결과, 건강 불평등, 심지어 사망에까지 기여하는 것은 인종이 아니라 인종차별입니다.19-21
  • 셋째, 캐나다 인구의 인종 및 민족적 다양성이 증가함에 따라 의료 전문가들이 변화하는 인구 통계의 요구를 충족하기 위해 새로운 성향과 기술을 개발하고 성장할 수 있는 기회와 책임이 생겼습니다.
  • 마지막으로, 의료계 내 인종차별에 맞서기 위해 캐나다의 모든 의사는 의학교육, 수련, 장학, 진료 등 어떤 분야에 종사하든 불편한 진실을 인식하고 받아들여야 합니다. 서양 의학의 토대는 인종차별적 관행과 이념에 기반을 두고 있으며2,3 이는 오늘날에도 지속되어 환자 치료와 인종차별적 동료와의 상호작용에 영향을 미치고 있다는 사실을 인정하고 받아들여야 합니다.

Anti-racism is important to physician competency for several reasons.

  • First, as outlined above, racism directly impacts the health and wellness of our learners, colleagues, patients, and communities.
  • Second, it is racism—and not race—that contributes to disparate health outcomes, health inequities, and even death.1921 
  • Third, the increasing racial and ethnic diversity of the Canadian population provides an opportunity and a responsibility for medical professionals to grow and develop new dispositions and skills to meet the needs of our shifting demographics.
  • Finally, in order to combat racism in medicine, all physicians in Canada –whether they are engaged in medical education, training, scholarship, or practice—must recognize and accept an uncomfortable truth; the foundation of Western medicine is grounded in racist practices and ideologies2,3 which persist today and affect patient care and interactions with racialized colleagues.


우리는 캐나다의 모든 의사가 인종주의가 환자 치료 결과와 흑인, 원주민 또는 유색인종으로 식별되는 의사 동료에게 미치는 영향을 해결하기 위해 반인종주의, 반억압적 진료에 대한 지속적인 역량을 입증해야 한다고 주장합니다.12 이러한 상황의 심각성은 새로운 진료 기준의 설정을 포함하되 이에 국한되지 않는 의료 전문가의 긴급한 조치와 시정 대응을 요구합니다.10 반인종주의 조치를 취하려면 명확한 언어 사용과 인종주의와 억압의 기원에 대한 확실한 개념적 이해, 이를 없애기 위한 대응이 필요하며 표 1은 자세한 용어집을 제시합니다. 표 1에서 의도적으로 누락된 용어는 의료 기관과 조직이 이 분야에서 달성하고자 하는 목표를 정의하기 위해 자주 사용하는 용어인 형평성, 다양성, 포용성입니다. 이러한 용어는 편안하고 열망을 불러일으키지만, 행동 지향적이지 않고 인종적 형평성이 필요한 집단이 앞으로 나아가는 것을 방해하는 문제인 인종차별과 억압을 명시적으로 언급하지 않기 때문에 인종차별의 맥락에서는 도움이 되지 않는다고 판단했습니다.
We argue that all physicians in Canada must demonstrate ongoing competence in anti-racist, anti-oppressive praxis, to address the impact of racism both on patient outcomes and on their physician colleagues who identify as Black, Indigenous, or people of color.12 The gravity of this situation demands urgent action and corrective responses from medical professionals, including but not limited to the setting of new practice standards.10 Anti-racist action requires the use of explicit language and solid conceptual understanding of the origins of racism and oppression, and counteractions to eliminate these; Table 1 provides a detailed glossary. Intentionally absent from Table 1 are terms that are frequently embraced by health care institutions and organizations to define what they aspire to achieve in this area: equity, diversity, and inclusivity. While these terms are comfortable and aspirational, we find them unhelpful within the context of racism because they are not action-oriented, and they do not explicitly name the issues that prevent racialized equity-deserving groups from moving forward: racism and oppression.

Term 용어 정의
Definition
Anti-Black racism 반흑인 인종 차별
교육, 의료, 사법 등 캐나다 기관에 뿌리를 둔 정책 및 관행으로 아프리카, 흑인, 카리브해 출신에 대한 신념, 태도, 편견, 고정관념 및/또는 차별을 반영하고 강화하는 것을 말합니다.24,25

The policies and practices rooted in Canadian institutions, such as education, healthcare, and justice, which mirror and reinforce beliefs, attitudes, prejudice, stereotyping, and/or discrimination towards people of African, Black, and Caribbean descent.
24,25
Anti-Indigenous racism 반원주민 인종 차별
이는 인디언법 및 기숙사 학교 제도와 같은 차별적인 연방 정책에서 분명하게 드러납니다. 또한 각 주의 형사 사법 및 아동 복지 시스템에서 원주민이 과대 대표되고 교육, 복지, 건강에서 불평등한 결과가 나타나는 것에서도 나타납니다. 원주민에 대한 적대감과 폭력 행위의 증가에서 반원주민 인종차별에 대한 개인의 생생한 경험을 확인할 수 있습니다.24

This is evident in discriminatory federal policies such as the Indian Act and the residential school system. It is also manifested in the overrepresentation of Indigenous peoples in provincial criminal justice and child welfare systems, as well as inequitable outcomes in education, well-being, and health. Individual lived-experiences of anti-Indigenous racism can be seen in the rise in acts of hostility and violence directed at Indigenous people.
24
Anti-oppression 반억압
반억압 접근법은 일부 집단에 혜택을 주고 다른 집단을 배제하는 사회 내 권력 불균형을 인식합니다. 이 접근법은 억압, 인종차별 및 기타 형태의 차별이 없는 환경을 조성하기 위한 전략을 개발하고자 합니다. 인종, 혈통, 출신지, 피부색, 출신 민족, 시민권, 신념, 성별(임신 포함), 성적 지향, 성 정체성, 성 표현, 나이, 범죄 기록, 결혼 여부, 가족 상태, 장애 등 눈에 보이거나 보이지 않는 정체성과 다양성의 교차점을 인정하고 다양한 정체성 간의 형평성을 증진하는 것을 목표로 합니다.24

An anti-oppression approach recognizes the power imbalance within society that attributes benefits to some groups and excludes others. This approach seeks to develop strategies to create an environment free from oppression, racism, and other forms of discrimination. It acknowledges the intersections of identity and diversity, both visible and invisible, including race, ancestry, place of origin, colour, ethnic origin, citizenship, creed, sex (including pregnancy), sexual orientation, gender identity, gender expression, age, record of offences, marital status, family status, and disability, and aims to promote equity between the various identities.
24
Anti-racism 반인종주의
반인종주의는 조직적 인종차별을 포함한 인종차별의 존재를 인식하는 데 뿌리를 둔 과정, 체계적인 분석 방법, 적극적인 행동 방침입니다. 반인종주의는 인종적으로 불평등한 결과와 집단 간의 권력 불균형을 식별, 제거, 예방, 완화하고 불평등을 지속시키는 구조를 바꾸기 위해 적극적으로 노력합니다.13

Anti-racism is a process, a systematic method of analysis, and a proactive course of action rooted in the recognition of the existence of racism, including systemic racism. Anti-racism actively seeks to identify, remove, prevent, and mitigate racially inequitable outcomes and power imbalances between groups and change the structures that sustain inequities.
13
Colonialism 식민주의
한 민족이나 권력이 다른 민족이나 지역을 지배하는 관행 또는 정책으로, 종종 식민지를 설립하고 일반적으로 경제적 지배를 목적으로 합니다. 식민지화 과정에서 식민지 지배자는 자국의 종교, 언어, 경제 및 기타 문화적 관행을 강요할 수 있습니다.26 식민주의는 크게 네 가지 유형으로 분류됩니다: 1) 정착민 식민주의, 2) 착취 식민주의, 3) 대리 식민주의, 4) 내부 식민주의.

A practice or policy of control by one people or power over other people or areas, often by establishing colonies and generally with the aim of economic dominance. In the process of colonization, colonizers may impose their religion, language, economics, and other cultural practices.
26 It is broadly classified into four types: 1) settler colonialism, 2) exploitation colonialism, 3) surrogate colonialism, and 4) internal colonialism.
Critical consciousness 비판적 의식 
파울로 프레이리에 의해 대중화된 개념으로, 현실에 개입하여 현실을 변화시키는 능력으로 정의됩니다. "의식 고양"이라고도 하는 이 개념에는 이러한 이해를 통해 깨달은 자신의 삶에서 억압적인 요소에 대항하는 행동이 포함됩니다.27 현대의 공식은 비판적 의식을 세 가지 요소로 나눕니다.

1) 비판적 성찰은 억압과 불평등이 존재하는 역사적, 제도적 방식에 대한 인식입니다.
2) 비판적 동기는 인지된 불평등을 해결하기 위해 인지된 능력 또는 도덕적 헌신입니다.
3) 비판적 행동은 인지된 불평등을 변화시키고, 도전하고, 이의를 제기하기 위한 개인적 또는 집단적 행동에 참여하는 것입니다.28

A concept, popularized by Paulo Freire, defined as the ability to intervene in reality to change it. Also known as “consciousness raising”, it includes taking action against the oppressive elements in one's life that are illuminated by that understanding.
27 Contemporary formulations divide critical consciousness into three components. 

1) Critical reflection is an awareness of both the historical and systemic ways oppression and inequity exist.
2) Critical motivation is the perceived capacity or moral commitment to address perceived inequalities.
3) Critical action is participation in individual or collective action to change, challenge, and contest perceived inequity.
28
Cultural safety 문화적 안전
문화적 안전은 치료를 받는 사람이 결정하는 결과입니다. 문화적 안전은 의료 서비스 제공자가 자신의 문화적 배경과 지배 문화가 소수 문화권에 서비스를 제공할 때 권력 관계의 본질을 성찰하여 내재된 위계를 해체하기 위해 노력할 것을 요구합니다. 서비스 제공자는 다른 집단의 신념과 문화적 관행을 조사하고 이해할 필요 없이 연령, 직업 또는 사회 계층, 인종적 배경, 성별, 성, 종교적 신념, 장애 등에서 자신과 다를 수 있는 사람들의 강점을 인정하고 장려해야 합니다. 문화적 안전을 위해서는 다수 문화권의 제공자가 소수 문화권에 대한 고정관념에 도전해야 합니다. 이는 다양성에 대한 긍정적인 인식을 촉진합니다.29

Cultural safety is an outcome determined by the recipient of care. It requires healthcare providers to reflect on their own cultural background and the nature of power relations in the provision of services to a minority culture by a dominant culture, so that the providers can work to dismantle the inherent hierarchy. Providers do not need to research and understand other groups’ beliefs and cultural practices; rather, they acknowledge and promote the strengths of those who may differ from them in age, occupation or social class, ethnic background, sex, sexuality, gender, religious belief, and disability. Cultural safety requires providers from the majority culture to challenge their own stereotyped views of a minority culture. It promotes positive recognition of diversity.
29
Epistemic racism 인식론적 인종주의
한 인종 집단의 지식을 다른 인종 집단보다 우월한 것으로 간주하는 것을 말하며, 여기에는 어떤 지식이 가치 있는 것으로 간주되는지뿐만 아니라 지식으로 간주되는지에 대한 판단도 포함됩니다.30

Refers to the positioning of the knowledge of one racial group as superior to another, it includes a judgment of not only which knowledge is considered valuable but is considered to be knowledge.
30
Intersectionality 교차성
사람들의 삶이 다중적이고 중첩된 정체성과 사회적 위치에 의해 형성되는 방식을 인정하는 프레임워크로, 이러한 요소들이 결합하여 추가적인 장벽이나 기회를 창출하는 등 해당 개인이나 집단에게 독특하고 뚜렷한 경험을 만들어낼 수 있습니다. 인종화의 맥락에서 이는 인종차별 또는 특권에 대한 사람들의 경험이 인종화된 한 집단 내를 포함하여 개인 또는 집단의 민족성, 원주민 정체성, 식민주의 경험, 종교, 성별, 시민권, 사회경제적 지위 또는 성적 지향과 같은 개인 또는 집단의 중첩된(또는 "교차하는") 사회적 정체성에 따라 다를 수 있고 달라질 수 있음을 인식하는 것을 의미합니다.31

A framework that acknowledges the ways in which people’s lives are shaped by their multiple and overlapping identities and social locations, which, together, can produce a unique and distinct experience for that individual or group, such as by creating additional barriers or opportunities. In the context of racialization, this means recognizing the ways in which people’s experiences of racism or privilege, including within any one racialized group, may differ and vary depending on the individual’s or group’s overlapping (or “intersecting”) social identities, such as ethnicity, Indigenous identification, experiences with colonialism, religion, gender, citizenship, socio-economic status or sexual orientation.
31
Microaggression 미시적 공격
소외된 집단(예: 성소수자, LGBTQ2S+, 장애)의 구성원에 대한 적대적, 경멸적, 부정적 비하 및 모욕미묘하게 그리고 종종 의도적 또는 비의도적으로 표현하는 일상적인 언어적, 행동적 또는 환경적 모욕, 발언 또는 행동으로, 일상적이고 일상적인 인종차별이라고도 합니다.24

Brief and common daily verbal, behavioral, or environmental indignities, comment or action that subtly and often intentionally or unintentionally expresses a hostile, derogatory, or negative slights and insults toward a member of a marginalized group (such as, but not limited to BIPOC, LGBTQ2S+, disability), also referred to as casual and everyday racism.
24
Positionality 지위
사회적 지위와 권력의 차이가 사회에서 정체성과 접근성을 형성하는 방식을 말합니다.32

Refers to the how differences in social position and power shape identities and access in society.
32
Power 권력
편안하고 안전하며 생산적이고 수익성 있는 삶을 사는 데 필요한 것을 얻을 수 있는 기회를 향상시키는 정보, 지식, 인맥, 경험 및 전문성, 자원, 의사 결정과 같은 특권에 대한 접근성.23

Access to privileges such as information, knowledge, connections, experience and expertise, resources, and decision-making that enhance a person’s chances of getting what they need to live a comfortable, safe, productive, and profitable life.
23
Privilege 특권
사회에서 지배적인 집단의 구성원에게 존재하는 불로소득의 권력, 혜택, 이점, 접근권 및 기회. 다른 집단에 비해 한 집단이 갖는 상대적 특권을 의미하기도 합니다. 페기 매킨토시의 글과 도구 '백인 특권'을 참조하세요: 보이지 않는 보따리 풀기"33를 참조하세요.

Unearned power, benefits, advantages, access, and opportunities that exist for members of the dominant group(s) in society. Can also refer to the relative privilege of one group compared to another. See Peggy McIntosh’s article and tool “White Privilege: Unpacking the Invisible Knapsack.”
33
Race 인종
주로 피부색이나 '내재적' 또는 '변하지 않는' 것으로 인식되는 기타 명백한 차이와 같은 신체적 특성(표현형)을 기준으로 사람들을 그룹으로 분류하는 데 사용되는 용어입니다.34 예를 들어, 사회 집단의 문화나 종교는 때때로 변하지 않고 내재적인 것으로 취급될 수 있습니다. 인종 범주는 과학이나 생물학에 근거한 것이 아니라 사회가 만들어낸 차이(즉, "사회적으로 구성된")에 근거하며, 사람들의 삶에 중대한 영향을 미칩니다. 인종 범주는 시간과 장소에 따라 달라질 수 있으며 민족, 문화 또는 종교적 집단과 겹칠 수 있습니다.

A term used to classify people into groups based principally on physical traits (phenotype), such as skin colour or other apparent differences perceived as “inherent” or “unchanging.”
34 For example, a social group’s culture or religion may sometimes be treated as unchanging and inherent. Racial categories are not based on science or biology but on differences that society has created (i.e., “socially constructed”), with significant consequences for people’s lives. Racial categories may vary over time and place, and can overlap with ethnic, cultural, or religious groupings.
Race Correction 인종 보정
'인종 조정', '인종 수정' 또는 '인종 규범화'라고도 합니다. 이러한 해로운 관행은 의료 서비스 제공자가 다른 기준, 즉 "환자의 인종 또는 민족에 따라 결과를 조정하거나 "수정"하는 진단 알고리즘 및 진료 지침을 사용함으로써 발생합니다."10 이러한 관행의 사용은 환자의 치료 및 지원 접근성에 영향을 미칩니다.

Also referred to as “race adjustment”, “race modification” or “race norming”. This harmful practice results from health care providers using different criteria, “diagnostic algorithms and practice guidelines that adjust or “correct” their outputs on the basis of a patient’s race or ethnicity.”
10 The use of this practice has implications for patient’s access to treatment and support.
Self-determination 자기결정권
각 개인이 스스로 선택하고 자신의 삶을 관리할 수 있는 능력을 말합니다. 이는 심리적 건강과 웰빙에 중요한 역할을 하며, 사람들이 자신의 선택과 삶을 통제할 수 있다고 느끼게 하여 참여 동기와 개인의 결과에도 긍정적인 영향을 미칩니다.35
Refers to each person's ability to make choices and manage their own life. It plays an important role in psychological health and well-being, and allows people to feel that they have control over their choices and lives, which positively impacts motivation to engage and even outcomes for individuals.
35
Settler Colonialism 정착민 식민주의
이러한 형태의 식민주의는 종종 종교적, 정치적 또는 경제적 이유로 식민지에 정착민을 대규모로 이주시키는 것을 포함합니다. 이러한 형태의 식민주의는 캐나다, 미국, 호주와 같은 국가의 원주민들이 정착민들이 기존 인구를 대체하면서 경험했습니다. 이러한 형태의 식민주의는 토지와 번영을 차지하기 위한 목적으로 많은 수의 정착민들이 식민지로 이주하는 것을 포함합니다.
This form of colonialism involves the large-scale immigration of settlers to colonies, often motivated by religious, political, or economic reasons. This form of colonialism has been experienced by Indigenous peoples in countries such as Canada, the United States and Australia as settlers supplant prior existing populations. This form of colonialism involves a large number of settlers emigrating to colonies for the expressed purpose of claiming land and prosperity.
Structural competency 구조적 역량
증상, 태도 또는 질병처럼 임상적으로 정의되는 다양한 문제(예: 우울증, 고혈압, 비만, 흡연, 약물 '비순응', 트라우마, 정신병)가 의료 및 음식 배달 시스템, 구역법, 도시 및 농촌 인프라, 의료화, 심지어 질병과 건강의 정의와 같은 문제에 대한 상위 결정의 하류 영향을 식별하는 훈련된 능력입니다.36
The trained ability to discern how a host of issues defined clinically as symptoms, attitudes, or diseases (e.g. depression, hypertension, obesity, smoking, medication “non-compliance”, trauma, psychosis) also represent the downstream implications of upstream decisions about such matters as health care and food delivery systems, zoning laws, urban and rural infrastructures, medicalization, or even the very definitions of illness and health.
36
Structural racism 구조적 인종차별
공공 정책, 제도적 관행, 문화적 표현 및 기타 규범이 인종화된 집단의 불평등을 강화하고 영속화하는 방식으로 작동하는 시스템. 이는 백인의 특권과 '피부색'과 관련된 불이익이 시간이 지남에 따라 지속되고 적응할 수 있도록 허용한 우리 역사와 문화의 차원을 식별합니다. 구조적 인종차별은 소수의 사람이나 기관이 선택한 것이 아니라 우리 모두가 존재하는 사회, 경제, 정치 시스템의 특징입니다.37
A system in which public policies, institutional practices, cultural representations, and other norms work in ways to reinforce and perpetuate racialized group inequity. It identifies dimensions of our history and culture that have allowed white privilege and disadvantages associated with “colour” to endure and adapt over time. Structural racism is not something that a few people or institutions choose to practice; instead, it has been a feature of the social, economic, and political systems in which we all exist.
37
Systemic racism 시스템적 인종차별
일부 인종 집단을 배제, 이동 또는 소외시키거나 가치 있는 혜택과 기회에 접근하는 데 불공정한 장벽을 만드는 조직 문화, 정책, 지침, 관행 또는 절차. 이는 종종 조직 문화, 정책, 지침, 관행, 절차가 중립적으로 보이지만 일부 그룹에 특권을 부여하고 다른 그룹에 불이익을 주는 영향을 미치는 제도적 편견의 결과입니다.19
Organizational culture, policies, directives, practices, or procedures that exclude, displace, or marginalize some racialized groups or create unfair barriers for them to access valuable benefits and opportunities. This is often the result of institutional biases in organizational culture, policies, directives, practices, and procedures that may appear neutral but have the effect of privileging some groups and disadvantaging others.
19
Trauma-informed care 트라우마 정보에 기반한 치료
치유를 지향하는 효과적인 의료 서비스를 제공하기 위해서는 환자의 과거와 현재 생활 상황에 대한 완전한 그림이 이해되어야 한다는 것을 인정하는 치료 접근법입니다. 이러한 접근 방식은 의료기관의 임상 및 조직 수준에서 모두 구현되어야 효과적입니다.22
An approach to care that acknowledges that a complete picture of a patient’s life situation — past and present — must be understood in order to provide effective health care services with a healing orientation. This approach must be implemented at both the clinical and organizational levels of institutions to be effective.
22
Upstander intervention 방관자Upstander 개입
방관자Upstander는 고위험 또는 유해한 결과를 초래할 수 있는 행동을 목격하고 상황을 개선하기 위해 개입을 선택하는 사람입니다. 업스탠더가 개입하는 상황에는 일상적인 피해 행위(예: 길거리 괴롭힘, 따돌림, 차별적 발언, 성차별적 농담) 또는 고위험 상황(예: 신체적 폭력, 성폭행, 관계 폭력으로 이어질 수 있는 상황)이 포함됩니다.38
An upstander is someone who witnesses a behavior that could lead to something high risk or harmful and makes the choice to intervene to make things better. Situations an upstander intervenes in include daily acts of harm (i.e., street harassment, bullying, discriminatory comments, sexist jokes), or high risk situations (i.e., situations that may lead to physical violence, sexual assault, relationship violence).
38

이 원고에서는 반인종주의적 진료와 관련된 주요 개념을 간략히 검토하고 이를 2025 CanMEDS 의사 역량 프레임워크에 통합하기 위한 권장 사항을 제공합니다. 중요한 것은 이전에 발간된 왕립대학 문서에서 원주민을 특별히 돌보는 데 있어 의료 제공자, 학습자 및 교육자를 위한 주요 접근법, 아이디어 및 배경 지식을 설명하고 있으며22, 이 시리즈의 관련 신흥 개념 원고에서는 다른 억압받는 인구와 관련된 형평성, 다양성, 포용성 및 사회 정의를 다루고 있다는 점입니다.23 우리는 또한 프레임워크 자체가 인종차별을 용인할 수 있음을 시사하는 설득력 있는 논평에 독자들의 주의를 환기시킵니다.15 우리는 이 일련의 문서가 2025년 버전의 CanMEDS로 효과적으로 번역되어 미래에 의사들이 환자와 지역사회를 돌보는 데 있어 행동 지향적이고, 반인종차별적이며, 반억압적이고, 구조적 역량을 갖추고, 문화적으로 안전한 진료에 유능하게 참여할 수 있기를 희망합니다. 

Within this manuscript we briefly review key concepts related to anti-racist praxis and provide recommendations for incorporating them in the 2025 CanMEDS Physician Competency Framework. Importantly, a previously published Royal College document describes the key approaches, ideas and background knowledge for health care providers, learners and educators in caring specifically for Indigenous Peoples22 while a related emerging concepts manuscript in this series addresses equity, diversity, inclusion, and social justice as they relate to other oppressed populations.23 We also draw our readers attention to a compelling commentary which suggest that the Framework, itself, may be permissive of racism.15 We are hopeful that this suite of documents will be effectively translated into the 2025 version of CanMEDS to ensure that — in the future – physicians are able to competently engage in action-oriented, anti-racist and anti-oppressive, structurally competent, and culturally safe praxis in the care of our patients and our communities.

2015 CanMEDS 역량 프레임워크에서 반인종주의는 어떻게 표현되어 있나요?
How is anti-racism represented in the 2015 CanMEDS competency framework?

2015 CanMEDS 의사 역량 프레임워크(표 2A)에는 반인종주의에 대한 명시적인 언급은 없지만, 관련 개념을 다루는 커뮤니케이터, 학자, 건강 옹호자 및 전문가 역량이 있습니다. 가장 근접한 역량인 커뮤니케이터 4.1은 환자와의 문화적으로 안전한 의사소통을 요구합니다(표 2B). CanMEDS에 반인종주의 역량이 없으면 인종이 다른 의료진의 부정적인 경험과 인종이 다른 환자의 예후 악화에 기여할 수 있습니다. 학자들은 인종차별과 관련하여 전문직건강 옹호자 역할 사이의 긴장을 강조하며, 전문직이 인종 형평성과 사회 정의를 옹호하는 인종 차별적 의사에 대해 무기로 사용된다고 지적했습니다.39 따라서 프레임워크 자체가 인종차별을 영속화할 수 있다는 설득력 있는 주장이 제기될 수 있습니다.15
While there are no explicit references to anti-racism within the 2015 CanMEDS Physician Competency Framework (Table 2A), there are Communicator, Scholar, Health Advocate, and Professional competencies that touch on related concepts. The closest enabling competency, Communicator 4.1, calls for culturally safe communication with patients (Table 2B). The absence of anti-racist competencies in CanMEDS may contribute to the negative experiences of racialized health care providers and the worsened outcomes of racialized patients. Scholars have highlighted a tension between the Professional and Health Advocate roles when it comes to racism, with professionalism being weaponized against racialized physicians who are advocating for racial equity and social justice.39 Thus a compelling argument can be made that the Framework may itself perpetuate racism.15

 

 

 

2025 CanMEDS 역량 프레임워크 내에서 인종 차별을 어떻게 표현할 수 있을까요?
How can anti-racism be represented within the 2025 CanMEDS competency framework?

인종주의는 의료 시스템과 더 넓은 사회의 문화와 구조 속에 존재하기 때문에 의료 및 의학교육 시스템 내에서 인종주의를 해결하는 것은 많은 과제를 안고 있습니다. 인종차별을 해결하기 위한 효과적인 접근법으로 조직 전체에 반인종주의적 접근 방식을 도입할 것을 제안합니다. 반인종주의적 접근법은 개인이 비판의식, 문화적, 심리적 안전, 트라우마 정보에 기반한 치료, 업스탠더 개입의 개념을 적절히 활용하여 학습 환경과 임상 치료 환경 모두에서 안전과 소속감의 문화를 조성하는 데 지식과 기술을 갖추고 있을 때 가능합니다.
Since racism exists within the culture and fabric of the health care system and broader society, addressing racism within the healthcare and medical education systems presents many challenges. We propose the incorporation of an anti-racist approach throughout the fabric of our organizations as an effective approach to addressing racism. An anti-racist approach is enabled when individuals are knowledgeable and skilled in appropriately utilizing concepts of critical consciousness, cultural and psychological safety, trauma-informed care, and upstander intervention to create a culture of safety and belonging both within the learning environment and the clinical care environment.

우리는 반인종주의가 의사의 모든 역량을 가로지르고 있으며, 기존의 CanMEDS 역량 프레임워크는 반인종주의, 반억압적 진료와 관련된 의사에 대한 기대치를 명시적으로 명시하여 발전할 필요가 있다고 믿습니다. 우리는 19개의 수정 사항과 24개의 추가 사항을 요구하며, 이를 통해 반인종주의가 CanMEDS의 각 역할에서 중요한 구성 요소가 될 수 있도록 할 것입니다. 이러한 역량과 표 2에 설명된 다른 역량들은 21세기 의사가 건강 형평성을 증진하고 보다 다양하고 포용적인 환경을 적극적으로 지원하는 데 필요한 역량입니다. 
We believe anti-racism is a cross-cutting physician competency, and that the existing CanMEDS competency framework requires evolution with explicit statements for expectations of physicians related to anti-racist, anti-oppressive praxis. We call for nineteen modifications and twenty-four additions that would make anti-racism a prominent component within each of the CanMEDS roles. These competencies, and others described in Table 2, are required for the 21st century physician to promote health equity and actively support more diverse and inclusive environments.

이러한 새로운 역량을 캔메드에 통합하는 것은 작은 진전이지만, 반인종주의 및 반억압적 실천의 근본적인 필요성을 보여주는 더 효과적인 방법은 인종차별적 요소로 오염된 토양에서 자랐다는 인식을 높이기 위해 CanMEDS의 '꽃'을 완전히 새롭게 상상하는 것입니다.2,3 반인종주의 토양에 단단히 뿌리를 내린 꽃의 이미지는 우리 공통의 인류를 대표하는 줄기를 지지하면서 인정과 가능성의 메시지를 전달하고 반인종주의로 가득한 꽃잎을 돋아나게 할 수 있을 것입니다. 우리 시대의 시급한 요구와 높아지는 기대에 부응하기 위해서는 새로운 의사의 정체성15이 필요하며, 반인종주의는 의사의 기본 역량이 되어야 합니다.
While incorporating these new competencies within CanMEDS would be a small step forward, a more effective way to demonstrate the fundamental need for anti-racist and anti-oppressive praxis would be a complete reimagining of the CanMEDS ‘flower’ to raise awareness that it was grown in soil corrupted by racist elements.2,3 The image of a flower with roots firmly established in anti-racist soil would deliver a message of acknowledgement and possibility while supporting a stem representative of our common humanity40 and sprouting petals infused with anti-racism. A new physician identity15 is required to meet the urgent needs and growing expectations of our times, and anti-racism must be a foundational physician competency.

 


Can Med Educ J. 2023 Mar 21;14(1):33-40. doi: 10.36834/cmej.75844. eCollection 2023 Mar.

Anti-racism in CanMEDS 2025

Affiliations

1Cumming School of Medicine, University of Calgary, Alberta, Canada.

2Collège des Médecins du Québec, Quebec, Canada.

3University of Toronto, Ontario, Canada.

4University of British Columbia, British Columbia, Canada.

5University of Saskatchewan, Saskatchewan, Canada.

6University of Ottawa, Ontario, Canada.

PMID: 36998489

PMCID: PMC10042777

DOI: 10.36834/cmej.75844

CanMEDS 2025에서 형평성, 다양성, 포용성, 사회적 정의 (Can Med Educ J. 2023)
Equity, diversity, inclusion, and social justice in CanMEDS 2025

 

소개
Introduction

의료 분야의 사회 정의각 개인이 속한 다양한 공동체에 관계없이 건강, 복지, 기회 및 특권에 대한 권리가 다른 사람과 동일하다는 것을 지지합니다.1 사회 구조의 모든 측면에서 배제는 다수 인구의 권력을 확보하는 수단이 되어 왔으며, 의료 인력 구성2과 다양한 인구 및 지역사회의 환자에게 제공되는 치료의 가용성과 질 모두에 명백한 영향을 미쳤습니다.3,4 의사 인력은 성별 및 젠더, 원주민으로서의 정체성 또는 비주류 인종, 민족, 문화 또는 종교 집단에 속하는 경우, 다양한 능력, 가족의 사회경제적 지위 등 여러 인구집단을 과소 대표하고 있는 것으로 알려져 있습니다.5,6 
Social justice in medicine upholds that each individual, regardless of the diverse communities to which they may belong, has the same right to health, wellbeing, opportunities and privilege as another.1 Exclusions in all aspects of societal structure have been a means to secure power for the majority population in Canada, and have unequivocally impacted medicine both through the composition of the medical workforce2 and the availability and quality of care provided to patients of diverse populations and communities.3,4 The physician workforce is recognized to have underrepresentation of several populations including those defined

  • by their sex and gender;
  • their identity as an Indigenous person or belonging to non-dominant racialized, ethnic, cultural or religious groups;
  • by various abilities; and
  • by familial socioeconomic status.5,6 

이들 및 기타 억압받는 인구집단의 구성원들은 의학 교육 과정에서 불이익, 차별, 배제를 경험하고 있습니다7. 의학교육 시스템은 능력주의를 강화하고, 위계를 지지하며, 인구 집단 간의 차이를 영속화하고, 다른 지식을 배제하는 생물의학 모델을 장려하기 위해 구축되었습니다.8 캐나다 의료 시스템은 보편적 접근의 원칙에 기초하고 있지만, 의료 시스템에서 인종주의, 성차별, 젠더차별, 능력주의, 연령주의 등 여러 형태의 차별과 그 교차점9에서 겪는 피해에 대해 이용 가능한 치료를 받는 것의 이점을 반드시 평가해야 하는 환자들의 실제 경험과는 다른 경우가 많습니다. 이러한 환자들은 다른 환자들과 동일한 수준의 치료를 받지 못하며10 그 결과 더 나쁜 건강 결과를 겪습니다.11 일부 지역에서는 결과의 격차가 더 커지고 있습니다.11
The experiences of members of these and other oppressed populations throughout their medical training are of disadvantage, discrimination, and exclusion 7. Medical education systems have been constructed to reinforce meritocracy, support hierarchy, perpetuate differences between population groups, and promote the biomedical model to the exclusion of other knowledges.8 The Canadian health care system is founded on the principle of universal access, yet this is often not the lived experience of patients who necessarily assess the benefits of seeking available care against the harms of racism, sexism, genderism, ableism, and ageism among other forms of discrimination, and all their intersections,9 they suffer in the healthcare system. These patients are not afforded the same quality of care as others10 and suffer worse health outcomes as a result.11 In some areas, the gap in outcomes is widening.11

의료계의 변화에 대한 오랜 요구가 행동의 우선순위로 떠오른 지금,12 기존의 CanMEDS 의사 역량 프레임워크는 형평성, 다양성, 포용성, 사회 정의의 개념과 관련된 의사에 대한 기대치를 명시적으로 제시하는 방향으로 발전할 필요가 있습니다.13 우리는 여기서 이러한 개념을 검토하고, 의학교육(모든 저자), 연구(CB, ND, KOT), 임상 진료(모든 저자)에서 형평성 및 반인종주의 이니셔티브의 리더로서의 경험과 캐나다의 원주민(CB), 아프리카계 캐나다인(KOT), 소수 인종(JMM, SR, BMW), 의학계 여성(CB, ND), 장애인(ND)으로서 살아온 경험을 바탕으로 CanMEDS 의사 역량 프레임워크에 통합하기 위한 권고사항을 제시합니다. 독자들은 반인종주의에 대해 구체적으로 다룬 원고도 참고하시기 바랍니다.14,15 
With the long call for changes in medicine now prioritized for action,12 the existing CanMEDS physician competency framework requires evolution with explicit statements for expectations of physicians related to concepts of equity, diversity, inclusion and social justice.13 We review these concepts here and provide recommendations for incorporating these in the CanMEDS physician competency framework drawn from our experiences as leaders in equity and anti-racism initiatives in medical education (all authors), research (CB,ND,KOT), and clinical practice (all authors) as well as our lived experiences as persons of Indigenous (CB), African-Canadian (KOT), and racial minorities in Canada (JMM, SR, BMW), women in medicine (CB, ND), and disabled (ND). We direct readers to also refer to the manuscripts that speaks specifically on the topic of anti-racism.14,15

형평성, 다양성, 포용성, 사회 정의란 무엇이며 의사의 역량에 중요한 이유는 무엇인가요?
What are equity, diversity, inclusion, and social justice and why are they important to physician competency?

형평성, 다양성, 포용성, 사회 정의는 차별을 해결한 결과로 간주되어야 합니다.

  • (모든 사람에게 동일한 분배를 제공하는 평등equality과는 구별되는) 형평성equity은 개인이 한 집단 또는 다수의 집단에 속해 있다는 이유만으로 기회가 줄어들 뿐만 아니라 다른 구조적, 제도적 장벽에 직면하기 때문에 공정성fairness을 달성하는 데 필요한 행동에 차이가 있을 수밖에 없음을 인정합니다.16
  • 다양성은 집단 내 모든 인구의 대표성을 높이고, 변화와 가능성이 무한하며, 다양한 지식과 경험에서 비롯된 관점과 혁신의 기회를 제공하는 것을 의미합니다.17
  • 포용성은 집단 내 다양한 인구 구성원의 존재뿐만 아니라 그 경험의 성격을 말하며, 그들이 그 집단에서 지지받고 가치 있는 구성원이라고 평가하는 것을 말합니다.18
  • 사회 정의는 각 개인이 다른 사람과 동일한 건강, 복지, 기회 및 특권을 가져야 한다는 권리 기반 접근 방식에서 개념화됩니다.1

Equity, diversity, inclusion, and social justice are to be considered outcomes of addressing discrimination.

  • Distinct from equality (which is the same distribution for everyone), equity necessarily accepts there will be variations in actions required to attain fairness, as individuals encounter different structural and systemic barriers as well as reduced opportunities simply by virtue of their membership in a or many population(s).16 
  • Diversity refers to enhanced representation of persons from all populations in a collective, limitless in variations and possibilities, and yielding the opportunity for perspectives and innovations arising from multiple knowledges and experiences.17 
  • Inclusion speaks not only to the presence of but also the nature of the experience of members of diverse populations in a collective, and that they assess they are a supported and valued member of that collective.18 
  • Social justice is conceptualized from a rights-based approach, valuing that each individual should have the same health, wellbeing, opportunities and privilege as another.1 

이러한 용어 및 관련 용어에 대한 간략한 정의는 표 1에서 확인할 수 있습니다.
Concise definitions of these and related terms are provided in Table 1.

용어 정의
Term
Definition
Ableism 에이블리즘
모든 형태의 장애(예: 신체적, 지적)를 가진 사람에 대한 차별 및/또는 편견. 장애를 가진 사람을 비장애인에 비해 열등한 존재로 규정하는 것입니다.

Discrimination and/or prejudice against persons with any form of disability (e.g., physical, intellectual). Persons are defined by their disabilities, and are characterized as inferior to the non-disabled.
Ageism 연령 차별
일반적으로 나이가 많은 사회 구성원에 대한 고정관념, 차별 및/또는 편견, 나이를 기준으로 한 사람에 대한 고정관념, 차별 및/또는 편견입니다.

Stereotyping, discrimination and/or prejudice against persons on the basis of their age, typically against older members of society.
Accessibility 접근성
참여의 장벽을 제거하여 모든 사람이 참여할 수 있는 환경을 조성하기 위한 편의 제공.

Accommodations are made to remove barriers to participation, resulting in the ability for all to participate.
Anti-oppression 반억압
억압 체제에 도전하는 전략, 이론, 행동 및 관행

The strategies, theories, actions, and practices that challenge systems of oppression
Cultural Safety 문화적 안전
서비스 제공자와 조직이 불평등을 야기하는 편견과 장벽을 해소하는 성찰적이고 인식 있는 관행과 정책의 결과물입니다. 문화적 안전의 존재 여부를 결정하는 것은 서비스 제공자가 아니라 서비스를 받는 사람입니다.

An outcome of reflective and aware practice and policy, where providers and organizations address bias and barriers contributing to inequities. It is the recipient of care, and not the provider, who determines if cultural safety is present or not.
Decolonization 탈식민지화
사회 구조, 서비스 및 투자에 대한 자결권을 위한 고유한 권리와 권한을 회복합니다.

Regaining inherent rights and power for self-determination of societal structure, services and investments.
Diversity 다양성
집단 내에 가능한 한 다양한 속성이 존재합니다.

The greatest possible representation of attributes are present within the collective.
Equity 형평성
불균형이 해소된 공정성과 정의의 달성.

Attainment of fairness and justice, with imbalances addressed.
Homophobia 동성애 혐오
레즈비언, 게이, 퀴어 또는 양성애자라는 이유로 사람을 향한 고정관념, 차별 및/또는 편견.

Stereotyping, discrimination and/or prejudice against persons on the basis of being lesbian, gay, queer or bisexual.
Inclusion 포용성
모든 사람이 동등하게 환영받고, 지원받고, 가치를 인정받고, 존중받을 수 있도록 보장하는 행동과 접근 방식.

Acts and approaches that ensure that all persons are equally welcomed, supported, valued and respected.
Intersectionality 교차성
불평등에 직면한 여러 인구집단의 일원이 됨으로써 발생하는 상호 작용 또는 시너지 효과로 인해 독특한 경험을 할 수 있습니다.

Interactions or synergies which arise from being a member of multiple populations facing inequities, and which results in a unique experience.
Meritocracy 능력주의
능력과 성과에 따라 기회와 지위를 부여하지만 모든 사회 구성원의 기회를 제한하는 구조적 불평등을 인정하지 않는 시스템.

System of awarding opportunities and status on the basis of ability and performance, but which neglects acknowledging structural inequities that limit opportunities for all members of society.
Racism 인종차별
인종을 이유로 한 사람에 대한 고정관념, 차별 및/또는 편견.

Stereotyping, discrimination and/or prejudice against persons on the basis of their race.
Sexism 성차별
성별에 따른 고정관념, 차별 및/또는 편견 일반적으로 여성으로 식별되는 개인에 대한 편견.

Stereotyping, discrimination and/or prejudice against persons on the basis of their sex typically against those individuals that identify as women.
Genderism 젠더차별
젠더에 근거한 사람에 대한 고정관념, 차별 및/또는 편견.

Stereotyping, discrimination and/or prejudice against persons on the basis of their gender.
Social justice 사회 정의
각 개인이 경제적, 정치적, 사회적 권리와 기회를 모두 누릴 권리가 있다는 것을 장려하는 접근 방식.

Approach promoting that each person has the right to the full spectrum of economic, political, and social rights and opportunities.

의사 인력 및 다양한 배경을 가진 환자에게 제공되는 의료 서비스 모두에서 권력과 특권의 불균형에 대한 인식이 증가하고 있지만, 의사는 기존의 불평등을 식별하거나 사회 정의 목표를 향한 진전을 방해하는 복잡한 요인을 이해하도록 타고나지 않았다.19 의사는 의사 동료와 환자 모두를 위한 동맹allies으로서 정보에 기반한 근거 기반 조치를 취하는 데 필요한 기술을 개발해야 합니다.2 
While there is increasing recognition of the power and privilege imbalances in both the physician workforce and the care provided to patients from diverse backgrounds, physicians are not intrinsically able to identify existing inequities, nor understand the complex contributors impeding progress towards social justice goals.19 Physicians need to develop the necessary skills to take informed and evidence-based action as allies for both physician colleagues and patients.2

형평성, 다양성, 포용성, 사회 정의는 2015 CanMEDS 역량 프레임워크에서 어떻게 표현됩니까?
How are equity, diversity, inclusion, and social justice represented in the 2015 CanMEDS competency framework?

CanMEDS 2015 의사 역량 프레임워크에는 의사 인력 및 환자 치료와 관련된 형평성, 다양성, 포용성 및 사회 정의와 관련된 추론이 있지만(표 2A 및 및2B),2B) 명시적으로 다루고 있는 경우는 드뭅니다. 특히, 이 역량들은 의료계, 더 넓게는 사회에 만연한 차별적 신념 및/또는 관행의 존재를 인정하지 않으며, 의사가 사회 정의를 증진하는 활동을 지원하거나 주도하는 데 유능해야 한다는 명시적인 기대치를 제시하지 않습니다. '리더' 및 '건강 옹호자' 측면의 기존 역량 중 적절하고 권한을 부여하는 의사 교육 및 근무 환경 유지와 관련된 내용은 없습니다. 이는 우리 직업에서 배제를 더욱 영속화하고, 다양하고 포용적인 의료 인력을 확보하기 위해 채용과 유지를 지원하는 데 형평성 이니셔티브가 필요하다는 점을 강조하지 않으며, 의학교육에 내재된 위계질서를 해체하는 데 기여하지도 않습니다.8 
There are inferences relevant to equity, diversity, inclusion and social justice related to both the physician workforce and patient care in the CanMEDS 2015 physician competency framework (Table 2A and and2B),2B), but they are rarely explicitly addressed. Specifically, the competencies do not acknowledge the existence of discriminatory beliefs and/or practices rampant in medicine specifically and more broadly in society, and do not set out explicit expectations for physicians to be competent in supporting or leading activities that promote social justice. None of the existing competencies in ‘leader’ and ‘health advocate’ aspects related to upholding appropriate and empowering physician training and work environments. This further perpetuates exclusions in our profession and does not reinforce that equity initiatives are necessary to support recruitment and retention to achieve a diverse and inclusive medical workforce, nor contribute to dismantling the hierarchy inherent in medical education.8

 

이번 호 캐나다 의학교육 저널에 게재된 논문에서 Adam 외21는 CanMEDS 역할 설명 및 역량에서 '환자'라는 용어가 어떻게 언급되는지를 강조합니다. 역할 설명에서 환자를 치료의 파트너로 포지셔닝하는 경우가 많지만, 건강 옹호자 역량을 제외한 다른 역할의 핵심 및 활성화 역량에서는 환자를 정보와 치료의 수동적인 수신자로 포지셔닝합니다. 이러한 관점에서 볼 때, 이러한 역량은 지속적인 차별을 허용하는 방식으로 기존의 권력 불균형을 악화시키는 것으로 볼 수 있습니다.
In a paper published in this issue of the Canadian Medical Education Journal, Adam et al21 highlights how the term ‘patient’ is referenced in the CanMEDS role descriptions and competencies. While the role descriptions frequently position patients as partners in care, with the exception of the Health Advocate competencies, the key and enabling competencies of the other roles position patients as passive recipients of information and care. Viewed from this perspective, the competencies could be seen to exacerbate existing power-imbalances in a way that allows ongoing discrimination.

2025 CanMEDS 역량 프레임워크 내에서 형평성, 다양성, 포용성, 사회 정의를 어떻게 더 잘 표현할 수 있을까요?
How can equity, diversity, inclusion, and social justice be better represented within the 2025 CanMEDS competency framework?

2025 CanMEDS 역량 프레임워크에 형평성, 다양성, 포용성 및 사회 정의와 관련된 역량을 명시적으로 포함할 것을 권장합니다. 이러한 역량을 포함하면 이 분야의 전문성 개발을 지원하여 의사가 불평등을 적극적으로 식별하고 대처하는 동시에 인력의 다양성과 포용성을 증진하는 데 도움이 될 것입니다. 또한 의사들이 형평성에 취약한 집단에 속하는 환자들과 효과적으로 소통할 수 있는 기술을 개발하여 의료 서비스 제공을 개선하고 체계적이고 구조적인 변화를 위한 옹호 활동을 지원할 수 있도록 하고자 합니다. 이러한 각 측면과 관련된 기존 CanMEDS 역량은 표 2A 및 및2B2B에 요약되어 있으며 표 2C 및 및2D2D에 수정이 제안되어 있습니다.
We recommend competencies explicitly related to equity, diversity, inclusion and social justice for the 2025 CanMEDS competency framework. Including such competencies will support professional development in this area that will help physicians to actively identify and combat inequities while promoting diversity and inclusion in the workforce. We also want to ensure physicians develop skills that allow them to effectively engage with patients belonging to equity-deserving groups to both improve the delivery of healthcare and assist in advocacy efforts for systemic and structural change. Existing CanMEDS competencies related to each of these aspects are outlined in Table 2A and and2B2B with suggested modifications in Table 2C and and2D2D.

의사 인력과 관련하여, 우리는 의사가 변화의 주체로 활동하여 인력의 다양성과 포용성을 지원한다는 기대를 강화하는 협력자 및 리더 역할에 새로운 역량을 추가할 것을 권장합니다. 건강 옹호자 역할에 새로 추가된 내용은 다양한 의사의 중요한 역할과 소외된 지역사회에 대한 의료 서비스 제공에 대한 증거를 반영합니다.22 학자 및 전문가 역할에 대한 수정 사항은 수련 중인 의사의 안전한 학습 환경의사 동료의 안전한 근무 환경을 보장하는 데 필요한 활동을 강화하는 한편, 의사가 학습자나 동료를 평가할 때 개인의 암묵적 편견을 인식하고 완화할 것을 촉구합니다. 
Related to the physician workforce, we recommend new competencies under the Collaborator and Leader roles that reinforce the expectation that physicians act as agents of change to support diversity and inclusion in the workforce. A new addition to the Health Advocate role reflects evidence on the critical role of diverse physicians and care delivery to underserved communities.22 Modifications to the Scholar and Professional roles reinforce necessary activities to ensure safe learning environments for physicians in training and safe working environments for physician colleagues, while also calling on physicians to recognize and mitigate their personal implicit biases when assessing learners or peers.

또한 사회 정의를 위해 노력하는 의사의 책임을 강화하고 의료 서비스에서 공평하고 문화적으로 안전한23 환자 경험의 결과를 지원하는 전략을 제정하는 새로운 역량에 대한 권장 사항을 제시합니다. 특히, 우리는 심리사회적 평가와 건강의 사회적 결정요인 및 구조적 요인에 대한 불평등 해결을 포함하도록 치료 상호작용에서 의사의 책임을 확대하는 수정을 제안했습니다. 이를 위해서는 차별을 중단하고, 의사와 환자 간 상호작용에서 권력 불균형을 최소화하며, 지역사회 참여와 파트너십을 구축하고, 각 환자의 현실에 맞게 치료를 맥락화하기 위한 구체적인 기술 개발이 필요합니다. 마지막으로, 아담 외21가 설명한 유형의 의사-환자 상호작용의 틀을 해결하고 환자를 의료 서비스의 능동적 주체 및 파트너로 인식하는 언어를 통합하는 방향으로 전환하는 것이 중요합니다.
We also make recommendations for new competencies that reinforce the responsibility of physicians to strive for social justice and enact strategies that support outcomes of equitable and culturally safe23 patient experiences in healthcare. Specifically, we have suggested modifications that broaden the responsibility of physicians in care interactions to include psychosocial assessment and addressing inequities in the social determinants of health and structural factors. This will require specific skill development in interrupting discrimination, minimizing power imbalances in the physician-patient interaction, community engagement and partnership, and contextualizing care to the realities of each patient. Lastly, it will be important to address the framing of physician-patient interactions of the type described by Adam et al21 and shift towards incorporating languages that recognizes patients as active agents and partners in their healthcare.

형평성, 다양성, 포용성, 사회 정의를 증진하기 위해 기존 CanMEDS 프레임워크에 제안된 추가 및 수정 사항은 의학교육에 필요한 중요한 개선 사항을 다루는 것을 목표로 합니다. 이러한 변화는 의료 인력의 대표성을 강화하는 동시에 의료진에게 지원적이고 안전한 근무 환경을 제공하는 동시에 환자 치료에 도움이 되는 향상된 기술을 명시적으로 요구할 것입니다.
The proposed additions and revisions to the existing CanMEDS framework to promote equity, diversity, inclusion, and social justice aim to address critical improvements needed in medical education. These changes will strengthen representation in the medical workforce while providing a supportive and safe work environment for practitioners, while also explicitly requiring enhanced skills to benefit patient care.


Can Med Educ J. 2023 Mar 21;14(1):27-32. doi: 10.36834/cmej.75845. eCollection 2023 Mar.

Equity, diversity, inclusion, and social justice in CanMEDS 2025

Affiliations

1University of Calgary, Alberta, Canada.

2University of Ottawa, Ontario, Canada.

3McGill University, Quebec, Canada.

4University of Toronto, Ontario, Canada.

5University of Saskatchewan, Saskatchewan, Canada.

6Collège des médecins du Québec, Quebec, Canada.

PMID: 36998498

PMCID: PMC10042795

DOI: 10.36834/cmej.75845

CanMEDS 의사 역량 프레임워크에서 새로 등장하는 개념들  (Can Med Educ J. 2023)
Emerging concepts in the CanMEDS physician competency framework (Can Med Educ J. 2023)

 

소개
Introduction

CanMEDS 역량 프레임워크는 1996년에 발표되어 2005년과 2015년에 업데이트되었습니다.3 캐나다4,5 및 국제적으로 의학교육에 큰 영향을 미쳤으며,6-8 커리큘럼 및 프로그램 설계를 변화시켜 과거에는 의학교육에서 충분히 다루지 않았던 역량에 초점을 맞추도록 했습니다. 캐나다 왕립 의사 및 외과의 대학에서 내부적으로 추적한 결과, 현재 전 세계 50개 이상의 관할 지역에서 최소 12개 전문 분야에서 CanMEDS를 사용하고 있으며 수백만 명의 수련생과 환자에게 영향을 미치고 있는 것으로 추산됩니다.
The CanMEDS competency framework was published in 19961 with updates in 20052 and 2015.3 It has had a major impact on medical education both in Canada4,5 and internationally,68 transforming curricular and program design to increase the focus on competencies that were historically not addressed adequately within medical education. Internal tracking by the Royal College of Physicians and Surgeons of Canada estimates that CanMEDS is now used in over 50 jurisdictions around the world by at least 12 professions, impacting millions of trainees and patients.

의학교육에서 의사의 역량 프레임워크가 수행하는 중심적인 역할을 고려할 때, 2025년에 계획된 개정판은 새로운 역량을 추가하고 오래된 역량을 제거함으로써 진화하는 사회적 요구에 부응해야 합니다. 이는 특히 코로나19 팬데믹으로 인해 의료 및 의학교육이 계속 차질을 빚고 있고,9-15 건강 및 행동 모니터링에 기술이 점점 더 많이 사용되고 있으며,16,17 식민주의,18 제도적 차별,19 성차별,20 인종차별,21 기후변화22가 건강에 미치는 영향이 점점 더 많이 인식되고 있는 현재 환경과 관련이 있습니다. 
Given the central role that the CanMEDS physician competency framework plays within medical education, the planned 2025 revision must respond to evolving societal needs through the addition of new competencies and the removal of outdated competencies. This is particularly relevant in the current environment as healthcare and medical education continue to be disrupted by the COVID-19 pandemic,915 technology is increasingly used to monitor health and behavior,16,17 and the impacts of colonialism,18 systemic discrimination,19 sexism,20 racism,21 and climate change22 on health are increasingly acknowledged.

2015년 개정된 CanMEDS 역량 프레임워크3는 저자 팀원(Van Melle)이 수행한 문헌 검색 및 주제별 분석을 통해 정보를 얻었습니다.23 이 분석은 7가지 새로운 개념(직업적 자아 정체성, 역량의 한 형태로서의 감정, 시스템 기반 실무/실무 기반 학습 및 개선, 인수인계, 글로벌 보건, 소셜 미디어, 의료 분야의 재정적 인센티브)을 확인하고 설명했습니다. 문헌에 초점을 맞추다 보면 발표되지 않은 개념을 놓칠 수 있지만,24 우리는 2025년 CanMEDS 역량 프레임워크의 개정을 위한 광범위한 환경 조사의 일환으로 이 작업을 복제하고 확장하고자 했습니다.  
The 2015 revision of the CanMEDS competency framework3 was informed by a literature scan and thematic analysis performed by a member of our authorship team (Van Melle).23 Their analysis identified and described seven emerging concepts (professional self-identity, emotion as a form of competence, systems-based practice/practice-based learning and improvement, handover, global health, social media, and financial incentives in health care). While focusing on the literature may miss concepts that have not been published,24 we sought to replicate and expand upon this work as part of a broader environmental scan that will inform the revision of the 2025 CanMEDS competency framework.

2015년 방법론을 기반으로,23 우리는 분석에 광범위한 이해관계자 그룹을 포함하여 방법론을 자세히 설명하고, 의학교육 커뮤니티의 검토와 논평을 위해 결과를 공개적으로 게시함으로써 검색 및 검토 프로세스의 엄격성, 포용성, 투명성을 높이는 것을 목표로 합니다. 
Using the 2015 methodology as a base,23 we aim to increase the rigor, inclusiveness, and transparency of the search and review process by outlining our methodology in detail, including a broad group of stakeholders in the analysis, and openly publishing our results for review and commentary from the medical education community.

 

방법론
Methodology

문헌 검색, 제목 및 초록 검토, 일반 주제별 분석25을 통해 문헌을 종합하여 CanMEDS 역할과 관련된 새로운 개념을 파악했습니다. CanMEDS 내에서 더 잘 표현되어야 할 개념을 식별하는 광범위한 작업 목표를 고려할 때, 일반적인 문헌 검토 전략으로는 우리의 목표를 달성할 수 없다는 사실을 발견했습니다. 그 대신 2015년 CanMEDS 개정 이전에 Van Melle 23에서 사용했던 실용적인 접근 방식을 기반으로 포함할 문헌을 결정하고 분석에 정보를 제공했습니다. 
We synthesized the literature using a literature scan, title and abstract review, and generic thematic analysis25 to identify emerging concepts related to the CanMEDS roles. Given the broad-based goal of our work to identify concepts that needed to be better represented within CanMEDS, we did not find that any common literature review strategies would meet our goals. Rather, we built upon the pragmatic approach previously used by Van Melle 23 prior to the 2015 CanMEDS revision to determine the literature to be included and inform its analysis.

2015년 신개념 검토는 한 명의 저자가 수행했지만,23 이 검토를 위해 2025년 CanMEDS 개정을 주도하는 기관/단체인 캐나다 왕립 의사 및 외과의사 대학, 캐나다 퀘벡 의과대학, 캐나다 가정의학과 대학, 캐나다 의학부 협회에서 회원 추천을 받아 실무 그룹을 구성했습니다. 검토의 목적상, 새로운 개념은 의사의 역할 및 역량과 관련된 동료 검토 문헌에서 논의된 아이디어 중 2015 CanMEDS 의사 역량 프레임워크에 없거나 과소 대표되는 개념으로 정의했습니다.3 
While the 2015 emerging concepts review was conducted by a single author,23 for this review we created a working group by soliciting nominations for members from the institutions/organizations steering the 2025 CanMEDS revisions: the Royal College of Physicians and Surgeons of Canada, Collège des Médecins du Québec, College of Family Physicians of Canada, and Association of Faculties of Medicine of Canada. For the purpose of our review, an emerging concept was defined as an idea discussed in the peer reviewed literature related to the role and competencies of physicians that is either absent or underrepresented in the 2015 CanMEDS physician competency framework.3

논문 포함 기준
Article inclusion criteria

Van Melle이 이전 검토에서 사용한 방법과 유사하게 우리는 CanMEDS와 관련된 새로운 개념을 논의할 가능성이 높은 의학 저널을 선택했습니다. 여기에는 저널 임팩트 팩터 기준 가장 영향력이 높은 의학교육 저널 3종(Academic Medicine, Medical Education, Medical Teacher)과 캐나다(Canadian Medical Education Journal) 및 대학원(Journal of Graduate Medical Education) 의학교육과 관련된 콘텐츠를 게재하는 저널이 포함되었습니다. 이러한 접근 방식은 영향력이 가장 높은 의학교육 저널을 중심으로 한 Van Melle의 연구와는 다소 차이가 있지만,23 캐나다 및 대학원 의학교육에 특별히 초점을 맞춘 저널을 포함시키는 것이 중요하다고 생각했습니다. 2018년 10월 1일부터 2021년 10월 1일 사이에 이러한 저널에 게재된 모든 논문이 포함 대상으로 고려되었습니다. 이 3년의 기간은 실현 가능한 범위 내에서 현재와 관련된 개념에 초점을 맞추기 위해 실용적으로 선택되었습니다.  
Paralleling the methods used by Van Melle in the prior review.23 we selected medical journals that would be likely to discuss emerging concepts related to CanMEDS. They included the three highest impact medical education journals by Journal Impact Factor (Academic Medicine, Medical Education, and Medical Teacher) and journals that publish content specifically related to Canadian (Canadian Medical Education Journal) and postgraduate (Journal of Graduate Medical Education) medical education. This approach differed somewhat from Van Melle’s work which was based specifically on the highest impact medical education journals,23 but we felt it was important to include journals focused specifically on Canadian and postgraduate medical education. All articles published within these journals between October 1st, 2018, and October 1st, 2021 were considered for inclusion. This three-year time period was pragmatically selected to focus on currently relevant concepts while still being feasible.

데이터 추출
Data extraction

제목과 초록 검토를 용이하게 하기 위해 선택한 저널에서 검토 기간 내에 출판된 모든 논문에 대해 저널 제목, 논문 제목, 인용 데이터를 포함한 메타데이터를 PubMed에서 추출했습니다. 이러한 데이터를 Zotero26으로 가져와서 각 논문의 초록을 포함한 추가 메타데이터를 추가했습니다. 그런 다음 확장된 메타데이터를 Zotero에서 Google 시트로 내보냈습니다. 토마는 예비 검토를 수행하여 새로운 개념에 초점을 맞추지 않을 것 같은 몇 가지 기사 유형을 제외했습니다. 이러한 기사에는 기관 보고서, 작가 성명서, 정정 및 정오표, 에세이 콘테스트 기사, 편집자에게 보내는 편지, 저널에 초점을 맞춘 사설, 다른 기사 목록을 요약한 기사, 심사자 및/또는 기획위원회 위원에게 감사를 표하는 기사 등이 포함되었습니다. 나머지 기사는 표준화된 형식으로 정리하여 개별 실무 그룹 구성원에게 검토를 맡겼습니다. 
To facilitate the title and abstract review, metadata including the journal title, article title, and citation data were extracted from PubMed for all articles published within the review period in the selected journals. These data were imported into Zotero26 which added additional metadata including each article’s abstract. The expanded metadata were then exported from Zotero into a Google Sheet. Thoma performed a preliminary review and excluded several article types because they were unlikely to focus on emerging concepts. These articles included institutional reports, artist’s statements, corrections and errata, essay contest articles, letters to the editor, editorials focused on the journal, articles summarizing lists of other articles, and articles focused specifically on thanking reviewers and/or planning committee members. The remaining articles were arranged in a standardized format and assigned for review to individual working group members.

논문 검토
Article review

15명의 리뷰어 각각은 2021년 10월 10일부터 2021년 11월 30일까지 검토할 기사를 배정받았습니다. 토마는 팀 또는 개별 가상 회의에서 각 리뷰어에게 방향을 제시했습니다. 각 검토자는 자신의 문서에 대해 다음 질문에 답변했습니다: 
Each of 15 reviewers was assigned articles for review between October 10, 2021 and November 30, 2021. Thoma oriented each reviewer in a team or individual virtual meeting. Each reviewer responded to the following questions for their articles:

  1. 이 기사가 CanMEDS 역할과 관련이 있습니까? (예/아니오/아마도)
  2. 이 문서가 위에서 정의한 새로운 개념을 설명하는가? (예/아니오/아마도)
  3. 예/아니요/아마도 해당한다면, 이 기사가 관련된 주요 역할은 무엇인가요? (의료 전문가, 커뮤니케이터, 협력자, 학자, 건강 옹호자, 리더, 전문가)
  4. 해당 역할과 관련된 추가적인 CanMEDS 역할이 있나요? (의료 전문가, 커뮤니케이터, 협력자, 학자, 건강 옹호자, 리더, 전문가)
  5. 새로운 개념을 간단한 제목으로 설명해 주세요. (자유 텍스트)
  6. 필요한 경우, 새로운 개념에 대한 간략한 설명을 제공하세요. (자유 텍스트)
  7. 이 개념은 2015년 버전의 CanMEDS에 (가) 없거나 (나) 과소 대표되는 개념인가요? (A 및 B 옵션이 있는 드롭다운)
  8. 이 글은 새로운 개념을 잘 요약한 예시적인 글인가요? (예, 아니오, 아마도)

 

  1. Does this article relate to the CanMEDS roles? (Yes/No/Maybe)
  2. Does this article describe an emerging concept as defined above? (Yes/No/Maybe)
  3. If yes/maybe, what is the primary role that it relates to? (Medical Expert, Communicator, Collaborator, Scholar, Health Advocate, Leader, Professional)
  4. Are there any additional CanMEDS roles that it relates to? (Medical Expert, Communicator, Collaborator, Scholar, Health Advocate, Leader, Professional)
  5. Please describe the emerging concept as a brief title. (free text)
  6. If necessary, provide a brief description of the emerging concept. (free text)
  7. Is this a concept (a) absent from or (b) underrepresented in the 2015 iteration of CanMEDS? (dropdown with A and B options)
  8. Is this an exemplar article that summarizes the emerging concept well? (Yes, No, Maybe)


질문 1 또는 2에 대한 답변이 '아니오'인 경우, 질문 3~8에 대한 답변이 없는 것으로 간주하여 추가 검토 대상에서 제외합니다. 검토자가 질문 1 또는 2에 대해 '아마도'로 표시한 기사는 두 번째 검토자(토마)가 검토한 후 응답에 따라 포함 또는 제외했습니다. 
If the response to questions 1 or 2 was ‘no,’ questions 3-8 were not answered and these articles were excluded from further review. Articles that the reviewer tagged as ‘maybe’ for question 1 or 2 were reviewed by a second reviewer (Thoma) and included or excluded based upon their responses.

주제별 분석
Thematic analysis

제목과 초록 검토가 완료된 후, 나머지 논문은 하나의 Google 스프레드시트에 통합되었습니다. 그런 다음 두 명의 저자(토마, 반 멜레)가 질문 5에 대한 검토자의 답변, 검토자가 파악한 새로운 개념, 필요한 경우 논문의 메타데이터를 바탕으로 주제별 분석25,27을 수행했습니다. 이 분석은 주제 분석의 단계를 따랐습니다.28

  • 예비 검토(숙지)를 거친 후 코드북을 개발하고 공동으로 다듬었습니다.
  • 그런 다음 필요한 경우 코드북을 수정하여 모든 기사를 코딩했습니다(코딩).
  • 모든 기사가 코딩된 후, 토마는 코드를 통합한 예비 주제 세트를 개발하고 정의했습니다(주제 검색).
  • 이렇게 만들어진 주제별 프레임워크는 Van Melle이 각 주제를 명확히 하고 정의하기 위해 수정하여 검토, 수정, 승인했습니다(주제 정의 및 이름 지정).
  • 그런 다음, 전체 워킹 그룹에게 피드백 및 수정을 위해 제시되어 구성원 점검(테마 검토)의 첫 번째 단계로 진행되었습니다.
  • 후속 조치로, 구성원 점검(테마 검토)의 두 번째 단계로 Google 설문조사를 통해 워킹그룹 구성원을 대상으로 설문조사를 실시했습니다. 이 설문조사에서는 각 테마에 대한 검토팀의 지지를 요청하고 테마를 어떻게 더 구체화할 수 있는지 물습니다. 이러한 제안은 분석에 반영되었습니다.


After the title and abstract reviews were completed, the remaining articles were amalgamated into a single Google Sheet. Two authors (Thoma and Van Melle) then conducted a thematic analysis25,27 of the reviewers’ responses to question 5, the emerging concept identified by the reviewing author and, when necessary, the article’s metadata. This analysis followed the phases of thematic analysis.28 

  • Following a preliminary review (familiarization), we developed and collaboratively refined a codebook.
  • We then coded all the articles with refinements to the codebook when necessary (coding).
  • Once all articles were coded, Thoma developed and defined a preliminary set of themes incorporating the codes (searching for themes).
  • The resulting thematic framework was reviewed, modified, and endorsed by Van Melle with modifications to clarify and define each theme (defining and naming themes).
  • It was then presented to the full working group for feedback and revision as the first part of the member check (reviewing themes).
  • In follow-up, a survey of the working group members was conducted via a Google Forms survey as the second part of the member check (reviewing themes). The survey requested endorsement for each of the themes from the review team and asked how the themes could be further refined. These suggestions were incorporated into the analysis.

분석 내내 토마스와 반 멜레는 각자의 위치를 고려했습니다. 토마는 응급 및 외상 전문 의사로, 기술 기반 의학교육(시뮬레이션, 온라인 교육 리소스, 학습 분석)에 중점을 둔 의학교육 연구를 수행하고 있습니다. Van Melle은 프로그램 평가 및 변화, 특히 역량 기반 의학교육 맥락에 대한 전문성을 갖춘 교육 과학 박사 학위자입니다. 두 사람 모두 캐나다 왕립 의사 및 외과의 대학과 계약을 맺고 캐나다 전역의 전문 의사 양성을 위한 교육 개발에 관한 자문을 제공하고 있습니다. 우리는 가상 회의와 다양한 의료 전문 분야 및 CanMEDS 이해관계자의 관점을 담은 저자 그룹과의 후속 설문조사를 통해 각자의 위치로 인한 편견을 완화하기 위해 노력했습니다.
Throughout the analysis, Thoma and Van Melle considered their positionality. Thoma is a practicing emergency and trauma physician who conducts medical education research with a focus on technology-enhanced medical education (simulation, online educational resources, and learning analytics). Van Melle is a PhD education scientist with expertise in program evaluation and change, particularly in competency-based medical education contexts. Both are contracted by the Royal College of Physicians and Surgeons of Canada to provide advice regarding educational developments in the training of specialty physicians across Canada. We attempted to mitigate the biases introduced by their positionality through member checks conducted both in a virtual meeting and a follow-up survey with an authorship group that contained perspectives from a range of medical specialties and CanMEDS stakeholders.

삼각측량
Triangulation

출판 지연과 의학 저널의 게이트키핑으로 인해 일부 새로운 개념이 문헌에 나타나지 않았을 수 있다는 점을 인식하고24, 문헌 검색과 병행하여 수행한 온라인 검색 및 주제별 분석의 결과와 결과를 상호 참조했습니다. 이 검색 및 분석의 전체 방법론과 결과는 보고되지 않았고 본 연구의 공식적인 부분은 아니었지만, 저자 중 한 명(Snell)은 새로운 개념을 식별하고 주제별로 분석하기 위해 여러 키워드로 Google 검색 엔진을 사용하여 회색문헌을 검색했습니다. 이 분석 결과를 자체 분석 결과와 상호 참조하여 검색에서 발견되지 않은 회색 문헌에서 떠오르는 개념을 식별함으로써 결과를 삼각측량했습니다.
Acknowledging that publication delays and the gatekeeping of medical journals could have prevented some emerging concepts from appearing in the literature24, we cross-referenced our results with the findings of an online search and thematic analysis that was conducted in parallel to our literature scan. While the full methodology and results of this search and analysis have not been reported and were not a formal part of our study, one of our authors (Snell) searched the grey literature using the Google search engine with multiple key words with the goal of identifying and thematically analyzing emerging concepts. We triangulated our results by cross-referencing the findings of this analysis with our own to identify emerging concepts from the grey literature that were not found in our scan.

전문가 검토
Expert review

앞서 설명한 분석이 완료된 후, 토마는 워킹 그룹 멤버와 캐나다 왕립 의사 및 외과의사 임상의사 교육자들에게 10가지 새로운 개념 각각에 대해 전문가를 추천해 달라고 요청했습니다. 이 원고의 저자 팀원과 설문조사에서 추천된 전문가를 결합하여 집필 그룹을 구성했습니다. 각 집필 그룹은 2022년 3월부터 2022년 7월까지 각 개념을 정의하는 간략한 원고를 작성하고, 2015 CanMEDS 의사 역량 프레임워크에서 어떻게 표현되는지 설명하고,3 CanMEDS 2025의 변경 사항을 제안하는 것을 목표로 2~4회 모임을 가졌습니다. 집필 그룹이 각 개념의 이름에 대해 제안한 변경 사항은 검토 및 승인을 위해 실무 그룹에 이메일로 전송되었습니다.
Following the completion of the described analysis, Thoma asked the working group members and the Royal College of Physicians and Surgeons of Canada Clinician Educators to nominate experts for each of the ten emerging concepts. Writing groups were formed, combining members of the authorship team for this manuscript and the experts that were nominated in the survey. Each writing group met 2-4 times from March 2022 through July 2022 with the goal of writing a brief manuscript defining each concept, outlining how it is represented in the 2015 CanMEDS physician competency framework,3 and proposing changes for CanMEDS 2025. The changes proposed by the writing groups to the name of each concept were emailed to the working group for review and approval.

결과
Results

그림 1에 요약된 바와 같이 관심 기간 동안 포함된 저널에 4973편의 논문이 게재되었습니다. 이 중 505개 논문은 기관 보고서, 저자의 성명서, 정정 또는 오타, 에세이 콘테스트 논문, 편집자에게 보내는 편지, 저널에 초점을 맞춘 사설, 논문 목록이 포함된 논문, 심사자에게 감사하는 논문, 위원회 위원에게 감사하는 논문으로 간주되어 예비 검토에서 제외되었습니다. 4468편의 논문이 제목 및 초록 심사를 받았습니다. 각 심사자는 142~385편의 논문을 검토했습니다(평균 = 298편). 두 차례의 검토를 거친 후 나머지 4468편의 논문 중 1017편(22.8%)이 주제별 분석에 포함되었습니다. 
As outlined in Figure 1, 4973 articles were published in the included journals during the period of interest. 505 of these articles were excluded in the preliminary review because they were deemed to be institutional reports, artist’s statements, corrections or errata, essay contest articles, letters to the editor, editorials focused on the journal, articles containing lists of articles, articles thanking reviewers, or articles thanking committee members. 4468 articles underwent title and abstract review. Each of the reviewers reviewed between 142 and 385 articles (mean = 298 articles). Following both rounds of review, 1017 of the remaining 4468 articles (22.8%) were included in the thematic analysis.

질적 분석에서는 81개의 코드를 9개의 예비 테마로 통합했습니다. 저자 검토자들과의 대규모 그룹 회의에서 나온 피드백을 바탕으로, 하나의 주제(형평성, 다양성, 포용성)를 두 개의 주제(1. 의사 형평성, 다양성, 포용성 및 2. 환자 접근성, 형평성, 포용성, 사회 정의)로 나눴다. 모든 워킹그룹 구성원이 구성원 점검 설문조사를 완료했습니다. 설문조사 피드백을 바탕으로 몇 가지 주제 제목의 문구를 수정했지만, 삭제되거나 통합된 주제는 없었습니다. 
Our qualitative analysis incorporated 81 codes into nine preliminary themes. Based upon feedback at the large group meeting with the author reviewers, one additional theme was created by splitting one of the themes (Equity, Diversity, and Inclusion) into two (1. Physician Equity, Diversity, Inclusion and 2. Patient Access, Equity, Inclusion, and Social Justice). All working group members completed the member check survey. Modifications in wording were made to several of the theme titles based upon survey feedback, but no themes were removed or amalgamated.


10명의 워킹 그룹 멤버와 9명의 임상 교육자가 설문조사를 완료하여 새로운 개념 각각에 대한 집필 그룹에 참여할 전문가를 추천했습니다. 각 개념에 대해 글쓰기 그룹을 구성하여 해당 개념이 CanMEDS에 보다 효과적으로 통합될 수 있는 방법을 설명하는 임무를 맡겼습니다. 환자와 의사의 형평성, 다양성, 포용성에 초점을 맞춘 두 개의 집필 그룹은 이 개념이 단일 원고를 작성하기에는 너무 광범위하다고 판단했습니다. 따라서 초기 분석에서처럼 환자와 의사 중심의 원고로 나누기보다는 반인종주의에 초점을 맞춘 원고와 형평성, 다양성, 포용성, 사회 정의에 초점을 맞춘 두 번째 원고로 나눌 것을 권장했습니다. 이 권고에 따라 집필 그룹은 수정되었고, 이후 집필팀의 승인을 받았습니다. 집필 그룹의 수정에 따른 주제별 분석 결과는 표 1에 제시되어 있습니다. 

Ten working group members and nine clinician educators completed the survey nominating experts to participate in a writing group on each of the emerging concepts. A writing group was formed for each concept and tasked with describing how it could be more effectively integrated into CanMEDS. The two writing groups focused on patient and physician equity, diversity, and inclusion determined that this concept was too broad for a single manuscript. However, rather than splitting it into patient and physician focused manuscripts as was done in the initial analysis, they recommended dividing it into one manuscript focused on anti-racism and a second manuscript focused on equity, diversity, inclusion, and social justice. The writing groups were modified in keeping with this recommendation which was subsequently endorsed by our authorship team. The results of the thematic analysis following modification by the writing groups are presented in Table 1.

 

표 1에는 각 주제에서 논의된 의사의 잠재적 역량에 대한 설명과 1차 구성원 점검 시 각 주제를 새로운 개념으로 지지한 실무 그룹 구성원의 비율도 포함되어 있습니다. 17명의 실무 그룹 구성원 중 최소 14명(82.4%)이 각 주제를 새로운 개념으로 지지했습니다. 주제를 지지하지 않은 이유에 대해서는 해당 주제가 참신하지 않다고 생각하거나, CanMEDS 역량과 어떻게 관련될 수 있는지 알지 못하거나, 주제를 분할하거나 확장하는 것을 선호하는 등 다양한 제안이 있었습니다. 회색 문헌 검색에서는 추가로 떠오르는 개념이 발견되지 않았습니다. 
Table 1 also includes a description of the potential physician competencies discussed within each theme and the proportion of working group members who endorsed each theme as an emerging concept during the first member check. At least fourteen (82.4%) of the 17 working group members endorsed each of the themes as an emerging concept. There were a variety of suggestions provided for why a member did not endorse a theme such as not identifying the theme as novel, not seeing how it could relate to a CanMEDS competency, or preferring themes be split or expanded. The grey literature search did not identify any additional emerging concepts.

10개 작성 그룹 중 6개 그룹은 개념을 더 정확하게 설명하기 위해 개념의 제목을 변경할 것을 제안했습니다. 새로운 개념의 이름과 정의에 대한 변경과 반인종주의 개념의 추가는 집필 그룹의 만장일치 동의로 승인되었습니다. 
Six of the 10 writing groups proposed changes to the title of their concept to describe it more accurately. The changes to the names and definitions of the emerging concepts, as well as the addition of the anti-racism concept, were approved by the authorship group with unanimous consent.

토론
Discussion

본 연구에서는 의학 문헌에서 CanMEDS 2025에 기술된 의사 역량에 통합될 수 있는 10가지 새로운 개념을 확인했습니다. 이러한 각 개념은 범위가 상당히 광범위하며, 대부분 현재 의학교육 문헌에서 논의되고 있는 여러 트렌드 또는 문제를 포괄하고 있습니다. 조사 결과 중 몇 가지 주제는 지난 3년간 광범위한 사회적, 경제적, 정치적, 환경적 담론에서 두드러진 현재 주제를 반영하고 있어 주목할 만합니다. 
Our study identified ten emerging concepts in the medical literature that could be incorporated into the physician competencies described by CanMEDS 2025.2938 Each of these concepts is quite broad in scope, with most encompassing several trends or issues currently being discussed in the medical education literature. Several themes within our results are notable as they mirror current themes of prominence in the broader social, economic, political, and environmental discourse over the past three years.

접근성, 형평성, 다양성, 포용성, 사회 정의, 반인종주의29,30와 관련된 주제가 분석에서 가장 많이 나타났습니다. 이 큰 주제는 궁극적으로 반인종주의에 초점을 맞춘 두 가지 주제로 분리되었습니다.30 의학 문헌에서 이러한 주제가 두드러진 것은 대중 담론에서 제도적 차별의 부정적인 영향에 대한 인식과 유사합니다.19 이 검토의 캐나다적 맥락, 캐나다 의료 시스템에서 원주민 건강의 중심적 중요성,39 그리고 캐나다 진실과 화해 위원회 최종 보고서에 발표된 건강 관련 7가지 행동 촉구를 고려할 때 원주민 건강에 초점을 맞춘 별도의 테마가 독립적으로 나타나지 않았다는 점이 주목할 만합니다. 이는 포함된 학술지들이 국제적인 초점(캐나다 의학교육 저널만이 캐나다 의학교육에 더 중점을 두었음)과 형평성, 다양성, 포용성, 사회 정의,29 및 반인종주의라는 주제 아래 많은 관련 구성 요소를 자주 코딩했기 때문인 것으로 생각됩니다.30 특히 원주민 건강과 관련된 제안 역량은 이러한 주제를 자세히 설명하는 두 원고의 중심입니다.29,30  
Themes relating to access, equity, diversity, inclusion, social justice, and anti-racism29,30 were prevalent in the analysis. This large theme was ultimately separated into two themes, with one focused specifically on anti-racism.30 The prominence of these themes in the medical literature parallels the acknowledgement of the negative impact of systemic discrimination in the public discourse.19 Given the Canadian context of this review, the central importance of Indigenous health in the Canadian healthcare system,39 and the seven calls to action related to health published in the final report of the Truth and Reconciliation Commission of Canada,40 it is notable that a separate theme focused on Indigenous health did not emerge independently. We suspect this is due to both the international focus of the included journals (only the Canadian Medical Education Journal focused more closely on Canadian medical education) as well as the frequent coding of many relevant constructs under the themes of equity, diversity, inclusion, social justice,29 and anti-racism.30 Notably, proposed competencies related to Indigenous health are central to both manuscripts further describing these themes.29,30

기후변화가 건강에 미치는 급격한 영향에 대한 과학적 공감대가 형성되고 있는 상황에서 '지구 보건' 주제38 의 존재는 놀라운 일이 아닙니다.22 특히 의사와 의료 수련생들은 기후변화가 인구의 건강과 복지에 미치는 영향에 관한 교육 및 옹호에서 두드러진 목소리를 내왔습니다.41-43 그러나 일부 저자는 개별 의사와 관련된 역량 틀 내에서 이러한 구조를 동원하는 것이 어려울 것이라는 우려를 표명했습니다.
The presence of the ‘Planetary Health’ theme38 is unsurprising given the scientific consensus building on the drastic impacts of climate change on health.22 Notably, physicians and medical trainees have had a prominent voice in education and advocacy relating to the impacts of climate change on the health and wellbeing of the population.4143 However, some authors were concerned that it would be challenging to mobilize this construct within a competency framework relevant to individual physicians.

데이터 및 기술 사용과 관련된 추가 역량의 필요성에 대해서는 폭넓은 공감대가 형성되었습니다.32,35 정밀 의학을 둘러싼 논의가 증가하는 가운데, '가상 진료' 및 '데이터 정보 의학' 테마로 통합된 이 코드는 신흥 기술의 보급16,17 과 개인 및 건강 데이터를 윤리적이고 안전하게 사용해야 할 필요성에 대한 사회적 인식의 증가와 병행합니다.44 
There was broad consensus regarding the need for additional competencies related to the use of data and technology.32,35 Beyond the growing dialogue surrounding precision medicine, the codes consolidated into the ‘Virtual Care’ and ‘Data-Informed Medicine’ themes parallel growing societal awareness of the pervasiveness of emerging technologies16,17 and the need for personal and health data to be used ethically and securely.44

코로나19 팬데믹의 영향은 여러 테마에서 확인할 수 있었습니다. 특히 '가상 치료' 주제35는 팬데믹 기간 동안 여행 및 모임 제한으로 인해 상당한 영향을 받은 개념인 가상 교육 및 가상 의료와 밀접한 관련이 있습니다.15,45 코로나19 팬데믹의 영향을 받은 다른 개념으로는 의료 시스템에 대한 복잡하고 지속적인 영향으로 인한 '복합 적응 시스템'33,46과 의사와 기타 의료 제공자에게 미치는 부담으로 인한 '의사 휴머니즘'31,47이 있습니다.
The influence of the COVID-19 pandemic was seen in numerous themes. In particular, the ‘Virtual Care’ theme35 relates strongly to virtual education and virtual healthcare, concepts that were substantially impacted by travel and gathering restrictions during the pandemic.15,45 Other concepts that were likely influenced by the COVID-19 pandemic included ‘Complex Adaptive Systems’33,46 due to its complex ongoing impacts on the healthcare system and ‘Physician Humanism’31,47 due to its strain on physicians and other healthcare providers.

'적응적 전문성'37 및 '임상적 추론'36 주제는 복잡한 임상 진료 영역에서 얼마나 빠른 변화가 일어나고 있는지를 인정합니다.48 의사가 이러한 과제를 해결하기 위해 진료 방식을 발전시킬 수 있는 역량이 있지만, 진화하는 의사의 역량을 적시에 CanMEDS에 통합하는 방법도 고려해야 합니다. 과거에는 약 10년에 한 번씩 개정을 실시하는 것이 CanMEDS에 도움이 되었지만, 이 원고의 발행과 CanMEDS 2025의 시행 사이에 새로운 역량이 등장할 것으로 예상할 수 있습니다. 정기적인 업데이트에서 지속적인 반복 프로세스로 전환하는 것을 고려해야 합니다. 이는 레지던트 수련 프로그램에서는 물류적으로 어려울 수 있지만, 미국심장협회와 같은 기관에서 지침을 지속적으로 업데이트하는 최신 프로세스에 부합하며, 더 작은 규모의 더 빈번한 업데이트가 가능할 수 있습니다.49 
The ‘Adaptive Expertise’37 and ‘Clinical Reasoning’36 themes acknowledge how rapidly changes are occurring within the complex realm of clinical practice.48 While there are competencies that enable physicians to evolve their practices to meet these challenges, it will also be important to consider how evolving physician competencies are integrated into CanMEDS in a timely manner. While performing revisions approximately once per decade has served CanMEDS well in the past, it is conceivable that new competencies will emerge between the publication of this manuscript and the implementation of CanMEDS 2025. Consideration should be given to transitioning CanMEDS from periodic updates to an ongoing iterative process. While this may be logistically challenging for residency training programs, it is in keeping with modern processes for the continuous updating of guidelines by organizations like the American Heart Association and could allow for smaller, more frequent updates.49

새롭게 등장한 각 개념이 CanMEDS 2025에 어떻게 통합될 수 있는지 설명하기 위한 추가 작업이 진행되었습니다. 집필 그룹은 각 개념을 정의하는 원고의 초안을 작성하고, CanMEDS 2015에 어떻게 표현되었는지 설명하고, CanMEDS 2025에 어떻게 통합할 수 있는지 제안했습니다.29-38 이 작업은 이 논문과 함께 캐나다 의학교육 저널 특별호에 게재되었으며, CanMEDS 2025 의사 역량 프레임워크 업데이트를 담당하는 전문가 실무 그룹에 정보를 제공할 것입니다. 이러한 새로운 개념이 공개됨으로써 캐나다 및 국제 의료계가 이 작업에 대해 논의하고 의견을 제시할 수 있는 기회가 될 것입니다.
Further work has been conducted to describe how each of the emerging concepts can be incorporated into CanMEDS 2025. Writing groups have drafted manuscripts that define each concept, outlined how it was represented in CanMEDS 2015, and proposed how it could be incorporated into CanMEDS 2025.2938 This work has been published along with this paper in this special issue of the Canadian Medical Education Journal and will inform the CanMEDS 2025 Expert Working Groups responsible for updating the CanMEDS physician competency framework. The open publication of these emerging concepts should provide Canadian and international medical communities with an opportunity to discuss and comment on this work.

강점과 한계
Strengths and limitations

이 검토의 강점은 상세하고 투명한 방법과 이해관계자 단체의 폭넓은 참여에 있습니다. CanMEDS 2015 개정 이전에 관련 문헌 검토가 수행되었지만,23 그 방법론이 자세히 설명되거나 동료 검토를 거치지 않았습니다. 저희는 상세한 방법론을 발표하고 동료 검토 과정을 거침으로써 이 작업을 개선했습니다. 또한 캐나다 왕립 의사 및 외과의사 대학, 퀘벡 의과대학, 캐나다 가정의학과 대학, 캐나다 의과대학 학부 협회의 대표를 포함한 광범위한 이해관계자 그룹에 의해 논문 검토가 수행되었습니다. 저자 팀의 다양성으로 인해 중요한 개념이 누락될 가능성이 줄어듭니다.
The strength of this review rests in the detailed, transparent methods and the broad engagement of stakeholder organizations. While a related literature review was conducted prior to the CanMEDS 2015 revision,23 its methodology was not described in detail or peer reviewed. We have improved on this work by publishing a detailed methodology and by putting our work through the peer review process. Additionally, the article review was conducted by a broader group oof stakeholders (including representatives from the Royal College of Physicians and Surgeons of Canada, Collège des Médecins du Québec, College of Family Physicians of Canada, and Association of Faculties of Medicine of Canada). The diversity of our authorship team decreases the chance that important concepts were missed.

이 리뷰에는 몇 가지 한계가 있었습니다. 

  • 첫째, 포함 기간을 3년으로, 출처를 5개 저널로, 특정 논문 유형을 제한하여 검토 범위를 제한했습니다. 이러한 제한은 새로운 개념을 파악하는 동시에 타당성을 유지하기 위해 내린 의도적이고 실용적인 결정이라고 생각합니다. 일부 주제를 놓쳤을 가능성에도 불구하고 회색 문헌 스캔을 통한 삼각 측량으로 추가 개념을 식별하지 못했다는 점은 안심할 수 있습니다.
  • 둘째, 수많은 리뷰어가 문헌 검색에 참여했기 때문에 새로운 개념 기준과 라벨링의 일관성을 보장하기 어려웠습니다. 또한, 대부분의 초록 검토는 추가 검토를 위한 플래그가 지정되지 않는 한 독립적으로 수행되었습니다. 멤버 점검을 통해 이러한 문제가 결과에 미치는 영향을 완화할 수 있을 것으로 기대합니다.
  • 마지막으로, 확인된 수많은 개념을 테마로 통합하는 것이 어려웠습니다. 우리의 분석은 일부 테마 내에서 광범위한 수의 코드를 집계했다는 비판을 받을 수 있습니다. 일부 워킹그룹 참가자들은 일부 테마가 새로운 개념이 아니거나 지나치게 광범위하다고 느꼈습니다. 이러한 도전은 연구 단계에 걸쳐 형평성, 다양성, 포용성과 관련된 주제가 진화하는 과정에서 잘 드러났습니다. 그럼에도 불구하고 대다수(실무 그룹 구성원의 82.4% 이상)는 각 주제가 더 깊이 탐구할 가치가 있는 중요한 새로운 개념이라고 생각했습니다.

This review had several limitations. First, we restricted the scope of our review by limiting the inclusion period to three years, the source to five journals, and specific article types. We see these limitations as intentional, pragmatic decisions that we made to maintain feasibility while identifying emerging concepts. Despite the potential to have missed some themes, it is reassuring that triangulation with the grey literature scan did not identify any additional concepts. Second, with numerous reviewers participating in the literature scan, it was difficult to ensure consistency in emerging concept criteria and labelling. Further, most abstract reviews were done independently unless flagged for additional review. We anticipate that the member checks mitigated the impact of this challenge on the results. Finally, consolidating the numerous identified concepts into themes was difficult. Our analysis could be criticized for aggregating a broad number of codes within some of the themes. Some of the working group participants felt that some themes were either not emerging concepts or were overly broad. This challenge is well-represented by the evolution of the themes related to equity, diversity, and inclusion throughout the phases of the study. This said, a large majority (≥82.4% of working group members) felt that each theme represented an important emerging concept that deserved further exploration.

결론
Conclusion

이 검토 및 분석을 통해 CanMEDS 2025의사 역량 프레임워크에 통합하기 위해 고려해야 할 10가지 새로운 개념이 확인되었습니다. 이 작업의 결과는 이번 특별호에 자세히 설명되어 있으며, 여기에는 각 개념에 대한 확장된 기사와 함께 CanMEDS 2025.29-38에 통합할 수 있는 방법에 대한 제안이 포함되어 있습니다. 이 작업의 공개를 통해 CanMEDS의 개정을 알리는 것 외에도, 개정 과정에 대한 투명성을 높이고 의사 역량에 관한 학술 문헌에서 조기 대화를 촉진할 수 있기를 바랍니다.
This review and analysis identified ten emerging concepts that should be considered for incorporation into the 2025 CanMEDS physician competency framework. The results of this work are elaborated upon in this special issue, which contains an expanded article on each concept along with suggestions for how it could be incorporated into CanMEDS 2025.2938. We hope that in addition to informing the revision of CanMEDS, the open publication of this work will create greater transparency around the revision process while facilitating an early dialogue in the academic literature on physician competence.

 


 

Can Med Educ J. 2023 Mar 21;14(1):4-12. doi: 10.36834/cmej.75591. eCollection 2023 Mar.

Emerging concepts in the CanMEDS physician competency framework

Affiliations collapse

1University of Saskatchewan, Saskatchewan, Canada.

2Royal College of Physicians and Surgeons of Canada, Ontario, Canada.

3University of Ottawa, Ontario, Canada.

4Association of Faculties of Medicine of Canada, Ontario, Canada.

5Université de Montréal, Quebec, Canada.

6Collège des médecins du Québec, Quebec, Canada.

7McMaster University, Ontario, Canada.

8University of Toronto, Ontario, Canada.

9McGill University, Quebec, Canada.

10College of Family Physicians of Canada, Ontario, Canada.

11NOSM University, Ontario, Canada.

12Queen's University, Ontario, Canada.

PMID: 36998506

PMCID: PMC10042782

DOI: 10.36834/cmej.75591

Free PMC article

Abstract 

Background: The CanMEDS physician competency framework will be updated in 2025. The revision occurs during a time of disruption and transformation to society, healthcare, and medical education caused by the COVID-19 pandemic and growing acknowledgement of the impacts of colonialism, systemic discrimination, climate change, and emerging technologies on healthcare and training. To inform this revision, we sought to identify emerging concepts in the literature related to physician competencies.

Methods: Emerging concepts were defined as ideas discussed in the literature related to the roles and competencies of physicians that are absent or underrepresented in the 2015 CanMEDS framework. We conducted a literature scan, title and abstract review, and thematic analysis to identify emerging concepts. Metadata for all articles published in five medical education journals between October 1, 2018 and October 1, 2021 were extracted. Fifteen authors performed a title and abstract review to identify and label underrepresented concepts. Two authors thematically analyzed the results to identify emerging concepts. A member check was conducted.

Results: 1017 of 4973 (20.5%) of the included articles discussed an emerging concept. The thematic analysis identified ten themes: Equity, Diversity, Inclusion, and Social Justice; Anti-racism; Physician Humanism; Data-Informed Medicine; Complex Adaptive Systems; Clinical Learning Environment; Virtual Care; Clinical Reasoning; Adaptive Expertise; and Planetary Health. All themes were endorsed by the authorship team as emerging concepts.

Conclusion: This literature scan identified ten emerging concepts to inform the 2025 revision of the CanMEDS physician competency framework. Open publication of this work will promote greater transparency in the revision process and support an ongoing dialogue on physician competence. Writing groups have been recruited to elaborate on each of the emerging concepts and how they could be further incorporated into CanMEDS 2025.

근거중심의학과 공유의사결정의 연결(JAMA, 2014)
The Connection Between Evidence-Based Medicine and Shared Decision Making

Tammy C. Hoffmann, PhD1,2; Victor M. Montori, MD, MSc3; Chris Del Mar, MD, FRACGP1

 

근거 기반 의학(EBM)과 공유 의사 결정(SDM)은 모두 양질의 의료 서비스를 제공하는 데 필수적이지만, 이 두 접근법 간의 상호 의존성은 일반적으로 잘 알려져 있지 않습니다. 근거 기반 의학은 환자로부터 시작하여 환자로부터 끝나야 합니다. 임상의는 근거를 찾아 평가하고 그 추론을 자신의 전문 지식과 통합한 후 환자의 가치와 상황을 반영한 결정을 내립니다. 환자의 가치, 선호도, 상황을 통합하는 것은 아마도 가장 어렵고 제대로 매핑되지 않은 단계일 것이지만, 가장 적은 관심을 받고 있습니다.1 이로 인해 EBM이 환자의 가치와 선호도를 무시한다는 비판이 제기되었는데, 이는 명백히 의도가 아닙니다.2 
E
vidence-based medicine (EBM) and shared decision making (SDM) are both essential to quality health care, yet the interdependence between these 2 approaches is not generally appreciated. Evidence-based medicine should begin and end with the patient: after finding and appraising the evidence and integrating its inferences with their expertise, clinicians attempt a decision that reflects their patient’s values and circumstances. Incorporating patient values, preferences, and circumstances is probably the most difficult and poorly mapped step—yet it receives the least attention.1 This has led to a common criticism that EBM ignores patients’ values and preferences—explicitly not its intention.2

공유 의사 결정임상의와 환자가 옵션, 이점과 해로움에 대해 논의하고 환자의 가치, 선호도, 상황을 고려한 후 공동으로 건강 결정에 참여하는 과정입니다. 이는 환자 중심의 의사소통 기술과 EBM의 교차점이며, 좋은 환자 치료의 정점입니다(그림).
Shared decision making is the process of clinician and patient jointly participating in a health decision after discussing the options, the benefits and harms, and considering the patient’s values, preferences, and circumstances. It is the intersection of patient-centered communication skills and EBM, in the pinnacle of good patient care (Figure).

 

 
 

하나만으로는 부족할까요?
One Without the Other?

이러한 접근 방식은 대부분 병행하여 발전해 왔지만, 어느 쪽도 다른 쪽 없이는 그 목표를 달성할 수 없습니다. SDM 없이는 진정한 EBM이 이루어질 수 없습니다.3 SDM은 근거를 상담에 명시적으로 가져와 환자와 논의할 수 있는 메커니즘입니다. 임상의가 환자의 선호도를 의사 결정에 반영하려고 해도 때때로 잘못 추측하는 경우가 있습니다. 그러나 근거에 기반한 숙고를 통해 환자는 정보에 입각한 선호도를 구축할 수 있습니다. 결정을 실행하고 그 결과를 감당해야 하는 환자의 입장에서는 이 과정을 통해 임상의의 근거와 전문성, 그리고 자신의 가치와 선호도를 의사 결정에 통합한다는 점을 인식하는 것이 더 적절할 수 있습니다. SDM이 없으면 EBM은 증거 폭압으로 변질될 수 있습니다. SDM이 없으면 근거가 실제 진료와 개선된 결과로 제대로 이어지지 않을 수 있습니다.
These approaches, for the most part, have evolved in parallel, yet neither can achieve its aim without the other. Without SDM, authentic EBM cannot occur.3 It is a mechanism by which evidence can be explicitly brought into the consultation and discussed with the patient. Even if clinicians attempt to incorporate patient preferences into decisions, they sometimes erroneously guess them. However, it is through evidence-informed deliberations that patients construct informed preferences. For patients who have to implement the decision and live with the consequences, it may be more pertinent to realize that it is through this process that patients incorporate the evidence and expertise of the clinician, along with their values and preferences, into their decision-making. Without SDM, EBM can turn into evidence tyranny. Without SDM, evidence may poorly translate into practice and improved outcomes.

마찬가지로, EBM의 원칙에 주의를 기울이지 않으면 SDM의 여러 단계가 근거와 불가분의 관계에 있기 때문에 한계가 있습니다. 예를 들어, 질환의 자연력, 가능한 옵션, 각각의 이점과 해로움에 대해 환자와 논의하고 이를 정량화할 때는 반드시 최상의 연구 증거를 기반으로 해야 합니다. SDM에 이러한 근거가 포함되지 않은 경우, 환자가 표현하는 선호도는 옵션의 위험과 이점에 대한 신뢰할 수 있는 추정치에 근거하지 않을 수 있으며, 그 결과 진정한 정보를 바탕으로 한 결정이 이루어지지 않을 수 있습니다.
Likewise, without attention to the principles of EBM, SDM becomes limited because a number of its steps are inextricably linked to the evidence. For example, discussions with patients about the natural history of the condition, the possible options, the benefits and harms of each, and a quantification of these must be informed by the best available research evidence. If SDM does not incorporate this body of evidence, the preferences that patients express may not be based on reliable estimates of the risks and benefits of the options, and the resulting decisions not truly informed.

단절이 있는 이유는 무엇인가요?
Why Is There a Disconnect?

EBM과 SDM 사이에 존재하는 단절의 원인 중 하나는 EBM의 리더, 연구자, 교사와 SDM의 리더, 연구자, 교사가 서로 다른 클러스터 출신이며, 역사적으로 서로 다른 클러스터에서 진료, 연구, 출판, 협업하는 경향이 있기 때문일 수 있습니다. 일부 형태의 SDM은 환자 커뮤니케이션에서 비롯되었으며, 이 분야의 컨퍼런스와 저널에서 많은 연구가 발표되었습니다. 1997년에 발표된 한 중요한 논문에서는 SDM을 치료 의사 결정의 모델이자 환자-임상의 간 의사소통 기술로 개념화했습니다. 그러나 이 논문은 EBM과 아무런 연관성이 없었는데, 그도 그럴 것이 EBM은 초기 단계였기 때문입니다.2
A contributor to the existing disconnect between EBM and SDM may be that leaders, researchers, and teachers of EBM, and those of SDM, originated from, and historically tended to practice, research, publish, and collaborate, in different clusters. Some forms of SDM have emerged from patient communication, with much of its research presented in conferences and journals in this field. A seminal paper in 19974 conceptualized SDM as a model of treatment decision making and as a patient-clinician communication skill. However, it did so without any connection to EBM—perhaps not surprisingly, because EBM was in its infancy.2

반대로 임상 역학에서 시작된 EBM은 증거를 찾고, 평가하고, 종합하는 것을 용이하게 하는 방법과 리소스에 중점을 두었습니다. 이러한 근거를 환자와 논의하고 그 사용에 환자와 함께 참여하는 것에는 훨씬 덜 집중해 왔습니다 (때로는 '소프트' 기술이라고 폄하되기도 함) . EBM에 대한 대부분의 관심은 스캔들(예: 미공개 데이터, 결과 '스핀', 이해관계 충돌)과 첨단 기술(예: EBM을 더 쉽고 편리하게 만드는 시스템)에 관한 것이었습니다. 환자와의 의사 결정에 근거를 사용하는 것에 대한 정보는 부족합니다.
Conversely, with its origins in clinical epidemiology, much of the focus of EBM has been on methods and resources to facilitate locating, appraising, and synthesizing evidence. There has been much less focus on discussing this evidence with patients and engaging with them in its use (sometimes even disparagingly referred to as “soft” skills). Most of the EBM attention has involved scandals (eg, unpublished data, results “spin,” conflicts of interest) and the high technology milestones (eg, systems to make EBM better and easier). Information about using evidence in decision-making with patients has been scant.

두 접근법 사이의 단절은 임상의와 학생에게 제공되는 교육에서도 분명하게 드러나며, 이 역시 교사의 배경을 반영하는 경우가 많습니다. EBM 교육을 수강할 수 있는 기회는 주로 질문을 형성하고 근거를 찾아 비판적으로 평가하는 데 초점을 맞춘 내용으로 가득합니다.5 근거를 적용하고 통합하는 방법을 배우는 것은 일반적으로 없거나 기술 훈련 없이 통과하는 과정으로 언급됩니다.
Disconnect between the 2 approaches is also evident in, and maintained by, the teaching provided to clinicians and students, again often reflecting the backgrounds of their teachers. Opportunities to attend EBM teaching abound with content largely focused on forming questions and finding and critically appraising evidence.5 Learning how to apply and integrate the evidence is usually absent, or mentioned in passing without skill training.

EBM과 SDM의 연결성 실현하기
Realizing the Connection Between EBM and SDM

논리적으로 시작할 수 있는 방법은 SDM 스킬 트레이닝을 EBM 트레이닝에 통합하는 것입니다. 이는 앞서 언급한 EBM 교육의 결함뿐만 아니라 현재 이용 가능한 SDM 교육 기회의 부족을 해결하는 데 도움이 될 것입니다. 또한 SDM의 활용을 촉진하고 더 넓게는 증거 번역을 촉진할 수 있습니다. 최근 의학교육에 SDM을 일상적으로 통합해야 한다는 요구는 두 접근법을 밀접하게 연계하여 활용할 수 있는 즉각적인 기회를 제공합니다. 
A logical place to start is by incorporating SDM skill training into EBM training. This will help to address not only the aforementioned deficits in EBM training but also the lack of SDM training opportunities presently available. Additionally, it may facilitate the uptake of SDM and, more broadly, evidence translation. Recent calls for SDM to be routinely incorporated into medical education present an immediate opportunity to capitalize on closely aligning the approaches.

EBM과 SDM을 통합하기 위한 또 다른 시작점은 임상 진료 지침의 개발과 실행입니다. 대부분의 가이드라인은 권고안을 수립할 때 환자의 선호도를 고려하지 않는 반면,6 일부 가이드라인은 임상의에게 환자와 옵션에 대해 이야기하라고 조언하지만, 이를 수행하는 방법과 환자가 이해할 수 있는 방식으로 근거를 전달하는 방법에 대한 지침은 제공하지 않습니다. 다음의 경우에 가이드라인에서 공동 의사 결정을 강력히 권장할 수 있습니다.

  • 옵션의 장단점이 거의 대등한 경우,
  • 근거의 불확실성으로 인해 명백히 우월한 접근법을 결정하기 어려운 경우,
  • 환자의 행동에 따라 혜택과 위험의 균형이 달라지는 경우(와파린을 사용하는 환자의 복약, 모니터링, 식이요법 준수 등)


Another place to start to bring EBM and SDM together is the development and implementation of clinical practice guidelines. Whereas most guidelines fail to consider patients’ preferences in formulating their recommendations,6 some advise clinicians to talk with patients about the options but provide no guidance about how to do this and communicate the evidence in a way patients will understand. Shared decision making may be strongly recommended in guidelines

  • when the options are closely matched in their advantages and disadvantages,
  • when uncertainty in the evidence impairs determination of a clearly superior approach, or
  • when the balance of benefits and risks depends on patient action, such as adherence to medication, monitoring, and diet in patients using warfarin.

결론
Conclusions

최근까지 EBM과 SDM 간의 연관성은 거의 없거나 기껏해야 암시적인 수준이었습니다. 그러나 상호 작용의 고무적인 징후가 나타나고 있습니다. 예를 들어, 두 접근법을 일부 통합하여 가르치거나,7 가이드라인을 SDM을 촉진하기 위해 어떻게 조정할 수 있는지에 대한 탐구,8,9 두 접근법을 모두 인정하는 연구 및 리소스 도구가 등장하고 있습니다. 후자의 예로는 의사결정에 적절한 근거를 제공하기 위해 환자 및 임상의와 협력하여 연구 의제 및 우선순위 설정, 체계적으로 수집된 최신 근거를 인용하고 그 신뢰성을 평가하는 국제 환자 의사결정 지원 표준의 새로운 근거 기준 등이 있으며,10 이를 통해 SDM 도구의 개발이 근거 기반 가이드라인의 수립에 대한 현대적 요구사항에 부합하도록 조정하고 있습니다. 또한, 근거 기반 의료의 실천과 공유 의사 결정의 과학에 초점을 맞춘 독립적인 대표 학회들이 공동 회의를 소집하고 있습니다.
Links between EBM and SDM have until recently been largely absent or at best implied. However, encouraging signs of interaction are emerging. For example, there has been some integration of the teaching of both,7 exploration about how guidelines can be adapted to facilitate SDM,8,9 and research and resource tools that recognize both approaches. Examples of the latter include research agenda and priority setting occurring in partnership with patients and clinicians to help provide relevant evidence for decision making; and a new evidence criterion for the International Patient Decision Aids Standards requiring citation of systematically assembled and up-to-date bodies of evidence, with their trustworthiness appraised,10 thus aligning the development of SDM tools with contemporary requirements for the formulation of evidence-based guidelines. Also, independent flagship conferences focused on the practice of evidence-based health care and on the science of shared decision making are now convening joint meetings.

의학은 최신 근거 없이 시행될 수 없고, 시행되어서도 안 됩니다. 또한 정보에 입각한 환자의 선호도를 파악하고 존중하지 않고 의학을 시행할 수도 없습니다. 임상의, 연구자, 교사, 환자는 이러한 접근법의 상호 의존적 관계를 인식하고 적극적으로 촉진해야 합니다. EBM에는 SDM이 필요하고, SDM에는 EBM이 필요합니다. 환자에게는 이 두 가지가 모두 필요합니다.
Medicine cannot, and should not, be practiced without up-to-date evidence. Nor can medicine be practiced without knowing and respecting the informed preferences of patients. Clinicians, researchers, teachers, and patients need to be aware of and actively facilitate the interdependent relationship of these approaches. Evidence-based medicine needs SDM, and SDM needs EBM. Patients need both.


JAMA. 2014 Oct 1;312(13):1295-6. doi: 10.1001/jama.2014.10186.

The connection between evidence-based medicine and shared decision making

Affiliations collapse

1Centre for Research in Evidence-Based Practice, Faculty of Health Sciences and Medicine, Bond University, Queensland, Australia2University of Queensland, Brisbane, Australia.

2Knowledge and Evaluation Research (KER) Unit, Mayo Clinic, Rochester, Minnesota.

3Centre for Research in Evidence-Based Practice, Faculty of Health Sciences and Medicine, Bond University, Queensland, Australia.

PMID: 25268434

DOI: 10.1001/jama.2014.10186

국제 교수개발에서 문화를 이해하고 포용하기(Perspect Med Educ. 2023)
Understanding and Embracing Culture in International Faculty Development
SARA MORTAZ HEJRI, RASHMI VYAS, WILLIAM P. BURDICK, YVONNE STEINERT

 

소개
Introduction

여러 국가의 보건 전문직 교육자들의 역량 강화를 촉진하기 위해 고안된 국제 교수진 개발 프로그램(IFDP)의 수가 증가하고 있습니다[1, 2]. 프로그램 평가에 따르면 참가자의 지식과 기술이 향상되고 초국가적 커뮤니티가 형성된 것으로 나타났습니다[1, 3, 4, 5, 6, 7]. 그러나 프로그램 참가자의 문화적 배경이 다양함에도 불구하고 참가자의 신념, 가치관, 행동이 참여와 학습에 미치는 영향을 조사한 연구[8, 9]는 거의 없습니다. 교육자들은 또한 교수진 개발 문헌에서 '모든 것에 맞는 하나의 크기'라는 개념이 널리 퍼져 있다고 한탄했습니다[10]. 
The number of international faculty development programs (IFDPs), designed to promote capacity-building among health professions educators across different countries, is growing [1, 2]. Program evaluations have demonstrated increases in participants’ knowledge and skills, and the creation of transnational communities [1, 3, 4, 5, 6, 7]. However, despite the diversity of program participants’ cultural backgrounds, very few studies [8, 9] have explored the influence of participants’ beliefs, values, and behaviors on participation and learning. Educators have also lamented that the notion of “one size fits all” prevails in the faculty development literature [10].

IFDP는 전 세계 여러 국가의 참가자를 한데 모으는 프로그램[3, 4, 6, 7]과 한 국가에서 다른 국가로 교수자 개발 프로그램을 이전하는 프로그램[5, 11, 12, 13, 14, 15]의 두 가지 유형을 지칭합니다. IFDP에 대한 24개 보고서를 대상으로 한 범위 검토에 따르면 "이들 보고서의 약 50%가 국가적 맥락이나 문화적 규범 및 신념의 중요성을 인정했지만", IFDP의 문화적 문제는 체계적으로 연구되지 않았습니다[1]. 또한 주요 출판물[3, 4, 5, 6, 7, 12, 13, 14, 15, 16]을 검토한 결과, 현재 IFDP 내에서 경험하는 문화적 차이의 유형에 대해 알려진 바가 거의 없는 것으로 나타났습니다. 다른 나라에서 온 참가자들이 IFDP에서 낯선 문화적 가치와 관행에 직면했을 때 어떻게 느끼거나 행동할 수 있는지, 그리고 교수진이 문화적 차이를 어떻게 다루는지에 대해서도 거의 연구되지 않은 상태입니다. 
IFDPs refer to two types of programs: those that bring together participants from different countries around the world [3, 4, 6, 7] and those that include the transfer of a faculty development program from one country to another [5, 11, 12, 13, 14, 15]. A scoping review of 24 reports of IFDPs stated that “although close to 50% of these reports acknowledged the importance of national contexts or cultural norms and beliefs”, cultural issues in IFDPs have not been systematically studied [1]. In addition, a review of primary publications [3, 4, 5, 6, 7, 12, 13, 14, 15, 16] revealed that little is currently known about the types of cultural differences experienced within an IFDP. How participants from different countries may feel or behave when facing unfamiliar cultural values and practices in an IFDP – and how faculty developers address cultural differences – has also remained largely unexplored.

이 연구의 목표는 IFDP에서 다양한 문화적 가치와 신념이 어떻게 인식되고 경험되는지 탐구하는 것이었습니다. 연구 질문은 다음과 같습니다:

  • (1) IFDP에서 문화는 어떻게 인식되었나요?
  • (2) 문화가 교육과 학습에 미치는 영향은 무엇인가? 

The goal of this study was to explore how different cultural values and beliefs were perceived and experienced in an IFDP. Our research questions addressed the following:

  • (1) How was culture perceived in an IFDP?
  • (2) What were the influences of culture on teaching and learning?

이 연구를 위해 우리는 다음과 같은 문화의 정의를 선택했습니다: "집단 간에 공유할 수 있고, [사고, 의사소통 스타일, 상호작용 방식, 역할 및 관계에 대한 견해, 가치관, 관행 및 관습을 포함하는] [학습된 신념과 행동]의 통합된 패턴"[17]. 후자는 또한 변혁적 학습 이론(TLT)에 설명된 대로 '마음의 습관'이라고 불리는 것을 지칭할 수도 있습니다[18, 19]. 비판적 성찰과 변증법적 담론의 역할을 포함하는 TLT는 개인이 자신의 신념을 보다 포용적이고 변화에 개방적으로 만들기 위해 가정과 기대를 변화시키는 방법을 설명합니다. 이 이론은 본질적으로 구성주의적이고, 사회적 관점에서 학습을 바라보며, 이문화 교육, 의사소통, 고등교육의 국제화를 연구하는 데 사용되어 왔기 때문에 데이터 분석과 해석에 TLT를 사용했습니다[20, 21].
For this study, we chose the following definition of culture: “An integrated pattern of learned beliefs and behaviours that can be shared among groups and include thoughts, styles of communicating, ways of interacting, views of roles and relationships, values, practices and customs” [17]. The latter can also refer to what has been called “habits of mind”, as described in Transformative Learning Theory (TLT) [18, 19]. TLT, which includes the role of critical reflection and dialectical discourse, describes how individuals transform assumptions and expectations to make their beliefs more inclusive and open to change. We used TLT to inform our data analysis and interpretation as this theory is constructivist in nature, views learning through a social lens, and has been used to study cross-cultural training, communication, and the internationalization of higher education [20, 21].

연구 방법
Methods

우리는 해석적 설명을 사용하여 질적 연구를 수행했습니다 [22, 23]. 해석적 기술(ID)은 "인간 경험의 구성적이고 맥락적인 본질을 인정하는, 해석적 지향에 기반을 둔" 귀납적 접근법입니다[22]. 이러한 설계를 통해 반구조화된 인터뷰를 통해 참가자의 경험과 인식에 대한 심층적인 이해를 확인하고 연구 결과를 교수진 개발의 맥락에서 적용할 수 있는 가시적인 결과로 변환할 수 있었습니다[24]. 이 연구는 맥길 대학교 의학 및 보건과학부 IRB의 윤리 승인을 받았습니다.
We conducted a qualitative study using interpretive description [22, 23]. Interpretive description (ID) is an inductive approach “grounded in an interpretive orientation that acknowledges the constructed and contextual nature of human experience” [22]. This design allowed us to ascertain an in-depth understanding of participants’ experiences and perceptions through semi-structured interviews and translate our findings into tangible outcomes that could be applied in the context of faculty development [24]. Ethics approval was obtained from the Faculty of Medicine and Health Sciences IRB at McGill University.

연구 설정 및 샘플링
Study Setting and Sampling

연구 환경은 국제 보건 전문직 교육자의 교육 성과 및 리더십 기술 향상을 목표로 2001년에 설립된 미국 기반의 교수진 개발 프로그램인 FAIMER(국제 의학 교육 및 연구 발전 재단) 연구소 펠로우십이었습니다[25]. 이 연구 당시, 변혁적 학습의 원칙[26]에 따라 2년 동안 진행된 이 프로그램은 필라델피아에서 두 차례의 현장 세션과 두 차례의 원격 학습 세션으로 구성되었습니다. 참가자의 소속 기관에서 실행될 교육 프로젝트는 학습의 중심지이자 초국가적 교육자 커뮤니티를 형성하는 매개체였습니다[27]. 40여 개국에서 약 2,000명의 교육자가 FAIMER 인스티튜트에 참여했습니다[25].
The study setting was the FAIMER (Foundation for Advancement of International Medical Education and Research) Institute Fellowship, a US-based faculty development program established in 2001 that aims to improve the teaching performance and leadership skills of international health professions educators [25]. At the time of this study, this two-year program, informed by principles of transformational learning [26], was composed of two onsite sessions in Philadelphia, followed by two distance learning sessions. An education project, to be implemented in participants’ home institutions, was a focal point for learning and a vehicle for creating a transnational community of educators [27]. Almost 2000 educators from over 40 countries have participated in the FAIMER Institutes [25].

저희는 2014년부터 2019년까지 5개 코호트(2014~2019년)의 FAIMER 펠로우와 프로그램에 적극적으로 참여한 FAIMER 교수진 중에서 참가자를 모집했습니다. 적극적인 참여의 기준에는 현장 프로그램 중 최소 한 세션 이상에서 '주임 교수진'으로 간주되는 교수진과 지난 3~5년 동안 FAIMER에서 강의한 경험이 있어 현재 프로그램 콘텐츠와 문화에 익숙한 교수진이 포함되었습니다. FAIMER는 여러 국가에 여러 지역 연구소를 운영하고 있지만, 필라델피아 연구소는 국제적인 코호트를 대표하기 때문에 연구 참가자를 선정했습니다. 연구와 연구팀을 소개하기 위해 WB와 RV는 대상자(72명의 펠로우와 10명의 교수진)에게 이메일을 보냈고, 이어서 SMH와 YS가 동일한 명단에 대한 자세한 이메일 초대장을 보냈습니다. 총 40명의 펠로우와 5명의 교수진이 참여에 동의했으며, 이 중 성별, 국적, 징계 배경, 코호트 연도의 이질성을 보장하기 위해 의도적으로 표본을 추출했습니다. 우리는 새로운 데이터가 새로운 주제나 데이터 해석의 수정으로 이어지지 않을 때 발생하는 정보적 충분성에 도달할 때까지 연구 참가자를 등록했습니다[28]. 
We recruited participants from five cohorts of FAIMER Fellows (2014–2019) and from FAIMER Faculty who were actively involved in the program. Criteria for active involvement included Faculty who were considered “lead faculty” in at least one session during the onsite program and Faculty who had taught in FAIMER within the last 3–5 years and were, therefore, familiar with the current program content and culture. While FAIMER offers several Regional Institutes in different countries, we selected study participants from the Philadelphia site because it represents an international cohort. To introduce the study and research team, WB and RV sent e-mails to the eligible population (72 Fellows and 10 Faculty); this was followed by a detailed e-mail invitation from SMH and YS to the same list. Altogether, 40 Fellows and 5 Faculty agreed to participate, from which we purposefully sampled participants to ensure heterogeneity in gender, nationality, disciplinary background, and cohort year. We enrolled study participants until we reached informational sufficiency, which occurred when new data did not lead to new themes or modifications of data interpretation [28].

데이터 수집 및 분석
Data Collection and Analysis

FAIMER 연구소와 관련이 없거나 이전에 펠로우들에게 알려지지 않았던 SMH는 2019년 가을과 겨울에 Zoom 또는 Skype를 사용하여 반구조화된 인터뷰를 진행하여 펠로우들의 경험과 인식을 이끌어냈습니다. 처음 두 차례의 인터뷰와 데이터 분석을 통해 얻은 조사자들의 개념 정립에 따라 인터뷰 가이드(온라인 부록 1)를 파일럿 테스트했습니다. 이러한 변경 사항에는 질문의 순서를 바꾸고 도입부를 짧게 하여 주요 질문으로 더 빨리 진입하고 인터뷰 시간에 맞출 수 있도록 했습니다. 또한 필요에 따라 다른 문구를 사용하여 FAIMER 교수진에 대한 인터뷰 가이드를 수정했습니다. 예를 들어, 교수진에게는 "당신의 가르침"을, 펠로우에게는 "당신의 학습"을 사용했습니다. 또한 FAIMER 교수진이 문화적 차이를 고려하기 위해 교육 내용이나 과정을 변경한 적이 있는지 조사했습니다. 차별, 판단, 고정관념을 암시할 수 있는 인터뷰 질문은 의도적으로 피했습니다. 인터뷰는 43분에서 74분 동안 진행되었으며, 오디오로 녹음되었고 외부 전문가가 그대로 필사했습니다. SMH는 모든 녹취록의 정확성을 검토했습니다.
SMH, who was not connected to the FAIMER Institute nor previously known to Fellows, conducted semi-structured interviews in the fall and winter of 2019 using Zoom or Skype to elicit Fellows’ experiences and perceptions. We pilot-tested an interview guide (Online Appendix 1), which was adjusted following the first two interviews and the investigators’ evolving conceptualizations that arose from data analysis. These changes included the order of questions asked and a shorter introduction so that we could dive into the main questions more quickly and accommodate the interview timeframe. We also modified the interview guide for FAIMER Faculty by using different wording, as needed. For example, for Faculty, we used “your teaching;” for Fellows, we used “your learning”. We also probed if FAIMER Faculty had ever changed their teaching content or process to take cultural differences into account. We deliberately avoided interview questions that could suggest discrimination, judgment, or stereotyping. Interviews lasted from 43 to 74 minutes, were audio-recorded, and were transcribed verbatim by an external professional. SMH reviewed all transcripts for accuracy.

데이터 분석은 데이터 수집과 함께 반복적으로 진행되었습니다. 귀납적 기법으로 시작하여 예비 패턴공통 스레드를 식별하고, 나중에 데이터를 가장 잘 표현하기 위해 병합했습니다. 지속적인 비교 분석을 통해 유사한 코드와 주제에 대한 전체론적, 교차 사례 분석을 수행했습니다. 또한 각 주제를 설명하는 예시적인 인용문도 적극적으로 찾았습니다. 분석 결과를 이해하기 위해 점차 해석 단계로 나아갔고, 이를 통해 의미 있는 주제에 집중하고 분석 결과의 실제 적용을 정교화할 수 있었습니다. 또한 기존의 이론적 프레임워크[22]를 사용하여 코드를 생성하지 않았기 때문에 데이터 분석은 '상향식'이었지만, 분석과 해석이 발전함에 따라 TLT를 활용하여 참가자 간의 상호 연결된 관계와 교육 및 학습의 맥락에 대한 의미 있는 시각을 제공했습니다[23]. 
Data analysis was iterative and started alongside data collection. We began with an inductive technique to identify preliminary patterns and common threads which were later merged to best represent the data. By using constant comparative analysis, we conducted a holistic and cross-case analysis of similar codes and themes. We also actively sought out exemplar quotations, illustrating each theme. To make sense of the findings, we gradually progressed to interpretation; this allowed us to focus on meaningful themes and elaborate on the practical application of findings. Moreover, though our data analysis was “bottom-up” as we did not generate codes using a pre-existing theoretical framework [22], we drew upon TLT as the analysis and interpretation evolved, to provide a meaningful lens on the interconnected relationship between participants and the context for teaching and learning [23].

두 명의 저자(SMH와 YS)가 처음 두 번의 인터뷰독립적으로 읽고 코딩한 후, 주요 관찰 사항과 예비 조사 결과를 논의했습니다. SMH는 YS의 의견을 바탕으로 나머지 녹취록을 계속 분석하여 패턴과 주제에 대한 합의를 이끌어냈습니다. 펠로우들과의 15개 인터뷰가 모두 코딩되었을 때, SMH는 나머지 FAIMER 교수진과의 인터뷰 5개를 코딩하기 시작했습니다. SMH와 YS는 정기적으로 만나 확인된 코드, 주제별 그룹, 연구 결과의 해석을 다듬고 합의했습니다. RV와 WB는 주제를 검토하고 주제, 예시 인용 및 연구 결과의 적용에 대한 피드백을 제공했습니다. 
Two authors (SMH and YS) independently read and coded the first two interviews, after which they discussed key observations and preliminary findings. SMH continued analyzing the remaining transcripts with input from YS, which led to consensus on patterns and themes. When all 15 interviews with Fellows had been coded, SMH started coding the remaining five interviews with FAIMER Faculty. SMH and YS met regularly to refine and agree on identified codes, thematic groupings, and interpretations of the findings. RV and WB reviewed the themes and gave feedback on themes, exemplar quotations, and applications of the findings.

반영성
Reflexivity

저자들은 학문적 배경, 교육적 책임, FAIMER 연구소에서의 참여도 등이 서로 달랐습니다. 저자 중 두 명은 미국 출신으로 FAIMER 펠로우십에 참여했습니다. 다른 두 명의 저자는 캐나다(다양한 문화가 공존하는 프랑스어권 지역) 출신입니다. 모든 저자는 국제 교수진 개발에 참여했습니다. 문화에 대해 묻고 있었기 때문에 우리가 직접 문화적 유사점과 차이점을 어떻게 경험하고 인식했는지에 대해 논의하는 것이 중요했습니다. 우리는 개별적으로 또는 함께 반성하고, 우리의 경험이 인터뷰 질문과 데이터 분석에 어떤 영향을 미칠 수 있는지 논의했습니다. 우리는 우리의 이전 경험이 연구의 질문과 설계를 형성했을 뿐만 아니라 데이터 수집과 분석에도 영향을 미쳤을 수 있다는 것을 알고 있었습니다. 정기적으로 모임을 갖고 지속적인 토론과 성찰의 과정에 깊이 참여함으로써 참가자들의 내러티브를 해석하는 데 있어 다양한 관점의 균형을 맞추는 데 도움이 되었습니다. 
The authors differed in their disciplinary backgrounds, educational responsibilities, and involvement in the FAIMER Institute. Two of the authors are from the US and have been involved in the FAIMER Fellowship. The two other authors are from Canada (from a French-speaking province with multiple cultures). All authors have been involved in international faculty development. Since we were asking about culture, it was important for us to discuss how we experienced and perceived cultural similarities and differences ourselves. We reflected individually and together, and we discussed how our experiences could influence the interview questions and data analysis. We were aware that our previous experiences not only shaped the questions and design of the research but might have also impacted data collection and analysis. Meeting regularly and becoming deeply engaged in the process of continuous discussion and reflection further helped us balance diverse perspectives on the interpretation of participants’ narratives.

연구 결과
Results

12개국에서 15명의 FAIMER 펠로우와 5명의 FAIMER 교수진이 연구에 참여했습니다(표 1). 아래에서는 두 가지 조사 영역에 따라 연구 결과를 제시합니다. 인용문은 펠로우(F) 또는 FAIMER 교수진(FF)으로 표기했습니다. 
Fifteen FAIMER Fellows and five FAIMER Faculty from 12 countries participated in the study (Table 1). Below, we present our findings according to the two areas of inquiry. Quotations are referenced by Fellows (F) or FAIMER Faculty (FF).

1) IFDP에서 문화는 어떻게 인식되었나요?
1) How was culture perceived in an IFDP?

FAIMER 펠로우와 교수진은 위계질서, 협업, 문서화, 자발성과 관련된 가치와 신념뿐만 아니라 특정 행동과 관행(예: 얼굴 표정, 몸짓, 서로를 대하는 방식, 의견 표현, 물리적 거리두기)에서 문화적 유사점과 차이점을 인식했다고 답했습니다. 
FAIMER Fellows and Faculty commented that they perceived cultural similarities and differences in terms of specific behaviors and practices (e.g., facial expressions, body gestures, ways of addressing each other, opinion expression, physical distancing) as well as values and beliefs related to hierarchy, collaboration, documentation, and spontaneity.

[X]의 사람들은 여러분과 대화할 때 실제로 여러분을 쳐다보지 않습니다. 그들은 아래를 내려다보거나 어깨 너머로 바라봅니다. 여러분은 서로 마주보고 있고, 여러분은 그들을 바라보고 있지만 그들은 여러분의 얼굴을 보고 있지 않습니다. (F-9)
They in [X] don’t actually look at you when they talk to you. They look down, or they look over your shoulder. You are facing each other, and you are looking at them, but they are not looking at your face. (F-9)

우리나라 문화의 특징 중 하나는 상급자에서 하급자로 내려오는 상명하복식 문화입니다. 이는 사람과 사람 사이의 상호작용, 즉 후배와 선배, 젊은이와 나이든 사람 사이의 상호작용과 관련이 있습니다. 이 나라에는 일정한 패턴이 있습니다. (F-6)
One characteristic in the culture in my country, one policy they use, is top-down, from the superior to the subordinate. That’s related to the interaction between people, with junior, with senior, with the younger, with the older ones. There is a certain pattern in this country. (F-6)

일부 문화적 차이는 초창기 FAIMER와의 만남에서 '눈에 보이는' 것으로 여겨졌지만, 일부는 펠로우와 교수진이 함께 시간을 보낸 후에야 분명해졌습니다. 또한 어떤 문화적 차이(예: 의견 표현)는 펠로우들이 온라인 세션을 시작할 때에도 분명했던 반면, 다른 문화적 차이(예: 주도권 잡기, 위계질서 존중)는 펠로우들이 각자의 나라에서 개별 프로젝트를 시작했을 때에 분명해졌습니다. 
While some cultural differences were deemed “visible” in the early FAIMER encounters, some were only evident after Fellows and Faculty had spent time together. Also, several cultural differences (e.g., opinion expression) were evident when Fellows started the online sessions, whereas others (e.g., taking initiative; respecting hierarchy) were apparent when Fellows began working on their individual projects in their own countries.

제가 소속된 기관으로 돌아갔을 때, 그곳의 교육 문화는 상당히 달랐습니다. 사람들은 혼자서 가르치고 다른 사람이 옆에 있는 것을 원하지 않기 때문에 교사들 간의 상호 협력은 FAIMER의 독특한 측면이었는데, 저는 여전히 본국 기관에서 이를 구현하는 데 어려움을 겪고 있습니다. (F-2)
When I went back to my institution, there the culture of how to teach is quite different. People teach in isolation; they don’t want any other person to be there, so the kind of mutual collaboration between teachers was a unique aspect of FAIMER, which I am still struggling with in trying to implement it in my home institution. (F-2)

참가자들은 일반화나 지나친 단순화를 피하기 위해 조심스럽게 문화라는 개념에 접근했습니다. 예를 들어, 참가자들은 한 국가 내에서도 서로 다른 문화적 가치관을 가질 수 있기 때문에 같은 나라에서 온 다른 참가자가 자신의 신념을 공유하지 않을 수도 있다는 점을 강조했습니다. 또한 참가자들은 '문화적 습관'으로 인식되는 것이 반드시 문화와 관련이 있는 것이 아니라 성격, 개인의 선택, 조직 구조 또는 직업적 가치관에 기인할 수 있다는 점에 주목했습니다. 
Participants approached the notion of culture with caution, trying to avoid generalizations or oversimplification. For example, participants highlighted that another participant from their country might not share their beliefs, as people within a single country could have distinct cultural values. Participants also noted that what was perceived as a “cultural habit” might not necessarily be related to culture; instead, it could be attributed to personality, individual choice, organizational structure, or professional values.

따뜻하고 외향적이며 사람들과 대화하는 것을 좋아하는 사람과 사람들과 교류하기 싫어하고 목이 뻣뻣한 사람은 다르게 행동할 수 있습니다. 하지만 이는 모든 문화에 존재합니다. 그게 바로 개성이죠. 저는 그것을 문화와 연결시킬 수 없다고 생각합니다. (F-9)
If you are warm and outgoing and you like to talk to people, if you are a stiff-necked person who doesn’t want to engage with people, you behave differently. But that’s in every culture. That’s personality. I don’t think I can connect it to culture. (F-9)

"제가 다니는 대학은 65년 정도 된 곳인데 [...] 변화에 대한 저항이 엄청나지만, 문화를 그 원인으로 돌리지는 않겠습니다." (F-14)
“My university is about sixty-five-years old […] the resistance to change is enormous, but I’m not going to attribute culture to this.” (F-14)

2) 문화적 다양성이 교수와 학습에 미치는 영향은 무엇인가요?
2) What were the influences of cultural diversity on teaching and learning?

IFDP에서 문화적 차이가 교수와 학습에 미치는 영향과 관련된 세 가지 주제를 확인했습니다. 
We identified three themes related to the influences of cultural differences on teaching and learning in an IFDP.

배움의 장벽이 아니라, 문화 인식과 네트워크 구축의 가교 역할
Not a barrier to learning, but a bridge to cultural awareness and network-building

펠로우들은 연수 프로그램에서 경험한 문화적 차이가 대부분 학습에 방해가 되지 않는다고 생각했습니다. 이들은 펠로우십을 최대한 활용하려는 의욕과 열정이 강했으며, 새로운 규칙과 경험을 이해하고 존중하며 수용하는 데 도움이 되는 새로운 '마음가짐'을 가지고 프로그램에 참여했습니다. 또한 자신의 생각에 의문을 제기하고 자신의 신념과 가정을 평가할 수 있는 기회도 소중하게 여겼습니다. 
Fellows believed that the cultural differences they experienced during the Institute program did not, for the most part, impede learning. They were highly motivated and passionate about making the most out of the Fellowship, and they came to the program with a fresh “mindset” that helped them understand, respect, and embrace new rules and experiences. They also valued the opportunity to question their thinking and assess some of their beliefs and assumptions.

몇 가지 사소한 것들이 눈에 띄지만 학습에 큰 영향을 미치지는 않는다고 생각합니다. 이런 것들은 우리가 주의해야 할 작은 문화적인 것들일 뿐이고... 저는 그 문화가 무엇이든 배우고 싶었습니다. (F-10)
You can see there are some small things, but I don’t think that they would drastically affect our learning. These are just small cultural things that we have to be careful about… and I wanted to learn whatever the culture was. (F-10)

또한 문화적 차이로 인해 펠로우들이 "다양성에 대한 학습"과 "네트워크 구축"과 같은 프로그램 목표 이상의 성과를 달성할 수 있었다는 점도 주목했습니다. 많은 참가자가 문화적 다양성의 존재를 높이 평가했으며, 한 펠로우는 이를 "삶의 아름다움"이라고 표현했습니다. 참가자들은 다른 문화에 대해 배우는 것 외에도 비판적 성찰과 변증법적 담론의 중요한 측면인 잘못된 가정을 바로잡을 수 있는 기회를 가졌다고 보고했습니다. 다양성은 참가자들이 자신이 "세계를 대표하지 않는다"는 것과 자신이 실천하는 것이 반드시 "유일한 것 또는 최선의 것"은 아니라는 것을 이해하는 데 도움이 되었습니다. 또한 다양한 문화에 대한 노출은 참가자들이 문화적으로 다양한 환경에서 일하는 데 더 유능해졌다고 느끼면서 전문성 개발에 도움이 되었습니다. 한 펠로우는 이 새로운 인사이트가 "눈을 뜨게 해줬다"고 말했는데, 펠로우십 이전에는 "비슷한 특성을 공유하는 특정 인구가 있는 특정 환경에 갇혀 있었기 때문에" "이야기의 다른 측면을 보지 못했다"고 합니다.
We also noted that cultural differences enabled Fellows to achieve outcomes beyond the program objectives, including “learning about diversity” and “network building.” Many participants appreciated the existence of cultural diversity, which one Fellow described as the “beauty of life.” In addition to learning about other cultures, participants reported having an opportunity to correct false assumptions, an important aspect of critical reflection and dialectical discourse. Diversity helped participants understand that they did “not represent the world” and that what they practiced was not necessarily “the only thing or the best thing.” Additionally, exposure to different cultures helped participants with their professional development, as they felt that they had become more competent working in culturally diverse settings. One Fellow called this new insight an “eye-opener,” as before the Fellowship, she was “confined to a certain environment with a certain population sharing similar characteristics” that prevented her from “seeing other aspects of stories.”

다양한 문화를 접하고 이러한 규칙을 배우면서 지난번에 무슨 일이 있었는지 기억하기 때문에 매번 더 효과적으로 대처할 수 있다고 느낍니다. 그래서 저는 바로 제 규칙을 적용하지 않습니다. 대신 한 발 물러서서 관찰만 하고 판단하지 않으려고 노력합니다. (FF-3)
As I encounter different cultures and learn these rules, I feel that I’m able to be more effective each time because I remember what happened last time. So, I don’t immediately jump to my set of rules. Instead, I try to stay back, just observe, and not judge. (FF-3)

몇몇 참가자들은 펠로우십을 마친 후에도 다른 펠로우들과 돈독한 관계를 맺었고, 문화적 이질감이 네트워크 구축의 장애물이 되지 않았다고 말했으며, 오히려 공통의 관심사와 공동의 목표를 가진 개인들의 네트워크에 대한 설명은 팀워크와 자원에 대한 접근 기회를 제공하는 실천 커뮤니티[29]의 정의와 일치한다고 말했습니다. 일부 펠로우들은 여러 면에서 다른 사람들과 "이렇게 오래 지속되는 관계를 구축"하거나 "개인적인 생각과 이야기를 공유"할 수 있을 거라고는 예상하지 못했기 때문에 네트워크 구축이 놀랍게 다가왔습니다. 한 펠로우는 국제 프로그램은 물론이고 좀 더 동질적인 환경에서 "매우 특별한 수준의 신뢰"를 쌓은 경험이 없었다고 설명하며, 과거의 경험에 대해 비판적으로 생각할 수 있는 기회를 소중히 여겼습니다. 
Several participants stated that they developed strong relationships with other Fellows that continued after completing the Fellowship, and that cultural dissimilarities were not experienced as barriers to network building; rather, their description of a network of individuals with common interests and shared goals was consistent with the definition of a community of practice [29], offering opportunities for teamwork and access to resources. Network-building came as a surprise to some Fellows, as they did not expect to “build such long-lasting relationships” or “share their personal thoughts and stories” with people who were different in many aspects. One Fellow explained that she had not previously experienced building “trust at a very special level” in more homogenous settings, let alone in an international program, and she valued the opportunity to think critically about past experiences.

우리는 서로 다른 환경, 다른 수준의 발전, [다른] 교육 분야에서 왔습니다. 그래서 우리는 서로에게서 배우고, 경험을 공유하고, 협력할 수 있는 기회를 가졌습니다. 저는 직장에서 어려운 일이 생기면 "여러분, 저에게 이런 문제가 있습니다. 어떻게 하면 좋을까요?"라고 묻습니다. (F-1)
We came from different environments, different levels of advancement, [different] education fields. So, we had the chance to learn from each other, to share our experiences, to collaborate. If I have some challenges at work, I just write a message asking, “Guys, I have this issue. What do you advise me?” (F-1)

적응, 수정, 중재로 이어지는 불안과 불확실성
Unease and uncertainty leading to adaptation, modification, and mediation

인터뷰 결과, 특정 문화적 차이는 때때로 불안감이나 불확실성을 야기하는 미묘한 도전으로 경험되는 것으로 나타났습니다. 
The interviews revealed that certain cultural differences were experienced as subtle challenges which, at times, created uneasiness or uncertainty.

위에서 강조한 바와 같이 프로그램 진행 중 문제가 되었던 문화적 행동의 예로는 다음 등이 있었습니다. 

  • 학습자의 반응을 중시하는 강사에게 특히 어려운 수업 중 무표정, 눈 맞춤 부족, 의견 보류,
  • 잘못 해석을 하기도 하고, 무슨 일이 일어나고 있는지 파악하기 위한 '속앓이inner talk'로 이어질 수 있는 몸짓,
  • '너무 친밀하다' 또는 '조금 무섭다'고 인식될 수 있는 물리적 거리,
  • 수업 기능이나 그룹 작업을 방해할 수 있는 시간에 대한 다른 태도, 프로그램 일정을 변경해야 하는 특정 종교 의식 

Examples of cultural behaviors that were challenging during the program, as highlighted above, included:

  • a flat facial expression, a lack of eye contact, and holding back opinions in class, which was especially challenging for instructors who valued learners’ reactions;
  • body gestures that might be interpreted incorrectly and lead to an “inner talk” to figure out what was happening;
  • physical distance that might be perceived as “too intimate” or “a little frightening”;
  • different attitudes towards time that could interrupt class function or group work; and
  • certain religious rituals that would require program rescheduling.

제가 질문을 했더니 그녀는 저를 붙잡고 제 귀에 대고 말하기 시작했습니다. [...] 그 순간 저는 배우고 이야기하는 것보다 그녀에게서 도망치는 데 더 관심이 있었습니다. [X의 문화는 상당히 개방적이고 매우 친밀합니다... 그리고 저는 포옹, 특히 이성과의 포옹을 좋아하지 않습니다(웃음) 포옹은 종교적인 것에 가깝기 때문입니다. 그래서 아마 그녀는 몰랐을 것이고 그것은 그녀의 사랑의 제스처 였지만 그 사랑의 제스처는 저에게 효과가 없었습니다. (F-10)
I asked her a question, and she just grabbed me and started talking in my ear. […] At that moment, I was more interested in getting away from her, instead of learning and talking. [X’s] culture is quite open, very intimate… And, I am not into hugging, especially with the opposite gender (laughs), because it’s more of a religious thing. So, probably she didn’t realize, and it was her gesture of love, but that gesture of love did not work for me. (F-10)

펠로우들은 고국으로 돌아간 후에도 문화적 차이로 인해 어려움을 겪었다고 말했습니다. 거의 모든 펠로우가 자신의 교육기관에서 FAIMER의 대화형 교수법을 사용하려고 했지만, 일부 펠로우들은 새로운 이니셔티브를 성공적으로 실행하지 못했습니다. 한 펠로우는 자신이 "위험을 감수하고 비판적 사고와 사회적 상호작용을 촉진하기 위해 비학문적 개인적 문제에 대해 성찰하고 이야기하는 데 보호된 시간을 할애하는 '학습 서클'과 같은 활동을 설계할 용기가 있는지 확신할 수 없었다"고 말했습니다. 참가자들은 또한 위계 존중, 협업, 문서화와 같은 문화적 관행이 각자의 가정 환경에서 프로젝트의 성공과 지속 가능성에 영향을 미칠 수 있다고 믿었습니다. 
Fellows also indicated some challenging cultural differences once they returned to their home countries. While almost all Fellows intended to use FAIMER’s interactive teaching methods in their own institutions, some were not able to successfully implement new initiatives. One Fellow specified that he was not sure he had “the courage to take a risk” and design an activity like “learning circles,” in which protected time was devoted to reflection and talking about non-academic personal issues to promote critical thinking and social interactions. Participants also believed that cultural practices, such as respecting hierarchy, collaboration, and documentation, could influence their projects’ success and sustainability in their home settings.

우리는 매우 즉흥적입니다. 아이디어가 떠오르면 이렇게 말합니다: "해보자. 멋지네요!" (웃음). 그리고 프로그램은 포기되거나 전혀 개발되지 않습니다. 사람들이 준비와 생각 없이 잘못된 방식으로 시작했기 때문입니다. (F-7)
We are very spontaneous. We have just the idea, and we are like: “let’s do it. It’s great!” (laughs). And programs are abandoned or not developed at all. Because people started in the wrong way without preparing and thinking. (F-7)

펠로우와 교수진은 문화적 이질감을 해소하기 위해 세 가지 접근 방식을 사용한다고 보고했습니다:
Fellows and Faculty reported using three different approaches to address cultural dissimilarities:

적응: 몇몇 참가자들은 서로 다른 억양을 사용하거나 서로 이름을 부르는 등 특정 문화적 차이를 다루는 것이 "처음에는 조금 힘들었다"고 말했습니다. 하지만 곧 새로운 상황에 익숙해졌고 큰 어려움 없이 적응할 수 있는 방법을 찾았습니다.
Adaptation: Several participants observed that it was a “bit tough in the beginning” to deal with certain cultural variations, including different accents or addressing each other by first names. However, they quickly became accustomed to the new situation and found ways to adapt without much trouble.

교수님이나 상대방의 이름을 불러야 한다는 사실 자체가 첫날에는 저에게 충격이었습니다. 적응하는 데 시간이 좀 걸렸어요. (F-11)
The fact that we had to speak to our professors or whoever they were by their first name itself was a shock for me on the first day. It took some time to adjust to that. (F-11)

한 FAIMER 교수진이 FAIMER에서 펠로우십을 시작했을 때의 이야기를 들려주었습니다. 그는 펠로우들이 팀 빌딩 연습을 위해 외부로 끌려갔을 때 "문화적 충격"이라고 표현한 "약간 굴욕적인" 경험 때문에 첫날 프로그램을 그만두기로 결심했습니다. 하지만 그는 "며칠 동안 머물면서 프로그램의 좋은 점들을 하나씩 알아가기 시작했고, 마침내 프로그램의 가치에 대해 확신을 갖게 되었다"고 합니다.
A FAIMER Faculty shared a story of the time when he started his Fellowship in FAIMER. He had decided to quit the program on the first day, because of a “little bit humiliating” experience that he described as a “cultural shock” when the Fellows were taken outside for a team-building exercise. But then, he “stayed for a few days, started to pick up the nice things in the program, and finally, was convinced about the program’s value”.

수정: 대부분의 경우, 참가자들은 불안감이나 불확실성에 대응하여 평소의 관행을 수정하고 해결책을 찾았습니다. 예를 들어, 펠로우가 자원봉사를 하거나 의견을 표현하는 것이 어렵다고 느꼈을 때, FAIMER 교수진은 펠로우에게 직접 말하거나 펠로우가 게시판에 질문을 적어두면 나중에 교사가 익명으로 답변하는 형식인 '주차장'을 사용하는 등 다른 접근 방식을 사용했습니다. 다른 예로는 더 천천히 말하기, 종교적 관습에 맞춰 프로그램 일정 변경하기, 교육 내용 수정하기 등이 있습니다.
Modification: In most cases, participants modified their usual practices and found solutions in response to unease or uncertainty. For instance, when Fellows found it difficult to volunteer or express their opinions, FAIMER Faculty used alternative approaches, such as addressing the Fellow directly or using “parking lots”, a format in which Fellows noted their questions on a board for teachers to address anonymously later. Other examples included speaking more slowly, shifting program schedules to accommodate religious practices, and modifying educational content.

저는 첫해에 피드백을 바탕으로 리더십 커리큘럼을 변경했습니다. 우리는 상당히 전통적인 하버드 비즈니스 스쿨의 리더십을 가르치고 있었는데, [X]의 일부 학생들에게는 공감을 얻지 못했습니다... 우리는 조정하고, 국제적인 그룹을 구성하고, 리더십 커리큘럼을 다시 만들었습니다. (FF-4)
I changed the leadership curriculum based on feedback in the first year. We were teaching fairly traditional Harvard Business School leadership, and it was not resonating with some from [X]… We adjusted, put together an international group, and redid the leadership curriculum. (FF-4)


중재: 중재는 덜 일반적이지만 그럼에도 불구하고 효과적인 접근법으로, 다른 사람이 상호 이해와 합의를 이끌어내기 위해 옹호자 역할을 하는 것입니다. 어떤 상황에서는 대화에 참여한 두 사람이 문화적 차이의 존재를 인식하지 못하고 있을 때, 제3의 사람이 오해가 있음을 깨닫고 도움을 주기 위해 나섰습니다. 다른 상황에서도 펠로우들은 인식된 차이를 숨기고 싶지 않았지만 직접 개입하는 대신 다른 사람에게 옹호해 달라고 요청하는 것을 선호했습니다.
Mediation: Mediation was a less common but nonetheless effective approach in which another person acted as an advocate to bring about mutual understanding and agreement. In one situation, while two individuals involved in a conversation were not aware of the existence of a cultural disparity, a third person realized that there was a misunderstanding and stepped forward to help. In other situations, Fellows did not want to hide a perceived difference but preferred to ask someone else to advocate for them instead of getting involved directly.

이 문화에서는 권위에 의문을 제기하는 것처럼 보일 수 있는 질문을 할 때 그 질문과 관련된 개인적인 이야기를 들려주어 부드럽게 만듭니다. 그래서 그는 자신의 이야기를 늘어놓았는데, 저는 그가 왜 그런 이야기를 하는지 이해하지 못했고, 그가 저에게 질문하는 것도 이해하지 못했습니다. 하지만 그곳의 문화를 잘 아는 몇몇 교직원이 저에게 와서 "상사와 마주치는 것이 걱정되나 봐요"라고 말했고, 저는 "네, 맞아요!"라고 대답했습니다. 그렇게 해서 그 문제를 해결할 수 있었습니다. (FF-4)
In their culture, when you are asking a question that might look like you’re questioning authority, you tell a personal story around the question to soften it. So, he told a whole story, and I didn’t understand why he was telling the story, and I didn’t understand that he was asking me a question. But there were some faculty members there who knew the culture and were able to come over to me and say,” He is worried about confronting his boss,” and I said, “Oh, yes, yes!” Then, I was able to address the question. (FF-4)

프로그램 및 직업 문화에 따른 완화
Mitigation by program and professional cultures

FAIMER 교수진과 펠로우는 맥락의 중요성에 주목하고, 문화적 차이가 교육과 학습에 미치는 영향이 프로그램의 문화(즉, 가치, 규범, 신념)와 참가자들의 공통된 직업적 배경 및 보건 전문직 교육자로서의 경험에 의해 완화되는 것을 관찰했습니다
FAIMER Faculty and Fellows noted the importance of context and observed that the influences of cultural differences on teaching and learning were mitigated by the program’s culture (i.e., values, norms, and beliefs) and participants’ common professional backgrounds and experiences as health professions educators.

프로그램 문화펠로우들은 주최자가 존중하는 의사소통에 대한 기본 규칙을 정하고, 펠로우들이 문화에 대해 명시적으로 이야기하도록 장려하며개인적인 이야기를 나눌 수 있는 보호된 시간을 마련함으로써 문화적 차이에 대한 인식을 보여주는 문화적으로 민감한 프로그램에 참가했다고 긍정했습니다. 또한 교수진은 팀워크를 위해 이질적인 그룹을 구성하는 것도 활용했습니다. 종교 의식을 위한 특별한 장소를 제공하고 다양한 식사를 제공하는 것도 문화적 다양성을 지원하는 또 다른 예입니다. 펠로우들은 FAIMER에서 자신들의 문화가 "놀라울 정도로 인정"되고 "대표"되고 있다고 믿었습니다. 두 명의 교수진은 FAIMER가 문화적 차이를 파악하고 해결하는 과정을 지속적으로 수행함으로써 안전하고 편견 없는 환경을 조성했다고 덧붙였습니다. 
Program culture: Fellows affirmed that they had entered a culturally sensitive program where organizers showed awareness to cultural differences by setting ground rules about respectful communications, encouraging Fellows to talk explicitly about culture, and scheduling protected time for sharing personal stories. Faculty also took advantage of forming heterogeneous groups for teamwork. Providing special places for religious rituals and making different meals available were other examples of how cultural diversity was supported. Fellows believed that their cultures were “amazingly acknowledged” and “represented” in FAIMER. Two Faculty added that FAIMER created a safe, non-judgmental environment by undertaking a continuous process of identifying and addressing cultural differences.

우리가 가장 먼저 하는 일 중 하나는 이 기본 규칙 세션에서 우리가 여기서 배우려고 하는 것에 동의하는 것입니다. 존중, 경청, 다양성 존중, 외부 호칭, 기밀 유지 등이 우리가 가장 먼저 합의한 사항입니다. (FF-1)
One of the first things that we have is this ground rules session to agree that here we are trying to learn. These are the first things that we agree on: respect, listen, diversity is good, titles outside, and confidentiality. (FF-1)

프로페셔널 문화모든 FAIMER 펠로우와 교수진은 광범위한 보건 전문직 교육자 커뮤니티의 일원으로서 비슷한 관심사와 가치관, 익숙한 경험과 관행을 공유했습니다. 한 펠로우는 "변화에 대한 교수진의 저항"이 자국만의 문화적 문제가 아니라 전 세계 어디에서나 일어날 수 있는 일이라는 것을 깨달았을 때가 "펠로우십 기간 중 가장 만족스러운 순간"이었다고 말했습니다. 참가자들은 다양한 방식으로 보건 전문직 교육과 관련된 '공통점'을 찾아내어 차이점을 최소화하고 각국의 직업 문화에 반영된 유사점에 집중할 수 있었습니다. 
Professional culture: All FAIMER Fellows and Faculty were members of the broader community of health professions educators and shared similar interests and values as well as familiar experiences and practices. One Fellow said that it was “the most satisfying moment during the Fellowship” when he realized that the “resistance of faculty members toward change” was not a cultural problem specific to his country but could happen anywhere around the world. In diverse ways, participants identified a “common ground” related to health professions education that enabled them to minimize differences and focus on similarities reflected in their professional culture.

교수진 부족, 자원 부족, 학생 수 과다, 전통적인 교육 및 학습 방식, 연구에 대한 높은 관심 등 보건 과학 교육은 어디를 가나 동일한 문제를 공유하고 있습니다. 이러한 문제는 거의 모든 사람이 공유하고 있습니다. (F-14)
Health science education is the same everywhere you go, we share the same problems: lack of faculty, lack of resources, too many students, a traditional way of teaching and learning, high interest in research. Those were shared by almost everyone. (F-14)

토론
Discussion

이 연구는 보건 전문직의 교수진 개발에서 자주 간과되는 측면, 즉 문화가 IFDP에 미치는 영향을 프로그램 참가자와 교수진의 시각에서 살펴봤습니다. 문화적 다양성이 교수와 학습에 미치는 영향을 살펴본 결과, 문화적 차이가 학습에 장애가 되는 것이 아니라 오히려 문화를 배우고 네트워크를 구축할 수 있는 기회로 작용한다는 사실을 발견했습니다. 동시에 특정 문화적 차이는 불안감과 불확실성을 야기하여 적응, 수정, 중재라는 세 가지 건설적인 반응으로 이어졌습니다. 참가자들의 관점은 프로그램 문화와 직업 문화의 영향을 받았기 때문에 맥락도 중요했습니다. 
This study explored a frequently overlooked aspect of faculty development in the health professions – the influence of culture on IFDPs – from the lens of program participants and faculty. Exploring the influences of cultural diversity on teaching and learning, we found that cultural differences were not a barrier to learning; rather, they acted as an opportunity to learn about culture and build networks. At the same time, certain cultural dissimilarities produced a sense of unease and uncertainty, which led to three constructive responses: adaptation, modification, or mediation. Context also mattered, as participants’ perspectives were affected by the program culture as well as their professional culture.

첫 번째 주제는 문화적 차이가 학습에 방해가 되지 않는다는 것을 보여주었습니다. 펠로우들은 FAIMER의 콘텐츠와 학습자 중심의 다양한 접근 방식에 대해 매우 높이 평가했습니다. 이들은 소그룹 토론에 적극적으로 참여하고 프로젝트를 완성했으며, 본국에서도 유사한 방법을 적용할 수 있도록 영감을 받았습니다. 이러한 결과는 TLT의 중요한 구성 요소인 관점을 바꾸고 변화의 주체가 되려는 참가자들의 의지를 강조했습니다[25]. 또한 비판적 성찰, 변증법적 담론, '안전한' 환경에서 변화할 수 있는 능력의 가치를 강조했습니다. 위에서 보고한 바와 같이, 이러한 변화 과정의 예로는 다른 문화권 동료의 행동에 대한 성찰, 개인의 가정과 행동을 더 잘 이해하기 위해 해당 문화에 익숙한 동료 및 교수진과의 토론 등이 있습니다. 이전의 여러 연구에서 위계질서에 대한 복종과 연공서열에 대한 존중과 같은 특정 문화적 측면이 참가자의 학습에 영향을 미쳤다고 보고한 바 있지만[30, 31, 32], 본 연구 결과는 다양한 국가에서 실시된 미국 기반의 IFDP에 대한 보고서[33, 34, 35, 36]와 일치하며 성공적인 결과를 도출했습니다. 우리의 연구 결과는 성취 성과를 개인의 기대 관련 신념 및 과제 가치 신념과 연결시키는 기대 가치 이론으로도 설명할 수 있습니다[37]. 우리 연구에서 펠로우들은 명성이 높은 미국의 명문 프로그램에서 "가치 있는 결과를 기대"하고 있었을 가능성이 높습니다. 또한 펠로우들은 이 프로그램이 자기계발에 대한 내적 신념(내재적 가치)에 부합할 뿐만 아니라 본국에서의 잠재적 경력 발전과 같은 외적 혜택(효용적 가치)의 가능성을 높이는 등 자신의 직업적 성장에 기여할 것이라고 믿었을 가능성도 있습니다. 다양한 방식으로, 펠로우들의 학습 동기는 인식된 문화적 격차와 관련된 문제보다 우선시되었을 수 있습니다
Our first theme showed that cultural differences did not impede learning. Fellows were very appreciative of the FAIMER content and variety of learner-centered approaches. They actively participated in small group discussions, completed their projects, and were inspired to apply similar methods in their home countries. This finding highlighted participants’ willingness to change perspectives and become change agents, important components of TLT [25]. It also underscored the value of critical reflection, dialectical discourse, and the ability to change in a “safe” environment. As reported above, examples of this transformative process included reflections on the behaviors of colleagues from other cultures and discussions with colleagues and faculty members familiar with that culture to better understand personal assumptions and behaviors. While several previous studies have reported that specific cultural aspects, like obedience to hierarchy and respect for seniority, influenced participants’ learning (and at times a tendency to prefer a more traditional lecture-dominated program) [30, 31, 32], our findings are consistent with reports on an established US-based IFDP which was delivered across a variety of countries [33, 34, 35, 36], with successful results. Our findings might also be explained by expectancy-value theory, which links achievement performance to individuals’ expectancy-related and task-value beliefs [37]. It is likely that in our study, Fellows were “expecting valuable outcomes” from a prestigious American program with a strong reputation. It is also possible that Fellows believed that this program would contribute to their professional growth, which not only resonated with their internal beliefs about self-improvement (intrinsic value) but would also increase the probability of external benefits such as potential career advancement in their home countries (utility value). In diverse ways, Fellows’ motivation to learn may have overridden challenges related to perceived cultural disparities.

두 번째 주제에서 언급했듯이, 일부 FAIMER 펠로우들은 주로 서로 다른 대인 커뮤니케이션 스타일에서 비롯된 특정 문화적 차이로 인해 불안과 불확실성을 경험했습니다. 다른 나라로 이전한 미국 기반 프로그램에서 참가자들의 정서적 자제력, 침묵, 직접적인 눈맞춤 회피로 인한 어려움에 직면했을 때도 비슷한 결과가 보고되었습니다[13]. 이번 연구에서는 이러한 문제에 대한 세 가지 건설적인 반응(적응, 수정, 중재)을 확인함으로써 이러한 증거를 추가했습니다. 

  • 적응은 펠로우가 더 자주 언급하는 반면, 
  • 수정은 FAIMER 교수진이 더 많이 사용했으며, 
  • 제3자의 중재는 펠로우와 교수진 모두 긴장을 해소하거나 문화적 충돌을 피하기 위한 효과적인 전략으로 확인되었습니다. 

TLT[25]는 사람들이 글로벌 인식과 역량을 개발하는 방법을 설명하는 데 유용성이 있는 것으로 나타났는데[21], 개인이 자신에게 익숙하지 않은 상황에 처하게 되면 이를 거부하거나 대부분의 경우 적응한다고 주장하며, 본 연구에서도 볼 수 있습니다. 비판적 성찰과 변증법적 담론도 이러한 변화 과정에 도움이 되었습니다.
As noted in our second theme, some FAIMER Fellows experienced unease and uncertainty due to certain cultural differences, primarily those emanating from different interpersonal communication styles. A similar finding was reported when a US-based program, transferred to another country, encountered challenges resulting from participants’ emotional self-control, silence, and avoidance of direct eye contact [13]. Our study adds to this evidence by identifying three constructive responses (adaptation, modification, and mediation) to these possible challenges.

  • While adaptation was mentioned more frequently by Fellows,
  • modification was used more by FAIMER Faculty, and
  • mediation by a third person was identified as an effective strategy for both Fellows and Faculty to resolve tensions or avoid cultural confrontation.

TLT [25], which has been shown to have utility in describing how people develop global awareness and competence [21], asserts that when an individual is involved in a situation that is unfamiliar to them, they will either reject it or, in most cases, adapt, as seen in our study. Critical reflection and dialectical discourse also helped this transformative process.

또한 연구 결과가 특정 프로그램과 직업 문화 속에서 이루어졌다는 점에 주목했습니다. FAIMER는 TLT의 원칙에 따라 신중하게 설계된 커리큘럼, 문화적으로 민감한 환경, 반응이 빠른 교수진, 동기 부여가 높은 학습자가 특징입니다. 이러한 (그리고 다른) 노력들이 문화적 차이에 대한 네트워크 구축과 솔루션 지향적 대응뿐만 아니라 혁신적인 학습을 가능하게 하여 일부 연구 결과에 기여한 것으로 보입니다. 또한 참가자들이 모두 보건 전문직 교육자였기 때문에 그들의 직업 문화가 문화적 차이를 인식하고 해결하는 방식에 영향을 미쳤을 수 있습니다. 우리는 다른 연구에서도 보건 전문직 교육자들이 학문적 배경, 경험, 가치, 신념, 요구 측면에서 공통점을 발견했으며, 다른 연구에서도 이를 보고한 바 있습니다[3, 33, 35]. 많은 대학이 커리큘럼 개혁을 수행하고 학생 중심 방식으로 전환하고 있고[16, 35], 글로벌 표준 및 인증 프로세스가 점점 더 인정받고 수용되고 있기 때문에[38], 교육 설계 및 전달의 원칙이 일반적으로 잘 받아들여지는 것은 놀라운 일이 아닙니다[16, 38]. 참가자들의 전문적 배경과 '공통점'을 찾고 행동에 전념하려는 열망은 비판적 성찰과 변증법적 담론에 대한 프로그램의 강조와 함께 문화적 차이를 초월하는 데 도움이 되었을 수 있으며[18], 이는 결과적으로 향후 긍정적인 행동과 의미 있는 관계로 이어질 수 있습니다[19].
We also noted that our findings were situated in a specific program and professional culture. FAIMER is informed by principles of TLT and characterized by a carefully designed curriculum, a culturally sensitive environment, responsive Faculty, and highly motivated learners. It is likely that these (and other) efforts contributed to some of our findings, enabling transformative learning as well as network building and solution-oriented responses toward cultural differences. Furthermore, since our participants were all health professions educators, their professional culture might have influenced how they perceived and addressed cultural differences. We found, and other studies have also reported, commonalities among health professions educators in terms of disciplinary backgrounds, experiences, values, beliefs, and needs [3, 33, 35]. As many universities are conducting curricular reforms and moving toward student-centered methods [16, 35], and global standards and accreditation processes are increasingly recognized and accepted [38], it is not surprising that principles of instructional design and delivery are generally well-received [16, 38]. Participants’ professional backgrounds and desires to seek “common ground” and commit to action, together with the program’s emphasis on critical reflection and dialectical discourse, may have helped to transcend cultural differences [18], which in turn, can lead to positive future action and meaningful relationships [19].

강점과 한계
Strengths and Limitations

우리가 FAIMER 펠로우십을 선택한 이유는 다양한 국가의 펠로우와 교수진으로 구성된 다문화 환경을 제공하고, 종단적 프로그램 내에서 현장 및 온라인 교육으로 구성되어 있어 다양한 교육 방법을 탐색할 수 있었기 때문입니다. FAIMER 교수진, 현재 펠로우, 이전에 펠로우십을 수료한 펠로우를 인터뷰하여 데이터 출처를 삼각 측량하고 신뢰도를 높일 수 있었습니다. 하지만 이 연구는 하나의 IFDP만 조사했고 참여가 자발적으로 이루어졌기 때문에 연구 결과를 다른 환경에 그대로 적용하기는 어려울 수 있습니다. 참가자들에게 과거에 있었던 경험을 회상하도록 요청했기 때문에 회상 편향도 또 다른 문제였습니다. 마지막으로, 교육과 학습의 일부 측면은 펠로우십 기간 동안 상호작용을 관찰했더라면 더 잘 이해할 수 있었을 것입니다. 
We chose the FAIMER Fellowship because it offered a multicultural environment with Fellows and Faculty from different countries and was composed of onsite and online training within a longitudinal program, thus enabling the exploration of diverse instructional methods. As we interviewed FAIMER Faculty, current Fellows, and Fellows who had previously completed their Fellowship, we were able to triangulate our data sources and enhance trustworthiness. However, this study investigated only one IFDP, and participation was voluntary; hence, findings might not be transferable to other settings. Recall bias was another challenge, since we asked participants to recall experiences that had occurred in the past. Lastly, some aspects of teaching and learning might have been better understood by observing interactions during the Fellowship.

교육 및 연구에 대한 시사점
Implications for Education and Research

본 연구의 결과는 문화가 IFDP에 미치는 영향에 대한 이해를 증진하고 교수진 개발 활동을 개선하고 향후 연구 방향을 제시할 수 있는 제안을 가능하게 합니다.

The findings of this study advance our understanding of the influence of culture on IFDPs and enable us to make suggestions for enhancing faculty development activities and recommending future research directions.

교수진 개발자와 IFDP 참가자를 위한 교육적 시사점을 표 2에 요약했습니다. 연구적 관점에서 볼 때, 교육자들은 다양한 지역의 IFDP를 조사해 볼 것을 권장합니다. 예를 들어, FAIMER 지역 연구소는 'home' 환경에서 문화의 영향력을 탐구하는 데 유용한 데이터 소스가 될 수 있습니다. 또한 민족지학과 같은 다른 방법을 사용하면 전문직 및 프로그램 문화의 영향과 상호 작용에 초점을 맞추어 IFDP에서 문화가 교수와 학습에 미치는 영향에 대해 더 많은 통찰력을 얻을 수 있습니다. TLT의 렌즈를 통해 교수진 개발에 대한 추가 탐구도 가치가 있을 것입니다. 
We have summarized the educational implications for faculty developers and participants in IFDPs in Table 2. From a research perspective, we encourage educators to examine IFDPs in different locations; for example, FAIMER Regional Institutes could be a valuable data source for exploring the influence of culture in “home” settings. Additionally, using other methods such as ethnography could provide more insights into the influence of culture on teaching and learning in IFDPs, with a focus on the influence – and interaction – of professional and program cultures as well. Further exploration of faculty development through the lens of TLT would also be worthwhile.

결론
Conclusion

IFDP에서 보건 전문직 교육자의 문화적 다양성은 학습에 장애가 되지 않는 것으로 보입니다. 인식되는 특정 문화적 차이는 미묘한 문제를 야기할 수 있지만, 긴장을 피하기 위해 건설적으로 관리되는 경우가 많습니다. 프로그램 문화와 직업 문화는 때때로 문화적 신념, 가치, 규범보다 우선할 수 있습니다.
The cultural diversity of health professions educators in an IFDP does not seem to be a barrier to learning. Certain perceived cultural differences may cause subtle challenges, but they are often managed constructively to avoid tension. Program culture and professional culture may, at times, override cultural beliefs, values and norms.

인간 본성은 같지만 우리는 문화에 따라 적응합니다. 우리는 그것을 우리에게 두른 다음 특정 문화에 가장 적합한 방식으로 행동하려고 노력합니다. 인간 본성의 개념, 치료자의 개념, 스승의 개념 등 많은 유사점이 있습니다. 이것들은 보편적인 것들 중 일부입니다. (F-10)
We have that same human nature, but then we adapt according to the culture. We wrap it around ourselves, and then we try to act accordingly what is most suitable in that specific culture. There are so many similarities, the concept of human nature, the concept of healer, the concept of teacher. These are some of the universal things. (F-10)


Perspect Med Educ. 2023 Jan 4;12(1):1-11. doi: 10.5334/pme.31. eCollection 2023.

Understanding and Embracing Culture in International Faculty Development

Affiliations collapse

1Institute of Health Sciences Education, Faculty of Medicine and Health Sciences, McGill University, Montreal, Canada.

2FAIMER (The Foundation for Advancement of International Medical Education and Research), member of Intealth, and Director FAIMER Global Programs, FAIMER Philadelphia, USA.

3The Network: Towards Unity for Health (TUFH) and Founding Co-Director Emeritus of the FAIMER Institute, USA.

4Family Medicine and Health Sciences Education, the former director of the Institute of Health Sciences Education, and the Richard and Sylvia Cruess Chair in Medical Education, Faculty of Medicine and Health Sciences, McGill University, Montreal, Canada.

PMID: 36908745

PMCID: PMC9997115

DOI: 10.5334/pme.31

Free PMC article

 

Abstract

Introduction: Research on international faculty development programs (IFDPs) has demonstrated many positive outcomes; however, participants' cultural backgrounds, beliefs, and behaviors have often been overlooked in these investigations. The goal of this study was to explore the influences of culture on teaching and learning in an IFDP.

Method: Using interpretive description as the qualitative methodology, the authors conducted semi-structured interviews with 15 Fellows and 5 Faculty of a US-based IFDP. The authors iteratively performed a constant comparative analysis to identify similar patterns and themes. Transformative Learning Theory informed the analysis and interpretation of the results.

Results: This research identified three themes related to the influences of culture on teaching and learning. First, cultural differences were not seen as a barrier to learning; instead, they tended to act as a bridge to cultural awareness and network building. Second, some cultural differences produced a sense of unease and uncertainty, which led to adaptations, modifications, or mediation. Third, context mattered, as participants' perspectives were also influenced by the program culture and their professional backgrounds and experiences.

Discussion: The cultural diversity of health professions educators in an IFDP did not impede learning. A commitment to future action, together with the ability to reflect critically and engage in dialectical discourse, enabled participants to find constructive solutions to subtle challenges. Implications for faculty development included the value of enhanced cultural awareness and respect, explicit communication about norms and expectations, and building on shared professional goals and experiences.

Keywords: Faculty development; cultural diversity; culture; transformative learning theory.

성찰을 통해 배우고 적응하기: 의과대학생의 환자 임종 경험 탐색(BMC Med Educ, 2019)
Learning and coping through reflection: exploring patient death experiences of medical students
Travuth Trivate1, Ashley A Dennis2, Sarah Sholl3 and Tracey Wilkinson4*

배경
Background

의학교육 문헌에 따르면 임상 실습 중 환자의 사망은 의대생에게 정서적으로 중요한 사건이 될 수 있다고 합니다[1,2,3,4,5,6,7,8,9]. 이러한 감정은 두려움[1], 죄책감[1,2,3,4], 슬픔[2,3,4,5,6,7,8], 충격[2,7,8,9], 분노[3,8]와 같은 부정적이거나 고양된 느낌[2,6]과 같은 긍정적인 것일 수 있습니다. 환자의 죽음에 대한 첫 경험은 가장 기억에 남는 것으로 보고되었으며[7], 감동적이고 고통스러우며 생각과 기억이 지속되는 것으로 나타났습니다[6]. 전임상에서 임상으로 넘어가는 과도기는 환자의 죽음으로 인한 강한 감정을 고조시키는 요인입니다. 이러한 과도기는 의대생이 환자 사망을 처음 경험하는 경우 자신의 역할[5]과 책임[9]을 파악하는 데 어려움을 겪을 수 있습니다. 환자의 죽음에 직면했을 때 스트레스에 대처하거나 감정을 조절하는 방법[10]을 배우는 것은 의대생이 반드시 배워야 할 핵심 기술입니다[11,12,13]. 따라서 이러한 초기 경험은 잠재적으로 어렵지만 중요한 학습의 기회를 제공합니다.
During clinical placements, medical education literature suggests that the death of a patient can be an emotionally significant event for medical students [1,2,3,4,5,6,7,8,9]. These emotions can be negative, such as fear [1], guilt [1,2,3,4], sadness [2,3,4,5,6,7,8], shock [2, 7,8,9] and anger [3, 8], or be positive, such as feeling uplifted [2, 6]. First experiences with patient death have been reported to be the most memorable [7], moving, painful, and persistent in thoughts and memories [6]. The transition period from preclinical to clinical years is a factor that heightens the strong emotions evoked by patient death. This transition period may increase the difficulty for medical students in identifying their roles [5] and responsibilities [9] when encountering their first experiences of patient death. Learning to cope or to regulate emotions under stress [10] when faced with patient death is a key skill that medical students must learn [11,12,13]. Therefore, although potentially difficult, these early experiences also provide significant opportunities for learning.

성찰: 임상 실습 경험을 통해 배우는 방법
Reflection: a way to learn from experiences in clinical practices

이러한 학습 경험을 탐구하는 한 가지 방법은 경험적 학습 이론과 성찰의 렌즈를 통해 살펴보는 것입니다. 경험적 학습 이론은 경험을 통해 지식이 형성될 수 있다는 점에서 임상 기반 학습을 지원합니다[14]. 콜브가 정의한 경험적 학습 주기에서 성찰은 기본 구성 요소입니다 [15, 16]. Atkins 등[17]은 성찰에는 첫째, 불편한 감정과 생각의 식별, 둘째, 그러한 감정과 생각에 대한 비판적 분석, 셋째, 학습에 대한 새로운 관점의 개발이 포함된다고 제안했습니다[17]. 따라서 성찰은 개인이 대처하는 법을 배우는 방법뿐만 아니라 환자의 죽음과 같은 어려운 임상 경험을 통해 더 일반적으로 배우는 방법을 탐구하는 데 유용한 메커니즘이 될 수 있습니다. 2018년 졸업생을 대상으로 한 일반의협의회 결과[18]에 따르면, 성찰은 의사가 임상에서 겪는 어려움에서 회복하는 데 필요한 대처 전략으로 간주됩니다. 
One way to explore these learning experiences is through the lens of experiential learning theory and reflection. Experiential learning theory supports clinical-based learning insofar as it suggests knowledge can be shaped through experiences [14]. In the experiential learning cycle as defined by Kolb, reflection is a fundamental component [15, 16]. Atkins et al. [17] proposed that reflection involves

  • first, the identification of uncomfortable feelings and thoughts;
  • second, a critical analysis of those feelings and thoughts; and
  • third, the development of a new perspective on learning [17].

Therefore, reflection may be a helpful mechanism to explore not only how individuals learn to cope, but also how they learn more generally from difficult clinical encounters such as patient death. According to General Medical Council outcomes for graduates 2018 [18], reflection is regarded as a coping strategy necessary for physicians to recover from difficulties in clinical practice.

환자 사망 경험 시 대처 전략
Coping strategies when experiencing patient death

의대생이 채택하는 다양한 대처 전략이 문헌에서 확인되었습니다(표 1 참조). 그러나 학습이라는 렌즈를 통해 대처 전략을 탐구하는 의학교육 연구는 제한적입니다. 한 가지 예로 Kelly 등[2]은 의대생들이 환자 사망에 대한 초기 경험을 되돌아보면서 대처 전략과 학습 경험을 모두 탐구했습니다. 예를 들어, 이들은 성찰을 통해 드러나는 학생들의 전문적인 정서적 반응과 학문적 지식에 대해 논의합니다. 
Various coping strategies adopted by medical students have been identified in the literature (see Table 1). However, there is limited medical education research that explores coping through the lens of learning. One example is Kelly et al. [2], who explore both the coping strategies and the learning experiences of medical students as they reflect on their early experiences of patient death. For example, they discuss both students’ professional emotional responses and academic knowledge that emerge through reflection.

의료진 및 교수진의 지원
Support from medical team and faculty

표 1에서 도출할 수 있는 한 가지 주요 특징은 일반적인 대처 전략이 누군가에게 이야기하는 것(광범위한 문헌에서는 '지원 구하기'[10])이라는 점입니다. 누군가, 특히 주치의나 컨설턴트와 대화하는 것은 학생들에게 토론하고 보고할 수 있는 기회를 제공합니다[2, 8]. 환자 사망 후의 토론은 반드시 학생이 주도적으로 시작해야만 대처에 도움이 되는 것은 아닙니다. 일부 연구에서는 교직원이 적극적으로 주도한 토론이 학생에게 특히 도움이 되었다고 보고했습니다[3, 9]. 토론은 학생들의 정서적 고통(예: 두려움, 불안, 죄책감)을 줄이고[20], 종결감을 가져다주는 것으로 밝혀졌습니다[5]. 중요한 것은 경험적 학습의 관점에서 볼 때, 문헌에 따르면 환자의 죽음에 대한 토론은 학생들이 학문적 관점, 정서적 조절 관점, 직업적 정체성 개발에서 성찰하고 학습하는 데 도움이 된다고 합니다[2, 5]. 예를 들어, 라타나웡사 등[5]은 경험이 많은 다른 팀원들이 환자 사망에 대해 어떻게 반응했는지를 학습함으로써 의대생들이 자신의 반응과 비교하고 적절한지 평가할 수 있다는 사실을 발견했습니다. 다른 문헌에 따르면, 직업적 정체성 발달 측면에서 환자 사망 후의 토론은 학생들이 "감정과 전문성 사이의 어려운 긴장에 참여"하는 데 도움이 되었다고 합니다([2], 426쪽). 다시 말해, 팀 상호작용은 성숙하고 전문적인 대응을 개발하는 데 도움이 되는 상호작용적이고 '피드포워드' 지향적인 과정으로서 학생들의 반성성을 향상시킵니다.
One main feature that can also be drawn from Table 1 is that a common coping strategy is speaking to someone (‘support seeking’ in the broader literature [10]). Talking to somebody, especially attending physicians or consultants, provides students with a chance to discuss and debrief [2, 8]. A discussion after patient death does not always need to be initiated by the student to be of value for coping. Some studies have reported that a discussion that was actively initiated by the staff member was especially helpful for students [3, 9]. Discussion has been found to lessen students’ emotional distress (e.g. fear, anxiety, guilt) [20] and bring a sense of closure [5]. Importantly, from an experiential learning perspective, the literature suggests that a discussion about patient death helps students reflect and learn from an academic perspective, from an emotional regulation perspective, and in the development of a professional identity [2, 5]. For example, Ratanawongsa et al. [5] found that learning how other more experienced team members reacted to a patient death enabled medical students to compare their own responses and assess whether they were appropriate. Other literature suggests that, in terms of the development of professional identity, a discussion following patient death has supported students to “engage in the difficult tension between emotion and professionalism” ([2], p.426). In other words, team interaction enhances students’ reflexivity as an interactive and ‘feed-forward’ orientation process to support the development of mature and professional responses.

문헌의 풍부함을 높이기 위해 본 연구는 성찰의 렌즈를 통해 학생들의 환자 죽음에 대한 경험을 탐구합니다. 이러한 경험에는 대처 전략, 환자 사망 후 교수진의 지원, 그리고 이러한 경험과 학습 간의 관계가 포함됩니다. 또한 환자 사망 후 학생들을 지원하는 시스템에 대한 권장 사항을 제공합니다. 우리의 연구 질문은 다음과 같습니다: 
To enhance the richness of the literature, our study explores students’ experiences with patient death through the lens of reflection. These experiences include coping strategies, support from faculty following patient death and the relationship between these experiences and learning. Furthermore, we provide recommendations for the systems supporting students following patient death. Our research questions were:

임상 실습 중 환자 사망에 대한 경험을 성찰함으로써: 
Through reflecting on their experiences with patient death during clinical placements:

  • 의대생들은 이러한 경험에 어떻게 대처하고 이를 통해 무엇을 배우는가?
  • 병동 직원과 의과대학의 지원이 이러한 경험에 대처하고 배우는 데 어떻게 도움이 되는가?
  • 환자 사망 후 학생들을 가장 잘 지원할 수 있는 방법은 무엇일까요?
  • How do medical students cope with and learn from their experiences?
  • How does support from ward staff and the medical school help them cope with and learn from these experiences?
  • How can students be best supported following patient death?

방법
Methods

이론적 관점
Theoretical perspective

이 질적 연구는 사회 구성주의 인식론에 의해 뒷받침됩니다. 사회 구성주의 인식론은 지식은 본질적으로 주관적이며, 각 개인의 앎의 방식, 즉 사람들과의 상호작용을 통한 지식 구성에 따라 다양한 현실이 존재한다는 것을 의미합니다[21]. 사회 구성주의는 해석주의적 패러다임으로 사람들의 경험을 수집하고 해석하여 현상을 이해하고자 하는 연구 패러다임으로 특징지어집니다 [21]. 특정 현상은 사회-관계적 맥락에 따라 사람마다 다르게 경험할 수 있으며, 따라서 해석도 다양할 수 있습니다[22]. 예를 들어, 환자의 죽음과 관련된 현실 또는 경험은 학생과 환자, 친척, 병동 직원, 동료 등의 다양한 상호작용에 따라 다양하게 나타날 수 있습니다. 
This qualitative study is underpinned by a social constructionist epistemology. Social constructionist epistemology means that knowledge is subjective in nature, and that multiple realities exist based on each individual’s way of knowing, or knowledge construction through his/her interactions with people [21]. Social constructionism is characterized as an interpretivist paradigm, which is a research paradigm that seeks to understand phenomena through gathering and interpreting people’s experiences [21]. A particular phenomenon can be experienced differently by different people based on their social-relational contexts; therefore, the interpretations can be varied [22]. For example, realities, or experiences with patient death in this regard, can be many according to different interactions between students and patients, relatives, ward staff, colleagues, etc.

방법론: 내러티브 탐구
Methodology: narrative inquiry

내러티브 탐구는 개인의 내러티브 또는 이야기를 분석하여 특정 경험을 이해하는 질적 방법론입니다[23]. 내러티브 탐구는 개인적 경험에 대한 학습을 위한 가장 중요한 방법론으로 여겨지기도 합니다[24]. 내러티브 탐구는 내레이터와 연구자의 관점에서 삶의 사건에 대한 해석을 다루기 때문에 해석주의 연구 패러다임 및 사회 구성주의와 일치합니다 [25]. 내러티브 기록의 본질은 과거 세계에 대한 사실적 재현이 아니라 과거, 현재, 상상된 미래를 연결하는 내레이터의 세계에 대한 해석과 구성에 있습니다 [26].
Narrative inquiry is a qualitative methodology that looks for an understanding of a particular experience through analyzing personal narratives or stories [23]. It is also seen as the first and foremost methodology for learning about personal experiences [24]. Narrative inquiry arguably aligns with an interpretivist research paradigm and with social constructionism, because it deals with an interpretation of life events, both from the narrator and the researcher perspective [25]. The essence of a narrative account is not about a truthful reproduction of the past world, but about an interpretation and a construction of the narrator’s world that connects his/her past, present, and imagined future [26].

방법: 내러티브 설문지
Method: narrative questionnaire

내러티브 데이터는 인터뷰, 자연주의적 의사소통, 현장 노트, 내러티브 글쓰기 등 다양한 방법으로 생성할 수 있습니다 [27]. 내러티브 설문지는 내러티브 글쓰기의 한 형태로, 신중하게 설계하면 스토리텔링을 통해 경험의 다양한 측면(예: 상황 설정, 감정, 기억, 행동)이 담긴 풍부한 정보를 얻을 수 있습니다[22]. Rees 등[22]은 그들의 연구에서 의대생들에게 임상 기간 동안 경험한 직업적 딜레마에 대해 질문하기 위해 내러티브 설문지를 활용했습니다. 설문지에는 참가자들이 경험한 딜레마에 관한 항목이 포함되어 있었습니다. 예를 들어, 딜레마의 간략한 본질, 장소, 관련자, 수행한 행동, 경험에 대한 느낌에 대해 질문했습니다. 각 질문 뒤에는 참가자가 답변을 작성할 수 있는 자유 텍스트 상자가 있었습니다. 이러한 형식을 사용하여 내러티브 설문지를 디자인했습니다. 

Narrative data can be generated in many ways: interviews, naturalistic communication, field notes and narrative writing [27]. A narrative questionnaire is a form of narrative writing that, when carefully designed, can yield rich information full of various aspects of an experience through storytelling (e.g. the setting of the situation, emotion, memory and action) [22]. In their research study, Rees et al. [22] utilized a narrative questionnaire to ask medical students about professional dilemmas that they had experienced during their clinical years. Their questionnaire included items around the dilemma the participants had experienced. For example, they asked about a brief essence of the dilemma, the location, the people involved, the actions done and the feelings about the experience. Each question was followed by a free-text box for the participants to write their answers. We used their format to inform the design of our narrative questionnaire.

설문지 디자인
Questionnaire design

임상 실무에서 어려운 경험에 관한 이전 연구[7, 22]의 설문 문구를 적용하여 환자 사망 후 '가장 기억에 남는' 경험을 중심으로 질문을 구성했습니다. 성찰을 촉진하고 서술에서 자기 인식을 장려하기 위해 질문과 함께 프롬프트가 제공되었습니다 [28]. 학생의 성찰을 통해 환자 사망 경험을 이해하는 것을 목표로 했기 때문에, 참가자들이 그 경험을 통해 배운 것을 묻는 문항에 포함시켰습니다. 또한 참가자들은 참가자 표본의 특성을 파악할 수 있도록 몇 가지 인구통계학적 질문도 작성했습니다. 설문지의 질문 항목은 추가 파일 1을 참조하세요. 
Adapting the questionnaire wording from previous studies concerning difficult experiences in clinical practice [7, 22], the questions were centered around the “most memorable” experience following patient death. Prompts were provided with the questions to promote reflection and encourage self-awareness in the narratives [28]. Because we aimed to understand the experience with patient death through student reflection, our prompts included what the participants had learned from the experience. Participants also completed a few demographic questions to enable characterization of our participant sample. Please see Additional file 1 for the question items in our questionnaire.

연구 절차
Study procedure

이 연구는 던디 대학교의 윤리적 승인을 받은 후 2016년 여름(2016년 6월~9월)에 걸쳐 진행되었습니다. 
After ethical approval was received from the University of Dundee, the study was conducted over the summer of 2016 (June–September 2016).

샘플링 및 모집
Sampling and recruitment

이 연구에서는 접근성과 근접성을 고려하여 피험자를 선정하는 편의표본추출법을 사용했습니다. 연구 대상은 던디 대학교 4학년과 5학년 의대생이었습니다. 연구에 참여하기 위해서는 임상 배치에서 환자 사망을 한 번 이상 경험한 적이 있어야 했습니다. '환자 사망 경험'의 정의는 상당히 광범위했습니다. 예를 들어, 학생들이 임종 중인 환자와 상호작용을 하거나 환자와 상호작용을 하다가 나중에 환자가 사망했다는 사실을 알게 된 경험이 포함될 수 있습니다. 학생들은 (1) 이메일, (2) 가상 학습 환경의 전자 공지, (3) 게시판의 실제 공지, (4) 스노우볼링(기존 참가자가 향후 참가자를 추천하거나 모집하는 방식), (5) 의과대학 소셜 네트워킹(예: 메드블로그) 등 다양한 모집 전략을 통해 참여하도록 초대받았습니다. 학생들은 연구에 대한 정보를 제공하는 초대장을 통해 온라인 설문지 링크를 따라갈 수 있었습니다. 내러티브 설문지(참가자 정보 시트가 첨부된)는 온라인 설문조사 소프트웨어인 브리스톨 온라인 서베이(BOS)에서 작성되어 배포되었습니다. 던디 대학교는 연구자들이 설문조사를 제작, 배포, 분석할 수 있는 사용하기 쉬운 플랫폼에 무료로 액세스할 수 있도록 BOS를 구독하고 있었습니다. 참가자들은 설문지를 작성함으로써 참여에 동의하는 것이며, 모든 데이터 보고서에서 기밀성과 익명성이 유지될 것임을 고지받았습니다. 또한 설문지의 디브리핑 페이지에서 지원 자료와 연구자의 연락처 정보를 제공받았습니다. 
We used a convenience sample for this study where subjects were selected because of their convenient accessibility and proximity. Our subjects were 4th and 5th year medical students at the University of Dundee. In order to participate, they needed to have had at least one experience with patient death on their clinical placements. The definition for ‘experience of patient death’ was quite broad. For example, this could have been an experience where students were interacting with a dying patient or had interacted with a patient to find out later that they had passed away. The students were invited to participate through multiple recruitment strategies, including: (1) email; (2) e-notices on virtual learning environments; (3) physical notices on notice-boards; (4) snowballing (existing participants recommend or help recruit future participants) and (5) medical school social networking (e.g. Medblogs). Through the invitation, which provided information about the study, students could follow a link to the online questionnaire. The narrative questionnaire (with the participant information sheet attached) was created and disseminated within the online survey software Bristol Online Survey (BOS). The University of Dundee had a subscription to BOS enabling researchers free access to the easy-to-use platform where researchers can produce, distribute and analyze surveys. Participants were informed that by completing the questionnaire they were consenting to participate, and that confidentiality and anonymity would be maintained in any data reports. They were also provided with supportive resources and the researchers’ contact information on the debriefing page of the questionnaire.

데이터 분석
Data analysis

주제별 분석
Thematic analysis

주제 분석은 일련의 데이터에 대한 체계적인 접근 방식으로 주요 이슈, 패턴 또는 주제와 이들 간의 연관성을 식별합니다[29, 30]. 이 분석을 통해 연구자들은 연구 질문 또는 기존의 관심사에 따라 데이터의 주요 이슈를 식별하고, 선별하고, 분류할 수 있습니다[29, 30]. 주제별 분석의 한 과정인 주제 매핑주제 간의 연결과 연관성을 시각화하는 데도 도움이 되었습니다[29, 30]. 우리는 주제별 분석을 사용하여 대처 전략에 대한 성찰, 학생들이 받은 지원, 이러한 문제와 학습 경험 간의 연관성을 탐색했습니다
Thematic analysis is a systematic approach to a set of data to identify key issues, patterns or themes and the association among them [29, 30]. This analysis allowed researchers to identify, sift and sort key issues in the data according to the research questions, or pre-existing interests [29, 30]. Thematic mapping, which is a process in thematic analysis, also supported the visualization of links and associations between themes [29, 30]. We used thematic analysis to explore reflections on coping strategies, the support students received, and the association between these issues and learning experience.

처음에는 응답 샘플을 사용하여 토론(TT 및 AD)을 통해 주제와 하위 주제를 식별했습니다. 그런 다음 TT는 이러한 주제와 하위 주제를 사용하여 초기 프레임워크를 개발했습니다. 그런 다음 전체 팀(TT, AD, TW, SS)이 모여 데이터와 코딩 프레임워크의 초안을 논의했습니다. 이 프레임워크는 반복적으로 수정되어 최종 프레임워크가 완성되었고, 이 프레임워크는 전체 데이터 집합을 코딩하는 데 사용되었습니다. 코딩 후, TT는 데이터의 최종 해석을 지원하기 위해 주제와 하위 주제 간의 패턴과 연관성을 탐색했습니다.
Initially, a sample of responses was used to identify themes and subthemes through discussion (TT and AD). Using these themes and subthemes, TT then developed an initial framework. The full team (TT, AD, TW, and SS) then met to discuss the data and the first draft of the coding framework. The framework was further iteratively revised to achieve the final framework, which was used to code the entire data set. After coding, TT explored patterns and connections between themes and subthemes to support the final interpretation of the data.

내러티브 분석
Narrative analysis

내러티브 분석에 Labov의 프레임워크를 사용하기로 한 이유는 내러티브 내용과 내러티브 전달 방식을 동등하게 강조하는 널리 인정받는 접근 방식이기 때문입니다[26]. Labov의 프레임워크[31]에서 내러티브는 다음과 같은 부분으로 구성됩니다:

  • (1) 추상(이야기할 내용이 있다는 표시),
  • (2) 방향(시간, 장소, 인물에 대한 설명),
  • (3) 평가(이 이야기를 할 가치가 있는 이유에 대한 화자의 표시와 감정 전달),
  • (4) 복잡한 행동(이야기의 핵심을 나타내는 일련의 행동),
  • (5) 해결(이야기의 마지막 사건),
  • (6) 코다(이야기가 끝났다는 표시와 이야기가 화자에게 미치는 영향)

즉, 학생들이 자신의 경험에 대해 성찰하고 무엇을 배웠는지 이해하기 위해 3~6단계에 특히 중점을 둡니다.
We chose to use Labov’s framework for narrative analysis because it is a widely recognized approach that equally emphasizes both the narrative content and the way the narrative is told [26]. In Labov’s framework [31], the narrative is organized into parts, including:

  • (1) Abstract (the indication that there is something to tell);
  • (2) Orientation (the description of time, place and person);
  • (3) Evaluation (the narrator’s indication on why this is worth telling and his/her communication of emotions);
  • (4) Complicating action (a sequence of actions representing the core of the story);
  • (5) Resolution (the final event in the story) and
  • (6) Coda (the indication that the story is finished and the effect of the story on the narrator).

By this means, we put a particular focus on steps 3–6 in order to understand students’ reflections on their experiences and what they learned.

또한 내러티브 분석은 학생이 반성하는 사건에서 정서적 요소를 포착하기 위해 감정적 단어부정, 강화, 반복과 같은 기타 극적 장치를 식별하여 추가로 수행할 수 있습니다 [22]. 따라서 우리는 주제별 분석에서 도출된 결과를 보완하기 위해 하나의 예시에 대해 Labov [31]와 Rees 등[22]이 제시한 내러티브 분석 접근법을 모두 적용했습니다. 
Additionally, narrative analysis can be further conducted by identifying emotional words and other dramatic devices such as negatives, intensifiers and repetitions to capture affective elements in the incident the student reflects upon [22]. Therefore, we applied both narrative analysis approaches informed by Labov [31] and Rees et al. [22] on one exemplar to complement the results derived from the thematic analysis.

결과
Results

참가자
Participants

12명의 의대생이 온라인 설문지를 작성했으며, 각 문항당 평균 약 400단어의 서술형 답변을 제공했습니다. 참가자의 특성은 표 2에 나와 있습니다. 
Twelve medical students completed the online questionnaire, with narrative responses averaging around 400 words per narrative. Participants’ characteristics are shown in Table 2.

환자 사망 경험에 대한 학생의 내러티브 내 테마
Themes within students’ narratives about patient death experience

주제별 분석 결과, 학생들의 환자 사망 경험에서 중요한 연구 질문과 관련된 네 가지 주요 주제가 확인되었습니다. 이러한 주제는 다음과 같습니다:

  • (1) 가장 기억에 남는 환자 사망 경험,
  • (2) 환자 사망의 여파,
  • (3) 환자 사망 후 지원에 대한 인식,
  • (4) 지원을 위한 제안.

이러한 주제에 대한 결과는 각 연구 질문의 맥락에서 논의될 것입니다.
The findings from thematic analysis identified four major themes within students’ experiences of patient death that related to the overarching research questions. These themes included:

  • (1) most memorable experience of patient death;
  • (2) the aftermath of patient death;
  • (3) perceptions of support following patient death and
  • (4) suggestions for support.

The findings from these themes will be discussed in the context of each research question.

환자 사망 경험에 대한 대처와 학습
Coping and learning from experiences with patient death

참가자들이 가장 기억에 남는 죽음으로 꼽은 것은 첫 번째 환자 사망, 예상치 못한 사망, 학생과 개인적으로 친분이 있던 환자의 예상치 못한 사망이었습니다. 참가자들은 환자의 죽음에 직면했을 때 환자와 환자의 동료 및 가족 등 관련된 다른 사람들에 대한 정서적 반응을 확인했습니다. 환자에 대한 정서적 반응은 다양했습니다. 이러한 감정 중 일부는 슬픔, 죄책감 등 부정적인 것이었고, 일부는 긍정과 부정이 혼합된 것이었으며, 일부는 긍정 또는 부정으로 해석될 수 있는 모호한 것이었습니다. 환자 가족과 관련하여 참가자들은 종종 가족에게 공감하는 감정을 느꼈다고 보고했습니다. 의료진의 다른 구성원에 대해 참가자들은 자신의 감정이 동료와 일치하는지 확인하려고 노력했다고 보고했습니다(표 3 참조). 
The deaths described by participants as the most memorable included their first patient death, unexpected deaths, and the expected death of a patient with whom the student had a personal connection. When confronted with the patient death, participants identified their affective responses towards both the patient and the other individuals involved, such as co-workers and family members of the patient. There were a range of affective responses towards patients. Some of these feelings were negative (e.g. sadness, guilt), some were a mix of both positive and negative, while others were ambiguous and could be interpreted as either positive or negative. In relation to the patients’ families, participants often reported feeling empathetic towards the family members. In response to other members of the healthcare team, participants reported trying to identify whether their own feelings were in alignment with their colleagues (see Table 3).

참가자들은 또한 이러한 경험 이후 어떤 일이 있었는지도 보고했습니다. 일부 참가자는 사건이 발생한 후 일정 기간 동안 이 경험으로 인해 영향을 받았다고 보고했습니다. 예를 들어, 두 명의 참가자는 반복되는 꿈을 경험했다고 보고했습니다:
Participants also reported what happened after these experiences. Some participants reported being affected by the experience for a period after the event transpired. For example, two participants reported experiencing recurring dreams:

"그 사건이 저에게 미친 영향 중 하나는 그 후 며칠 동안 그 사람이 제 꿈에 나타난다는 것이었습니다."(5번 학생).
“One of the ways it affected me was he was in my dreams for several days after that” (student no. 5).


참가자들은 이러한 경험에 대응하기 위해 사용한 몇 가지 대처 전략에 대해서도 논의했습니다. 이러한 전략은 외적 대처 전략과 내적 대처 전략으로 분류할 수 있습니다.

  • 우리는 내적 대처 전략을 학생들이 경험을 스스로 해결하기 위해 개별적으로 관여하는 내적 과정으로 정의했습니다.
  • 반면에 외적 대처 전략은 학생들이 비공식적 또는 공식적 토론을 통해 다른 사람들과 경험을 외부적으로 공유하는 것으로 정의했습니다.

Participants also discussed some of the coping strategies they used in response to the experience. These strategies could be categorized as external and internal coping strategies.

  • We defined internal coping strategies as internal processes that students individually engaged in to achieve their own resolution of the experience.
  • On the other hand, we defined external coping strategies as those where students shared the experience externally with others, for example, through an informal or formal discussion.

참가자들은 몇 가지 내부 대처 전략을 확인했습니다. 예를 들어, 일부 참가자들은 환자의 죽음에 대한 경험을 해결하기 위해 종교에 의지하여 종교에 비추어 죽음을 해석하는 것에 대해 논의했습니다. 한 참가자는 이렇게 말했습니다:
Participants identified several internal coping strategies. For example, some participants discussed turning to religion, interpreting the death in the light of religion to help resolve their experiences with patient death. One participant said:

"저는 어렸을 때 기억하는 대로 기독교적인 방식으로 해석하여 대처했습니다. 이런 식으로 제가 경험한 모든 것은 사후 세계가 없는 직접적인 실존적, 증거적 부패가 아닌 신학적인 의미를 갖게 되었습니다."(7번 학생).
“I coped with this by interpreting it in a Christian way (as I remembered it in my youth). In this way all I had experienced took on a theological meaning rather than one of a directly existential and evidential decay with no after life” (student no. 7).

일부 학생들은 개인적인 과정을 통해 그 경험을 개인적인 삶에 대한 교훈으로 삼았습니다. 예를 들어, 참가자 중 한 명은 환자의 죽음을 삶을 온전히 살아야 한다는 경각심을 일깨워주는 것으로 인식했습니다: 
Some students employed a personal process to turn the experience into a lesson learned for personal life. For example, one of our participants perceived the patient death as a reminder to live life to the full:

"인생은 참으로 짧기 때문에 우리는 최선을 다해 살려고 노력해야 하고, 우리를 도와준 사람들에게 감사해야 합니다."(1번 학생).

“Life is indeed a short episode and we should try to live to the full and be thankful to people who supported us” (student no. 1).

참가자들은 개인적인 삶에 국한되지 않고 자신의 직업적 역할이라는 렌즈를 통해 자신의 경험을 되돌아보기도 했습니다. 한 참가자는 병동 바닥에서 갑자기 사망한 환자를 목격한 후 그 경험을 떠올렸습니다: 
The lessons learned were not limited to personal life; participants also reflected on their experience through the lens of their professional role. One participant reflected on the experience after seeing a patient die suddenly on the floor of the ward:

"하지만 좀 더 효과적으로 대처했더라면 상황을 훨씬 더 잘 관리할 수 있었을 것이라는 생각이 듭니다. [돌이켜보면 분명 인재가 개입된 비극이었지만요. 이런 경험을 통해 더 나은 의대생이 될 수 있었다고 생각하며, 적시에 '적절한' 장소에 있었던 것이 다행이라고 생각합니다."(4번 학생). 
“I do however feel that the situation could have been managed much better as there were several areas that should have been dealt with more effectively. [ … ] In retrospect, although there was clearly human tragedy involved. I believe being exposed to this has made me a better medical student, and I feel fortunate for having been in the "right" place at the right time” (student no. 4).



다른 대처 전략도 확인되었습니다. 예를 들어, 일부 학생들은 사망 후 다른 일에 몰두함으로써 부정적인 감정으로부터 주의를 돌렸습니다
Other strategies for coping were also identified. For instance, some students shifted their attention from negative emotions after the death by being busy with other tasks:

"다른 일로 바쁘게 지내는 것으로 대처했습니다. 바쁘게 지내다 보니 그 환자의 죽음을 받아들이기가 더 쉬워졌어요."(7번 학생).
“I coped by staying busy with other responsibilities. Staying busy made it easier to accept his death” (student no. 7).

어떤 학생들은 그 경험을 누구와도 공유하지 않고 혼자서 감정을 견뎌냈다고 말했습니다. 한 참가자는 이렇게 말했습니다,
Some students reported just enduring the emotion without sharing the experiences with anybody. One participant said,

"저는 아버지의 죽음에 대해 누구와도 이야기하지 않고 혼자서 슬픔을 감당했습니다. [...] 저는 항상 제 생각과 [사망한 환자에 대한] 반복되는 꿈에 대해 조용히 지냈습니다. 사실 저는 항상 그 꿈에서 깨어나서 그런 경험을 했다는 사실을 자각하는 것을 즐겼습니다."(5번 학생).
“I just handled my sadness internally without discussing his death with anyone. [...] I was always quiet about my thoughts and the recurring dreams [about the patient who died]. I actually always enjoyed waking from them and being so aware of having had the experience” (student no. 5).


또한 학생들은 내적 대처 전략으로 정상화를 사용하는 것으로 나타났습니다. 이는 환자의 죽음에 대한 자신의 감정적 반응이 정상적이라고 인식할 때 나타났습니다. 외부 대처 전략과 관련해서는 대다수의 참가자가 직장이나 가족에게 환자의 죽음에 대해 이야기한다고 답한 점이 흥미롭습니다. 주목할 만한 점은 동료나 가까운 지인에게 이야기하는 경우가 많았다는 점입니다.
We also found students employed normalization as an internal coping strategy. This was shown when they recognized that their emotional responses towards patient deaths were normal. Regarding external coping strategies, it is interesting that the majority of participants mentioned speaking to someone at work or in the family about patient death. Of note, many spoke to peers and/or near-peers.

분석을 통해 외적 대처 전략과 내적 대처 전략이 동시에 나타나는 몇 가지 사례를 발견했으며, 이는 이들 주제 간의 관계를 시사합니다. 특히, 동료와의 상호작용이 환자 사망 후 학생들의 대처 메커니즘에 영향을 미치는 것으로 나타났는데, 이는 사망 경험을 교훈으로 정상화하고 재해석하는 것과 관련이 있습니다(그림 1 참조).
Through the analysis, we found several instances where external and internal coping strategies co-occurred, suggesting a relationship between these themes. In particular, we found that an interaction with colleagues influenced the students’ coping mechanisms following patient death (see Fig. 1) in relation to normalization and reinterpretation of the death experience as a lesson learned.

참가자들은 누군가와 대화를 나눌 때 그 경험에 대한 자신의 감정을 정상화하는 데 도움이 되었다고 종종 보고했습니다. 한 참가자는 동료와의 대화와 관련하여 다음과 같이 썼습니다:
When participants spoke with someone, they often reported that this helped normalize their feelings about the experience. One participant wrote in relation to speaking to a colleague:

"[나]의 감정이 혼자가 아니며 시나리오에 대한 정상적인 반응이라는 것을 인식했기 때문에 더 긍정적인 경험이 되었을 것입니다."(9번 학생).
“[It] probably made it [a] more positive experience as [I] recognized [I] wasn't alone in my feelings and that this was [a] normal response to the scenario” (student no. 9).


정상화 외에도 병동에서 누군가와 대화하는 것이 부정적인 감정을 학습의 기회로 재해석하는 것을 촉진한다는 사실을 발견했습니다. 이는 말기 암 환자에 대해 화가 나서 컨설턴트가 환자에게 나쁜 소식을 전하러 왔을 때 병동을 떠났던 한 학생의 반성에서 확인할 수 있습니다. 한 주니어 의사가 그 학생의 행동에 대해 이야기하면서 앞으로는 배울 점이 많으니 이런 경험을 피하지 말라고 조언했습니다
In addition to normalization, we found that speaking to someone on the ward promoted the re-interpretation of negative emotions into learning opportunities. This can be seen in the reflection of a student who was upset about an advanced-stage cancer patient and so left the ward when a consultant came to break the bad news to the patient. A junior doctor then spoke with them about their behavior, suggesting that in future they should not avoid the experience because there was a lot they could learn:

"그는 컨설턴트가 [... ] 오는 동안 왜 내가 [... ] 사라졌는지 물었습니다. 그는 사라지는 것이 저에게 아무런 도움이 되지 않을 것이라며 "병동 회진에서 겁먹지 말라"고 조언했습니다. 저는 그의 말이 굴욕감이 아니라 제 자신에 대한 [사실]을 지적하는 측면에서 [매우] 강하다고 느꼈습니다. 제가 병동 회진 중에 사라지지 않았다면 나쁜 소식이 어떻게 알려지는지 봤을 것입니다. 놓치지 말았어야 할 경험을 놓쳤어요 [...] 제가 상담했던 기초의학과 의사는 훌륭한 롤모델이었어요"(2번 학생).
“He asked me why I [had] vanished while the consultant [ … ] came. He suggested vanishing wouldn’t help me with anything, and “don’t be chicken when it comes to ward rounds”. I felt his comment was [very] strong, not in terms of humiliation, but in terms of pointing out [a fact] about myself. If I hadn’t vanished during the ward round, I would have seen how the bad news was brought to light. I did miss an experience I shouldn’t have missed [...] The foundation doctor I spoke to was a great role model” (student no. 2).

2번 학생은 환자가 나쁜 소식을 듣는 것을 목격하는 것이 불쾌한 일이라는 것을 기초 의사가 알고 있었다고 말했습니다. 그러나 그는 학생에게 부정적인 감정을 받아들이고 병동 회진에 참석할 것을 제안했고, 학생은 그 조언이 도움이 되었다고 말했습니다. 또한 이 학생의 의견은 학생들이 협상하는 법을 배워야 하는 환자 사망 경험의 개인적 측면과 직업적 측면의 상호 작용을 강조합니다. 아래 댓글을 보면 동료들의 지원이 학생에게 환자 사망의 개인적, 직업적 측면을 협상하는 데 어떻게 도움이 되었는지 알 수 있습니다: 
The student no. 2 suggested that the foundation doctor knew it was unpleasant to witness the patient being told bad news. However, he suggested that the student embrace their negative emotions and be present during the ward round, which the student found helpful advice. Furthermore, the student’s comments highlight the interplay of both personal and professional aspects of patient death experiences that students must learn to negotiate. In the comment below, we see how support from colleagues helped students negotiate the personal and professional aspects of patient death:

"제 기분을 잘 아는 병동 자매와 이야기를 나누며 대처했습니다. 우리는 [일을 계속해야 한다는 것을 알았기 때문에] 너무 오래 대화하지 않았습니다. 개인적인 감정과 업무를 분리하는 데 도움이 되었죠. 저는 모든 사람이 어떤 식으로든 환자의 죽음에 영향을 받는다고 생각하지만, 환자 중 한 명이 죽었다고 해서 다른 환자를 구할 수 없다는 뜻은 아니라고 생각합니다."(3번 학생).
“I coped by talking to a ward sister who seemed to know how I felt. We didn't spend too long chatting because we knew we had to move on [with doing our jobs]. It helped me separate the personal feelings from my work. I believe everybody is affected by patient death in some way, but [that] one of our patients died doesn't necessarily mean we can't save the others” (student no. 3).

병동 직원과 의과대학의 지원이 대처와 학습에 미치는 영향
Impact of support from ward staff and medical school on coping and learning

환자 사망 후 학생들의 지원에 대한 인식에는 (1) 병동 직원으로부터 받은 지원과 (2) 의과대학으로부터 받은 지원이라는 두 가지 주제가 있었습니다. 

  • 전자는 컨설턴트, 레지던트, 재단 의사, 동료, 간호사, 기타 의료진 등 병동에서 함께 일한 사람들로부터 받은 지원에 대한 참가자들의 반응에 초점을 맞췄습니다.
  • 후자는 체험이 이루어진 병동 환경에 관여하지 않은 교직원의 지원에 대한 반응에 관한 것이었습니다.

There were two themes for students’ perceptions of support following patient death: (1) Support received from ward staff and (2) Support received from the medical school.

  • The former focused on participants’ responses towards support they received from people with whom they worked on the ward, which included consultants, registrars, foundation doctors, colleagues, nurses and other medical staff.
  • The latter concerned the responses towards support from academic staff who were not involved in the ward setting where the experience took place.

병동 직원과 관련해서는 병동 직원의 지원 정도에 대한 학생들의 인식에 차이가 있었는데, 만족스럽다는 의견이 있는 반면 만족스럽지 않다는 의견도 있었습니다. 지원이 만족스러웠다고 응답한 경우, 또래 및 가까운 동료의 개인적인 정서적 지원이 공통적인 특징이었습니다. 예를 들어, 한 참가자는 이렇게 썼습니다:
In terms of ward staff, there was a range in students’ perception of how supportive the ward staff were; some people felt that it was satisfactory, while others did not. In instances where the support was satisfactory, personal emotional support from peers and near-peers was a common feature. For instance, one participant wrote:

"5학년 학생과 FY1(파운데이션 닥터 1년차)이 매우 공감해 주었고, 사건에 대해 이야기할 시간을 주었습니다."(7번 학생).
“The 5th year student and the FY1
 [foundation doctor Year 1] were very sympathetic and gave me time to discuss the event” (student no. 7).

7번 학생의 만족스러운 지원에 대한 의견은 정서적 지원에만 국한되지 않는다는 것을 알 수 있었습니다병동 직원과의 대화는 직업 생활에서 활용할 수 있는 학문적 교훈과도 관련이 있었습니다. 같은 학생이 계속 말했습니다:
We found that the student no. 7’s comments about satisfactory support were not limited to fostered emotional support. Their discussion with ward staff was also associated with academic lessons that they could take forward in their professional life. The same student continued:

"... 제 동료들은 죽음에 대한 제 반응에 당황하기도 하고 공감하기도 하면서 스스로 대처해 나갔던 것 같아요. 우리는 누구를 비난하지 않고 실질적인 방법으로 사건에 대해 논의하면서 서로를 지지했습니다."
“ … my colleagues were, I think, both baffled by and sympathetic to my reactions to the death, while coping with it themselves. We supported each other by discussing the event in practical ways without blaming anybody”.


여기서 우리는 앞서 설명한 것, 즉 누군가와 대화하는 것과 같은 외적 대처 방법이 개인적인 감정과 직업적 역할의 측면에서 내적 대처 과정을 지원한다는 상호 작용을 볼 수 있습니다.
Here we can see the interplay between what we illustrated earlier – that external coping measures (i.e. speaking to someone) supported the internal coping process both in terms of personal feelings and professional roles.

또는 한 참가자는 자신이 지지받지 못했다고 느꼈던 환자의 죽음에 대한 이야기를 들려주었습니다: 
Alternatively, one participant narrated a story about a patient death where they did not feel supported:

"의료계 종사자들은 모두 죽음에 익숙해져 있고 그들에게는 죽음이 자연스러운 일이지만, 처음 경험한 저는 그렇지 않았습니다."(6번 학생).
“Everyone in medicine is so hardened by deaths and it came like nature to them, but not me who had that first experience” (student no. 6).

학생들이 지지를 받지 못한다고 느낀 경험은 개인적인 정서적 지원이 제공되지 않은 상황인 경우가 많았습니다. 여기서 흥미로운 점은 환자의 죽음에 대한 초기의 개인적 정서적 경험(예: 처음)과 학생들의 감정에 민감하게 반응하는 사람들의 지원에 대한 감사 사이의 연관성입니다. 한 참가자의 경험은 이 아이디어를 더욱 발전시켜 총체적인 지원의 중요성을 강조했습니다. 그들은 이렇게 썼습니다: 
Often, experiences where students did not feel supported were situations where personal emotional support was not provided. An interesting point that can be drawn here is the link between early personal emotional experience (i.e. first time) with patient death and the appreciation for support from those who were sensitive to students’ emotions. One participant’s experience took this idea further, highlighting the importance of holistic support. They wrote:

"... [지원]이 많지 않았습니다. 대화는 주로 배운 교훈과 더 잘했어야 하는 부분에 초점을 맞추었는데, 이 부분은 제가 매우 중요하게 생각하는 부분입니다. 개인적/도덕적/실존적 측면은 별로 논의되지 않았습니다."(4번 학생).
“ … not much [support]. The conversations focused primarily on the Lessons Learned, and what should have been done better, which is an area I feel very strongly about getting right. The personal/moral/existential aspects were not really discussed” (student no. 4).


의과대학에서 받은 지원과 관련하여 참가자들은 만족스러운 지원과 불만족스러운 지원의 지점도 파악했습니다. 참가자들은 병동 근무를 준비할 때 받은 지원에 대해 자주 이야기했습니다. 예를 들어 한 참가자는 이렇게 썼습니다: 
In relation to support received from the medical school, participants also identified points of satisfactory and unsatisfactory support. Participants often talked about the support they received in preparing for their work on the wards. For instance, one participant wrote:

"EOL(임종) 문제를 관리할 수 있도록 최선을 다해 준비해 주었습니다. 하지만 거기까지가 그들이 할 수 있는 전부인 것 같고, 나머지는 직접 경험을 통해 배워야 합니다."(2번 학생).
“They did their best in preparing me for managing the EOL [end of life] issues. But that's the farthest they could go, I think; the rest of the lessons we have to learn from hands on experience” (student no. 2).

2번 학생은 의과대학의 지원을 높이 평가했지만, 전임상 단계에서 받은 준비 과정에만 국한된 시각을 가지고 있었습니다. 이 점과 관련하여 참가자들은 환자 사망을 경험한 구체적인 사례에서는 의과대학의 지원이 적절하지 않다고 생각한다는 의견도 언급했습니다. 한 참가자는 이렇게 썼습니다: 
Although the student no. 2 appreciated the support from medical school, their view towards it was limited only to the preparation course they received in preclinical years. Related to this point, participants also referred to the idea that they felt that medical school support was not relevant in these specific instances of experiencing patient death. One participant wrote:

"... 환자를 잃은 순간 학교는 도움을 요청하기에는 너무 멀리 떨어져 있다고 생각했습니다. 학교가 도움을 줄 수도 있겠지만, 저는 함께 일하는 사람들과 이야기하고 싶었습니다."(8번 학생).
“ … at the moment of losing a patient, I thought the school was [too far away] to reach for support. They might be supportive, but I'd rather talk to the people I am working with” (student no. 8).

지원을 위한 제안
Suggestions for support

참가자들은 지원을 위한 두 가지 주요 제안에 대해 이야기했습니다. 첫 번째는 타이밍에 관한 것이었습니다. 학생들은 환자의 임종 경험을 준비할 때 실질적인 측면과 정서적인 측면을 모두 포함한 지원의 중요성에 대해 논의했습니다. 예를 들어, 한 참가자는 기술 준비에 대해 비판했습니다:
Participants talked about two key suggestions for support. The first was around timing. Students discussed the importance of support in the preparation for patient death experiences, including both practical and emotional aspects. For example, one participant criticized the skills preparation:

"죽음과 그에 따라 의료진이 수행해야 하는 업무가 의과대학에서 잘 다루어지거나 가르쳐지지 않는 것 같아요. 언젠가는 임상 술기 세션에서 죽음을 확인하고 인증하는 과정, 즉 실질적이고 감정적인 측면을 다루어야 한다고 생각합니다."(10번 학생).
“I don't think death and the accompanying tasks that need to be performed by medical staff are well covered/taught by the medical school. I think we should have a clinical skills session at some point taking us through confirming and certifying death - the practical and emotional aspects” (student no. 10).


둘째, 좋은 지원의 특징이 강조되었는데, 이는 표 4에 요약되어 있습니다. 이러한 주제는 구조, 활동, 감수성, 포용적 지원과 관련된 것이었습니다. 즉, 

  • 환자 사망 후 지원은 구조적이고 공식적이며 조직적이어야 하고,
  • 일부 학생은 스스로 지원을 요청하지 않을 수 있으므로 보다 적극적이어야 하며,
  • 오랜 기간 환자 사망을 경험한 의료인은 굳어져 의대생의 초기 경험에 대한 민감성을 잃을 수 있으므로 민감해야 하고,
  • 동료 및 가까운 동료의 지원이 도움이 될 수 있으므로 포용적이어야 한다는 것입니다.

Second, the characteristics of good support were highlighted; these are summarized in Table 4. These themes concerned structure, activity, sensitivity, and embracing support. In other words, the support following patient death should be structured, formal and organized; it should be more active, as some students might not seek support themselves; it should be sensitive, since over a period of time experiencing a number of patient deaths, a healthcare professional may become hardened and sensitivity to medical students’ early experiences might be lost; and support from peers and near-peers can be helpful and should be embraced.

내러티브 분석의 추가 결과
Further findings from narrative analysis

학생들이 보고한 경험의 유형을 보여줄 뿐만 아니라 이러한 경험을 통한 성찰과 학습 간의 상호작용에 대한 통찰을 제공하기 때문에 10번 학생이 작성한 내러티브의 예시(표 5 참조)를 분석 대상으로 강조했습니다. 
We highlight the exemplar narrative written by the student no. 10 (See Table 5) for analysis, as it not only illustrates the types of experiences that students reported, but also provides insight into the interplay between reflection and learning through these experiences.

화자는 환자의 사망 사건에 대한 성찰을 시간, 장소, 사람(5학년 마지막 주, 병동에서 갑자기 사망한 환자, 1∼2행)에 대한 오리엔테이션으로 시작합니다. 다음 다섯 문장은 병실에 있던 간호사가 환자의 친척인 줄 모르고 사망 확인을 하는 과정에 관한 내용입니다. 이 문장들은 참가자가 감정 단어(예: 모멸감, 끔찍함, 8행)를 사용하여 사건에 대한 평가를 표현한 것으로 보아 라보프[31]의 평가 단계를 나타낸다고 볼 수 있습니다.
The narrator begins their reflection on the patient death incident with an orientation of time, place and person (last week of 5th year, on the ward, a patient who had suddenly died, line1–2). The following five sentences concern the death confirmation process without realizing that the nurse in the room was a patient’s relative. These sentences arguably represent Labov’s [31] evaluation phase as the participant expressed their evaluation of the incident through using emotion words (i.e. mortified and terrible, line 8).

내러티브는 화자가 foundation doctor와 함께 사망 인증을 수행하는 복잡한 행동 단계 [31]로 진행됩니다. 여기서 일종의 미묘한 감정 장치[22]인 반복이 발견되는데, 환자의 친척이 있다는 사실과 그 상황을 어떻게 보았는지(속상하다, 10행)에 대해 반복해서 언급하는 장면이 그것이다. 성찰의 렌즈를 통해 이러한 부정적 감정이 상황의 불편함을 확인하는 학생의 성찰의 첫 단계라고 해석할 수 있습니다.
The narrative proceeds to the complicating action phase [31] in which the narrator performed death certification with a foundation doctor. A repetition, which is a kind of subtle emotional device [22], is found here when they repeated mentioning about the presence of patient’s relative and how they saw the situation (upset, line 10). Through the lens of reflection, we interpreted that these negative feelings were the first stage of the student’s reflection in which they identified discomfort in the situation.

그러나 내러티브가 이야기의 결말을 나타내는 결말과 코다 부분으로 넘어가면[30], 감정 단어가 부정적 감정에서 긍정적 감정으로 전환됩니다. 이 전환점은 이야기 내에서 학생이 자신이 배운 것을 반성하고 그 상황이 자신에게 '가르친 것'에 대해 이야기하는 시점과 일치합니다(11행). 화자의 현재 시점('나는 느낀다', 11행과 12행의 반복으로 명시적으로 알 수 있음)에서 볼 때, 이 상황은 기초 박사 수준의 준비라는 측면에서 좋은 상황입니다. 긍정적(11행), 유익한(12행, 16행), 더 나은(12행) 등의 형용사와 부사가 느낌을 나타내기 위해 사용되었습니다. 유익한(12행과 15행) 앞에 매우라는 강화어가 반복적으로 사용되었는데, 이는 그들이 배운 것 때문에 전반적으로 상황을 긍정적으로 생각하고 있음을 보여줍니다. 
However, when the narrative moves to the resolution and coda parts, indicating the ending of the story [30], the emotion words shift from negative to positive ones. This transition point aligns with the point within the story where the student reflects on what they have learned; they discuss what the situation has taught them (line 11). From the narrator’s current point of view (explicitly signaled by the repetition of ‘I feel’, lines 11 and 12), this situation is good in terms of the preparation for the foundation doctor level. Adjectives and adverbs such as positive (line 11), beneficial (lines 12 and 16) and better (line 12) were used to indicate their feeling. The intensifier very was used repeatedly before beneficial (lines 12 and 15), demonstrating how they positively think about the situation overall, arguably because of what they have learned.

이 장면을 통해 참가자가 임상 연도 실습에서 죽음을 인정하기 위한 실질적이고 감정적인 준비에 대해 어떻게 느꼈는지 알 수 있습니다. 화자는 '죽음과 의료진이 수행해야 하는 그에 수반되는 업무가 의과대학에서 잘 다루어지거나 가르쳐지지 않는 것 같다'(17-18행)고 명시적으로 말하며 부정의 표현을 사용합니다. 여기서 부정의 형태로 나타나는 사실과 반대되는 사고를 확인할 수 있는데, 이는 일어났어야 하지만 일어나지 않았다고 생각하는 사건에 대한 실망감을 반영합니다[32]. 성찰 단계의 틀 안에서, 우리는 그들이 사건에 대한 분석을 통해 두 가지 새로운 관점, 즉 사망 인증 과정에서 얻은 교훈과 의과대학이 이 부분에 대한 준비 과정을 개선할 수 있는 기회가 있다고 해석합니다. 
Reflecting on what happened, we also see how the participant felt about the practical and emotional preparation for certifying death in clinical year practice. The narrator employs the use of negation by explicitly saying, I don’t think death and the accompanying tasks that need to be performed by medical staff are well covered/taught by the medical school (lines 17–18). A counterfactual thinking in the form of negation can be identified here, reflecting disappointment at an event that they think should have happened but did not [32]. Within the framework of reflection stages, we interpret that there are two new perspectives emerging from their analysis of the event: the lessons they have learned from the death certification process, and the opportunity for the medical school to improve their preparation course regarding this aspect.

토론
Discussion

이 연구는 의대생들의 환자 사망 경험에 대한 내러티브를 탐색함으로써 연구 질문과 관련된 많은 문제를 확인했습니다. 
By exploring narratives of medical students about their experiences with patient death, this study identified many issues related to the research questions.

환자 사망에 대한 대처 전략
Coping strategies for patient death

주제별 분석을 통해 학생들이 사용하는 많은 대처 전략이 이전 연구의 결과와 유사하다는 것을 관찰했습니다. 이제 각 전략에 대해 논의하여 연구 결과가 기존 논문의 결과를 어떻게 반영하거나 확장하는지 보여드리겠습니다. 
Through thematic analysis, we observed that many coping strategies that students employed resembled the results of previous studies. Each of the strategies will now be discussed to show how the findings reflect or extend the results of existing articles.

종교에 의지하기
Turning to religion

종교에 의지하는 것은 의대생[3, 5, 7], 의사[33], 간호사[34]가 환자가 사망한 후 흔히 취하는 대처 전략으로 확인되었습니다. 종교가 환자 사망 후 의대생의 대처 메커니즘에 어떻게 도움이 되는지 명확하게 설명하는 논문은 많지 않지만, 루디실 등[35]은 의대생의 죽음 이후의 삶에 대한 강한 종교적 신념과 낮은 죽음 불안 사이의 관계를 발견했습니다. 이는 학생들의 종교적 배경에 따라 죽음을 해석하기 때문일 수 있습니다. 예를 들어, 일부 학생들은 죽어가는 환자가 더 높은 힘에 의해 보호받고 있다고 느끼고 죽음을 긍정적인 방식으로 해석할 수 있습니다[36]. 

Turning to religion was identified as a common coping strategy for medical students [3, 5, 7], physicians [33] and nurses [34] after a patient had died. Although there are not many articles clearly explaining how religion helps with medical student coping mechanisms following patient death, Rudisill et al. [35] found a relationship between medical students’ strong religious beliefs about life after death and lower death anxiety. This could be due to the interpretation of death based on the students’ religious background; for example, some students might feel that the dying patient was being protected by a higher power, and then interpret the death in a positive way [36].

바쁘게 지내기
Staying busy

다른 업무로 바쁘게 지내는 것이 환자의 죽음에 대처하는 방법이라는 이전 연구[3, 7, 19]의 결과와 일치하는 것으로 볼 수 있습니다. 그러나 이번 연구에서는 약간의 차이가 있을 수 있습니다. 페사그노 등[3]은 의사의 업무를 계속하는 것이 전문적 속성의 식별과 관련이 있다는 것을 발견했습니다. 즉, 그들의 연구에 참여한 학생들은 "의사와 의대생이 얼마나 많은 환자 사망을 경험하든 간에 다른 사람을 계속 돕는 것이 의학의 필수적인 부분"이라고 생각하기 때문에 스스로를 바쁘게 유지했습니다([3], 54페이지). 우리 연구에 참여한 일부 학생들은 바쁘게 지내는 것을 대처 전략으로 설명했지만, 직업적 속성은 반드시 확인되지 않았습니다. 한편으로는 바쁘게 지내는 것이 그들이 겪고 있는 슬픔으로부터 인식을 전환하는 방법이었습니다. 반면에 바쁘게 지내는 것은 직업적 속성의 발달에 따른 행동일 수도 있습니다. 
Coping by staying busy with other responsibilities might be seen to agree with the results of previous studies [3, 7, 19], showing that keeping on with clinical tasks was a way to cope with patient death. However, there might be a minor difference in this current study. Pessagno et al. [3] found that carrying on with physicians’ tasks was linked to the identification of professional attributes; that is, the students in their study kept themselves busy because they saw that “continuing to help others is an essential part of medicine regardless of how many patient deaths physicians and medical students experience” ([3], p. 54). Some students in our study described staying busy as a coping strategy, without the professional attributes necessarily being identified. On the one hand, staying busy was a way to shift awareness away from sadness they were experiencing. On the other hand, staying busy could be an action following the development of professional attributes.

부정적인 감정 견디기
Enduring negative emotions

이 연구에 따르면 일부 학생들은 부정적인 감정을 내부적으로 처리하는 것으로 나타났습니다. 이러한 대처 방법은 Firth-Cozens 등의 연구를 제외하고는 이전 연구에서 많이 언급되지 않았습니다 [19]. 그들의 연구에서 의대생의 2.3%는 반드시 도움을 구하지 않고 시간이 지나면 그 감정이 사라지도록 내버려 두는 방법을 선택했습니다 [19]. 또한 이러한 대처 전략은 죽음에 대한 불안 수준을 측정하는 심리 측정 도구인 죽음에 대한 두려움(FOD) 점수가 높은 것과 관련이 있다는 것을 발견했습니다 [19]. 흥미롭게도 이 전략을 사용한 현재 연구에 참여한 한 참가자는 사망한 환자에 대한 꿈이 한동안 계속 되풀이되는 경험을 했다고 보고했습니다. 사망한 환자에 대한 이러한 반복적인 꿈은 사망에 큰 영향을 받는 의료진에게 발생할 수 있는 방해가 되는 생각이나 기억으로 해석될 수 있습니다 [37, 38]. 저희가 아는 한, 저희의 연구는 환자 사망 후 의대생들의 방해가 되는 기억을 보고한 최초의 연구입니다. 연구 결과를 통해 우리는 이러한 특징들(지속되는 부정적 감정, 죽음에 대한 불안, 침입적 사고) 사이에 어떤 연관성이 있을 수 있으며, 이러한 연관성은 추가 연구를 위한 영역을 남길 수 있다고 제안합니다. 
This study has revealed that some students processed their negative emotions internally. This method of coping has not been mentioned much in previous studies except that of Firth-Cozens et al. [19]. In their study, 2.3% of medical students chose to let the feeling fade away through time without necessarily seeking support [19]. They also found that this coping strategy was associated with a high Fear of Death (FOD) score, which is a psychometric tool measuring the level of death anxiety [19]. Interestingly, a participant in the current study who used this strategy reported that they experienced dreams about the deceased patient that kept returning for some time. These recurrent dreams about patients who died may be interpreted as intrusive thoughts or memories that can occur to healthcare providers who are powerfully affected by the death [37, 38]. To our knowledge, our study is the first to report intrusive memories in medical students after patient death. From our findings, we also propose that there might be some connections between these features (enduring negative emotions, death anxiety and intrusive thoughts), and that these connections leave an area for further research.

개인적인 삶과 직업적 역할에 대한 교훈으로 경험을 재해석하기
Personal re-interpretation of the experience into a lesson learned for personal life and professional role

생명의 가치를 고려하여 죽음을 긍정적으로 바라보는 것은 의대생의 대처 전략을 연구하는 기존 문헌에서 일반적으로 언급되지 않았습니다. Kelly 등[2]은 젊은 환자의 죽음을 계기로 학생들이 삶의 가치에 대해 고민하게 되었다고 간략하게 언급했습니다. 본 연구는 학생들이 삶을 온전히 살아가고 도움을 준 사람들에게 감사하는 삶의 가치에 대해 어떻게 생각하는지 보여줌으로써 그들의 연구 결과를 뒷받침합니다. 
Seeing the death in a positive light by considering life value has not generally been mentioned in the existing literature studying medical students’ coping strategies. Kelly et al. [2] briefly mentioned that the death of a young patient led to students contemplating their life values. Our study supports their finding by demonstrating how life values were contemplated, in terms of living life to the full and being thankful to people for their support.

환자의 죽음 경험에 대한 또 다른 개인적 재해석은 직업적 역할 측면에서 발견되었습니다. 여기서 학생들은 죽어가는 환자와 환자의 죽음에 대한 향후 접근 방식에 대한 직업적 역할에 대한 교훈으로 경험을 전환하는 방법을 보여주었습니다. 이러한 특징은 주제 분석에서 4번 학생의 의견(심폐소생술 카트와 환자 사망 주변 다른 환자의 존재에 대한 교훈)과 내러티브 분석에서 10번 학생의 의견(사망 인증 과정과 그 준비에 대한 교훈)에서 발견되었습니다. 이는 학생들 스스로가 EOL 실습을 긍정적이고 의미 있는 방식으로 해석할 수 있는 잠재력을 가지고 있음을 보여주며, 이는 맥레오드 외[39]의 연구 결과와도 일치합니다. 
Another personal re-interpretation of patient death experience was found in terms of professional role. Here, students have shown how experiences were turned into lessons learned for their professional role in future approaches to dying patients and patient death. This feature was found in the student no. 4’s comment in the thematic analysis (lessons about CPR cart and the presence of other patients around the death of a patient) and the student no. 10’s comment in narrative analysis (lessons about death certification process and its preparation). This shows that students themselves have the potential to interpret the EOL practice in a positive and meaningful way, aligning with the findings of MacLeod et al. [39].

학생들이 경험의 재해석을 통해 얻은 교훈은 앞서 논의한 성찰의 마지막 단계, 즉 새로운 관점의 개발을 보여준다고 간단히 제안할 수 있습니다. 그러나 4번 학생(환자의 죽음에 대한 경험에서 대처하고 배우기 섹션 참조)과 10번 학생(표 5 참조)의 의견에서 흥미로운 측면을 발견할 수 있었습니다. 두 학생 모두 마지막에 정서적 요소가 부정적에서 긍정적 요소로 바뀌면서 직업적 역할에 대한 귀중한 교훈을 얻게 되었습니다. 특히 10번 학생의 성찰에 대한 내러티브 분석을 통해 학습이 의대생이 경험하는 부정적 감정과 긍정적 감정 사이의 가교 역할을 하는 것을 알 수 있습니다. 부정적인 감정을 생산적인 결과로 전환하는 이러한 특성은 학습뿐만 아니라 정서적 민첩성이라고 주장합니다 [40]. David 외[40]는 정서적 민첩성이란 사람들이 부정적인 감정에 대해 사려 깊고 생산적이며 가치 중심적인 방식으로 접근하는 자질이라고 제안합니다.  
It can be simply proposed that the lessons students have learned through the re-interpretation of experiences display the final stage of reflection we discussed earlier, which is the development of new perspectives. However, we found an interesting aspect from the comments of the student no. 4 (see section Coping and learning from experiences with patient death) and the student no. 10 (see Table 5). In both comments, the affective elements shifted from negative to positive at the end, resulting in valuable lessons for their professional role. In particular, through narrative analysis of the student no 10’s reflections, we can see how learning provides a bridge between the negative and positive emotions that medical students experience. This quality of transforming a negative emotion into a productive consequence is not only learning but, we argue, emotional agility [40]. David et al. [40] propose that emotional agility is the quality by which people approach negative emotions in a mindful, productive and value-driven way. Through reflection there is an opportunity for individuals not only to learn, but to cope through learning. More importantly, there is also an opportunity to learn to cope through developing skills such as emotional agility and mindfulness.

정상화
Normalization

이 연구는 환자의 죽음에 대한 강한 감정이 정상적인 반응으로 인식되어 대처 메커니즘이 생길 수 있음을 보여주었습니다. 우리가 아는 한, 의대생이 환자 사망을 경험한 후 정상화를 대처 전략으로 보고한 기존 논문은 없습니다. 그러나 일부 연구에서는 간호사[41]와 구급차 서비스 종사자[37]가 이 전략을 광범위하게 사용한다고 제안했습니다. 의료 사회학에서 정상화는 의대생과 같은 새로운 사회 구성원이 실무 커뮤니티에 참여함으로써 사고 방식을 배우고 직업적 가치와 행동을 내면화하는 과정입니다[42]. 이 정의는 내적 과정으로서의 정상화를 의미하지만, 우리의 데이터는 다른 사람들과의 상호 작용이 정상화에 영향을 미친다는 것을 시사합니다(그림 1 참조). 로즈-크로프 등[7]은 또한 의대생에게 환자의 사망 후의 감정에 대해 일상적으로 질문하고 임상 실습에는 강한 감정이 수반된다는 사실을 깨닫도록 도와줌으로써 시니어 의사가 이 과정을 향상시킬 수 있다고 주장했습니다. 
This study showed that strong feelings towards the death of a patient could be recognized as normal responses, resulting in coping mechanisms. To our knowledge, none of the existing articles have reported normalization as a coping strategy after medical students’ experiences with patient death. However, some studies suggested that nurses [41] and ambulance service workers [37] broadly employed this strategy. In medical sociology, normalization is the process in which a new social member (such as a medical student) learns the ways of thinking and internalizes the professional values and behaviors through participating in a community of practice [42]. Although this definition implies normalization as an internal process, our data suggest that interaction with other people influences normalization (see Fig. 1). Rhodes-Kropf et al. [7] also advocated that senior staff members could enhance this process by routinely asking medical students about their feelings following the death of a patient and helping them to realize that clinical practices involve strong emotions.

누군가에게 이야기하고 그 경험을 직업적 역할에 대한 교훈으로 재해석하기
Speaking to someone and re-interpretation of the experience into a lesson learned for professional role

누군가와 대화하는 것이 환자 사망 후 대처하는 일반적인 방법으로 나타났으며, 이는 로즈-크로프 외[7] 및 존스 외[8]의 연구 결과와 일치합니다. 또한 환자의 죽음에 대해 누군가에게 이야기하는 것이 정서적 위안과 학문적 이익(2, 5, 8)을 제공할 뿐만 아니라 전문적 특성의 개발에도 도움이 된다는 기존 연구 결과와도 일치합니다[5]. 그러나 이 연구는 전문직 역할에 대한 학습을 지원하는 개인적 성찰과 집단적 성찰 간의 상호작용을 보여줌으로써 기존 문헌을 확장합니다. 특히 학생과 숙련된 전문가 간의 팀 상호작용은 학생들이 환자의 죽음 경험과 관련하여 학습하고 균형을 잡아야 할 개인적 측면과 전문적 측면이 모두 존재한다는 것을 인식하는 데 도움이 되는 것으로 나타났습니다. 
Speaking to someone was shown to be a common method of coping after patient death, aligning with the findings of Rhodes-Kropf et al. [7] and Jones et al. [8]. Our results also correspond with existing research in that speaking to someone about patient death provided not only emotional comfort and academic benefit (2, 5, 8], but also the development of professional attributes [5]. However, this study extends existing literature as it illustrates the interplay between personal and group reflection supporting learning about professional roles. Team interaction, especially between students and experienced professionals, was shown to help the students recognize that there were both personal and professional aspects to learn and balance in relation to patient death experiences.

이전 연구와 일치하는 이번 조사의 또 다른 특징은 참가자들이 죽음에 대해 동료 및 가까운 사람들에게 가장 자주 이야기했다는 점입니다. 이는 동료 지원이 비슷한 맥락과 지적 수준을 가진 사람들이 상호 작용하는 비공식적인 환경에서 발생하기 때문일 수 있습니다[43]. 그러나 다른 문헌에 따르면 동료, 가까운 동료 또는 선배 동료는 종종 의대생에게 실질적인 조언과 지침을 제공하는 신뢰할 수 있는 출처로 간주됩니다 [44]. 
Another feature of this investigation that aligns with previous studies is that participants most often spoke to peers and near-peers about the death. This could be due to the fact that peer support occurs in an informal environment in which people of similar context and intellectual status are interacting [43]. However, other literature has found that peers, near-peers or senior peers are often considered trusted sources of practical advice and guidance for medical students [44].

지원을 위한 제안
Suggestions for support

본 연구에서는 좋은 지원의 핵심 특성으로 타이밍을 확인했습니다. 학생들은 환자 사망을 경험하기 전과 후에 모두 지원이 필요하다고 느꼈습니다. 학생들은 사전에 의과대학에서 이러한 경험에 대비할 수 있도록 준비시켜야 한다고 생각했습니다. 학생들은 준비 과정에 죽음에 대한 실질적인 측면(예: 사망 확인 및 인증)과 정서적인 측면이 모두 포함되어야 한다고 강조했습니다. 이는 환자 사망의 실제적 측면과 정서적 측면을 모두 다루는 것이 중요하다고 밝힌 Jones 등[8]의 연구와도 일치합니다.

Our study has identified timing as being a key characteristic of good support. It was clear that students felt support was needed both before and after an experience with patient death. Beforehand, students felt that medical schools ought to prepare them for these experiences. They highlighted that the preparation process should include both the practical (e.g. death confirmation and certification) and emotional aspects around death. This aligns with work from Jones et al. [8], who also identified the importance of addressing both practical and emotional aspects of patient death.

환자 사망을 경험한 후 학생들은 경험이 많은 동료와 함께 디브리핑할 수 있는 비공식적인 기회를 중요하게 여겼으며, 이는 다른 문헌과도 일치합니다[8]. 중요한 것은 학생들이 환자 사망 경험의 전문적 측면뿐만 아니라 개인적, 정서적 측면을 성찰할 수 있는 기회의 중요성을 강조했다는 점입니다. 또한 학생들이 환자 사망 후 구조화되거나 공식화된 지원이 필요하다는 것을 확인했습니다. 이는 환자 사망 후 공식적인 토론을 통해 학생들이 자신의 경험을 이야기하고 공유할 수 있는 기회를 늘릴 수 있다는 Kelly 등[2]의 연구를 반영한 것입니다. 다른 문헌에서는 숙련된 교수진이 참여하는 정기적인 그룹 세션이 도움이 될 수 있으며[45], 슈와츠 회진[46]과 같은 비문서적 성찰의 형태로도 이루어질 수 있다고 제안합니다.
After experiencing a patient death, students valued informal opportunities to debrief with more experienced colleagues, which aligns with other literature [8]. Importantly, our study highlighted the importance of the opportunity for students to reflect on not only the professional but also the personal and emotional aspects of patient death experiences. We also identified that the students needed structured or formalized support following the death of a patient. This reflects work by Kelly et al. [2], which suggests that formalized discussions after patient death could enhance opportunities for students to talk and share their experiences. Other literature has suggested that regular group sessions where trained faculty is included may be helpful [45] and can also be done in the form of non-written reflection e.g. Schwartz rounds [46].

지원 시기를 고려하는 것 외에도 학생의 경험에 공감하는 것이 중요한 것으로 확인되었습니다. 1961년 로저스가 정의한 공감 능력[47]은 다른 사람의 감정에 민감하게 반응하고 그들과 민감하게 소통할 수 있는 능력입니다. 다른 연구에서 논의된 바와 같이, 환자의 죽음을 여러 번 목격한 의료진의 반응은 경험이 없는 의대생의 반응과 매우 다를 수 있습니다 [9]. 본 연구와 로즈-크로프 외[7]의 연구 결과(표 2 참조)에도 반영된 바와 같이, 학생들은 의료진이 죽음에 대해 너무 굳어져서 학생들과 함께 죽음을 인정할 수 있는 감수성이 부족하다고 느끼는 경우가 있었습니다. 위에서 언급했듯이 정상화는 이러한 의대생들이 대처하는 데 있어 중요한 부분이며, 교직원staff이 학생들의 경험에 공감하는 것이 이 과정에서 도움이 될 수 있습니다. 이러한 연구 결과 중 일부는 학생들이 동료 및 가까운 동료와 대화하는 것이 특히 중요하다고 강조하는 이유도 설명할 수 있습니다. 가까운 동료는 의대생이 직면하고 있는 도전과 어려움을 더 잘 이해하고 공감할 수 있으므로 더 잘 지지하고 적절한 제안을 할 수 있습니다[48, 49]. 

In addition to considering the timing of support, being empathetic to the student experience was identified as important. Empathy, defined by Rogers in 1961 [cited in [47]], is the capacity to be sensitive to what others are feeling and to be able to communicate sensitively with them. As discussed in other work, the reaction of the medical staff, who may have seen patient death many times, could be very different from that of inexperienced medical students [9]. As reflected in both our study and Rhodes-Kropf et al.’s [7] findings (see Table 2), students sometimes felt the healthcare team had become so hardened by death that they lacked the sensitivity to acknowledge it with students. As mentioned above, normalization is an important part of coping for these medical students, and staff being empathetic to students’ experiences may be helpful in this process. Some of these findings might also explain why students highlight speaking to peers and near-peers as being particularly valuable. Near-peers may be able to understand and empathize better with the challenges and difficulties medical students are facing; therefore, they are more able to support and give appropriate suggestions [48, 49].

좋은 지원의 마지막 특징은 의학교육자가 적극적인 역할을 하는 것입니다. 학생은 지원이 필요함에도 불구하고 대화를 시작하지 않을 수 있습니다. 이 결과는 Pessagno 등[3] 및 Smith-Hans 등[9]의 연구 결과와 일치하며, 학생들은 교수진이 적극적으로 시작한 지원을 높이 평가한다는 것을 보여줍니다. 일반적으로 많은 학생들이 어려움을 겪고 있어도 교수진에게 도움을 요청하지 않는 경우가 많기 때문에 교수진의 정기적이고 적극적인 지원이 도움이 되는 것으로 나타났습니다[50]. 
The last characteristic of good support is for medical educators to take an active role. A student might not initiate a conversation, even though he or she needs to be supported. This finding aligns with the results of Pessagno et al. [3] and Smith-Hans et al. [9], showing that students appreciated the support actively initiated by faculty members. In general, regular active support from faculty has been shown to be beneficial, because many students are unlikely to seek help from faculty even if they are having difficulties [50].

본 연구의 강점 및 주요 한계점
Strengths and key limitations of this research

이 연구에는 몇 가지 강점이 있습니다.

  • 첫째, 엄격한 질적 연구의 중요한 특성인 내적 일관성을 보여주기 위해 인식론, 방법론, 연구방법이 일치하도록 세심하게 설계되었습니다[51].
  • 둘째, 데이터 분석 과정에서 그룹 접근법을 사용했습니다. 따라서 데이터 분석의 신뢰성[52]과 해석의 엄밀성은 연구자 삼각측량[53]을 통해 향상되었습니다.
  • 셋째, 주제별 프레임워크 분석과 내러티브 분석 등 다양한 분석 방법을 사용했습니다. 이는 데이터 분석 삼각측량법을 채택하여 관심 있는 현상에 대한 보다 심층적이고 총체적인 이해를 도모하고 질적 연구의 타당성을 높이는 데 도움이 됩니다[54].
  • 마지막으로, 해석주의 연구 패러다임을 통해 사회 세계에 대한 다양한 해석이 존재하며 단일한 앎의 방식은 존재하지 않는다는 인식이 있습니다 [21]. 따라서 연구자의 배경과 입장이 연구에 영향을 미칠 수 있으며, 이를 해결하기 위해 반사성을 높이기 위한 여러 가지 접근 방식을 취했습니다. 또한, 연구 과정 전반에 걸쳐 지도교수 및 연구팀의 모든 의견은 연구자의 반성 일지에 기록하였습니다.

This study has several strengths. First, it was carefully designed to have an alignment between epistemology, methodology and method to show internal coherence, which is an important quality of rigorous qualitative research [51]. Second, we employed group approaches in the data analysis process. The reliability of data analysis [52] and interpretative rigor were therefore enhanced by researcher triangulation [53]. Third, multiple analytical methods, including thematic framework analysis and narrative analysis, were used. This denotes the adoption of data analysis triangulation, which helps promote a deeper and holistic understanding of an interested phenomenon and enhance the validity of qualitative research [54]. Finally, through an interpretivist research paradigm, there is a recognition that there are multiple interpretations of our social world and no single way of knowing [21]. Therefore, the researcher’s background and position could have impacted the research and a number of approaches were taken to enhance reflexivity to address this. In addition, every comment from the supervisor and research team was kept in the main researcher’s reflexive journal throughout the study process.

그럼에도 불구하고 본 연구에는 한계가 있습니다. 12명의 참가자는 표본 규모가 작고, 표본 추출 방식에 따라 극단적인 사례나 이상값과 같은 주요 인물을 놓쳤을 수 있습니다[55]. 그러나 이 연구는 질적 연구로서 가장 기억에 남는 환자의 죽음에 대한 의대생들의 경험을 심층적으로 탐구하는 것을 목표로 했습니다. Malterud 등[56]은 질적 연구자는 표본 크기와 관련하여 정보력을 고려해야 한다고 제안합니다. 우리는 정보력에 기여하는 요인이 유리하다고 주장할 수 있습니다. 우리의 데이터는 초점이 좁았던 연구 질문을 다룰 수 있었을 뿐만 아니라 그 내용도 풍부했습니다. 특정 참가자 그룹의 응답은 평균 400단어에 달하는 환자 사망 경험에 대한 내러티브였으며, 이는 우리가 받은 응답이 단순히 짧은 의견이 아님을 보여줍니다. 단어 수가 데이터의 풍부함을 직접적으로 반영하지는 않지만, 단어 수가 길수록 데이터의 풍부함의 다른 차원(예: 개인적인 응답 및 특정 지식이 있는 응답 [57, 58])과 연관성이 있습니다. 우리의 내러티브는 환자의 죽음에 대한 개인적인 성찰과 특정 의학 지식(예: 심폐소생술 카트의 준비 상태 및 사망 인증 절차)이 포함된 내용으로 채워져 있다고 주장할 수 있습니다. 따라서 통계적 대표성이나 논란의 여지가 있는 포화도 기준을 충족하는 것보다 연구 질문에 답할 수 있는 풍부한 데이터와 엄격한 분석 방법이 가장 중요한 요건입니다[56, 59].
Nevertheless, our study contains limitations. Twelve participants is a small sample size and our method of sampling could have missed some key people, such as extreme cases or outliers [55]. However, being qualitative research, this study aimed to explore deeply the medical students’ experiences around their most memorable patient death. Malterud et al. [56] suggest that qualitative researchers should consider information power in relation to sample size. We would argue that the factors which contribute to information power are favorable. Not only were our data able to address our research questions, which were narrow in focus, but the content was rich. Responses from our specific group of participants were narratives about experiences with patient death, averaging 400 words, demonstrating that the responses we received were not just short comments. Although word counts do not directly reflect data richness, there is an association between longer word counts and other dimensions of data richness (e.g. personal responses and responses with specific knowledge [57, 58]). It can be argued that our narratives were filled with personal reflections on patient death and with specific medical knowledge included (such as the readiness of the cardiopulmonary resuscitation cart and the death certification process). Therefore, the richness of data that answers the research questions and our rigorous analytical methods are a prime requirement, rather than statistical representativeness or meeting the controversial criteria of saturation [56, 59].

단일 기관에서 수행된 연구이기 때문에 광범위한 일반화에 주의를 기울여야 합니다. 질적 연구 자체는 일반화 가능성과 관련이 없지만[60], 이전 가능성은 중요합니다. 이전 가능성은 연구 결과가 다른 유사한 환경이나 맥락에서 관련성, 유용성 또는 정보를 제공할 수 있는 정도를 말합니다[53]. 이전 가능성을 높이기 위해 연구 결과와 기존 문헌의 결과를 철저히 비교하여[61], 기존 지식에 추가할 수 있는 많은 관련 사항을 발견했습니다. 또한 설정, 이론적 관점, 방법론, 표본 추출 및 분석과 같은 연구의 세부 사항을 제공하여 독자가 자신의 상황에 이 연구가 적용될 수 있는지 스스로 판단할 수 있도록 하였습니다[61]. 본 연구의 잠재적 이전 가능성은 특히 환자 사망에 관한 학생 지원 영역에서 다양한 학습자 그룹(예: 의대생, 간호대생)과 관련된 광범위한 보건의료 교육 실무에 기여할 수 있기를 바랍니다. 
Being a study conducted at a single institution, caution must be taken in making widespread generalizations. Qualitative research in itself is not concerned with generalizability [60], but transferability is important. Transferability is the extent to which research findings are relevant, useful or able to inform other similar settings or contexts [53]. To enhance transferability, comparisons between our results and those of the existing literature were thoroughly considered [61], and we found many relevant points that added to existing knowledge. Also, the study’s details, such as settings, theoretical perspective, methodology, sampling and analyses, were provided in order for readers to judge themselves whether this study can be applied to their contexts [61]. We hope that the potential transferability of this study could contribute to a wider range of healthcare education practices relevant to multiple learner groups (e.g. medical students, nursing students) especially in the area of student support regarding patient deaths.

또 다른 한계는 임상 연도 교육 전에 참가자들에게 죽음과 임종에 대한 과거 경험에 대해 명시적으로 묻지 않았다는 점입니다. 기존 문헌에 따르면 친한 친구나 가족의 죽음과 같은 개인적인 사별 경험은 사별을 얼마나 잘 처리하느냐에 따라 의대생이 환자의 죽음에 접근하는 방식에 긍정적으로 또는 부정적으로 영향을 미칠 수 있다고 합니다[62]. 또한, 임상 전 단계의 사체 해부는 학생들이 죽음에 대한 감정에 대처하고 배우는 방식에 영향을 미칠 수 있습니다[3, 7]. 설문지는 개방형이었지만, 참가자 중 임상 환자 사망 경험을 가족의 사망과 같은 개인적인 경험이나 해부학 해부 실습 경험에 연결시킨 사람은 없었습니다. 향후 연구에서는 이 부분을 보다 명확하게 조사할 수 있습니다. 
Another limitation is that we did not explicitly ask participants about past experiences with death and dying prior to the clinical year training. Existing literature suggests that previous close personal bereavement, like the death of a close friend or a family member, could both positively or negatively affect how medical students approach patient death [62], depending on how well the bereavement is processed. Furthermore, cadaveric dissection in preclinical years can affect how students cope and learn from their feelings around death [3, 7]. Although our questionnaire was open ended in nature, none of our participants connected their clinical patient death experiences with personal experiences such as a death in the family or with previous experiences in anatomy dissection. Future research could explore this area more explicitly.

향후 연구를 위한 권장 사항
Recommendations for future research

해부학 해부뿐만 아니라 향후 연구에서 주목할 만한 세 가지 핵심 영역이 있습니다. 첫째, 일부 참가자는 환자 사망 후 정서적 고통을 견뎌냈으며, 반복되는 꿈의 형태로 나타나는 방해가 되는 기억을 보고했습니다. 퍼스-코즌스 등[19]은 부정적인 감정을 견디는 것과 죽음 불안 사이의 연관성을 발견했으며, 저희는 부정적인 감정을 견디는 것과 방해가 되는 기억 사이의 잠재적 연관성을 발견했습니다. 이러한 관계를 더 탐구하는 것은 향후 연구의 영역이 될 수 있습니다. 둘째, 의대생들의 대처 경험과 마음챙김 및 정서적 민첩성과의 연관성을 확인했습니다. 스트레스와 고통 관리 측면에서 의학교육에서 마음챙김의 활용을 언급한 논문이 많이 있습니다[63, 64]. 그러나 이들 중 어느 누구도 정서적 민첩성의 일부로 환자의 죽음에 대처하기 위한 마음챙김과 연습을 어떻게 가르칠 수 있는지에 대해 구체적으로 연구하지 않았습니다. 
As well as anatomy dissection, there are three key areas worth highlighting for further research. First, some participants endured emotional pain after patient death and reported intrusive memories that came in the form of recurrent dreams. Firth-Cozens et al. [19] found an association between enduring negative emotions and death anxiety, and we found a potential link between enduring negative emotions and intrusive memories. Further exploring this relationship may be an area for future research. Second, we identified links to mindfulness and emotional agility with the coping experience of medical students. Many articles refer to the use of mindfulness in medical education in terms of stress and distress management [63, 64]. However, none of them specifically studied how mindfulness and practices for coping with patient death could be taught as a part of emotional agility.

셋째, 본 연구는 병동 직원과 의과대학이 환자 사망 후 지원할 준비가 어느 정도 되어 있는지에 대해서는 다루지 않았습니다. 이와 관련하여 의대생의 관점을 연구한 결과, 의대생들은 전공의 등 동료 및 가까운 동료의 지원을 높이 평가하는 것으로 나타났습니다. 그러나 다른 사람들의 지지적 역할에 관한 데이터는 아직 수집되지 않았습니다. 이들의 데이터는 Wear [65]가 요구한 것과 같이 기존 문헌에 풍부함을 더하고 부족한 부분을 해결할 수 있습니다. 
Third, our study has not resolved the extent to which ward staff and the medical school are ready to be supportive after patient death. We have studied medical student perspectives in this regard and found that they appreciated support from peers and near-peers, including foundation doctors. However, data from the perspectives of others concerning their supportive roles have not yet been collected. Data from them could add richness to – and address gaps in – the existing literature, such as that called for by Wear [65].

결론
Conclusion

이번 연구를 통해 의대생들이 임상 실습에서 가장 기억에 남는 환자 사망 경험에 대한 인사이트를 얻을 수 있었습니다. 여기에는 학생과 관계를 맺은 환자의 첫 사망, 예상치 못한 사망, 예상되는 사망이 포함되었습니다. 그 결과 학생들은 다양한 내적 및 외적 대처 전략을 사용한다는 사실을 발견했습니다. 이러한 대처 전략 중 학생들의 성찰은 환자의 죽음에서 얻은 교훈을 발전시키는 데 중요한 것으로 나타났습니다. 일부 학생들은 성찰을 통해 이 두 가지 측면의 균형을 맞출 수 있었고 사건에 대해 성숙하게 행동하는 법을 배웠습니다. 또 다른 학생들은 처음에는 부정적인 감정을 가졌지만 스스로 귀중한 교훈을 얻을 수 있다는 것을 보여주며 정서적 민첩성을 보여주었습니다. 중요한 것은 병동 직원의 지원이 학생들이 경험의 개인적/정서적 측면과 직업적 측면을 모두 인식하도록 도와 반성을 강화하는 데 도움이 되었다는 점입니다. 그러나 참가자들은 성찰적 학습 환경에서 성찰적 학습자로서 더 나은 지원을 받을 수 있는 방법에 대해서도 목소리를 냈습니다. 
Our research provides insights into medical students’ most memorable experiences with patient death in clinical placements. This included the first death, unexpected death and expected death of a patient with whom the student had a connection. We found that students employed many internal and external coping strategies. Within those coping strategies, student reflections were shown to be important for the development of lessons learned from patient death. Through their reflections, some students were able to balance these two sides and learned to act upon the incident maturely. Others showed they could come up with valuable lessons on their own, despite having negative feelings at the beginning, demonstrating emotional agility. Importantly, our results also showed that support from ward staff often enhanced reflections by helping students recognize both personal/emotional and professional aspects of the experience. Yet, participants also gave voice to how they could be better supported as reflective learners in a reflective learning environment.

이러한 조사 결과를 통해 의대생들이 경험을 통해 배우고 성장하는 데 있어 성찰의 중요성이 강조되기를 바랍니다. 학부 의대 교수들은 의대생이 전문가로 성장하는 과정에서 유용한 대처 전략을 갖추도록 하는 것을 궁극적인 목표로 삼아 환자의 죽음 경험에 대한 성찰적이고 지원적인 환경을 조성하는 것을 고려할 것을 제안합니다. 
We hope that these finding will highlight the importance of reflection for medical students to learn and grow from experience. We suggest that undergraduate medical teachers consider promoting a reflective and supportive environment around patient death experiences with the ultimate aim of equipping medical students with helpful coping strategies as they develop as professionals in their own right.

 


BMC Med Educ. 2019 Dec 4;19(1):451. doi: 10.1186/s12909-019-1871-9.

Learning and coping through reflection: exploring patient death experiences of medical students

Affiliations collapse

1Bangkok Metropolitan Administration Hospital, 514 Luang Road, Pomprabsattrupai District, Bangkok, 10100, Thailand.

2Office of Medical Education, Billings Clinic, 801 N. 29th St. Billings, Montana, 59101, USA.

3Business School, Edinburgh Napier University, 219 Colinton Road, Edinburgh, EH14 1DJ, UK.

4Centre for Anatomy and Human Identification, University of Dundee, Dow Street, Dundee, DD1 5EH, UK. a.t.wilkinson@dundee.ac.uk.

PMID: 31801494

PMCID: PMC6894273

DOI: 10.1186/s12909-019-1871-9

Free PMC article

 

Abstract

Background: Existing studies have explored many aspects of medical students' experiences of patient death and propose the importance of faculty support for coping. However, UK-based literature on this subject and research concerning learning through reflection as part of coping are relatively limited. This study, through the lens of reflection, aims to explore students' experiences with patient death in a UK context. These include coping strategies, support from faculty following patient death and the relationship between these experiences and learning. Our research questions were: How do medical students cope with and learn from their experiences?How does support from ward staff and the medical school help them cope with and learn from these experiences?How can students best be supported following patient death?

Methods: We employed narrative inquiry to explore how medical students made sense of their experiences of patient death. Twelve students participated in our study via an online narrative questionnaire. Thematic analysis and complementary narrative analysis of an exemplar were applied to address our research aim.

Results: Coping strategies comprised internal and external strategies. Internal strategies included (1) re-interpretation of the death into a meaningful experience including lessons learned; (2) normalization; (3) staying busy and (4) enduring negative emotions. External strategies included speaking to someone, which was found to influence normalization, and lessons learned. Both satisfactory and unsatisfactory support from ward staff was identified. Satisfactory support was characterized by the inclusion of emotional and professional support. Unsatisfactory support was often characterized by a lack of emotional support. Narrative analysis further demonstrated how the experience with patient death was re-interpreted meaningfully. Students suggested that support should be structured, active, sensitive, and include peers and near-peers.

Conclusion: Many coping strategies, internal and external, were employed in students' experiences with patient death. Student reflections, enhanced by support from ward staff, were shown to be important for learning from patient death. We encourage faculty to have regular sessions in which medical students can reflect on the death incident and discuss appropriately with others, including peers and near-peers.

Keywords: Coping strategies; Medical students; Patient death; Reflection; Student support.

데이터로서 스토리, 스토리로서 데이터: 임상교육에서 내러티브 탐구의 의미찾기(Med Educ, 2005)
Stories as data, data as stories: making sense of narrative inquiry in clinical education* 
Alan Bleakley

숫자를 넘어서는 통찰력
Getting beyond the numbers

마크 해든의 소설 '밤에 일어난 개들의 기이한 사건'은 자폐 스펙트럼에 속하는 아스퍼거 증후군을 앓고 있는 소년의 마음을 들여다볼 수 있는 통찰력을 제공합니다. '수학에 대해서는 아주 많이 알지만 인간에 대해서는 거의 알지 못한다'는 소년의 세계는 사람이 아닌 숫자입니다. 주인공 크리스토퍼는 자신의 욕망에 대한 메시지를 담고 있는 타인의 이야기에 공감하지 못하는 놀라운 능력을 보여줍니다. 이와 같은 내러티브 통찰력의 부족은 의학계에서 체계적으로 배양된 것으로 볼 수 있는데, 의료 사례 연구의 특징적인 평면적이고 분리된 설명2은 의료 행위가 '이야기 속에 자리 잡고 있는' 현실을 감추는 역할을 합니다.3 또한 이러한 이야기는 연구를 위한 풍부한 소스를 제공합니다. 이야기의 중요성을 부정하는 의학계의 자폐증에 대한 도전이 제기되면서 임상 교육 연구자들은 '숫자가 아니라 환자를 치료하라'는 격언을 실천하기 위해 이야기 탐구 방법론을 연마한 인류학,4 심리학5 및 사회학6과 같은 학문을 활용하기 시작했습니다. 이는 숫자 치료의 가치를 부정하는 것이 아닙니다. 해든의 소설에서 크리스토퍼가 말했듯이 '많은 것이 미스터리입니다. 하지만 그렇다고 해답이 없다는 뜻은 아닙니다. 단지 과학자들이 아직 답을 찾지 못했을 뿐입니다. 
Mark Haddon's1 novel The Curious Incident of the Dog in the Night-Time offers an insight into the mind of a boy suffering from Asperger's syndrome, a condition on the autistic spectrum. The boy's world is numbers, not people: ‘he knows a very great deal about maths and very little about human beings’. The central character, Christopher, shows a striking inability to empathise with the narrative accounts of others that carry messages about their desires. An equivalent lack of narrative acumen can be seen to have been systematically cultivated in medicine, where the telling symptom is the characteristically flat, detached account of the medical case study2 serving to conceal the reality that medical practice is ‘entrenched in stories’.3 Further, such stories afford rich sources for research. As a challenge emerges to medicine's self-imposed institutional autism that is a denial of the importance of story, clinical education researchers are beginning to draw on disciplines such as anthropology,4 psychology5 and sociology6 that have honed their narrative inquiry methodologies, to address the maxim: ‘treat the patient, not the numbers’. This is not to deny the value of treating numbers. As Christopher, in Haddon's novel, says: ‘Lots of things are mysteries. But that doesn’t mean there isn't an answer to them. It's just that scientists haven't found the answer yet.'

이전에는 인문학의 영역으로 여겨졌던 건강과 질병에 대해 정서적, 관계적, 윤리적, 상상력적인 측면에 초점을 맞춘 질적 연구 형태가 등장했습니다. 일부 내러티브 연구자들은 질병에 대한 환자의 이야기와 같은 이야기를 데이터로 수집하고 분석하는 데 관심이 있으며,6,7 다른 연구자들은 삶의 경험 데이터를 내러티브 형식으로 재구성하여 연구 결과물로 제시합니다.8,9 데이터로서의 이야기, 이야기로서의 데이터 - 내러티브 탐구의 세계에 오신 것을 환영합니다. 
Qualitative forms of research have come to engage with health and illness in ways that were considered previously the province of the humanities, focusing upon affective, relational, ethical and imaginative aspects of experience. Some narrative researchers are interested in taking stories, such as patients' accounts of illness, and analysing them as data,6,7 while other researchers re-present the data of life experience in narrative form as a research product.8,9 Stories as data and data as stories − welcome to the world of narrative inquiry.

단일 사례에 초점을 맞춘 '내러티브 기반 의학'10은 인구 연구에서 도출되고 정량적이며 일반화 가능성을 추구하는 지배적인 증거 기반 접근법을 보완할 수 있습니다. 후일러9는 뉴멕시코 대학교 시체 안치소에서 감사 연구 프로젝트를 수행한 사례를 소개합니다. 그는 사망 연도별로 정리된 법정 크기의 마닐라 봉투 더미를 뒤지며 '자동' 모드로 전환하여 컴퓨터에 데이터를 입력하고 '잔여물을 증류'하는 작업을 했습니다. 하일러는 '숲속의 여인이 저지른 행위'라는 범죄 현장 사진에 적나라하게 드러난 비극에 마취된 듯 눈앞에 펼쳐진 비극에 빠져들었습니다. 자살로 의심되는 숲속의 죽은 여인의 사진이 주는 감정적 충격은 그의 '감사' 사고방식과 냉정하게 일치하는 객관적인 의학적 설명과 극명한 대조를 이룹니다: '받은 시신은 진행 단계에있는 중년 백인 여성의 시신입니다 ...'. 갑자기 하일러는 자살한 할머니를 떠올립니다. 그는 이를 떨쳐버리고 당면한 과제에 집중합니다: '그만해, 이건 설명적 통계, 역학이야...'라고 스스로에게 말했습니다.
A ‘narrative-based medicine’10 focused on the single case can complement the dominant evidence-based approach that draws from population studies, is quantitative and seeks generalisability. Huyler9 describes carrying out an audit research project at the morgue of the University of New Mexico. Working through a pile of legal-sized manila envelopes arranged by year of death, he would go into ‘automatic’ mode, typing data into a computer, ‘distilling them to a residue’. Huyler found himself anaesthetised to the tragedies that spread out before him, explicit in the photographs of crime scenes: ‘the act done … the woman in the woods’. The emotional impact of the photograph of the dead woman in the woods, a suspected suicide, contrasts starkly with the concurrent, objective medical description that chillingly correlates with his ‘audit’ frame of mind: ‘The body received is that of a middle-aged Caucasian woman in the advanced stages of…’. Suddenly Huyler thinks of his grandmother, who also committed suicide. He shrugs this off and stiffens to the task at hand: ‘Enough of that, I told myself …This is descriptive statistics, epidemiology …’.

질적 연구는 종종 '하드' 과학과 달리 '소프트'하다는 고정관념이 있지만,11 내러티브 조사의 소프트 데이터는 하드한 현실을 조명할 수 있습니다. 예를 들어, 리치와 그레이(Rich와 Grey)12는 '숫자를 넘어서'라는 부제가 붙은 연구에서 '관통 폭력'을 당한 48명의 젊은 흑인 생존자에게 외상 수술이 미친 영향을 고려했습니다. 심층 인터뷰를 통해 사회적으로 소외된 환자들의 삶에 대한 극적인 통찰력을 제공하는 내러티브 스토리를 발굴했으며, 참가자들은 그들의 세계에 진정한 관심을 보이는 연구자들을 통해 힘을 얻었습니다. 객관적인 이환율과 사망률 데이터는 특징적으로 얼굴이 드러나지 않지만, 내러티브 조사는 사람을 대상으로 하는 연구가 아니라 사람에 대한 연구로서 의도적인 개입을 통해 연구 대상자를 개인화할 뿐만 아니라 능동적으로 참여시키려고 노력하는 경우가 많습니다. 
Qualitative studies are often stereotyped as ‘soft’, in contrast to the ‘hard’ sciences11 but narrative inquiry's soft data can illuminate hard realities. For example, Rich and Grey,12 in a study subtitled ‘getting beyond the numbers’, consider the impact of trauma surgery on 48 young, black survivors of ‘penetrating violence’. In-depth interviews stimulated narrative accounts that give dramatic insight into the lives of these socially marginalised patients, while the participants are empowered through researchers showing a genuine interest in their worlds. While objective morbidity and mortality data characteristically remain faceless, narrative inquiry often seeks not only to personalise but also to engage proactively with its research population through deliberate intervention, as research with, not on, people.

양적 대 질적 논쟁을 넘어, 두 가지 질적 접근법 사이의 명백한 긴장을 다룸으로써 임상 교육에서 내러티브 탐구 분야를 이해할 수 있습니다.

  • 첫 번째는 내러티브의 구조를 분석하고 분류를 도출하는 방법입니다.
    • '이야기에 대해 생각하기'6를 하는 이 접근법은 연구자가 분석적 사고방식을 채택해야 하며, 원시 데이터에서 귀납적으로 범주를 도출하거나 설명 또는 조명을 위해 내러티브에 다시 범주를 적용하는 것을 포함합니다.
  • 두 번째는 내러티브에 대한 보다 총체적이고 통합적인 이해를 추구하는 방법으로, 주어진 내러티브에 공감적으로 '들어가거나' 구조적 분석에서 놓칠 수 있는 요소를 포착하기 위해 연구 결과물로서 스토리를 만드는 것입니다.
    • 이러한 '이야기로 사고하기'6의 접근 방식은 수사학과 같은 문학적 장치에 대한 응용 지식으로서 '내러티브 역량'13을 종합하고 개발할 수 있는 연구자를 필요로 합니다. 이야기에 대한 분석적 접근이 환자를 객관화할 수 있다면, 종합적 접근은 이야기라는 매체를 통해 연구자와 실무자를 환자의 세계에 더 가까이 다가가게 하고, 은유적으로 청진기 역할을 한다고 주장합니다. 

Beyond the quantitative vs. qualitative debate, we can make sense of the field of narrative inquiry in clinical education by addressing an explicit tension between two qualitative approaches.

  • First are methods that analyse the structure of narratives and derive classifications. This approach of ‘thinking about stories6 requires a researcher to adopt an analytical mindset and involves deriving categories inductively from raw data, and/or applying such categories back to narratives for explanation or illumination.
  • Second are methods that seek a more holistic and integrative understanding of narratives, either empathically ‘entering’ a given narrative, or creating a story as research product as a way of capturing elements otherwise lost to a structural analysis. This approach of ‘thinking with stories6 requires a researcher who can synthesise and has developed ‘narrative competence’13 as applied knowledge of literary devices such as rhetoric. Where analytical approaches to stories may lead to objectifying the patient, approaches of synthesis claim to bring researchers and practitioners closer to the patient's world through the medium of story, acting − metaphorically − as stethoscope.

내러티브 연구에 대한 이러한 서로 다른 접근 방식 사이의 긴장은 의학에서 분석의 수준으로서 집단과 단일 사례 사이의 익숙한 구분을 반영합니다. 의학교육이 주로 과학적 방법을 지향하는 경우, 이야기에 대한 사고가 이야기로 생각하는 것보다 특권을 누릴 수 있습니다. 실제로 모리스14는 내러티브 탐구에서 이야기를 통한 사고는 '철저하게 무시'된다고 말합니다. 예를 들어, 의대생들은 자신이 들은 환자의 이야기를 분석적이고 양식화된 설명2으로 변환하는 방법을 빠르게 배우는데, 이는 객관화된 임상적 시선과 청각적으로 동등한 것입니다. 이러한 편향에 대응하기 위해 '균형 잡힌' 의학 교육은 임상 전문 지식내러티브 통찰력, 즉 전자는 과학에 기반하고 후자는 인문학에 기반한 통찰력을 종합적으로 제공할 수 있습니다.
The tension between these differing approaches to narrative research echoes the familiar distinction in medicine between population and the single case as levels of analysis. Where medical education is orientated primarily to scientific method, thinking about stories may be privileged over thinking with stories. Indeed, Morris14 suggests that thinking with stories is ‘thoroughly neglected’ in narrative inquiry. For example, medical students rapidly learn to translate the patients' stories they hear into analytical and stylised accounts2 as an aural equivalent of the objectifying clinical gaze. To counter this bias, a ‘rounded’ medical education could synthesise clinical expertise and narrative acumen − the former based on science, the latter on the humanities.

내러티브란 무엇인가요?
What is a narrative?

브루너15는 과학적 앎내러티브적 앎은 근본적으로 다르다고 주장합니다. 과학이 진리를 규명하는 데 관심이 있다면, 내러티브는 경험에 의미를 부여하는 데 관심이 있습니다. '내러티브'(L. narrare)는 '알다'라는 뜻으로, 스토리텔링은 단순히 사건의 투명한 서술이 아니라 지식 생산과 경험의 형성을 포함합니다.3 현대의 내러티브 탐구는 1980년대 사회과학에 대한 재고가 활발했던 시기에 사빈16이 과학의 지배적인 기계 은유에 도전하면서 내러티브를 인간 활동의 근본 조직 원리라고 설명하면서 공고화되었습니다. 내러티브에 대한 정통적인 견해는 '일련의 사건에 대한 조직화된 해석'이라는 Murray의 정의17를 따릅니다. 이야기무질서한 사건의 시리즈로 경험될 수 있는 것에 시간적 질서를 부여합니다.18 플롯사건을 시퀀스로 배치하여 내러티브를 구성하지만, 일반적으로 주체, 의도, 수단, 목표 및 설정과 같은 이야기 요소 간의 불일치를 통해 긴장감을 조성합니다. 따라서 내러티브는 종종 기대에 반하여 익숙한 것을 낯설게 만들며, 어떤 이야기도 단일한 읽기나 의미를 갖지 않습니다.3 Strawson19은 내러티브 연구에서 모든 경험을 이야기로 취급하는 경향에 대해 경고하며, 일부 개인은 삶을 '통시적'(시간)이 아닌 '에피소드적'(공간)으로 경험할 수 있다고 말합니다.
Bruner15 argues that scientific and narrative ways of knowing are fundamentally different. Where science concerns itself with the establishment of truth, narrative's concern is to endow experience with meaning. ‘Narrative’ (L. narrare) means ‘to know’ and storytelling involves knowledge production and shaping of experience, not simply transparent recounting of events.3 Contemporary narrative inquiry was consolidated in the 1980s, during a fertile period of rethinking the social sciences, where Sarbin16 challenged the dominant machine metaphor of science, describing narrative as a root organising principle of human activity. The orthodox view of narrative follows Murray's17 definition: ‘an organized interpretation of a sequence of events’. Story brings temporal order to what would otherwise be experienced as a series of chaotic events.18 Plot structures narrative by putting events into a sequence, but tension is usually created through misfit between the elements of story such as agency, intention, means, goal and setting. Narratives thus often run counter to expectations, making the familiar unfamiliar, and no story has a single reading or meaning.3 Strawson19 warns against a tendency in narrative studies to treat all experience as story, where some individuals may experience life as ‘episodic’ (in space) rather than ‘diachronic’ (in time).

  • 내러티브에 대한 초기 학문적 연구는 내러티브의 내용, 즉 내러티브의 내적 구조에 관심을 두어 '내러톨로지'라는 용어를 탄생시켰습니다.18 내러톨로지스트들은 시간 속에서 전개되는 이야기의 패턴이나 인물과 행동 사이의 관계를 분석하여 이야기를 일련의 형식적 규칙과 전형적인 구조로 축소시켰습니다. 라보프20는 '장면 설정'과 같은 내러티브 요소의 기능에 따라 내러티브를 기능적으로 분류하면서 이러한 전통을 발전시켰습니다.
  • 그러나 다른 이론가들은 이야기의 구조에 대한 추상적 분석에는 관심이 적었고, 내러티브를 사회적 맥락에서 이야기가 수사학적으로 사용되는 방식과 같은 담론으로 취급하는 것을 선호했습니다. (수사학은 은유와 같은 장치를 통해 언어를 표현적이고 설득력 있게 사용하는 것을 말합니다.) 이러한 담론적 접근 방식은 스토리의 내용보다는 방식, 즉 사건보다는 스토리의 의미가 연구의 초점을 제공하는 것으로 생각할 수 있습니다.
  • 구조적 접근이 이야기를 투명하고 설명적인 것으로 취급하는 경향이 있다면, 현대 담론적 접근은 이야기를 사회적 의미를 구성하는 것으로 봅니다.3,21,22 앞서 소개한 이야기에 대해 생각하는 것이야기로 생각하는 것 사이의 긴장, 즉 내용으로서의 이야기(구조) 대 과정으로서의 이야기(담론) 사이의 긴장을 '이야기의 이중 논리'라고 부릅니다.22
  • Early academic work on narratives concerned itself with story content − the what, or internal structure, of narrative − giving rise to the term ‘narratology’.18 From analyses of patterns of stories unfolding in time, or the relations between characters and actions, narratologists reduced stories to a set of formal rules and typical structures. Labov20 refined this tradition, categorising narratives functionally according to what an element of narrative does, such as ‘scene-setting’.
  • Other theorists, however, were less interested in abstract analysis of the structure of story, preferring to treat narrative as discourse − for example, how stories are used rhetorically in social contexts. (Rhetoric is the expressive and persuasive use of language through devices such as metaphor.) This discursive approach can be thought of as the way, rather than the what, of stories, where the meaning of a story offers the focus for research, rather than events.
  • Where structural approaches tend to treat story as transparent and descriptive, contemporary discourse approaches see narratives as constructing social meanings.3,21,22 Echoing the tension, introduced earlier, between thinking about a story and thinking with a story, story as content (structure) vs. story as process (discourse) has been termed the ‘double logic of narrative’.22

에드워즈21은 '내러티브에 대한 연구는... 특정 이야기 내용이... 이야기 속에서 어떻게 사회적 행동을 수행하는지를 다루기보다는... 내러티브 구조의 일반화된 유형과 범주를 추구하는 경향이 있다'는 분석 중심의 연구로 인해 내러티브의 수사학적 목적의 중요성이 놓치고 있다고 지적합니다. 분석적 접근 방식은 이상화 경향이 있어 연구자들이 이야기를 미리 설정된 범주에 억지로 끼워 맞추도록 부추기며, 대안에 대응하여 자신의 신뢰성을 수사학적으로 관리하는 텍스트와 같이 특정 사회적 상황에서 특정 내러티브가 어떻게 작동하는지를 놓치게 됩니다. 예를 들어, 서로 다른 전문 분야의 의사들이 작성한 의뢰서 및 상담서의 수사학에 대한 연구에서 링가드 등은23 정신과 의사가 전문직 협회의 육성을 선호하는 의료-법적 문제에 대한 외과 의사의 편견과 같이 특정 관점을 홍보하는 데 사용되는 내용과 스타일의 차이에 주목했습니다. 이러한 편지는 중립적인 문서가 아니라 입장을 가정하고 저항을 예측하며 협력을 촉진하는 텍스트입니다. 의학교육에서 수사학은 세 가지 주요 영역에서 그 역할에 대해 연구되어 왔습니다.26,27

  • 상호작용 관리,24 
  • 은유를 사용한 임상적 판단,25 
  • 정체성 구축이라는 

Edwards21 suggests that the importance of narrative's rhetorical purpose is missed by analytically orientated research where ‘studies of narrative… have tended to pursue generalized types and categories of narrative structure, rather than dealing with how specific story content… may perform social actions in-the-telling’. Analytical approaches tend to idealisation, encouraging researchers to force stories into preset categories, again missing how specific narratives work for specific social occasions, such as a text rhetorically managing its own credibility by countering alternatives. For example, in a study of the rhetoric of doctors' referral and consultation letters from differing specialties, Lingard et al.23 note differences in content and style used to promote certain favoured perspectives, such as surgeons' biases to medico-legal issues where psychiatrists favoured the cultivation of professional associations. Such letters are not neutral documents, but texts assuming positions, predicting resistance and promoting collaboration. In medical education, rhetoric has been studied for its role in 3 main areas:

  • management of interactions,24 
  • clinical judgements employing metaphor25 and
  • identity construction.26,27

내러티브 방법
Narrative methods

이 섹션에서는 내러티브 조사에 대한 주요 방법론적 접근법을 비판적으로 살펴봅니다. 폴킹혼28은 연구 데이터를 수치, 짧은 형식, 내러티브의 세 가지 유형으로 구분합니다. 연구 질문이 공식화되면 연구자는 연구 질문을 다루기 위한 적절한 방법론을 결정해야 합니다. 내러티브 방법은 만성 질환과 같이 시간에 따른 경험을 연구하는 데 특히 적합합니다. 
This section examines critically the main methodological approaches to narrative inquiry. Polkinghorne28 distinguishes between three kinds of research data: numerical, short form and narrative. Once a research question has been formulated, the researcher must then decide on the appropriate methodology to address the research question. Narrative methods are particularly appropriate for researching experiences through time, such as chronic illness.

데이터 생성
Data generation

데이터는 비디오 또는 오디오 녹취록, 자연스러운 형태의 의사소통에 대한 현장 기록, 자극 회상과 같은 다양한 형태의 인터뷰, 사례 노트와 같은 인공물을 포함한 글쓰기 사례를 통해 생성할 수 있습니다. 확장 인터뷰는 내러티브 데이터를 생성하는 데 정기적으로 사용되지만 인터뷰 자체가 비판적으로 고려되는 경우는 드물며, 연구자들은 인터뷰가 어떻게 계획되고 수행되었는지 또는 효과적인 인터뷰에 필요한 자질에 대해 언급하지 않습니다.29,30 인터뷰는 너무 자주 목적을 위한 수단으로 취급되며, 지식(데이터 생성)을 적극적으로 구성하기 위한 사회적 매체가 아니라 정보 수집(데이터 수집)의 투명한 과정으로 취급됩니다. 미슬러30는 인터뷰의 몇 가지 잠재적 문제점으로 피험자의 이야기 흐름을 방해하여 내러티브가 단절되고, 인터뷰의 어떤 부분을 보고할지 선택할 때 편견이 생길 수 있다는 점을 지적합니다. 
Data can be generated through video or audio transcripts; field notes of naturalistic forms of communication; differing forms of interview such as stimulated recall; and examples of writing, including artefacts such as case notes. Extended interview is employed regularly in generating narrative data, but the interview itself is rarely considered critically, where researchers fail to comment on how the interview was planned and conducted, or on the qualities needed for effective interviewing.29,30 Interviewing is too often treated as a means to an end, and as a transparent process of information-gathering (data collection), rather than a social medium for active construction of knowledge (data generation). Mishler30 notes several potential problems with interviewing, such as stemming the flow of the subject's talk so that the narrative is fractured, and bias in selection of which parts of the interview are reported.

데이터 분석
Data analysis

후투넨 등31은 '내러티브 연구는 하나의 방법이 아니라 오히려 아메바처럼 느슨한 참조 프레임'이라고 도발적으로 제안합니다. 이와는 대조적으로, 내러티브 연구를 체계화한 설득력 있는 설명들이 많이 있습니다.6,7,32,33 내러티브 방법론은 '연구라기보다는 예술'이며 '거의 가르칠 수 없다'고 제안하지만, Lieblich 등은 내러티브 분석을 위한 명확한 틀을 제시하면서 전체론적 대 범주적, 내용 대 형식의 두 차원을 격자로 결합하여 네 가지 접근법을 형성할 수 있다고 말합니다(그림 1).

  • 전체론적 접근법은 문화와 역사에 맥락을 둔 전체로서의 이야기를 취하고 전체적인 패턴이나 지도적 은유를 파악하려고 시도하는 반면,
  • 범주적 접근법은 특정 에피소드를 분석합니다.
  • 내용은 내러티브에서 일어나는 일이고
  • 형식은 어떤 일이 일어나는 방식입니다. 

 

Huttunen et al.31 suggest, provocatively, that ‘narrative research is not a method… Rather, it is a loose frame of reference… amoeba-like’. In contrast, a number of eloquent accounts systematise narrative research.6,7,32,33 Despite suggesting that narrative methodology is ‘more art than research’ and ‘can hardly be taught’, Lieblich et al.33 proceed to offer a clear framework for narrative analysis, invoking two dimensions − holistic vs. categorical and content vs. form − that can be combined as a grid to form four approaches (Fig. 1). Holistic approaches take a story as a whole, contextualised in a culture and history, and attempt to grasp the overall pattern or guiding metaphors, where categorical approaches dissect particular episodes. Content is what happens in a narrative, form is how something happens.

폴킹혼28은 '내러티브의 분석'과 '내러티브적 분석'을 구분합니다. 내러티브적 분석은 실제로 내러티브를 통해 경험과 아이디어를 종합하는 과정이며, 여기서는 '내러티브를 통한 종합'으로 재구성합니다('내러티브 종합'은 무작위 대조 시험 이외의 방법으로 생산된 연구 데이터를 종합하는 방법으로 특정한 의미를 획득한 용어입니다). 내러티브의 복잡한 분야를 분석과 종합의 접근 방식 간의 단순한 대립으로 축소할 수는 없습니다. 그러나 이 관계를 생산적인 긴장으로 표현하면 내러티브 방법에 관심이 있는 의학교육자가 채택된 접근법의 한계를 비판적으로 고려하고, 거부된 경쟁 접근법도 제공할 수 있는 것을 참조하는 데 도움이 될 수 있습니다. 자물쇠와 열쇠의 비유를 통해 분석과 종합의 접근 방식은 서로 다른 것처럼 보이지만 함께 하나의 단위를 구성하며 혼합 방법론의 가치를 시사합니다. 
Polkinghorne28 distinguishes between the ‘analysis of narratives’ and ‘narrative analysis’. Narrative analysis is actually a process of synthesis of experience and ideas through narrative, and is reconfigured here as synthesis through narrative’ (rather than ‘narrative synthesis’ − a term that has acquired a particular meaning as a way of synthesising research data produced by methods other than randomised controlled trials). The complex field of narrative cannot be reduced to a simple opposition between approaches of analysis and synthesis. However, articulating this relationship as a productive tension could help medical educators interested in narrative methods to critically consider the limitations of an adopted approach by reference to what the rejected, competing approach can also offer. Drawing on the analogy of lock and key, approaches of analysis and synthesis look different apart but constitute a unit together, suggesting the value of mixed methodologies.

내러티브의 분석: 스토리에 대해 생각하기
The analysis of narratives: thinking about a story

원시 내러티브 데이터에서 귀납적으로 카테고리 생성하기
Inductively producing categories from the raw narrative data

원시 내러티브에서 귀납적으로 범주를 도출하여 추가 조사를 위한 관리 가능한 데이터를 제공할 수 있으며, 종종 프레임워크 또는 유형학을 다시 적용하여 단일 사례를 조명할 수 있습니다. 그 예로 Frank가 제시한 '질병 내러티브'의 세 가지 유형인 '회복', '혼돈', '탐구'를 들 수 있습니다. 회복 이야기는 질병을 일시적인 것으로 합리화하고, 혼돈 이야기는 상황에 압도당하는 것을 묘사하며, 탐구 이야기는 질병을 변화의 기회로 받아들이는 것을 묘사합니다. 프랭크는 '지저분하게' 이야기되는 질병 이야기는 이러한 범주에 깔끔하게 속하지 않으며, 한 사람의 account에서 세 가지 범주가 모두 작용할 수 있다고 경고합니다. 만약 그렇다면 프랭크의 방식은 그저 유용한 휴리스틱에 지나지 않을 수 있다. 그리고 그는 개별 사례의 특수성이 유형학에 포함될 수 있다는 점을 최초로 지적한 사람이다. 그러나 그는 개인이 복잡한 이야기를 제시하는 경우, 실무자와 연구자가 어떤 종류의 틀 없이는 '실타래를 정리'하기 어렵다고 주장합니다. 중요한 것은 '(유형학의) 장점은 아픈 사람들이 말하는 이야기에 세심한 주의를 기울이게 하고, 궁극적으로는 아픈 사람들의 이야기를 경청하는 데 도움을 준다는 점'이라고 프랭크는 주장합니다. 그러나 이야기에 직접적으로 몰입하는 것이 아니라 추상적인 분석 프레임이 어떻게 더 공감적으로 경청할 수 있도록 준비시키는지는 알기 어렵습니다.
Categories may be drawn inductively from the raw narratives to provide manageable data for further investigation − often a framework or typology that can then be applied back to illuminate single cases. An example of this is Frank's6 three types of ‘illness narratives’: ‘restitution’, ‘chaos’ and ‘quest’. Restitution stories rationalise illness as transitory; chaos stories describe being overwhelmed by the situation; and quest stories describe acceptance of illness as opportunity for change. Frank warns that ‘messy’ recounted illness stories do not fall neatly into such categories and all 3 categories may be at work in any one person's account. Frank's scheme may then offer no more than a useful heuristic, and he is the first to note that particularity of the individual story may be subsumed in the typology. However, he argues that where individuals present messy stories, it is difficult for practitioners and researchers to ‘sort out the threads’ without some kind of framework. Importantly, Frank claims that ‘The advantage (of typologies) is to encourage close attention to the stories ill persons tell; ultimately, to aid listening to the ill.’ It is, however, difficult to see how an abstract analytical frame, rather than direct immersion in a story, prepares one to listen more empathically.

Bal18은 '분류, 유형화... 모두 혼돈-불안에 대한 치료법으로서 매우 좋지만, 과연 어떤 통찰력을 얻을 수 있을까요? '라고 경고합니다. 분류 방법은 여전히 '다루기 힘든' 데이터를 정리할 때 자연과학에서 영감을 얻습니다. 일반적으로 내러티브는 시간에 따라 전개되는 통시적으로 나타납니다. 통시적 내러티브 데이터는 범주 분석을 거치면 역사적 또는 발전적 차원이 없는 공시적 데이터로 축소됩니다. 예를 들어, 내러티브 인터뷰 데이터에 대한 Crossley의5 분석은 HIV 양성 판정을 받고 5년 이상 살아온 38명이 사용한 생존 전략을 자세히 설명합니다. 데이터에서 '현재에 대한 철학을 가지고 살기'(낙관적), '미래에 대한 철학을 가지고 살기'(거짓 낙관적), '공허한 현재에 살기'(비관적)라는 세 가지 전략이 '시간적 지향성'의 뚜렷한 범주로 나타났습니다. 이러한 범주는 위에서 설명한 Frank6의 범주와 매우 유사합니다.
Bal18 warns that ‘classification, typology… is all very nice as a remedy to chaos-anxiety, but what insights does it yield?’. Categorisation methods still look to the natural sciences for inspiration in organising otherwise ‘unruly’ data. Typically, narrative presents diachronically, unfolding in time. When subjected to category analysis, the diachronic narrative data is collapsed into synchronic data with no historical or developmental dimension. For example, Crossley's5 analysis of narrative interview data details the survival strategies used by 38 people who had been living with HIV positive diagnosis for at least 5 years. Three strategies emerged from the data as distinct categories of ‘temporal orientation’: ‘living with a philosophy of the present’ (optimistic); ‘living in the future’ (falsely optimistic); and ‘living in the empty present’ (pessimistic). These categories conveniently resemble those of Frank6 described above.

원시 내러티브 데이터에 카테고리 적용하기
Applying categories to the raw narrative data

원시 데이터에서 귀납적으로 도출된 범주는 이제 설명력을 주장할 수 있지만 종종 단순한 설명에 불과한 새로운 사례에 적용되는 프레임워크가 됩니다. 위의 Crossley의 범주는 Gergen과 Gergen의34 만성 질환의 시간적 모델과 수렴하여 퇴행성, 진행성, 안정성의 세 가지 내러티브를 다시 설명합니다. Murray17는 이 체계를 유방암 생존자의 이야기에 적용하여 세 가지 전략을 설명합니다:

  • '안정적/퇴행적': 암을 고난의 삶에서 또 다른 좌절로 구성하는 이야기,
  • '진보적': 질병을 기회로 여기고 종종 믿음을 찾거나 현재에 새롭게 집중하는 이야기,
  • '안정적': 질병을 금욕적으로 받아들이고 통합하여 삶을 이어나가는 이야기입니다.

이러한 이야기는 절망, 희망, 수용이라는 핵심적인 은유를 드러냅니다. 머레이는 다발성 경화증 환자에게서도 같은 패턴을 볼 수 있다고 주장합니다. 작은 연구 영역 내에서 이러한 세 가지 분류 체계가 반복되는 것은 연구자들이 데이터에서 작동하는 어떤 원형적 패턴을 발견하는 것이 아니라 (모델의 교차 감염을 통해) 유형학을 구성하고 있을 수 있음을 시사합니다. 
Categories inductively derived from raw data now become frameworks applied to new cases that may claim explanatory power but are often merely descriptive. Crossley's categories, above, converge with Gergen and Gergen's34 temporal model of chronic illness, again describing three kinds of narratives: regressive, progressive and stable. Murray17 applies this scheme to stories of breast cancer survivors to delineate three strategies:

  • ‘stable/regressive’, where stories configure cancer as yet another setback in a life of difficulties;
  • ‘progressive’, where illness is seen as an opportunity and people often find faith, or a renewed focus upon the present; and
  • ‘stable’, where illness is accepted stoically and integrated, in getting on with life.

These stories reveal central guiding metaphors: despair, hope and acceptance. Murray claims that the same pattern can be seen in people suffering from multiple sclerosis. The recurrence of such triple classificatory systems within a small arena of research suggests that researchers may not be uncovering some archetypal pattern at work in the data, but constructing typologies through cross infection of models.

내러티브를 통한 종합: 스토리로 사고하기
Synthesis through narrative: thinking with a story

내러티브 분석은 환자의 경험에 공감하는 강력한 방법을 제공할 수 있는 객관적인 주제에 스토리의 정서적 영향력을 잃는 경향이 있습니다. 내러티브를 통한 종합에는 두 가지 기본 접근법이 있습니다: 

  • 첫째, '질병 내러티브'35 또는 '병리학'36,
  • 둘째, 사회 현실주의 회고록입니다.8,9,37 

전자는 종종 연구자의 해석이 수반되는 경우가 많습니다. 후자는 일반적인 연구 장치를 버리고 저자가 서사적 라이선스를 최대한 활용하여 일관성 있고 자기 참조적인 사회 현실주의 소설을 제공합니다.
Narrative analysis tends to lose the affective impact of the story to objective themes, where such impact may offer a powerful way of empathising with the patient's experience. There are two basic approaches to synthesis through narrative:

  • first, the ‘illness narrative35 or ‘pathography’;36 and
  • secondly, the social-realist memoir.8,9,37 

The former is often interpreted, with an accompanying gloss from the researcher. The latter abandons the usual research apparatus, where the author provides a coherent, self-referencing, social-realist fiction enjoying the full use of narrative licence.

질병 내러티브
Illness narratives

질병 내러티브 데이터는 일반적으로 공동 인터뷰를 통해 수집되며, 자서전적 글쓰기로 보완되며, 독자의 공감을 불러일으키기 위해 당사자의 목소리를 포착하려고 시도합니다. 이러한 내러티브는 특징적으로 의료 사례로 환원되는 것을 피하고 더 넓은 심리사회적 맥락에 해당 인물을 배치합니다. 예를 들어, 쿠저38는 '암은 의학적인 이야기가 아니라 감정의 이야기'라고 말하며 유방암 생존자의 내러티브에 나타나는 특징적인 '플롯마크'로 혹을 발견하고 진단을 받은 순간, 유방 절제술과 재건술을 포함한 치료법 선택의 어려움, 탈모, 식은땀, 조기 폐경 등 화학요법의 부작용, 성에 대한 혼란, 미래에 대한 결심 등을 제시합니다. 
Illness narrative data are usually collected through collaborative interviewing, supplemented by autobiographical writing, attempting to capture the voice of the person to invoke empathy in the reader. Such narratives characteristically avoid reduction to a medical case, situating the person in a wider psychosocial context. For example, Couser38 suggests that ‘cancer is more than a medical story, it's a feeling story', and notes characteristic ‘plotmarks’ to narratives of breast cancer survivors, such as the moments of discovery of a lump and diagnosis; difficulties of choice about treatments including mastectomy and reconstruction; the side effects of chemotherapy such as loss of hair, night sweats and early menopause; confusion over sexuality; and resolutions about the future.

이러한 내러티브는 자칫 자기만족적일 수 있는 고백적 이야기에 국한되지 않고39 질병을 안고 살아가는 데 있어 지적, 영적, 실제적 측면은 물론 정서적, 관계적 측면에 대한 통찰력을 제공합니다. 질병 내러티브를 통한 현대 연구는 상호작용과 정체성 형성의 수사학적 관리에 대한 명시적인 관심을 보여줍니다. Riessman40은 최근 이혼한 다발성 경화증을 앓고 있는 한 백인 노동계급 남성의 이야기를 들려줍니다. 이 남성은 강한 남성적 정체성을 유지하는 데 중점을 둔 자신의 삶을 이야기하면서 주요 사건을 선택적으로 수사학적으로 이야기함으로써 좌절에도 불구하고 자신의 정체성을 재구성합니다. 그가 청중을 대하는 방식은 우리가 그에 대한 인상을 형성하는 데 영향을 미칩니다. 
Such narratives are not limited to confessional accounts, that can be self-indulgent39 but give insight into the intellectual, spiritual and practical, as well as the emotional and relational, aspects of living with illness. Contemporary research through illness narratives shows explicit interest in rhetorical management of interaction and identity formation. Riessman40 offers an account of a recently divorced white working class male with advanced multiple sclerosis. The man reconfigures his identity, despite his setbacks, by selectively and rhetorically recounting key incidents in a narrative retelling of his life that centres on maintaining a strong masculine identity. The way he addresses the listener guides the impressions we form of him.

사회 현실주의 문학 작품
Social-realist literary accounts

이야기를 통한 사고의 더 강력한 버전은 회고록과 같은 사회현실주의적 '소설'을 쓰는 것입니다.8,9,37 이러한 기록은 체계적이고 엄격한 방식으로 데이터를 수집 및 분석하거나, 결론이나 권고에 도달하거나, 가설을 옹호하거나 위조하거나, 귀납적으로 테스트할 모델을 생성하거나, 타당성 및 신뢰성 점검을 유도하지 않기 때문에 기존의 정의에 따르면 '연구'로 분류되지 않습니다. 내러티브는 문학적 라이선스를 사용하여 이러한 엄격함을 노골적으로 과시할 수도 있지만, 도덕적 상상력을 명시적으로 다루는 중요한 '증인' 문서를 제공합니다. 
A stronger version of thinking with a story is to write a piece of social-realist ‘fiction’, as memoir.8,9,37 These accounts do not count as ‘research’ by conventional definitions, because they do not collect and analyse data in a systematic and rigorous way; reach conclusions or recommendations; uphold or falsify hypotheses; inductively generate models to be tested; or invite validity and reliability checks. While they may explicitly flaunt such rigour through employment of literary licence, such narratives offer important ‘witness’ documents explicitly addressing the moral imagination.

내러티브 연구자에게 필요한 속성
Attributes needed by narrative researchers

연구자의 특성은 내러티브 조사의 품질을 좌우하는 핵심 요소입니다. 내러티브 연구는 높은 수준의 윤리적, 비판적 참여와 사람과 유물을 모두 텍스트로서 면밀히 관찰하고 '읽는' 내러티브 감수성의 배양을 요구합니다.13 반사성모호성에 대한 관용은 이러한 집중적인 읽기에 매우 중요합니다.39 Gherardi와 Turner11는 '진짜 남자들은 부드러운 데이터를 수집하지 않는다'는 말을 뺨에 혀를 깨물며 상기시켜 줍니다. 이야기는 어린이(또는 여성)를 위한 것이 아닐까요? 내러티브 연구자의 가장 중요한 덕목은 아마도 연구 참여자뿐 아니라 데이터에 대해서도 여성 특유의 감수성을 발휘하는 것일 것입니다. 연구 대상이 숫자와 사물로 변할 수 있듯이, '부드러운' 질적 데이터도 무딘 상상력을 통해 쉽게 뭉개지거나 함부로 다룰 수 있습니다.  
The attributes of researchers are central to the quality of narrative inquiry. Narrative research demands high levels of ethical and critical engagement and the cultivation of a narrative sensibility, in which both people and artefacts are witnessed and ‘read’ closely, as text.13 Reflexivity and tolerance of ambiguity are critical to such intensive reading.39 Gherardi and Turner11 remind us, tongues firmly in cheeks, that ‘Real men don’t collect soft data'. Are stories not for children (or women)? Perhaps the most important virtue of a narrative researcher is the stereotypically feminine attribute of sensitivity − not simply to research participants, but also to data. As research subjects can be turned into numbers and objects, so ‘soft’ qualitative data can be readily squashed or treated in a ham-fisted way through a blunt imagination.

결론
Conclusion

이 글은 내러티브 탐구를 이해함에 있어 분석 접근법종합 접근법 사이의 명백한 분열을 비판적으로 다루었으며, 이를 생산적인 긴장으로 재구성하고 두 접근법 간의 동등성을 높일 것을 촉구했습니다. 내러티브 연구에 대한 접근 방식의 유형론을 제시하고, 이러한 프레임워크가 복잡한 탐구 분야를 이해하는 데 도움이 된다는 점을 인정합니다. 이야기의 구조(내용)를 강조하는 접근 방식이야기의 의미(담론)를 강조하는 접근 방식 사이의 긴장을 다루고 있습니다. 이는 두 가지 예를 통해 설명할 수 있습니다. 긴즈버그 외.41는 의대생들이 직업적 갈등을 해결하는 과정을 묘사합니다. 학생들은 직업적 실수를 보고할 때 특정 추론 전략을 통해 실수를 '다시 이야기'하는데, 그 중 하나가 바로 '내러티브 태도'입니다. 어떤 학생들은 중요한 사건을 다시 이야기하면서 책임을 회피deflect하는 반면, 다른 학생들은 반성reflect하는 모습을 보이기도 합니다. Good42는 의대생들이 사례 발표라는 장르를 학습함으로써 환자의 특이한 이야기를 의학적인 설명으로 바꾸고, 현상학적 묘사를 구체적인 병태생리학적 분석으로 전환하는 과정을 설명합니다. 이 과정은 진단과 치료에 대한 과학적 고려에 필요한 집중을 유도하고 의사를 불필요한 친밀감으로부터 보호하는 역할을 하지만, '사람을 의학적 문제로 공식화'하는 관행으로 이어집니다. 
In making sense of narrative inquiry, this article has critically addressed the apparent split between approaches of analysis and synthesis, reframing this as a productive tension and calling for greater parity between the two approaches. A typology of approaches to narrative research is offered, acknowledging that such a framework helps to make sense of a complex field of inquiry. A further tension is addressed − between approaches that emphasise the structure of a story (content) and those emphasising the meaning of a story (discourse). This can be illustrated by two examples. Ginsburg et al.41 describe medical students resolving professional conflicts. As they report professional lapses, students ‘re-story’ the lapse through certain reasoning strategies, one of which is ‘narrative attitude’. Some students characteristically deflect responsibility through re-storying critical incidents, where others reflect. Good42 describes how, through learning the genre of case presentations, medical students come to transform patients' idiosyncratic stories into medicalised accounts, where phenomenological description is translated into specific pathophysiological analysis. While this process encourages necessary focus upon scientific consideration of diagnosis and treatment, and also serves to protect the doctor from unnecessary intimacy, it results in a practice where ‘persons are formulated as… medical problems’.

첫 번째 예에서 의학교육자가 직면한 과제는 편향적 전달이 아닌 성찰적 전달 전략을 촉진하는 방법입니다. 두 번째 예에서 의학교육자의 역할을 하는 의사가 전통적으로 학생과 후배를 표준 사례 발표의 구조와 내용에 익숙하게 만드는 경우, 환자의 이야기를 변형하는 과정에서 공감을 저해할 수 있는 담론과 수사학적 관행 같은 사례 발표에 대해 어느 정도 인식하고 있을까요? 또한, 의학교육자들은 이야기를 매개로 민감한 경청을 교육하여 다시 공감을 이끌어낼 수 있는 종합적인 접근 방식보다 이야기에 대한 수렴적이고 분석적인 접근 방식을 선호하는 경향이 있을까요? 내러티브는 실습의 우연한 부산물로 여겨지기보다는 체계적인 임상 학습을 위한 강력한 매체가 될 수 있습니다. 의학교육의 또 다른 과제는 이러한 주장을 평가하는 것입니다. 

In the first example, the challenge to medical educators is how to promote reflective, rather than deflective, strategies of telling. In the second example, where doctors, in the role of medical educators, traditionally socialise students and juniors into the structure and content of the standard case presentation, to what extent are they aware of such case presentation as discourse and rhetorical practice that, in transforming the patient's story, may compromise empathy? Further, will medical educators tend to privilege convergent and analytical approaches to story over divergent approaches of synthesis that may educate sensitive listening through story as a medium, again leading to empathy? Rather than being seen as an accidental by-product of practice, narratives can offer a potent medium for structured clinical learning. A further challenge for medical education is to evaluate such a claim.

 


Med Educ. 2005 May;39(5):534-40. doi: 10.1111/j.1365-2929.2005.02126.x.

Stories as data, data as stories: making sense of narrative inquiry in clinical education

Affiliation

1Peninsula Medical School, University of Exeter, Knowledge Spa, Royal Cornwall Hospital, Truro, UK. alan.bleakley@pms.ac.uk

PMID: 15842721

DOI: 10.1111/j.1365-2929.2005.02126.x

Abstract

Background: Narrative inquiry is a form of qualitative research that takes story as either its raw data or its product. Science and narrative can be seen as two kinds of knowing, reflected in the distinction between evidence-based medicine derived from population studies and narrative-based medicine focused upon the single case. A similar tension exists in the field of narrative inquiry between cognitive-orientated analytical methods and affective-orientated methods of synthesis.

Aims: This paper aims to make sense of narrative inquiry in clinical education through definition of 'narrative', articulation of a typology of narrative research approaches, and critical examination in particular of analytical methods, the dominant approach in the literature. The typology is illustrated by research examples, and the role of medical education in developing expertise in narrative inquiry is discussed. An argument is made that the tension between analysis of the structure of stories and empathic use of stories can be seen as productive, stimulating expertise encompassing both approaches.

Discussion: Analytical methods tend to lose the concrete story and its emotional impact to abstract categorizations, which may claim explanatory value but often remain descriptive. Stemming from discomfort with more integrative methods derived from the humanities, a science-orientated medical education may privilege analytical methods over approaches of synthesis. Medical education can redress this imbalance through attention to 'thinking with stories' to gain empathy for a patient's experience of illness. Such an approach can complement understanding of story as discourse - how narratives may be used rhetorically to manage both social interactions and identity.

서사 탐구: 의학교육의 관계적 연구 방법론(Med Educ, 2017)
Narrative inquiry: a relational research methodology for medical education
D Jean Clandinin,1 Marie T Cave2 & Charlotte Berendonk3

 

소개
Introduction

의학계에서는 환자의 이야기에 귀를 기울이는 것이 중요하기 때문에 내러티브와 스토리에 대한 관심이 항상 있어 왔습니다. 작가이자 의사였던 콜스1는 의사가 환자의 이야기에 귀를 기울여야 환자의 질병뿐만 아니라 자신의 지식에 대해 더 많은 것을 배울 수 있다고 주장했습니다. 그러나 이러한 이야기를 연구할 수 있는 가능성을 발견한 것은 그보다 역사가 짧습니다. 이러한 가능성은 브루너2가 내러티브 지식패러다임적 지식의 차이를 강조한 이후 가시화되기 시작했습니다.

  • 전자(내러티브)는 '인간 또는 인간과 유사한 의도와 행동, 그리고 그 과정을 표시하는 우여곡절과 결과를 다룹니다. 본질적으로 시간을 초월하기보다는 일시적인temporal 것'입니다.
  • 후자(패러다임)는 현상 관찰, 경험적 데이터 수집 및 분석, 결과의 일반화 가능성 가정 등 재현 가능한 단계에 의존하는 논리-과학적 추론 과정을 사용합니다. 이러한 특징 때문에 패러다임적 사고는 물질 과학과 생물 과학에서 지배적인 추론 접근법으로 자리 잡았습니다.

There has always been interest in narrative and story in medicine through the importance of attending to what patients say. Coles,1 a writer and physician, called for physicians to listen to patients’ stories so as to learn more about their own knowledge as well as about their patients’ illnesses. However, seeing the possibilities for researching those stories has a shorter history. These possibilities began to become visible after Bruner2 highlighted the distinction between narrative and paradigmatic knowledge.

  • The former ‘deals in human or human-like intention and action and the vicissitudes and consequences that mark their course. It is essentially temporal rather than timeless’.
  • The latter employs a logico-scientific reasoning process, which relies on replicable steps, including observation of phenomena, empirical data collection and analysis and the assumption of generalisability of findings. These features led to paradigmatic thinking as the dominant reasoning approach in material and biological sciences.

의학교육에서 패러다임적 지식은 Flexner3가 임상 전 기초과학 학습을 도입한 이후 다른 형태의 지식을 압도해 왔습니다. 패러다임적 지식이 임상적 사고의 필수적인 부분이라는 데는 동의하지만, 임상의가 임상 진료의 불확실성과 모호성, 특히 진단의 불확실성에 직면할 때 패러다임적 지식만으로는 충분하지 않습니다.
In medical education, paradigmatic knowledge has swamped other forms of knowing since Flexner's3 introduction of pre-clinical basic science learning. Although we agree that paradigmatic knowledge is an essential part of clinical thinking, it is not enough when clinicians face the uncertainty and ambiguity of clinical practice, particularly the uncertainty of diagnosis.

브루너의 구분은 부분적으로 내러티브 연구 방법론의 개발을 가능하게 했습니다. 사회과학 연구자들은 1980년대 후반 '내러티브 혁명'이라고도 불리는 내러티브 연구 접근법의 급속한 수용과 함께 내러티브를 연구 방법론으로 발전시키기 시작했습니다.4 그러나 내러티브 연구 접근법으로 빠르게 전환하면서 연구자들이 내러티브라고 부르는 것이 무엇인지, 내러티브 연구 안에 무엇이 포함되는지 아직 잘 이해되지 않았습니다.5, 6 '이야기'와 '내러티브' 같은 용어는 종종 혼용되어 사용되지만 내러티브 연구에서는 다른 의미를 지니고 있습니다.7, 8 내러티브 연구에서 내러티브가 무엇을 의미하는지에 대한 명확한 정의가 없는 상황에서,7 학자들은 현재 진행 중이거나 다가오는 사건에 대한 대화와 같이 상호작용이 이루어지는 '작은 이야기' 접근법,9,10 또는 인생 이야기나 자서전에 초점을 맞춘 '큰 이야기' 접근법을 언급합니다.7 또한 내러티브는 '내면에 기반한 현상'과 '사회적 행동'11으로 간주되며 구체화된 경험으로 간주됩니다.5 1990년대에 Mishler12 가 내러티브 연구의 경계를 정하는 것을 경계했지만, 명확한 정의가 부족하여 약간의 혼란이 발생하고 있습니다. 
Bruner's distinction, in part, made it possible to develop narrative research methodologies. Social science researchers began to develop the idea of narrative as a research methodology in the late 1980s with a rapid uptake of narrative research approaches, sometimes called the ‘narrative revolution’.4 However, with the rapid turn to narrative research approaches, just what researchers were referring to as narrative, or what was included within narrative research, is not yet well understood.5, 6 Terms like ‘story’ and ‘narrative’ are often used interchangeably yet carry different meanings in narrative research.7, 8 With no clear definition of what narrative means in narrative research,7 scholars refer to ‘small story’ approaches such as conversations about ongoing or upcoming events, in which interaction is in the midst,9, 10 or big story approaches focusing on life stories or autobiographies.7 Furthermore, narratives are viewed as both ‘an interior based phenomenon’ and ‘as social actions11 as well as embodied experiences.5 Although Mishler12 cautioned in the 1990s against policing the boundaries of narrative research, the lack of delineation has resulted in some confusion.

이 논문에서는 내러티브 연구에 대한 한 가지 접근 방식인 내러티브 탐구의 방법론을 뒷받침하는 존재론적 및 인식론적 약속을 제시합니다.13 이러한 약속을 바탕으로 내러티브 탐구를 적용한 의학교육의 두 가지 연구를 소개합니다. 마지막으로, 내러티브 탐구 참여의 어려움에 대해 논의하고 유망한 연구 및 실천 가능성을 제시합니다. 
In this paper, we lay out the ontological and epistemological commitments that underlay the methodology of one approach to narrative research, narrative inquiry.13 Based on these commitments we introduce two studies in medical education applying narrative inquiry. Finally, we discuss challenges in engaging in narrative inquiry and point toward promising research and practice possibilities.

내러티브 탐구의 정의
Definition of narrative inquiry


우리는 다음과 같은 내러티브 탐구의 정의를 바탕으로 작업합니다.
We work from the following definition of narrative inquiry.

'사람들은 자신과 타인이 누구인지에 대한 이야기를 통해, 그리고 이러한 이야기의 관점에서 자신의 과거를 해석하면서 일상생활을 형성합니다. 현재의 관용구에서, 이야기story는 사람이 세상으로 들어가고, 세상에 대한 자신의 경험을 해석하고 개인적으로 의미 있게 만드는 통로입니다."14
‘People shape their daily lives by stories of who they and others are and as they interpret their past in terms of these stories. Story, in the current idiom, is a portal through which a person enters the world and by which their experience of the world is interpreted and made personally meaningful.’14

이 정의는 경험을 스토리로 생각하는 방법으로서 내러티브 탐구에 주목합니다. 방법론으로서의 내러티브 탐구는 경험 현상에 대한 내러티브적 관점에서 작동하며, 듀이의 경험에 대한 이해15를 기반으로 합니다.
The definition draws attention to narrative inquiry as a way of thinking about experience as a story. Narrative inquiry as methodology works from a narrative view of the phenomenon of experience, building on Dewey's15 understanding of experience.

듀이의 경험의 두 가지 기준(상황 속에서 이루어지는 상호작용과 연속성)은 시간성, 장소성, 사회성의 차원을 가진 3차원 내러티브 탐구 공간을 통해 경험에 대한 내러티브 개념의 근거를 제공합니다.

  • 시간성사건과 인물의 과거, 현재, 미래에 주의를 기울이며, 항상 진행 중인 사건에 초점을 맞춥니다.
  • 장소는 탐구와 삶의 사건이 일어나는 장소의 특수성에 주목합니다.
  • 사회성은 사람의 욕망, 미적, 도덕적 반응뿐만 아니라 실존적 조건, 즉 환경과 맥락적 힘에 대한 내적, 외적 초점을 말합니다. 

Dewey's two criteria of experience (interaction and continuity enacted in situations) provide grounding for a narrative conception of experience through the three-dimensional narrative inquiry space with dimensions of temporality, place and sociality.

  • Temporality draws attention to the past, present and future of events and people, with a focus on them as always in process.
  • Place draws attention to the particularities of where inquiry and life events take place.
  • Sociality refers to the inward and outward focus on a person's desires, aesthetic and moral reactions, as well as on the existential conditions, that is, the environment and contextual forces.

'이야기에 대한 사고'와 '이야기로 사고하는 것'을 구분한 모리스(Morris)16는

  • '이야기에 대한 사고내러티브를 대상으로 생각하는 것'인 반면,
  • '이야기로 사고하는 것은 생각하는 사람으로서 우리가 내러티브에 대해 작업하는 것이 아니라, 내러티브가 우리를 위해 작업하도록 허용하는 과정'이라는 내러티브 탐구의 근거를 제시하기도 합니다.

Morris’16 distinction between thinking about stories and thinking with stories also grounds narrative inquiry

  • where ‘thinking about stories conceives of narrative as an object’
  • whereas ‘thinking with stories is a process in which we as thinkers do not so much work on narrative as of allowing narrative to work on us’.

모리스의 구분은 내러티브 연구에 대한 다양한 접근 방식에서 작동하는 서로 다른 존재론적, 인식론적 가정에 주목하는 데 도움이 됩니다.

  • 일부 내러티브 연구자들은 시간성, 장소, 사람과 분리된 객체로서 서면 및 구어 텍스트를 분석하는 반면, 
  • 내러티브 탐구자들은 시간, 장소, 관계 속에서 연구 대상인 경험에 주목합니다.

클랜디닌과 로지크17가 지적했듯이, '내러티브 탐구의 초점은 개인의 경험뿐만 아니라 개인의 경험이 구성, 형성, 표현, 제정되는 사회적, 문화적, 제도적 내러티브에도 있다'고 할 수 있습니다. 내러티브 탐구자는 살아 숨 쉬고 이야기되는 개인의 경험을 연구하며, 듣고, 관찰하고, 다른 사람과 함께 생활하고, 텍스트를 작성하고 해석하는 과정을 통해 연구합니다. 
Morris’ distinction helps us draw attention to the different ontological and epistemological assumptions at work in different approaches to narrative research.

  • Some narrative researchers analyse written and spoken texts as objects separate from temporality, place and people, whereas
  • narrative inquirers attend to the experience under study in time, place and relationships.

As Clandinin and Rosiek17 pointed out, ‘the focus of narrative inquiry is not only on individuals’ experience but also on the social, cultural, and institutional narratives within which individuals’ experiences are constituted, shaped, expressed, and enacted’. Narrative inquirers study an individual's experience, storied in the living and telling, and studied through listening, observing, living alongside another, and writing and interpreting texts.

내러티브 탐구는 개인의 삶에 주목하면서도 삶이 중첩된 더 큰 맥락과 관계에 주의를 기울이는 경험에 대한 탐구 방식입니다. 이야기는 시간, 장소, 관계 속에서 각 개인의 삶과 이야기와 분리되지 않고 살아 있고, 이야기되며, 삶과 이야기와 분리되어 분석되고 해부되어야 할 텍스트로 간주되지 않습니다.
Narrative inquiry is a way of inquiring into experience that attends to individuals’ lives but remains attentive to the larger contexts and relationships within which lives are nested. Stories are lived, and told, not separated from each person's living and telling in time, place and relationships, not seen as text to be separated from the living and telling and analysed and dissected.

의학교육 연구의 내러티브 전환
Narrative turn in research in medical education

'보건의료의 내러티브적 전환'18, 즉 치료와 돌봄은 항상 삶의 예술에 대한 더 넓은 관점 안에 포함된다는 관점의 일환으로, 보건의료는 사람이 주어진 상황에서 자신의 경험을 이해하고 자신을 표현하고 자신의 이야기를 들려줄 수 있도록 하는 것으로 주목받고 있습니다. 돌봄제공자와 돌봄수혜자는 공존하는 관계로, 돌봄제공자가 특정 상황에서 도움이 되는 지식을 가지고 있다고 해도 그것이 돌봄제공자가 일반적인 삶의 문제, 더 나아가 다른 사람의 삶에 대한 전문가가 되는 것은 아닙니다. 돌봄제공자와 돌봄을 받는 사람은 '전기적 만남'의 공동 저자입니다.19
As part of a ‘narrative turn in health care’,18 that is, that cure and care are always embedded within a broader perspective on the art of living, there is attention to health care as enabling a person to make sense of their experiences in a given situation and to express themselves, to tell their stories. Caregiver and care-receiver are both co-existent in the sense that although a caregiver may have knowledge helpful in a given situation at a particular time, this does not make the caregiver an expert on issues of life in general or, more importantly, on the life of another person. Caregiver and care-receiver are co-authors in ‘biographical encounters’.19

이러한 보건의료의 내러티브 전환(의학교육의 내러티브 전환에 대해서는 예를 들어20, 21 참조) 속에서 방법론으로서 내러티브 연구로의 전환도 이루어지고 있습니다. Bleakley22가 지적했듯이, 내러티브 연구에서 사용되는 이야기의 존재론적, 인식론적 지위에 대한 가정을 탐구할 필요가 있습니다. 내러티브 연구에서 스토리는 데이터로 사용되는데, 이는 아마도 패러다임적 지식의 힘에서 비롯된 결과일 것입니다. 폴킹혼23이 내러티브의 분석이라고 표현한 것처럼, 모리스가 이야기에 대해 생각하는 것과 비슷한 의미에서 이야기를 일종의 object으로 삼아 분석의 대상subjected으로 삼습니다이야기를 탈맥락화되고 객관적인 연구 데이터로 취급하는 내러티브 분석은 현재 의학교육에서 내러티브 연구로 전환하는 과정에서 가장 널리 퍼져 있는 방식입니다. 그러나 최근 의학교육의 연구 방법론으로서 삶과 이야기, 맥락과 사람, 연구자와 연구 참여자의 관계적 측면에 주목하는 방법론인 내러티브 탐구로 전환하는 것에 대한 관심이 높아지고 있습니다.6
Within this narrative turn in health care (for the narrative turn in medical education, see, for example,20, 21), there is also a turn to narrative research as methodology. As Bleakley22 pointed out, assumptions about the ontological and epistemological status of stories as used in narrative research need to be explored. Stories are used by some in narrative research as data, perhaps a lingering result of the power of paradigmatic knowledge. Stories are taken as a kind of object and subjected to analysis in the sense that Polkinghorne23 described as analysis of narratives, akin to what Morris described as thinking about stories. Analysis of narratives, where stories are treated as decontextualised and objective study data, is currently what is most prevalent in the turn to narrative research in medical education. However, there is recent attention to shifting to narrative inquiry as a research methodology in medical education, a methodology that attends to the relational aspects of living and telling stories, of context and person, of researcher and research participant.6

의학 학습자의 정체성 형성을 위한 내러티브 연구
Narrative research in medical learners’ identity making

내러티브 연구가 의학 교육에서 사용되는 분야 중 하나는 의학 학습자의 정체성 발달에 관한 연구입니다. Monrouxe24가 강조했듯이

  • '정체성은 중요하다. 우리가 누구인지, 그리고 우리가 누구로 비춰지는지는 의학교육에서 우리가 하는 일의 많은 부분의 근간이 된다.

의학 학습자의 정체성 발달은 이론 및 개념 논문(예: 24-27)과 최근에 발표된 경험적 연구(예: 28, 29)의 초점입니다. 여기서는 내러티브 연구 접근법을 사용하여 직업 정체성 발달을 탐구한 세 가지 연구를 참조합니다.

  • Konkin과 Suddards의 종단적 통합 사무직 프로그램에 대한 평가30에서는 해석학적 현상학적 설계를 사용하여 환자에 대한 경험에 대해 학습자를 인터뷰했습니다. 이들은 근거 이론 방법을 사용하여 데이터를 분석하고 정체성 발달을 위한 학습자 경험의 의미를 찾아냈습니다. 그들은 '[...] 이 연구에서 학생들의 이야기는 환자 치료에 의미 있게 참여하는 행위에서 탄생하고 발전하는 돌봄의 윤리에 기반을 둔 새로운 직업적 정체성을 반영한다'는 사실을 발견했습니다.30
  • Wong과 Trollope-Kumar31는 사전 실습 기간 동안 작성된 의료 학습자의 포트폴리오에 대한 내러티브를 조사했습니다. 주제별 내러티브 분석은 '복잡한 학습 환경 속에서 환자, 멘토, 동료와의 상호작용을 통해 전문직 정체성을 능동적으로 구축'하는 학생들에 대한 통찰력을 제공했습니다. 다섯 가지 주제는 '역동적이고 담론적이며 반복적인' 과정으로서의 전문직 정체성 개발에 대한 통찰력을 제공했습니다.
  • Monrouxe32는 의료 학습자의 종단적 오디오 일기를 통해 그들의 근본적인 내러티브 줄거리에 대해 탐구하고 학습자의 내러티브에서 정체성 형성의 복잡성을 설명하는 8가지 주제를 확인했습니다.
  • Monrouxe33는 내러티브를 제시하고, 이 방법을 사용할 때 윤리적 차원을 논의했으며, 개인적 및 직업적 이야기가 의대생의 정체성을 형성한다는 점을 강조했습니다. 그녀는 내러티브의 맥락적, 사회적, 시간적 내재성을 고려하는 것이 중요하다고 설명했습니다. 몬루즈는 '의대생들의 개인적, 직업적 자아는 [...] 의사가 끊임없이 그 사이를 오가며 인간의 상태를 바라보는 두 가지 방식, 즉 생의학적 시선과 내러티브 시선의 예시'라고 결론지었습니다.

One area in which narrative research is used in medical education is in studies of medical learners’ identity development. As Monrouxe24 highlighted:

  • ‘Identity matters. Who we are, and who we are seen to be, underlies much of what we do in medical education’.

The identity development of medical learners is the focus of theoretical and conceptual articles (e.g.24-27) and recently published empirical research (e.g.28, 29). We refer to three studies, which used narrative research approaches to inquire into professional identity development.

  • In Konkin and Suddards’30 evaluation of a longitudinal integrated clerkship programme they interviewed learners regarding their experiences with patients using a hermeneutic phenomenological design. They analysed data using grounded theory methods and carved out the meaning of learners’ experiences for identity development. They found: ‘[…] students’ stories in this study reflect an emerging professional identity, born in the act of meaningful engagement in patient care, and grounded in a developing ethic of caring’.30 
  • Wong and Trollope-Kumar31 inquired into narratives of medical learners’ portfolios written during pre-clerkships. Thematic narrative analysis provided insights into students’ active construction ‘of their professional identities through interactions with patients, mentors and colleagues, within complex learning environments’. Five themes gave insights into professional identity development as a ‘dynamic, discursive and iterative’ process.
  • Monrouxe32 explored longitudinal audio-diaries of medical learners regarding their underlying narrative plotlines and identified eight themes in learners’ narratives that illustrate the complexity of identity formation.
  • Monrouxe33 presented narratives, discussed ethical dimensions in employing this method, and emphasised that personal and professional stories shape the identity of medical students. She illustrated the importance of considering contextual, social and temporal embeddedness of narratives. Monrouxe concluded that ‘their personal and professional selves […] exemplify the two ways of gazing on the human condition – biomedical and narrative – that doctors constantly oscillate between’.


이러한 연구에서 전문직 정체성 발달 개념에 대한 정교한 분석을 제공하지는 않지만, 우리는 전문직 정체성에 대한 보다 포괄적인 이해가 중요하다는 점을 강조합니다. 예를 들어, Bleakley 등34 은 '의대생에게 있어 전문직 정체성은 의학교육을 통해 '만들어지는 것''이라고 말했습니다. 인용된 경험적 연구 중 두 건은 전문직 정체성의 기초가 되는 전문직 경험에 초점을 맞췄습니다.30, 31 정체성에 대한 내러티브적 관점으로 보면, 전문직 정체성은 전문직 환경 내에서 형성될 뿐만 아니라 시간이 흐르면서 다양한 장소, 맥락, 관계 속에서 개인의 삶의 이야기에 의해 형성되며, 또한 사람에 의해 체화embodied됩니다.35 내러티브 경험 개념에서 정체성 발달은 내러티브 현상이며, 내러티브 탐구는 '내러티브 풍경에서 내러티브된 삶을 사는 것'에 주목하고 '우리가 누구인지, 우리가 살아갈 이야기가 근본적으로 내러티브적이라는 것'을 이해함으로써 의학 학습자의 정체성 발달을 연구하기에 적합한 방법론입니다.5 의학교육에서 정체성 형성 연구에 대한 내러티브 접근법 중 대부분은 이야기를 데이터로 사용했습니다.22 내러티브 탐구로 전환하면서 이야기에 대한 사고 대신 이야기와 함께 사고하는 것으로 전환했습니다.

Without providing a sophisticated analysis of notions of professional identity development in these studies, we emphasise the importance of even more comprehensive understandings of professional identity. For example, Bleakley et al.34 stated that ‘for the medical student, a professional identity is something that is ‘‘made’’ through a medical education’. Two of the cited empirical studies focused on professional experiences as the foundation of professional identity.30, 31 Working with a narrative view of identity, professional identities are formed within professional landscapes, as well as shaped by personal life stories, over time and in multiple places, contexts and relationships and are also embodied in persons.35 Within a narrative conception of experience, identity development is a narrative phenomenon and narrative inquiry is an appropriate methodology to study medical learners’ identity development with its attention to the living of ‘storied lives on storied landscapes’5 and through understanding ‘that who we are, our stories to live by, is fundamentally narrative in nature’.5 Of the narrative approaches to research in identity making in medical education, most used stories as data.22 With the turn to narrative inquiry, the shift is to thinking with stories instead of about stories.

의료 학습자의 정체성 형성 연구에서의 내러티브 탐구
Narrative inquiry in the study of medical learners’ identity making

2000년대 초반에 이미 교육에 대한 연구는 있었지만,36 의료 학습자의 정체성 형성에 대한 내러티브 탐구는 초기 단계에 있었습니다. 의학교육에서 내러티브 글쓰기에 초점을 맞춘 Charon,20은 의학 학습자가 임상 경험에 대한 이야기를 작성하고 이를 촉진된 소그룹에서 '밀착 읽기'를 통해 공유하는 과정인 평행 차트 작성이라는 교육적 접근법을 개발했습니다. 클랜디닌, 케이브, 케이브35도 의사들의 진료 경험을 글로 쓰고 공유함으로써 얻을 수 있는 변화의 효과에 관심을 가졌습니다. 우리 역시 의료 전문가들이 내러티브 성찰적 실천에 참여할 기회를 얻었을 때 그들의 삶에 미치는 영향을 목격했습니다.37 우리는 내러티브 탐구의 방법론과 방법을 사용하여 Charon의 평행 차트 교수법을 사용하여 의료 학습자의 정체성 형성에 대한 연구를 시작했습니다. 이 연구에서는 학부 학습자의 경험과 레지던트 과정을 밟는 졸업생의 경험의 차이, 새로운 임상 책임이 의대생의 전문적 정체성 형성에 미치는 영향, 소규모 동료 그룹에서 촉진된 내러티브 반성적 실습이 전문적 정체성 형성에 미치는 영향에 관한 수수께끼를 탐구했습니다. 
In the early 2000s, although there were already studies in teaching,36 narrative inquiry into the identity making of medical learners was in its initial years. Charon,20 with her focus on narrative writing in medical education, developed a pedagogical approach called parallel charting, a process where medical learners write stories of their experiences of clinical encounters and share them in ‘close reading’ in facilitated small groups. Clandinin, Cave and Cave35 were also interested in the transformative effects of writing and sharing physicians’ stories of practice. We, too, witnessed the impact on medical professionals’ lives if they received opportunities to participate in narrative reflective practices.37 Using methodologies and methods from narrative inquiry, we began studies into medical learners’ identity making using Charon's parallel chart pedagogy. The research explored puzzles around differences in undergraduate learners’ experiences to those of graduates undertaking residency, the impact of new clinical responsibilities on medical students’ professional identity making, and the impact of facilitated narrative reflective practice in small groups of peers on professional identity making.

우리는 참가자들에게 평행 차트를 작성하도록 한 다음 Clandinin과 Connelly의39 3차원 내러티브 탐구 공간을 사용하여 촉진된 내러티브 탐구 그룹에 참여하도록 한 두 개의 연구35, 38 에 대해 설명합니다. 두 연구 모두 각 참가자가 10개의 차트를 작성하고 공유하는 5번의 회의가 있었다는 점에서 유사했습니다. 이동 시간을 줄이고 장소의 중요성에 주목하기 위해 각 회의는 같은 병실 내에서 진행되었습니다. 그러나 집, 병원, 침대 옆, 강의실 등 특정한 상황에 스토리를 배치했기 때문에 장소도 중요했습니다. 모든 세션에서 각 참가자는 한두 개의 차트를 공유했고, 전체 그룹은 진행자나 연구자의 의견을 들어 각 차트에 대해 질문했습니다. 모든 참가자가 자신의 차트를 공유할 시간을 갖도록 함으로써 시간성의 차원을 인정했지만, 가정과 학교에서의 어린 시절 경험과 최근의 경험 모두에서 나온 이야기였기 때문에 시간성이 존재했습니다. 시간이 지남에 따라 이야기가 바뀌면서 임상 실습의 순간에 대한 경험은 때때로 다른 시간과 장소에서의 경험과 연결되기도 했습니다. 퍼실리테이터는 참가자들이 이야기를 나누는 동안 각 참가자에게 주의를 기울였으며, 참가자들이 글을 쓰고, 공유하고, 그룹 질문에 참여하고, 그룹 회의 사이의 시간에 자신의 경험에 대해 계속 질문하면서 많은 내러티브 탐구가 발생한다는 점을 염두에 두었습니다. 사회성의 차원이 인정되는 한 가지 방법은 그룹의 포용성과 일단 형성된 그룹의 배타성에서 확인할 수 있었습니다. 의학 학습자들이 특정 상황과 사건에 대한 이야기를 하면서 개인적인 감정적, 도덕적, 심미적 반응을 드러낼 때에도 사회성은 존재했습니다.  
We describe two studies35, 38 where participants were invited to write parallel charts, and were then asked to engage in facilitated narrative inquiry groups using Clandinin and Connelly's39 three-dimensional narrative inquiry space. Both studies were similar in that there were five meetings, with each participant writing and sharing 10 charts. Each meeting was held within the same hospital room to avoid travel time and to draw attention to the importance of place. However, place was also present as the stories themselves were placed in particular contexts, such as homes, hospitals, bedsides and lecture halls. At every session, each participant shared one or two charts and the entire group, with facilitator or researcher input, inquired into each chart. Ensuring all participants had time to share their charts acknowledged the dimension of temporality, but temporality was present as the stories told came from both childhood experiences in homes and schools and more recent experiences. As stories shifted over time, the experiences of moments of clinical practice were sometimes linked to experiences in other times and places. Facilitators were attentive to each participant as they shared, mindful that much narrative inquiry occurred as participants wrote, shared, engaged in the group inquiry and continued to inquire into their experiences in the time between group meetings. One way the dimension of sociality was acknowledged was in the inclusivity of the groups, and, once formed, their exclusivity. Sociality was also present as the medical learners told stories situated within particular circumstances and events and also made visible their personal emotional, moral and aesthetic reactions.

차트를 구두로 공유한 후 이어진 내러티브 질문에는 차트를 작성하는 사람과 듣는 사람 모두 반성적인 방식으로 참여하여 동료의 이야기와 함께 자신의 이야기를 겹쳐서 이야기했습니다. 참가자들은 이야기 속에서 자신을 상상하고40 모리스가 제안한 방식으로 이야기와 함께 생각할 수 있었습니다.16 
The narrative inquiry following the oral sharing of a chart involved all participants in reflexive ways, both chart writers and those listening, who layered their stories alongside their colleagues’ stories. Participants could imagine themselves within the story40 and think with the story in ways suggested by Morris.16

두 연구 모두에서 퍼실리테이터는 각 참가자의 용기를 인정하고 공유된 이야기에 대한 초기 검증validation을 제공했습니다. 퍼실리테이터는 다른 참가자들과 함께 각 이야기가 시간적 한가운데로 들어갈 수 있게 해준다는 것을 인식하고, 들려준 이야기에 대해 더 자세히 물어보면서 내러티브 탐구에 참여했습니다. 내러티브 탐구가 진행됨에 따라 참가자들은 환자와 자신의 삶, 삶이 내재된 장소와 관계, 환자, 의사, 서로의 정서적, 도덕적, 미적 반응에 주의를 기울이게 되었습니다. 
In both studies facilitators recognised each participant's courage and provided initial validation of the shared story. The facilitators, along with other participants, engaged in the narrative inquiry by asking for further details about the stories told, recognising that each story allowed them to enter into the temporal midst. As the narrative inquiry proceeded, participants became attentive to the unfolding lives of their patients and themselves, to the places and relationships within which lives were embedded, and to the emotional, moral and aesthetic responses of patients, physicians and each other.

두 연구 모두에서 참가자들은 자신의 직업적 정체성을 구성하는 경험에 주목했습니다.

  • 첫 번째 연구에 참여한 사라(Sarah)38는 레지던트 프로그램에서 경험이 풍부한 국제 의대 졸업생이었습니다. 그녀의 병렬 차트는 어느 새 엄마가 아이를 입양 보내기로 결정한 경험을 묘사했는데, 이는 사라의 문화적 내러티브에서 유기에 가까운 행동이었습니다.
    • 차트의 일부에는 사라가 그 어머니에게 어떻게 반응했는지 재검토하고 의사로서 자신이 누구인지에 대한 자기 성찰을 하게 된 동료의 반응이 기술되어 있었습니다.
    • 그룹의 반응에서 한 참가자는 사라의 자기 판단, 즉 도덕적 반응에 주목했습니다.
    • 또 다른 참가자는 아기의 미래에 초점을 맞춘 시간적 반응을 보였습니다.
    • 또 다른 참가자는 이 상황에서 누가 도와줄 수 있는지에 대한 질문을 던졌습니다.
  • 세 가지 측면에 주의를 기울이며 그룹 질문이 진행되자 사라는 결국 자신이 환자와의 관계 형성에 탁월하다는 것을 알고 있는 자신의 이야기를 다시 들려주었습니다. 참가자들은 사라의 전개되는 삶, 환자의 전개되는 삶, 그리고 의료 학습자로서의 자신의 삶에 주의를 기울였습니다. 

In both studies participants attended to their experiences of composing their professional identities. Sarah, a participant in the first study,38 was an experienced international medical graduate in the residency programme. Her parallel chart described her experience with a new mother's decision to offer her child for adoption, an action akin to abandonment in Sarah's cultural narrative.

  • Part of her chart described a peer's response that caused Sarah to re-examine how she responded to the mother and also to engage in self-facing about who she was as a physician.
  • In the group's responses, one participant attended to Sarah's self-judgement, her moral response.
  • Another response was temporal, focused on the baby's future.
  • Another raised questions about who else could help in this situation.

As the group inquiry proceeded with attention to the three dimensions, Sarah eventually retold her story as one in which she knew she excelled in establishing relationships with patients. Participants were attentive to Sarah's unfolding life, to her patient's unfolding life and to their own lives as medical learners.

내과 레지던트들이 참여했던 두 번째 연구35에서 우리는 한 참가자 레슬리의 경험을 공유했습니다. 레슬리와의 마지막 종료 대화에서 그녀는 자신의 직업적 정체성이 어떻게 변화하고 변화하는지에 대해 간략하게 설명했습니다. 레슬리가 자신의 경험에 대해 썼을 때, 우리는 그것을 그녀의 개인적인 실용적 지식을 '실천하고, 말하고, 되살리고, 다시 말하는 것'5으로 이해했습니다.41

  • 레슬리는 첫 번째 평행 차트에서 좋은 의사가 되는 것에 대한 자신의 고민을 설명했습니다. 당시 그녀는 환자에게 간호사로 인식되는 것부터 자신을 의사로 소개하는 데 어려움을 겪는 것까지 다른 사람의 인식에 크게 의존하고 있었습니다. 그녀는 의사로서의 역할과 의사로 인식되지 않는 것에 대한 우려 사이에서 부조화를 느끼는 경험을 다섯 번의 세션 동안 반복했습니다. 그녀는 좋은 의사가 된다는 것이 무엇을 의미하는지에 대해 의문을 품고 그룹에 속한 다른 사람들이 자신의 상황에서 어떻게 반응하고 행동했을지 확인했습니다.
  • 마지막 그룹 세션에서 레슬리는 의사로서 자신의 이야기를 담은 차트를 공유했습니다. 그녀는 자신을 자만하지 않는다고 설명하며 '어떤 일을 처리할 수 있다는 자신감이 생겼다'고 말했습니다. 그리고 그것은 단지 경험일 뿐입니다...'.35 이러한 변화는 신체적, 정서적으로, 말과 행동으로 나타났고, 레슬리는 이에 대해 자세히 기록했습니다. 그녀는 의사로서 자신의 이야기가 일반적인 임상 환경이 아닌 다른 곳에서 어떻게 실현되는지 보여주었습니다.
  • 레슬리는 시간이 지남에 따라 다양한 개인적, 직업적 환경에서 자신의 직업적 정체성이 어떻게 발전했는지 보여주었습니다. 그녀의 직업적 정체성은 한 인간이자 의사로서 그녀가 누구인지 구체화되어 있으며, 새로운 경험을 겪으면서 계속 발전하고 있습니다. '뒤돌아보는 이야기'를 하기 위해 마지막 차트를 작성하기로 결정함으로써 그녀는 자신의 이야기를 다시 들려주고 되새기며 의사가 되고 의사가 되는 미래 지향적인 이야기를 구성할 수 있었습니다. 

In an article from the second study,35 in which participants were internal medicine residents, we shared the experience of one participant, Lesley. In the final exit conversation with Lesley, she gave a brief account of how she understood her professional identity as shifting and changing. As Lesley wrote of her experience we understood it as the ‘living out, telling, reliving and retelling’5 of her personal practical knowledge.41 

  • In her first parallel chart Lesley described her concerns about being a good physician. At this time she was heavily reliant on the perceptions of others, from being perceived as a nurse by patients to her difficulty with introducing herself as a physician. Her experience of feeling dissonance (between her role as a physician and her concern at being not perceived as a physician) was recurrent over the five sessions. She questioned what it meant to be a good physician and checked with others in the group as to how they would have responded or acted in her situations.
  • In the last group session, Lesley shared a chart in which she was now telling the story of herself as a physician. She described herself as not complacent, ‘it's just more of a comfort that, more confidence that I can handle some stuff. And it's really just experience …’.35 The changes manifested themselves physically and emotionally, in speech and behaviours, and Lesley wrote of them in some detail. She made visible how her story of herself as a physician was lived out in places other than the usual clinical settings.
  • Lesley showed how her professional identity developed over time and in different personal and professional landscapes. Her professional identity is embodied in who she is as a person and physician and continues to develop as she undergoes new experiences. Her decision to write her final chart looking back, to tell a ‘backward looking story’, allowed her to retell and relive her stories and compose her forward looking story of becoming and being a physician.

이 두 가지 내러티브 탐구에서 우리는 내러티브 탐구가, 데이터로서의 이야기가 아니라, 시간이 지남에 따라 다양한 사람들과 다양한 상황에서 살아가고 이야기하는 각 의료 학습자의 지속적인 경험 이야기에 관계적인 방식으로 접근한다는 것을 알 수 있었습니다.
In these two narrative inquiries we showed that narrative inquiry does not work with stories as data but attends in relational ways to each medical learner's ongoing narratives of experiences as they are lived and told in multiple situations with diverse people over time.

의학교육에서 내러티브 탐구 참여의 도전 과제
Challenges of engaging in narrative inquiry in medical education

의학교육 연구에서 내러티브 탐구를 사용하는 데는 많은 어려움이 있으며, 그 중 네 가지를 강조합니다. 우리는 의학교육에서 내러티브 탐구에 참여했던 방식을 바탕으로 이러한 과제를 명확하게 설명합니다.
Using narrative inquiry in medical education research has many challenges and we highlight four. We draw on the ways that we have engaged in narrative inquiry in medical education to make explicit these challenges.

우리가 경험한 첫 번째 도전은 탐구의 시작점에 관한 것입니다. 내러티브 탐구는 경험을 연구하는 방법론이기 때문에, 우리는 개인의 경험담에 초점을 맞추는 데서 출발합니다. 우리는 우리 모두가 속한 사회적, 문화적, 제도적, 언어적, 가족적 내러티브의 중요성을 인정하지만, 연구의 초점은 개인의 경험입니다. 이러한 점에서 내러티브 탐구는 특정 현상에 대한 이론적 이해에서 출발하는 연구와 대조를 이룹니다. 내러티브 탐구자의 존재론적, 인식론적 가정에서 비롯되는 이러한 방법론적 출발점의 근본적인 차이는 많은 의학교육 연구와 대조적입니다.
The first challenge we experienced concerns the starting point for inquiry. Because narrative inquiry is a methodology for the study of experience, our starting point is the focus on an individual's stories of experience. Although we acknowledge the importance of social, cultural, institutional, linguistic and familial narratives within which we are all embedded, the focus of study is an individual's experience. In this way narrative inquiry stands in contrast with studies that begin in theoretical understandings of a particular phenomenon. This fundamental difference in the methodological starting point, which follows from narrative inquirers’ ontological and epistemological assumptions, is in contrast with many medical education studies.

예를 들어, 의사의 정체성 형성 연구에 대한 우리의 내러티브 탐구 접근법은 이론적으로 정립된 정체성 개념이 아니라 의학 학습자의 경험에서 출발했습니다. 내러티브 탐구에서는 각 의료 학습자의 경험담에서 시작합니다. 연구 설계에서는 참여자가 자신의 이야기를 들려줄 수 있는 방법을 채택하거나 참여자의 경험을 함께 할 수 있는 방법을 채택해야 합니다. 앞서 설명한 두 가지 내러티브 조사에서 우리는 차트 작성과 공유를 병행하여 의료 학습자가 자신의 이야기를 들려줄 수 있는 공간을 만들었습니다. 그런 다음 참가자들과 함께 이야기에 대해 질문하고 마지막 출구 대화에서 자신의 경험을 되돌아보도록 요청했습니다. 참가자를 포함하여 사람들로 하여금, 설문지나 반구조화된 인터뷰 프로토콜을 만들 수 있는 이론적 이해가 아니라, 질문의 출발점이 참가자의 경험에 있다는 것을 이해하도록 돕는 것은 어려운 과제였습니다.
As an example, our narrative inquiry approach to research into physician identity making began with the experiences of medical learners rather than with pre-set notions of identity developed in theory. In narrative inquiry we begin with each medical learner's stories of experience. Research designs require that we adopt methods to enable participants to tell their stories or to allow us to live alongside them as they undergo their experiences. In the two narrative inquiries described earlier we used parallel chart writing and sharing to create spaces within which medical learners could tell their stories. Alongside the participants, we then engaged in inquiries into the stories and finally we asked them to look back over their experiences in a final exit conversation. It is a challenge to help others, including participants, understand that the inquiry starting point is in the experience of participants, and not in theoretical understandings that would have allowed us to create a questionnaire or semi-structured interview protocol.

내러티브 탐구는 정체성 형성에 관여하는 요소를 더 깊이 이해할 수 있게 해주고, 의료 학습자가 임상 실습의 필수 출발점으로서 자신의 경험에 주목하는 법을 배울 수 있는 교육적인 방법을 만들어주기 때문에 그 중요성을 잘 알고 있습니다. 탐구 과정을 통해 학생들은 실습에 참여하기 위해 외부의 전문 지식뿐만 아니라 자신의 개인적 실무 지식을 활용하는 방법을 배우게 됩니다.
We see the importance of engaging in narrative inquiry as it allows deeper understandings of what is involved in identity making and creates educative ways for medical learners to learn to attend to their experiences as an essential starting point in their clinical practice. Through the inquiry process they learn to look to their own personal practical knowledge, as well as to outside expert knowledge, to engage in practice.

의학교육에서 내러티브 탐구를 활용하는 두 번째 과제는 의사와 의사가 되기 위해 배우는 사람들에게 (임상 사례나 사례 연구를 사용하여 글을 쓰고 생각하도록 요구하는) 일반적인 의학교육 관행에서 관심을 전환하는 것입니다. 의사들은 일상적인 연습으로 회진에 참석하고 그 일환으로 사례 연구를 준비하고 발표하도록 훈련받습니다. 사례 연구는 대부분 객관적인 생물의학 데이터를 바탕으로 한 집중적인 프레젠테이션입니다. 임상 사례 연구에서는 환자에 초점을 맞추고 의사의 경험은 포함되지 않습니다. 즉, 발표를 위해 사례를 준비할 때 의사는 객관적인 데이터를 수집하고 각 임상 사례에서 자신을 행위자 및 대리인으로 생각하지 않고 환자를 사례로 제시하는 방법을 배웁니다. 임상 사례와 사례 연구는 패러다임적 사고와 연결됩니다. 관계적 방법론인 내러티브 탐구에서는 모든 참여자(의사, 환자, 연구자)의 경험을 연구 대상으로 삼습니다. 패러다임적 지식에서 내러티브 지식으로의 전환, 즉 데이터로서의 이야기에서 이야기로의 사고로의 전환은 자신의 경험을 연구에서 배제하도록 잘 훈련된 의사에게는 어려운 전환입니다. 예를 들어, 의료 학습자의 정체성 형성 경험에 대한 내러티브 탐구에서 참가자들은 처음에는 환자와 관련된 자신의 이야기를 쓰는 것이 어렵다고 느꼈습니다. 처음에는 환자에 대한 사례 연구를 쓰는 것이 더 편하다는 것을 알게 되었습니다. 시간이 지나고 노력을 기울인 끝에 그들은 환자에 대한 자신의 경험내러티브 탐구에서 연구 대상이 될 수 있다고 생각하기 시작했습니다. 
The second challenge to engaging in narrative inquiry in medical education is shifting attention from the more common medical education practice of asking physicians and those learning to be physicians to write and think using clinical cases or case studies. They attend rounds as routine practice and are trained to prepare and present case studies as part of that practice. Case studies are, most frequently, intensive presentations that draw on objective biomedical data. In a clinical case study, the focus is on the patient and the physician's experience is not included; that is, in preparing cases for presentation, physicians are taught to collect objective data and to present patients as cases without thinking of themselves as actors and agents in each clinical encounter. Clinical cases and case studies are linked to paradigmatic thinking. In narrative inquiry, a relational methodology, the experiences of all participants (physicians, patients and researchers) are under study. This shift from paradigmatic to narrative knowledge, from thinking about stories as data to thinking with stories, is a challenging shift for physicians who have been well trained to leave their own experiences out of research. For example, in our narrative inquiries into medical learners’ experiences of identity making, our participants initially found it challenging to write stories of themselves in relation to patients. At first, they found it more comfortable to write case studies of the patients. It was over time and with effort that they began to see their experiences with patients as what was also under study in narrative inquiry.

내러티브 탐구 사용의 세 번째 과제는 방법론methodology으로서 내러티브 탐구와 일치하는 방법methods을 만들어야 한다는 것입니다. 다른 전문가와의 내러티브 탐구에서 사용되는 연구 방법은 여러 가지가 있지만,39 의사와 의료 학습자가 시간이 지남에 따라 관계를 형성하고 임상 환경에서 평가 체크리스트와 다른 전문가의 시선에 의해 형성된 '두려움/불확실성'을 내려놓을 수 있는 내러티브 탐구 방법을 만드는 것은 특히 어렵습니다. 이렇게 잘 정립된 관행에 맞서기 위해서는 의료 학습자가 자신의 이야기를 하고 자신의 이야기에 대해 질문할 수 있을 만큼 안전하다고 느끼는 공간을 만들어야 합니다. 예를 들어, 정체성 형성에 대한 내러티브 탐구에서 병렬 차트를 사용하고 공동 탐구 방식을 공유 및 촉진함으로써 참가자들에게 안전한 공간을 만들 수 있었습니다.
The third challenge in using narrative inquiry is the need to create methods that are congruent with narrative inquiry as a methodology. Although there are a number of research methods that are used in narrative inquiries with other professionals,39 it is particularly challenging to create methods of narrative inquiry to use with physicians and medical learners that allow for relationships over time and that will allow them to set aside their ‘fears/uncertainties’ shaped by assessment checklists and the expert gaze of others in their clinical environments. Working against these well-established practices requires that we create spaces where medical learners feel safe enough to tell their stories and to inquire into their stories. For example, in our narrative inquiries into identity making, it was the use of parallel charts followed by the shared and facilitated method of collaborative inquiry that created a safe space for the participants.

네 번째 도전 과제는 의사가 의료 상황에서 개별 행위자가 아닌 의료팀의 일원으로서 자신의 위치를 파악하는 데서 비롯됩니다. 내러티브 탐구에 참여하는 데 있어 어려운 점은 다양한 팀원들의 경험을 고려하는 것입니다. 앞서 언급했듯이 의사의 정체성은 다른 의사, 간호사, 치료사, 약사, 사회복지사, 관리자 등 다른 의료 서비스 제공자들과의 관계 속에서 살아가며 들려주는 이야기를 통해 형성됩니다. 내러티브 탐구에 참여할 때 우리는 학제 간 및 전문직 간 업무 환경에서 만나는 다양한 의료 서비스 제공자의 경험을 포함할 수 있는 방법을 개발해야 합니다. 내러티브 탐구에 참여하려면 다른 전문가들의 이야기뿐만 아니라, 개인이 포함된 더 큰 사회적, 제도적, 직업적 내러티브에 주의를 기울여야 합니다. 여러 참여자의 경험에 주의를 기울이는 다관점 내러티브 탐구에 참여하는 것은 쉽지 않은 일입니다. 따라서 각 의료 학습자의 경험을 이해하려면 각 분야와 각 전문직에 고유한 더 큰 사회적 전문직 내러티브제도적 내러티브서로 다른 전문가를 어떻게 포지셔닝하는지 이해해야 합니다.
The fourth challenge emerges from attending to physicians’ positioning as members of health care teams and not as individual actors in health care contexts. The challenge of engaging in narrative inquiry is in considering the experiences of different team members. As noted earlier, physicians’ identities are shaped through stories they live and tell in relation to other allied health care providers, including other physicians, nurses, therapists, pharmacists, social workers, managers, etc. As we engage in narrative inquiry we need to develop ways to include the experiences of different health care providers as they meet in their interdisciplinary and interprofessional work environments. Engaging in narrative inquiry requires attentiveness to the stories of other professionals as well as to the larger social, institutional and professional narratives in which individuals are embedded. Engaging in what often becomes multi-perspectival narrative inquiry that attends to the experiences of multiple participants is challenging. Consequently, to understand each medical learner's experiences we need to understand the larger social professional narratives that are unique to each discipline and each profession, and also how institutional narratives position different professionals in relation to each other.

향후 연구 및 실천 방향
Future research and practice directions

내러티브 탐구가 의학교육 연구에서 계속 채택됨에 따라 위에서 언급한 바와 같은 도전과제가 있을 것입니다. 우리는 내러티브 탐구와 다른 형태의 내러티브 연구의 기초가 되는 인식론적, 존재론적 가정을 계속해서 명확히 하는 것이 중요하다고 봅니다. 또한 의학교육과 다른 환경에서 내러티브 탐구의 근간이 되는 관계 윤리에 대한 이해를 계속 발전시켜야 할 필요성이 있습니다. 교육학으로서의 내러티브뿐만 아니라 내러티브 반성적 실천과 같은 전문적 실천에서 내러티브 탐구의 사용을 확장할 수 있는 가능성도 고려해야 합니다.
As narrative inquiry continues to be taken up in research in medical education, there will be challenges as noted above. We see the importance of continuing to clarify the epistemological and ontological assumptions underlying narrative inquiry and other forms of narrative research. There is also a need to continue to develop understandings of the relational ethics that underlie narrative inquiry in medical education and in other settings. Consideration must be given to the possibilities of extending the uses of narrative inquiry in professional practice, such as narrative reflective practice as well as narrative as a pedagogy.


 

Med Educ. 2017 Jan;51(1):89-96. doi: 10.1111/medu.13136. Epub 2016 Nov 2.

Narrative inquiry: a relational research methodology for medical education

Affiliations collapse

1Faculty of Education, University of Alberta, Edmonton, Alberta, Canada.

2Faculty of Medicine, University of Alberta, Edmonton, Alberta, Canada.

3Faculty of Nursing, University of Alberta, Edmonton, Alberta, Canada.

PMID: 27807868

DOI: 10.1111/medu.13136

Abstract

Context: Narrative research, an inclusive term for a range of methodologies, has rapidly become part of medical education scholarship.

Concepts: In this paper we identify narrative inquiry as a particular theoretical and methodological framework within narrative research and outline its characteristics. We briefly summarise how narrative research has been used in studying medical learners' identity making in medical education.

Uses of narrative inquiry: We then turn to the uses of narrative inquiry in studying medical learners' professional identity making. With the turn to narrative inquiry, the shift is to thinking with stories instead of about stories.

Conclusions: We highlight four challenges in engaging in narrative inquiry in medical education and point toward promising future research and practice possibilities.

학부의학교육에서 지속적 질 개선의 문화 만들기(Acad Med, 2020)
Shaping a Culture for Continuous Quality Improvement in Undergraduate Medical Education 
Guy W.G. Bendermacher, MSc, Willem S. De Grave, MSc, PhD, Ineke H.A.P. Wolfhagen, MSc, PhD, Diana H.J.M. Dolmans, MSc, PhD, and Mirjam G.A. oude Egbrink, MSc, PhD 

 

지속적인 질 향상(CQI) 전략은 커리큘럼이 발전하는 대중의 요구, 근거 기반 의학, 효과와 효율성에 대한 관심 증가에 발맞춰야 한다는 점에서 의과대학에 매우 중요합니다.1,2 1990년대 초부터 의과대학은 질 관리 접근 방식에 상당한 투자를 해왔으며, 그 결과 직원 성과에 대한 더 나은 통찰력, 개선이 필요한 주제를 알리는 기회, 더 명확하게 정의된 책임 등을 확보할 수 있었습니다.3,4
Continuous quality improvement (CQI) strategies are of key importance to medical schools, given the need for curricula to keep pace with advancing public demands, evidence-based medicine, and an increased attention to effectiveness and efficiency.1,2 Since the early 1990s, medical schools have invested considerably in quality management approaches, resulting in better insights in staff performance, further opportunities to signal topics for improvement, and more clearly defined responsibilities.3,4

CQI는 학교가 교육의 질에 대한 다양한 개념을 체계적으로 다루기 위해 도전하는 상태와 유사합니다. 이해관계자의 관점에 따라 교육의 질은 다음과 같이 이해할 수 있습니다.5 

  • 목적 적합성(유능한 미래 의사 교육),
  • 비용 대비 가치(교육에 대한 투자 수익),
  • 완벽성(결함 제로에 집중),
  • 탁월성(최고의 프로그램으로 우뚝 서기),
  • 혁신성(교육 학습 효과에 집중)

계획-실행-점검(PDCA) 사이클과 같은 실행 프레임워크는 CQI가 반복적이고 점진적인 과정임을 설명해줍니다. PDCA 사이클에서는 교육 결과를 미리 정해진 목표와 비교하고, 평가를 바탕으로 격차를 해소하기 위한 조치를 취할 수 있습니다.6
CQI resembles a state in which schools take on the challenge to address different notions of educational quality in a methodical manner. Depending on the perspective of a stakeholder, educational quality can be understood as

  • fitness for purpose (educating capable future physicians),
  • value for money (a return on investment in education),
  • perfection (focusing on zero defects),
  • exceptional (standing out as the best program), or
  • transformative (focusing on the educational learning effect).5 

Implementation frameworks such as the plan–do–check–act (PDCA) cycle illustrate that CQI is a repetitive and incremental process. The PDCA cycle holds that educational outcomes are compared to predetermined goals and that based on evaluations, action can be taken to address disparities.6

현재의 품질 관리 접근법의 긍정적인 효과에도 불구하고 교육 개선이 간단한 과정이 아니라는 공감대가 확산되고 있습니다.7 이 개념을 뒷받침하는 세 가지 주요 주장이 있습니다. 

  • 첫째, 질 관리 시스템과 프로세스의 성공 여부는 교수진, 지원 직원 및 학생이 이를 구현하고 수용하는 방식에 달려 있습니다.8 예를 들어, 학문적 자유의 전통은 프로그램 평가의 판단적 성격과 상충될 수 있습니다.9
  • 둘째, 의과대학은 다양한 하위 문화를 포함하는 복잡하고 계층적인 조직입니다.10 이러한 특성은 개선 노력에 대한 집단적 참여를 방해합니다.11
  • 셋째, CQI는 이용 가능한 정보, 시간 및 인센티브에 의해 제한됩니다.12 교육 개선을 위한 추가 노력에 대한 보상이 부족하면 평가가 구체적인 행동으로 이어지지 않는 이유를 설명할 수 있습니다.13

Notwithstanding the positive effects of current quality management approaches, there is a growing consensus that educational improvement is not a straightforward process.7 Three main arguments underpin this conception.

  • First, the success of quality systems and processes depends on the way they are implemented and received by faculty, support staff, and students.8 Traditions of academic freedom, for instance, can stand at odds with the judgmental character of program evaluations.9 
  • Second, medical schools are complex, hierarchical organizations, which include different subcultures.10 These characteristics hamper a collective engagement in improvement efforts.11 
  • Third, CQI is bounded by available information, time, and incentives.12 The lack of rewards for extra efforts to improve education can explain why evaluations are not followed up with concrete actions.13

CQI를 위한 구조와 프로세스는 교육의 질에 대한 관심을 핵심으로 하는 조직(하위)문화에 의해 보완되어야 한다는 생각질 문화 개념의 토대를 형성합니다.14,15 이 개념은 조직문화가 "외부 적응내부 통합의 문제에 대처하는 방법"16 을 반영한다는 관점과 연결되며 조직(하위)문화와 성과에 대한 이전 연구를 기반으로 합니다.17

  • 질 문화신뢰와 참여를 촉진하고
  • 리더십과 커뮤니케이션하드(시스템 또는 프로세스 지향)와 소프트(심리적 또는 가치 관련 지향) 차원 간의 연결을 강화할 수 있습니다.18,19

The idea that structures and processes for CQI should be complemented by an organizational (sub)culture with care for educational quality at its core forms the foundation of the quality culture concept.14,15 This concept is linked to the perspective that organizational cultures reflect “a way to cope with problems of external adaptation and internal integration,”16 and builds on previous research on organizational (sub)culture(s) and performance.17 

  • A quality culture promotes trust and involvement, while
  • leadership and communication can reinforce the link between its hard (system or process oriented) and soft (psychological or value-related oriented) dimensions.18,19

여러 연구가 질 문화 개념을 더 잘 정의하는 데 기여했으며20,21 고등 교육 내 하위 문화를 식별했습니다.22,23 또한 이전 연구는 질 관리의 성공이 문화적 요인 및 교수진 선호와의 얼라인먼트로 설명될 수 있다는 통찰에 기여했습니다.24,25 그러나 품질 문화에 대한 대부분의 연구는 이론적이고 설명적인 성격을 띠고 있습니다.19 질 정책 및 절차에 참여하는 학생과 교수진이 질 문화를 개념화하는 방식에 대한 실증적 연구가 부족합니다. 특히, 업무와 관련된 심리적 태도가 질 관리 및 책임 절차에 어떤 영향을 미치는지에 대한 교육기관 전반의 통찰력 강화가 필요합니다. 이 분야에 대한 지식의 증대는 품질 문화 육성의 장점을 뒷받침할 것입니다. 본 연구는 조직의 질 문화의 주요 특징이 무엇이며 이러한 특징이 학부 의학교육의 지속적인 개선에 어떻게 기여하는지에 대한 질문을 다룹니다. 의과대학의 CQI 시스템과 프로세스가 점점 더 유사해지고 있는 가운데, 우리는 주로 업무 관련 심리적 태도와 조직 가치 지향성이 CQI에 어떤 차이를 만드는지에 초점을 맞춥니다.
Several studies have contributed to a better definition of the quality culture concept20,21 and have identified subcultures within higher education.22,23 Moreover, previous research has contributed to the insight that the success of quality management can be explained by its alignment with cultural factors and faculty preferences.24,25 Yet most studies on quality culture are of a theoretical, descriptive nature.19 There is a lack of empirical research on the way students and faculty involved in quality policies and procedures conceptualize a quality culture. Specifically, an enhanced insight across institutions is needed in how work-related psychological attitudes counterbalance quality control and accountability procedures. Increased knowledge in this area will support the merit of nurturing a quality culture. Our study addresses the questions what are the key features of an organizational quality culture and how do these features contribute to continuous improvement of undergraduate medical education? As systems and processes for CQI in medical schools are increasingly becoming alike, we mainly focus on how work-related psychological attitudes and organizational value orientations make a difference in CQI.

방법
Method

연구 설계
Study design

우리는 사회 구성주의적 관점에서 다기관 포커스 그룹 연구를 수행했습니다. 연구 설계는 연역적(이론에서 실천으로) 접근법과 귀납적(실천에서 이론으로) 접근법을 결합한 것이 특징입니다.26 포커스 그룹 토론은 참가자들의 다양한 경험, 관점, 태도를 포착하는 데 도움이 되었습니다.27 데이터 수집, 분석, 해석은 순차적으로 진행되어 분석 절차를 반복적으로 조정하고 개선할 수 있었습니다. 
We conducted a multicenter focus group study, operating from a social constructivist stance. The study design is characterized by combining a deductive (from theory to practice) and inductive (from practice to theory) approach.26 Focus group discussions served to capture the broad array of participants’ experiences, perspectives, and attitudes.27 Data collection, analysis, and interpretation were conducted sequentially, allowing for iterative adaptations and refinement of the analysis procedure.

설정
Setting

네덜란드의 8개 의과대학의 교육 품질 자문위원회(EC)가 이 연구에 참여하도록 의도적으로 선정되었습니다. EC 위원은 공식적으로 선출된 교수진과 학생 대표로 구성됩니다. EC의 주요 임무는 교육의 질에 영향을 미치는 모든 문제에 대한 권고 사항을 프로그램 경영진에 제공하는 것입니다. EC는 프로그램 평가가 조직되는 방식에 대해 공식적으로 동의하고 조언할 수 있는 권한을 갖습니다. 위원회는 동일한 수의 학생과 교수진으로 구성됩니다(각 EC는 8~12명의 위원으로 구성). EC를 대상으로 포커스 그룹을 실시하는 데는 네 가지 이유가 있습니다.

  • 첫째, EC 교수진은 일반적으로 교육의 개발, 실행, 평가 및 개선에 대한 풍부한 경험을 가진 학자들입니다. 임상, 비임상 또는 전임상, 연구 및 교육 분야의 교수진이 참여 EC에 포함되어 있습니다.
  • 둘째, EC 구성원은 현재 시행 중인 품질 시스템 및 절차에 대해 잘 알고 있으며, EC 구성원은 교육 및 평가 및 정책 부서와의 협력을 통해 이러한 시스템 및 절차에 대한 추가 지식을 습득했습니다.
  • 셋째, EC에는 학생 대표가 포함됩니다. 이러한 대표성을 통해 품질과 품질 개선에 대해 교수진 외에 다른 관점을 반영할 수 있습니다.
  • 넷째, 교수진과 학생의 공통된 EC 멤버십을 통한 친분은 이질적인 그룹 환경에서 열린 토론을 촉진합니다. 

Education quality advisory committees (ECs) of the 8 medical schools in the Netherlands were purposively selected to participate in this research. EC members are formally elected faculty and student representatives. The central task of ECs is to provide the program management with recommendations on all matters influencing quality of education. ECs have formal rights of consent and advice on the manner in which program evaluations are organized. The committees consist of an equal number of students and faculty (each EC has 8 to 12 members). Four reasons underpin the choice to conduct focus groups among ECs.

  • First, EC faculty generally are academics with ample experience in the development, implementation, evaluation, and improvement of education. Faculty with clinical, nonclinical or preclinical, research, and education backgrounds are represented in the participating ECs.
  • Second, EC members are familiar with the quality systems and procedures in place; EC members have derived additional knowledge on these systems and procedures through training and cooperation with evaluation and policy departments.
  • Third, ECs include student representatives. This representation allows for the incorporation of a different perspective on quality and quality improvement (besides that of faculty).
  • Fourth, the acquaintance of faculty and students through their shared EC membership facilitates open discussion in a heterogeneous group setting.

네덜란드의 각 의과대학은 학교의 역사, 연구 및 교육 실적, 학생의 다양성에 따라 다소 독특한 조직 문화를 가지고 있습니다. 또한 학교 내에서 적용되는 교육적 접근 방식도 어느 정도 다릅니다. 그러나 학교마다 적용되는 교육 조직 구조와 품질 관리 접근 방식은 비슷합니다. 모든 프로그램은 학술 의료 센터에서 제공하거나 학술 의료 센터와 긴밀히 협력하여 제공합니다. 모든 참여 학교에서는 PDCA 사이클이 CQI의 지침 구조로 사용되며, 모든 프로그램은 네덜란드 학부 의학교육 프레임워크를 준수합니다.28 
Each medical school in the Netherlands has a somewhat distinct organizational culture based on its history, record in research and education, and diversity of student intake. Moreover, educational approaches applied within the schools vary to some extent. The educational organization structures and quality management approaches applied across schools are comparable, however. All programs are offered by, or in close cooperation with, academic medical centers. In all participating schools, the PDCA cycle serves as guiding structure for CQI, and all programs adhere to the Dutch Framework for Undergraduate Medical Education.28

데이터 수집
Data collection

우리는 PDCA 주기에 따라 질문이 구조화된 초기 포커스 그룹 가이드를 개발했습니다. 이 구조를 따르기로 한 것은 이 주기의 각 단계를 최적화하는 것이 CQI로 이어질 것이라는 전제하에 결정되었습니다. 이 가이드(부록 디지털 부록 1: https://links.lww.com/ACADMED/A909)는 연구자의 소속 기관의 전직 EC 위원들과 함께 파일럿 포커스 그룹 세션에서 테스트한 후 질문 순서를 수정하고 이론적 개념의 사용을 제한하는 등 조정을 거쳤습니다. 2018년 5월, 네덜란드에 있는 8개 의과대학의 EC에 모두 연락을 취했습니다. 8개 위원회 중 6개 위원회가 참여하기로 동의했습니다. 참여하지 않은 위원회는 회의 계획에 필요한 시간 투자 및 어려움, EC 구성의 변화 등을 이유로 들었습니다. 총 40명의 EC 위원(학생 18명, 교수진 22명)이 6개의 포커스 그룹에 참여했습니다(표 1 참조). 회의는 2018년 7월부터 12월까지 진행되었습니다. 모든 회의(평균 시간 79분)는 있는 그대로 기록되었고, 익명화 및 코딩된 녹취록이 참가자들에게 전송되어 승인을 받았습니다. 각 포커스 그룹을 시작할 때 참가자들에게 양질의 문화에서 가장 중요한 요소가 무엇이라고 생각하는지 적어달라고 요청했습니다. 이 활동은 사전 지식을 활성화하고 데이터 삼각 측량의 한 형태로 사용되었습니다. 포커스 그룹 회의가 끝날 무렵에는 참가자들에게 그들이 작성한 의견이 토론에 포함되었는지 물어보았습니다. 
We developed an initial focus group guide, with questions structured according to the PDCA cycle. The choice to follow this structure was based on the presupposition that optimizing each step of this cycle will lead to CQI. The guide (Supplemental Digital Appendix 1 at https://links.lww.com/ACADMED/A909) was tested in a pilot focus group session with former EC members from the researchers’ home institution and thereafter adjusted; the order of questions was revised, and the use of theoretical concepts was limited. In May 2018, we approached the ECs of all 8 medical schools in the Netherlands. Six of the 8 committees agreed to participate. Reasons given for not participating were the required time investment and difficulty in planning a meeting and changes in the composition of the ECs. In total, 40 EC members (18 students and 22 faculty) participated in the 6 focus groups (see Table 1). The meetings were conducted between July and December 2018. All meetings (average duration, 79 minutes) were transcribed verbatim, and anonymized and coded transcripts were sent to participants for approval. At the start of each focus group, we asked participants to write down what they considered the most important elements of a quality culture. This exercise activated prior knowledge and was used as a form of data triangulation. At the end of the focus group meeting, we asked participants if their written comments were covered in the discussion.

acadmed_2020_04_08_iler_sdc-00805_sdc1.pdf
0.11MB

데이터 분석
Data analysis

데이터 분석을 위해 단계적 주제 분석의 일종인 템플릿 분석을 사용했습니다. 템플릿 분석에서는 계층적으로 구조화된 테마로 구성된 일련의 코딩 템플릿을 개발하여 데이터에 반복적으로 적용합니다. 분석이 진행됨에 따라 테마는 지속적으로 수정됩니다.29

  • 첫 번째 단계로, G.W.G.B.와 W.S.d.G.는 코딩된 2개의 트랜스크립트를 독립적으로 열어 트랜스크립트 내 및 트랜스크립트 간에 반복되는 코드를 검색합니다. 품질 문화에 관한 최근 문헌고찰에 기술된 주제가 민감성 개념으로 사용되었습니다.19
  • 그 후, 두 저자는 만나서 코드를 비교하고 병합하여 주제를 식별했습니다.
  • 초기 코딩 템플릿은 24개의 주제와 183개의 코드로 구성되었습니다.
  • 그 후 G.W.G.B.는 나머지 기록의 분석을 바탕으로 템플릿을 반복적으로 변경했습니다. 이러한 변경 사항에는 코드의 세부 사항 추가, 병합 및 연결이 포함되었습니다.
  • 이후 G.W.G.B.와 W.S.d.G.가 다시 만나 코딩 템플릿의 최종 버전(25개의 테마와 199개의 코드 포함)을 확정했습니다.
  • 마지막 단계로, G.W.G.B.는 최종 버전의 템플릿을 사용하여 모든 트랜스크립트를 다시 코딩했습니다.

분석 프로세스는 Atlas-ti 8.3 소프트웨어(ATLAS.ti, GmbH, 독일 베를린)를 사용하여 지원되었습니다. 포커스 그룹이 시작될 때 수행한 서면 연습의 코딩, 녹취록 분석, 전체 연구팀과의 녹취록 토론을 통해 5가지 중요한 주제를 식별하는 데 기여했습니다. 
We used template analysis, a stepwise type of thematic analysis, to analyze the data. In template analysis, a succession of coding templates consisting of hierarchically structured themes is developed and iteratively applied to the data. Themes are modified continuously as the analysis progresses.29 

  • As a first step, G.W.G.B. and W.S.d.G. open coded 2 transcripts independently, searching for codes that recurred within and between transcripts. Topics described in a recent literature review on quality culture served as sensitizing concepts.19 
  • Subsequently, these 2 authors met to compare and merge codes and identify themes.
  • The initial coding template comprised 24 themes and 183 codes.
  • Iterative changes to the template were then made by G.W.G.B. based on the analysis of remaining transcriptsThese changes concerned further detailing, merging, and linking codes.
  • Hereafter, G.W.G.B. and W.S.d.G. met again to establish the final version of the coding template (including 25 themes and 199 codes).
  • As a last step, G.W.G.B. recoded all transcripts using the final version of the template.

The analysis process was supported by application of Atlas-ti 8.3 software (ATLAS.ti, GmbH, Berlin, Germany). The coding of the written exercise performed at the start of focus groups, analysis of transcripts, and discussion of transcripts with the full research team contributed to the identification of 5 overarching themes.

반사성
Reflexivity

데이터 수집 및 분석 단계에서 다양한 해석을 반영하기 위해 전체 연구팀은 여러 차례 회의를 가졌습니다. 이 회의에서는 익명화 및 코딩된 녹취록, 현장 노트, CQI에 대한 각자의 이해에 대해 논의했습니다. 모든 연구팀원은 네덜란드 마스트리흐트 대학교 보건직업교육대학의 보건, 의학 및 생명과학 학부에 소속되어 있습니다. 포커스 그룹 진행자(W.S.d.G.)는 개인 및 그룹 인터뷰 경험이 풍부한 교육 과학자입니다. 양질의 문화 개발 박사 과정 중인 또 다른 팀원은 관찰자이자 메모 작성자(G.W.G.B.)로 활동했습니다. 연구팀에는 품질 보증 경험이 있는 교육 과학자(I.H.A.P.W.), 의학교육 혁신가인 의학 생리학자(M.G.A.o.E.), 혁신적인 학습 준비에 중점을 둔 교육 과학자(D.H.J.M.D.)가 추가로 포함되었습니다. 품질 관리, 정책 결정, 의학교육 등 다양한 전문성을 바탕으로 다양한 관점을 포함할 수 있었습니다. 팀 토론을 통해 연구 결과에 대한 합의에 도달했습니다.  
To reflect on diverse interpretations in the data gathering and analysis stages, the full research team held several meetings. In these meetings, we discussed the anonymized and coded transcripts, field notes, and our individual understandings of CQI. All research team members are affiliated with the Faculty of Health, Medicine and Life Sciences at the School of Health Professions Education at Maastricht University in the Netherlands. The focus group moderator (W.S.d.G.) is an educational scientist with extensive experience in interviewing individuals and groups. Another team member, who is a doctoral candidate in quality culture development, acted as observer and note taker (G.W.G.B.). The research team further included an educational scientist with experience in quality assurance (I.H.A.P.W.), a medical physiologist who is a medical education innovator (M.G.A.o.E.), and an educational scientist focusing on innovative learning arrangements (D.H.J.M.D.). Our diverse expertise (in quality management, policy making, and medical education) enabled inclusion of multiple perspectives. From team discussions, we reached a negotiated consensus on the study’s results.

윤리적 고려 사항
Ethical considerations

이 연구는 네덜란드 의학교육윤리심의위원회(NVMO ERB-1046)의 승인을 받았습니다. 
The study was approved by the Dutch Association for Medical Education Ethical Review Board (NVMO ERB-1046).

연구 결과
Results

연구 결과 CQI의 질적 문화 구성 요소를 반영하는 5가지 주요 주제를 확인했습니다.

  • (1) 개방적 시스템 관점 육성,
  • (2) 교육 (재)설계에 이해관계자 참여,
  • (3) 교육과 학습의 가치인정,
  • (4) 소유권과 책임 사이의 탐색,
  • (5) 통합적 리더십 구축 

지원적인 커뮤니케이션 환경(조직의 리더에 의해 촉진될 수 있음)은 처음 네 가지 주제에 기여하고 그 안에 통합되어 있습니다. 다음 섹션에서는 이러한 주제와 그 특징적인 요소에 대해 논의하고(그림 1 참조), 포커스 그룹 회의(FGn)에서 도출된 학생(Sn) 및 교수진(Fn)의 발언을 인용하여 설명합니다. 
We identified 5 main themes that reflect quality culture constituents to CQI:

  • (1) fostering an open systems perspective,
  • (2) involving stakeholders in educational (re)design,
  • (3) valuing teaching and learning,
  • (4) navigating between ownership and accountability, and
  • (5) building on integrative leadership.

A supportive communication climate (which can be fueled by the organization’s leaders) contributes to, and is integrated within, the first 4 themes. In the following sections, we discuss the themes and their characterizing elements (see Figure 1) and illustrate them with student (Sn) and faculty (Fn) quotes derived in the focus group meetings (FGn).

조사 결과 참가자들이 양질의 문화에서 가장 중요하게 생각하는 요소에 대한 일관된 그림이 드러났습니다. 따라서 결과는 통합된 방식으로 제시됩니다(학생과 교수진의 의견을 번갈아 제시). 참가자들의 응답은 PDCA 모델에 따라 구조화된 질문과 일치했으며, 이는 교직원과 학생들이 개선을 위한 체계적인 작업 방식을 내재화했음을 의미합니다. 이러한 결과는 포커스 그룹을 시작할 때 실시한 서면 연습에서 응답자들이 체계적인 접근 방식이나 PDCA 주기의 중요성을 자주 언급한 결과로 뒷받침됩니다(보충 디지털 부록 2: https://links.lww.com/ACADMED/A910).
Our findings reveal a consistent picture on what participants considered the most important aspects of a quality culture. The results are therefore presented in an integrated manner (alternating student and faculty quotes). Participants’ responses were in line with prompts structured under the PDCA model, which implies that faculty and students have internalized a systematic way of working on improvement. This finding is supported by results from the written exercise performed at the start of the focus groups in which respondents frequently referred to the importance of systematic approaches or the PDCA cycle (Supplemental Digital Appendix 2 at https://links.lww.com/ACADMED/A910).

개방적 시스템 관점 육성
Fostering an open systems perspective

참가자들은 개방적인 분위기와 외부 지향성, 실험과 혁신의 자유가 CQI에 필수적인 요소로 꼽았습니다. 이들은 내부적 연계(과정 간)와 외부적 연계(현장의 발전 및 요구사항)에 대한 끊임없는 필요성을 표명합니다. 다음 교수진의 사례에서 알 수 있듯이 개방형 시스템 관점을 취하는 것은 다양한 수준에서 관련이 있습니다:
Participants identify a general atmosphere of openness, combined with an external orientation and freedom to experiment and innovate as essential for CQI. They express a constant need for internal alignment (between courses) and external alignment (with developments in, and requirements of, the field). As the following faculty member illustrates, taking an open systems perspective is relevant on different levels:

[코스 코디네이터는 엄밀한 의미에서 자신의 임무보다 조금 더 멀리 내다볼 필요가 있습니다. 저는 항상 이를 미시적 수준이라고 부르는데, 교육으로의 전환이 이루어지는 곳입니다. 하지만 중급 수준도 있습니다. 프로그램 내에서 무엇이 선행되고 무엇이 뒤따르는지를 고려해야 합니다. 수평적, 수직적 연결이 어떻게 이루어지고 있는지, 거시적 수준에서는 프로그램에서 사회적 맥락과 전문성에 대한 수요, 대학원 교육과의 연계까지....... 이렇게 복잡한 상황에서 교육을 발전시켜야 한다는 것이죠. 이것이 바로 도전입니다. (FG1, F5)
[Course coordinators] need to look a bit further than just their assignment in the strict sense. I always call that the microlevel, where the translation to education is being made. But then, there’s also the mesolevel. Within the program, you should consider what precedes and what comes after. How the horizontal and vertical connections are made, and at the macrolevel, from the program to the societal context and the demand for specialisms, the alignment with postgraduate training…. That you have to develop education in such a complicated situation. That is the challenge. (FG1, F5)

참가자들은 조직의 적응력에 중요한 기여 요인으로 개방성과 외부 지향성을 언급합니다. 프로그램 환경 내부 또는 외부에서 얻은 새로운 인사이트는 CQI의 변화 동인으로 간주됩니다. 인증 패널의 권고, 의료계의 다른 프로그램과의 벤치마킹, 노동 시장의 의견, 교육자, 동료 및 학생의 조언 등을 통해 얻은 새로운 인사이트가 그 예입니다. 이러한 인사이트를 실행에 옮기려면 유연성, 창의성, 실험과 혁신의 여지가 필요합니다:
Participants refer to openness and an external orientation as important contributors to the organization’s adaptivity. New insights obtained from within or outside the program environment are seen as drivers of change for CQI. Examples are new insights derived through recommendations by accreditation panels; benchmarking with other programs in medicine; inputs from the labor market; and advice from educationalists, peers, and students. To translate these insights into practice, flexibility, creativity, and room to experiment and innovate are needed:

실수를 해도 괜찮고 실수에 대해 솔직하게 말할 수 있는 문화가 조성되어야 합니다. 모든 것이 완벽하게 이루어지기를 서로에게 요구한다면 이를 억제해야 합니다....... 또한 ... 교육 자료가 완전히 고정되어 있지 않고 약간 유연한 형식으로되어있어 모든 사람이 자신의 길을 찾을 수 있습니다. 이렇게 하면 사람들이 더 많이 참여하고 다양한 접근 방식이 어떻게 작동하는지에 대한 정보를 수집할 수 있습니다. (FG1, F2)
The related culture is that it is all right to make mistakes and that you can be open about it. If we demand from each other that everything goes perfect, then you suppress that…. Also … that the educational material is not completely set in stone, but has a bit of a flexible format, so that everyone can find their way in it. That way, people thrive and you can collect information on how different approaches work. (FG1, F2)
교수진이 교육 접근 방식을 과감하게 바꾸는 것이 중요합니다. 예를 들어, 강의가 그 자체로 효과적이지 않다는 말은 종종 있습니다. 그러나 우리는 여전히 강의가 일반적인 관행인 상황에 처해 있습니다. (FG3, S1)
It is important that faculty dares to change educational approaches. For instance, it has often been said that lectures are not per se effective. However, we are still in a situation in which lectures are common practice. (FG3, S1)

학생과 교수진은 특히 커리큘럼 내용의 일관성을 유지하기 위해 교수팀 내, 교수 코디네이터 간의 소통에 주의를 기울여야 한다고 지적합니다. 
Students and faculty point out that communication within teaching teams and between teaching coordinators requires attention, especially to safeguard alignment of the curriculum content.

교육 (재)설계에 이해관계자 참여하기
Involving stakeholders in educational (re)design

EC 위원들은 교육을 (재)설계할 때 교수진과 학생의 참여와 공동의 목표 지향성에 주목해야 한다고 설명합니다. 이러한 참여는 다양한 시나리오, 기회, 과제를 고려하는 데 기여합니다: 
The EC members elaborate that involvement and a shared goal orientation of faculty and students in the (re)design of education require attention. This involvement contributes to considering various scenarios, opportunities, and challenges:

모든 사람은 그것[교육 발전]에 대해 다른 관점을 가지고 있으며, 모든 사람이 다른 가능성이나 구현을 봅니다. 예를 들어, 일이 잘못될 수도 있습니다. 서류상으로는 훌륭해 보이지만 학생과 교수진이 경험하는 것은 완전히 다를 수 있습니다. 그렇기 때문에 모든 사람의 참여가 필요하다고 생각합니다. (FG5, S2)
Everyone has a different view on it [development of education], and everyone sees different possibilities or implementations. Also, for example, where things can go wrong. Something can look great on paper but can be experienced totally different by students and faculty. That is why I think we need everyone on board. (FG5, S2)

응답자들은 또한 기대치를 명확히 하고 불확실성을 제거하여 교육 변화 활동에 대한 지원을 창출하기 위해서는 교직원과 학생의 참여가 중요하다고 언급합니다. 그러나 실제로는 더 큰 규모의 커뮤니티를 참여시키고 소통하는 목표를 달성하기가 어려울 수 있습니다:
Respondents further note that involvement of faculty and students is important to clarify expectations, take away uncertainty, and therewith create support for educational change activities. However, in practice, the objective to involve—and communicate with—the larger community can be difficult to meet:

계획 개발의 전체 기간 동안 우리는 계획을 실행해야 하는 사람들을 충분히 참여시키지 못했습니다. 이들에게 충분히 알리지 않았고, 갑자기 이런 일이 발생하니 압박감이 컸습니다. 반드시 필요한 일이었습니다. 사람들이 우리가 왜 이런 일을 하는지 충분히 알지 못했습니다. 이유가 무엇이었는지요. (FG6, F6)
During the whole period of plan development, we insufficiently involved the people who eventually needed to implement it. We did not inform them sufficiently and what happens all of a sudden, the pressure was on. It needed to happen. Without people being fully aware of why we were doing this. What the reason was. (FG6, F6)

상향식 개선 접근 방식과 하향식 개선 접근 방식 사이의 균형이 자주 언급됩니다. 한편으로는 기본 틀과 (재)개발 구조를 제공하기 위해 조직 고위층에서 시작된 접근 방식이 필요하고, 다른 한편으로는 교수진과 학생들이 계획에 기여할 수 있는 능력이 중요합니다: 
The balance between bottom-up and top-down improvement approaches is often referred to. On the one hand, approaches initiated at a high level in the organization are needed to provide a basic framework and (re)development structure, while on the other hand, the ability of faculty and students to contribute to plans is important:

하향식도 상향식도 아닌, 모든 이해 당사자들과 협의하여 문제를 파악하고 그런 방식으로 합의에 도달할 수 있어야 한다는 것이죠. 적어도 그것이 철학입니다. 코디네이터나 교수가 커리큘럼을 설계하고 "이렇게 하세요."라는 식으로 실행하는 것이 아니라 말이죠. (FG5, S3)
That it is neither a top-down nor a bottom-up implementation, but that we identify issues in consultation with all parties involved, and that a consensus can be reached that way. At least, that is the philosophy. Instead of a coordinator or professor designing a curriculum and implementing it like “here you go.” (FG5, S3)

구체적인 가치는 대표 기관에 귀속됩니다. 이러한 기구들은 새로운 아이디어를 발굴하고 모범 사례에 대한 통찰과 공유를 바탕으로 조언을 제공함으로써 개선에 기여합니다: 
Specific value is attributed to representative bodies. These bodies contribute to improvement by identifying new ideas and providing advice based on insights in—and sharing of—best practices:

저는 학생들이 교육 분야의 강점과 모범 사례를 잘 파악하고 있다고 생각합니다..... 제 생각에 학생[대표]으로서 우리가 해결해야 할 과제 중 하나는 이러한 품질 측면이 실제로 무엇인지 인식하고 "이것이 아이디어였고, 이것이 구현되는 방식이며, 품질을 개선하기 위해 주목해야 할 지점"을 고려하는 것입니다. (FG3, S1)
I believe that students are very well equipped to map the strong aspects and best practices within education…. In my opinion, part of the challenge for us as students [representatives] is to be aware of what those quality aspects actually are … to consider “this was the idea, this is how it is implemented, and these are the points of attention to improve quality.” (FG3, S1)

응답자들은 이해관계자의 참여를 보장하기 위해 변화 리더에게 프로젝트 관리 및 커뮤니케이션 기술이 필요하다고 지적합니다. 의학교육 환경에서 이러한 기술에 대한 관심이 부족하기 때문에 참여가 부족한 것으로 나타났습니다.
The respondents indicate that to safeguard stakeholder involvement, change leaders require project management and communication skills. A lack of attention for such skills in the medical education setting is expressed as reason for experiencing a shortfall of involvement.

교육과 학습의 가치
Valuing teaching and learning

교육을 개선하려는 교수진의 동기는 학생을 향한 (학문적, 직업적) 발전에 대한 헌신에서 비롯됩니다. 예를 들어, 교수진은 교육 활동 후 학생들과의 만남을 통해 동기 부여 효과를 얻을 수 있으며, 이 시간 동안 학생들은 특정 활동의 학습 효과에 대해 감사를 표합니다. 학생들에 따르면 교수진의 헌신은 여러 가지 방식으로 나타납니다(예: 가시성 및 접근성, 학생 질문에 대한 응답, 강의에 대한 열정). 헌신적인 교수진은 자기 성찰적인 태도를 취하는 경향이 더 강하고 피드백에 더 잘 반응하며, 이는 내부적인 개선 의지에서 비롯됩니다: 
The motivation of faculty to improve education stems, to an important degree, from commitment to the (academic and professional) development of students. Faculty, for example, point to the motivating effect of meeting with students after teaching activities, during which time students express appreciation for the learning effect of certain activities. According to students, faculty’s commitment manifests in several ways (e.g., through visibility and approachability, responsiveness to student questions, enthusiasm in lecturing). Committed faculty are more inclined to attain a self-reflective attitude and are more responsive to feedback, stemming from an internal drive to improve:

네, 우선은 개선하려는 교사의 의지가 중요하다고 생각합니다. 이것은 매우 다양한 개선 계획을 통해 얻을 수 있는 것이기도 합니다: 한 교사는 다른 교사보다 비판에 훨씬 더 개방적입니다. (FG2, S2)
Yes, I believe for one thing, it involves the willingness of the teacher to improve. This is also what you get from many very different improvement plans: One teacher is far more open to criticism than the other. (FG2, S2)

교사와 학생 모두 의사 결정 과정에 참여하고 목표를 공유하는 것이 헌신에 기여한다고 보고합니다. 일반적으로 교육에 대한 관심과 인정은 교사의 가르침에 대한 동기를 부여합니다. 다음 교직원의 말은 교육보다 연구를 중시하는 것이 교육에 투자하려는 노력을 방해할 수 있음을 보여줍니다:
Both teachers and students report that their participation in decision-making processes and shared goal formulation contribute to commitment. In general, attention to and appreciation of education fuel faculty’s motivation to teach. The following quote from a staff member illustrates that valuing research over education can hamper efforts to invest in education:

학과장은 교수진의 연구와 논문을 작성하는 것에 훨씬 더 많은 관심을 보입니다. 블렌디드 러닝을 구현하거나 새로운 학습 프로그램 청사진을 만들면 그것도 성과가 되지만, 부서장을 행복하게 하는 성과는 아닙니다. (FG5, F2)
A head of department is far more interested in faculty members’ research and you producing manuscripts. That is something which pays off … if you implement blended learning or create a new study program blueprint … that’s also output, but it’s not the kind of output that makes the department head happier. (FG5, F2)

6개 기관의 응답자 모두 과도한 업무량이 교육에 전념하는 데 방해가 된다고 답했습니다. 이에 반해 상급 경영진의 교육에 대한 관심과 인정은 헌신을 자극합니다: 
Respondents from all 6 institutions report that excessive workload hinders the commitment to education. As a counterbalance, attention to—and appreciation of—education from upper management stimulates commitment:

교육에 대한 관심과 열의에 따라 교육의 질이 만들어지기도 하고 깨지기도 하는데, 교육은 하다 보면 좋아지기 시작하고 자동적으로 향상되기 때문입니다. 그래서 주로 시간과 관심, 주변 사람들의 관심이 중요하다고 생각합니다. 그래서 네, 교육 경력 경로와 연례 평가에서 상사의 관심... 그런 모든 종류의... 교육에 대한 조직 내 관심. (FG3, F2)
The quality of education is made or broken by the attention to and enthusiasm for education, because if you do it, you will start to like it, and you will also automatically improve. So I think it mostly concerns time and attention and interest from other people. So yes, educational career paths and your superior’s attention during the annual appraisal … all that kind of … attention within the organization for education. (FG3, F2)

교수진은 헌신적인 교직원들이 동료 교사와의 대화나 비공식적으로 학생의 의견을 묻는 등 (표준 교육 평가 외에) 추가적인 피드백을 적극적으로 구한다고 설명합니다. 학생 대표들은 교수와 학생 간의 짧은 거리와 학생이 비공식적으로 피드백을 제공할 수 있는 기회가 이러한 측면에서 매우 중요하다고 덧붙였습니다. 참가자들은 교직원-학생 커뮤니티 구축을 통해 교수와 학습을 중시하는 것이 중요하며, 이는 대면 상호작용의 여지가 많은 소규모 단위의 교육을 통해 달성할 수 있다고 지적합니다. 
Faculty explain that committed staff members actively seek additional sources of feedback (other than the available standard educational evaluations), for instance, by sparring with peers and by informally asking for student opinions. The student representatives add that a short faculty–student distance and student opportunities to provide feedback informally are instrumental in this respect. Participants point to the importance of valuing teaching and learning through staff–student community building, which can be reached through teaching in smaller units with more room for face-to-face interaction, for example.

소유권과 책임 사이에서 탐색하기
Navigating between ownership and accountability

교수진에 따르면, 자신의 통찰력을 바탕으로 교수 학습 활동을 설계하면 주인의식을 고취하고 지속적으로 개선하려는 동기를 강화할 수 있다고 합니다. 교육 (재)개발을 (팀) 공동의 과제로 삼고 전문가적 자율성의 여지를 두면 팀이 헌신한다면 교육의 질에 영향을 미칩니다:
According to faculty, designing teaching and learning activities based on their own insights stimulates ownership and reinforces their motivation to continuously improve. Making the (re)development of education a shared (team) task, with room for professional autonomy, affects educational quality, provided that the team is committed:

최고의 기획 그룹은 광범위한 토론이 이루어지고 토론을 피하지 않는 팀입니다..... 헌신과 함께한다는 느낌이 있어야 합니다. "우리는 무언가를 창조할 것이다." "우리는 무언가를 만들 것이다." 이것이 바로 퀄리티를 얻는 방법입니다. 때때로 관리 기관에서 정한 경계를 넘을 수도 있습니다. 하지만 그런 경우에도 그들은 어쨌든 당신에게 호루라기를 불 것입니다. (FG1, F4)
The best planning groups are teams in which extensive discussions take place, and where discussion is not avoided…. There has to be commitment and a feeling of togetherness. “We are going to create something.” “We are going to build something.” That is how you get quality. Maybe you will sometimes cross the boundaries set by the management institute. But yes, in that case, they will blow the whistle on you anyway. (FG1, F4)

자율성과 주인의식은 동료 지원 및 책무성 절차(즉, 체계적인 평가 및 개선 접근 방식으로서의 PDCA 주기)와 연계되어야 합니다. 결과 평가보다 단기적인 학생 만족도와 과정에 중점을 두는 것은 교수진이 선호하는 학생의 종단적 발달에 초점을 맞추는 것과 모순됩니다. 또한 평가가 혁신과 연계되지 않거나 학습 효과를 포착하지 못할 수도 있습니다:
Autonomy and ownership need to be linked with peer support and accountability procedures (i.e., the PDCA cycle as systematic evaluation and improvement approach). An emphasis on short-term student satisfaction and process over outcome evaluations contradicts faculty’s preferred focus on the longitudinal development of students. Moreover, evaluations might not be aligned to innovations or fail to capture learning effects:

목표에 도달했는지 훨씬 더 광범위하게 측정해야 하고, 그러면 다시 제 목표로 돌아와야 합니다. 제 목표는 시험이 끝나거나 학기가 끝난 후 학생들이 행복해하는 모습을 보는 것이 아닙니다. 제 목표는 학생들이 훌륭한 의사가 되는 것을 보는 것입니다. 그것이 제 목표입니다. (FG6, F5)
You need to measure to a much larger extent if you reached your goals, and then I come back to my goal again. My goal is not to see happy students after their exam or after the semester. My goal is to see them become a great doctor. That is my goal. (FG6, F5)

학생과 교수진은 피드백이 건설적인 것이 중요하다고 말합니다. 여기서 응답자들은 내러티브 피드백의 부가가치에 대해 구체적으로 언급합니다. 평가의 후속 조치에 대한 정보와 커뮤니케이션이 부족하면 피드백의 질이 떨어집니다. 평가의 목적과 후속 조치에 대한 추가 설명은 학생이 양질의 피드백을 제공할 가능성을 높일 수 있습니다. 또한 명확한 책임 구조를 포함시키는 것이 중요합니다
Students and faculty report that it is key for feedback to be constructive. Herein, respondents refer specifically to the added value of narrative feedback. A lack of information and communication on the follow-up of evaluations mitigates feedback quality. Further explanation of the purposes and follow-up of evaluations can enhance the likeliness of students’ provision of high-quality feedback. Moreover, embedding clear responsibility structures is key:

저희는 1년 넘게 무슨 일이 일어나고 있는지 알아내려고 노력해 왔습니다. "저 사람은 이렇게 하고, 저 사람은 저렇게 하고, 저 사람은 저렇게 피드백을 준다"라고 말할 수 있는 사람은 아무도 없습니다. 코스를 보면 개선하는 것이 번거롭고 몇 년이 걸립니다. 하지만 어떤 방식으로 구조화되어 있고 누가 무엇을 책임지는지 위에서부터 아래로 명확해야 한다고 생각합니다. (FG4, S1)
We have been trying to find out what is going on for more than a year now. There is nobody who can tell us “that person does this, that one does that, and that one gives feedback,” etc. If you look at a course, it is cumbersome to improve and it will take years. But I think it should be clear from the top down that it is structured in a certain way and who is responsible for what. (FG4, S1)

복잡한 조직 구조와 많은 교직원이 조정 및 실행 역할에 관여하는 것평가 결과에 대한 후속 조치를 어렵게 하는 요인으로 꼽힙니다.
Complex organizational structures and involvement of many faculty members in coordination and implementation roles are considered to frustrate the follow-up on evaluation results.

통합적 리더십 구축
Building on integrative leadership

도표 1은 통합적 리더십이 앞서 설명한 4가지 주제에 존재하는 긴장을 엮어내는 데 어떻게 기여하는지를 요약한 것입니다. 개방적 시스템 관점을 키우는 것과 관련하여 학생을 포함한 다학제적 팀을 구성하는 것이 중요한 것으로 간주됩니다. 이러한 팀을 구성하면 책임과 의사결정을 공유하고 다양한 관점을 포용할 수 있습니다:

Chart 1 summarizes how integrative leadership contributes to intertwining the tensions present within the 4 themes described previously. With regard to fostering an open systems perspective, the formation of multidisciplinary teams (including students) is considered important. Such teams allow for shared responsibilities, decision making, and the inclusion of different perspectives:

작업을 수행해야 하는 그룹부터 시작해야 한다고 생각합니다. 그룹은 너무 크지 않고 가능한 한 이질적이어야 하며, 교육 조직 내 모든 구성원으로부터 해당 과제를 수행하도록 승인을 받아야 합니다. (FG3, S1)
I think you should start with the group that has to perform the task. The group should not be too large, but as heterogeneous as possible … and it should get approval from everyone within the educational organization to perform that task. (FG3, S1)

리더는 중요한 (프로그램 또는 조직) 목표와 개별 교수진의 야망, 동기 부여 및 전문성 사이의 연결 고리를 자극해야 합니다. 팀이나 개별 교사에게 너무 많은 자유를 제공하면 커리큘럼의 일관성을 해칠 수 있습니다:
Leaders ought to stimulate connective links between the overarching (program or organization) goals and individual faculty members’ ambitions, motivations, and specialties. Offering too much freedom to teams or individual teachers can endanger the curriculum coherence:

현재 많은 교사들이 [학습 프로그램 관리자가] 내용을 무시하고 자기 방식대로 교육을 개발하는 것을 볼 수 있습니다. 물론 좋은 의도로 하는 일이라는 점을 분명히 말씀드리지만, 그렇게 되면 모든 것이 뒤죽박죽이 되죠. (FG4, F2)
What you see now is that when those people [the study program management] ignore the content, a lot of teachers develop education in their own way. Of course they do this with the best intentions, let me make that clear, but yeah, then everything becomes all jumbled. (FG4, F2)

팀이 자유롭게 운영할 수 있는 일반적인 프레임워크는 혁신을 위한 유연성과 효과와 효율성을 유지하기 위한 안정성 사이에서 균형을 잡는 데 도움이 됩니다:
A general framework within which teams have freedom to operate helps to establish a balance between flexibility to innovate and stability to remain effective and efficient:

예, 의도된 목표에 따라 프레임워크가 결정되고, 결국 프레임워크에 따라 사람들에게 "여러분에게는 레버리지와 일정한 자유가 있지만, 주어진 프레임워크의 경계 내에서만 가능합니다."라고 말할 수 있습니다. (FG6, F5)
Yes, the intended aims determine the framework and the framework in the end determines that you tell people, “You have leverage and a certain freedom, but within the boundaries of the provided framework.” (FG6, F5)

지지적인 커뮤니케이션 분위기를 조성하는 리더의 능력은 개선 이니셔티브의 성공을 결정하는 중요한 요소로 간주됩니다. 리더는 커뮤니케이션 프로세스를 통해 조직 변화의 비전에 대한 투명성을 높이고 참여를 촉진할 수 있습니다:
Abilities of leaders to establish a supportive communication climate are considered important determinants of the success of improvement initiatives. Through communication processes, leaders can foster transparency on the vision behind organizational change and foster involvement:

좋은 비전일지라도 그 비전이 정확히 무엇을 수반하는지 알고 싶을 수 있습니다. 비전을 공유하면 오류가 발생할 여지가 있습니다. 그러면 적어도 지평선에 있는 점이 무엇인지 알 수 있습니다. 누군가 이메일을 보내는 것을 잊어버려도 "괜찮아. 우리가 무엇을 위해 노력하고 있는지 알아요. 왜 이런 일이 지금 일어나야 하는지 알아요."라고 생각할 수 있습니다. (FG2, F2)
It could just happen to be a good vision, but then you would like to know what it entails exactly. If you share it, this vision, you have some room for errors. Then at least you know what the dot on the horizon is. It could happen that somebody forgets to send you an email, and you think, “It’s okay. I know what we are working towards. I know why this needs to happen now.” (FG2, F2)

EC 멤버들은 지원과 협력적인 교사 네트워크, 교육 커리어 트랙 구축, 교육에 대한 추가 시간 또는 보상 제공리더가 교수와 학습의 가치를 증진하고 동기를 유발하는 다양한 사례를 제시합니다. 리더는 교육에 대한 인식과 건설적인 피드백 제공을 통해 소유권(및 자율성)과 책임(및 통제)의 균형을 맞추는 데 영향을 미칩니다. 
The EC members provide various examples of the way leaders promote valuing of teaching and learning and trigger motivation: through support and collaborative teacher networks, establishing education career tracks, and offering additional time or reimbursement for teaching. Leaders affect the balance of ownership (and autonomy) with accountability (and control) through appreciation of education and the provision of constructive feedback.

토론
Discussion

이 연구에서 우리는 의학교육에서 CQI 문화를 형성하는 5가지 주제를 확인했습니다. 차트 1에서 볼 수 있듯이, 조직 내에는 교육의 질을 유지하고 발전시키기 위한 방향성 사이에 다양한 긴장이 존재합니다. 이러한 경쟁적 지향점 간의 통합과 시너지 창출을 위해서는 다학제적 협업, 개방적 의사소통, 교수진 개발 및 인적 자원에 대한 투자, 품질 관리보다 품질 향상에 초점을 맞춘 책임 절차가 강화되어야 합니다. 
In this study, we identified 5 themes that in concert shape a culture for CQI in medical education. As Chart 1 illustrates, various tensions within organizations exist between orientations to maintain and further develop the quality of education. The integration and creation of a synergy between these competing orientations call for increased multidisciplinary collaboration, open communication, investments in faculty development and human resources, and accountability procedures focusing on quality enhancement over quality control.

개방형 시스템 관점(주제 1)지식 공유 강화와 창의성 및 혁신의 자극을 통해 질적 발전을 촉진하는 데 도움이 됩니다. 이러한 관점을 취하면 학교가 점진적 단일 루프 학습과 유사한 CQI 노력에 전념하는 것에서 진정한 학습 조직(수정된 목표, 의사결정 규칙 또는 양쪽 모두의 이중 루프 학습과 유사한)으로 전환하는 데 도움이 됩니다.30 CQI 문화를 촉진하려면 의과대학의 현재 가치관을 변화시킬 필요가 분명합니다. 교수진은 일반적으로 유연하고 인간 중심적인 조직을 선호하지만,31,32 의과대학은 여전히 보수적이고, 위계적이며, 단절되고, 인본주의적 성향을 저해하는 것으로 인식되는 경우가 많습니다.22,33-35 
An open systems perspective (Theme 1) aids in promoting quality development through intensified knowledge sharing and the stimulation of creativity and innovation. Taking such a perspective aids schools in their shift from devoting to CQI efforts (resembling incremental, single-loop learning) to becoming true learning organizations (resembling modified goals, decision-making rules, or both—double-loop learning).30 To stimulate a culture for CQI, a need for change in current values of medical schools is apparent. Whereas faculty typically prefer a flexible and human centered organization,31,32 medical schools are still often seen as conservative, hierarchical, disconnected, and discouraging humanistic orientations.22,33–35

광범위한 이해관계자의 참여(주제 2)는 지식, 기술 및 (잠재적으로 경쟁적인) 가치의 다양성이 교육의 질에 대한 다면적인 개념을 다루는 데 도움이 되므로 CQI의 핵심입니다.36 그러나 의료 전문가와 학계는 (임상 서비스 활동, 부서, 학문 분야 등과 관련된) 사일로 내에서 일하고 학습하는 데 익숙하며, 이는 고유한 태도, 문제 해결 능력 및 공통 언어 사용을 강화합니다.37 또한 의사와 연구자는 일반적으로 상당한 자율성을 가지고 있습니다. 이들은 개별적인 책임을 지는 데 익숙하고 자신의 학문 분야의 이익을 옹호하는 데 열심입니다.37 다양한 배경을 가진 교수진이 참여하는 철저한 토론과 공동 창작 세션은 집단적 개선 활동을 시작하는 데 핵심적인 역할을 합니다.38 또한 CQI는 학생을 고객 또는 소비자에서 조직 문화의 적극적인 구성원 및 기여자로의 역할 변화를 수반합니다.
Broad stakeholder involvement (Theme 2) is key to CQI, as diversity in knowledge, skills, and (potentially competing) values helps to address the multifaceted notion of education quality.36 However, medical professionals and academics are used to working and learning within silos (relating to their clinical service activities, departments, disciplines, etc.), which reinforce distinct attitudes, problem-solving skills, and the use of a common language.37 Moreover, physicians and researchers typically have a large degree of autonomy. They are used to taking individual responsibility and are keen to defend interests of their own discipline.37 Thorough discussions and cocreation sessions involving faculty from different backgrounds form a keystone to initiate collective improvement activities.38 Additionally, CQI entails a role change from students as clients or consumers to active members of and contributors to the organizational culture.

인센티브와 교수진의 요구 만족도에 대한 연구와 함께,39,40 우리는 CQI의 외재적 동기에 대한 몇 가지 예시를 발견했습니다. 자금 지원, 개발 시간, 정보 공유, 교수진 개발 이니셔티브 증가를 통한 교육 및 학습의 가치인정(주제 3)은 조직의 개선 잠재력에 영향을 미칩니다.41,42 따라서 조치를 취하는 데 필요한 자원이 부족하다면 단순히 주인의식과 책임감을 고취하는 것(주제 4)은 유익한 결과로 이어지지 않을 것입니다. 
In line with research on incentives and faculty needs satisfaction,39,40 we found several illustrations of extrinsic motivators for CQI. The valuing of teaching and learning (Theme 3) through increased funding, time for development, information sharing, and faculty development initiatives influence the organization’s improvement potential.41,42 Hence, a mere promotion of ownership and accountability (Theme 4) will not lead to beneficial results if required resources to take action are lacking.

우리의 연구 결과는 통합적 리더십(주제 5)이 양질의 문화 발전을 지원한다는 것을 보여줍니다. 의학교육의 리더십은 개별 교수진의 감독, 지도, 지원에서 더 넓은 집단에 초점을 맞추는 것으로 변화하고 있습니다. 리더는 강력한 리더십에 CQI의 책임을 돌리는 대신 동기부여자, 멘토, 촉진자 역할을 해야 합니다. 지식 집약적인 의과대학 환경에서 리더와 직원들은 조직의 문제에 대한 의미와 해결책을 재구성합니다.43 의학교육의 CQI는 다양한 스타일을 결합할 수 있고 다양한 이해관계자의 목표와 야망을 통합할 수 있는 리더가 가장 잘 수행할 수 있습니다.44 
Our findings indicate that integrative leadership (Theme 5) supports further quality culture development. Leadership in medical education is changing from individual faculty supervision, guidance, and support to a focus on the broader collective. Instead of attributing responsibility for CQI to strong leadership, leaders are expected to be motivators, mentors, and facilitators. In the knowledge-intensive setting of medical schools, leaders and employees coconstruct meaning and solutions to organizational issues.43 CQI of medical education is best served with leaders who are able to combine multiple styles and who are able to coalesce different stakeholder goals and ambitions.44

실천 커뮤니티(CoP)45를 구현하면 지속적인 개선을 위한 문화 주제를 다룰 수 있는 기회의 창이 열립니다. CoP는 다양한 학문적 배경을 가진 교사들 간의 상호작용을 촉진하고 새로운 관점을 얻을 수 있는 기회를 창출합니다. 또한 이러한 커뮤니티 내에서 조직된 (연수) 활동은 참여 의식을 강화하고 역량, 자신감, 신뢰성, 유대감을 향상시킵니다.46 CoP는 교수 학습, 개인 개발, 교사 정체성 확립을 중시하는 환경을 형성합니다. 교육 및 학습 커뮤니티의 건설적인 동료 피드백 프로세스는 교육 개선에 대한 주인의식과 책임감의 균형을 맞추는 데 도움이 됩니다. CoP의 개방적이고 종적인 성격은 그 부가가치에 필수적이며, 단순히 우수한 챔피언들의 일시적인 리그를 구축하는 것을 넘어서야 한다는 점에 유의해야 합니다.
The implementation of communities of practice (CoPs)45 opens a window of opportunity to address the themes of a culture for continuous improvement. CoPs facilitate interaction between teachers from diverse disciplinary backgrounds and create opportunities to gain new perspectives. Moreover, (training) activities organized within these communities strengthen a sense of involvement and enhance competence, confidence, credibility, and connection.46 CoPs form an environment in which the valuing of teaching and learning, personal development, and teacher identity building is central. Constructive peer feedback processes in teaching and learning communities help to balance ownership and accountability for educational improvement. It should be noted that an open and longitudinal character of CoPs is essential to their added value; they should go beyond the mere establishment of a temporal league of quality champions.

이론적 및 실무적 의의
Theoretical and practical significance

본 연구는 표준 품질 관리 접근 방식이 품질 문화 개념에서 도출된 통찰력으로 보완되어야 한다는 교수진과 학생들의 의견을 대변합니다. 보고된 결과는 이 이론적 개념에 대한 더 깊은 통찰력을 얻는 데 기여했습니다. 우리의 연구 결과는 책무성을 강조하는 정적인 접근 방식에서 전문가의 자율성과 커뮤니티 구축의 여지가 있는 유연한 접근 방식으로의 전환을 시사합니다. CQI는 교육의 가치, 교수진의 동기 부여, 학생의 개인적, 학업적, 전문적 개발에 더욱 집중할 것을 요구합니다. 
The present study gives voice to the opinion of faculty and students that standard quality management approaches should be complemented by insights derived from the quality culture concept. The reported results have contributed to gaining a deeper insight into this theoretical notion. Our findings imply a shift from static approaches emphasizing accountability toward flexible approaches with room for professional autonomy and community building. CQI requires a stronger focus on the valuing of teaching; faculty motivation; and student personal, academic, and professional development.

강점과 한계
Strengths and limitations

이 연구는 질적 문화를 경험적으로 연구한 몇 안 되는 연구 중 하나이기 때문에 이용 가능한 문헌에 기여합니다. 여러 기관에서 데이터를 수집하고 교수진과 학생의 관점을 모두 고려했습니다. 연구 결과에 따르면 의과대학 내 가치 지향성이 품질 관리 접근 방식과 교수진의 업무 관련 심리적 태도가 상호 작용하는 방식을 설명하는 데 매우 관련이 있는 것으로 나타났습니다. 참여 의과대학에서는 이러한 지향점이 다소 동질적이었지만, 다른 상황에서는 다를 수 있습니다. 네덜란드 의과대학(8개 중 6개)만 조사 대상에 포함되었다는 점도 결과의 해석과 적용 가능성에 있어 고려해야 할 사항입니다. 또 다른 한계는 EC 위원들의 견해가 다른 중요한 이해관계자들과 다를 수 있다는 점입니다. 그러나 EC는 일반적으로 조직 중간 수준에서 운영되기 때문에 경영진과 풀뿌리 관점을 모두 파악했다고 가정합니다. 세 번째 (잠재적) 한계는 응답자들이 익명성이 보장되었음에도 불구하고 다른 대학에 소속된 연구자들과 특정 경험을 공유하는 것을 꺼렸을 수 있다는 점입니다. 
This study contributes to the available literature as it is one of few studies that research quality culture empirically. We gathered data from multiple institutions and took into account both faculty and student perspectives. The findings suggest that value orientations within medical schools are highly relevant in explaining how quality management approaches and work-related psychological attitudes of faculty interact. Whereas these orientations were rather homogeneous in the participating medical schools, they might vary in other contexts. The fact that we included only medical schools from the Netherlands (6 of 8) should be taken into account in the interpretation and transferability of results. A further limitation is that views of EC members might differ from other important stakeholders. However, as ECs typically operate on the organizational mesolevel, we assume to have captured both management and grassroots perspectives. A third (potential) limitation is that the respondents, despite guaranteed anonymity, might have been reluctant to share particular experiences with researchers affiliated with another university.

향후 연구를 위한 제언
Recommendations for future research

CQI의 새로운 길을 개척하기 위해서는 개별적인 교육 개선 접근 방식을 집단적인 노력으로 전환하는 개입에 대한 향후 연구가 특히 유용할 것입니다. 또한 이 연구에서 확인된 5가지 주제와 관련된 모범 사례에 대한 사례 연구는 CQI를 위한 조직의 변화와 개발을 시작하는 데 지렛대가 될 것입니다. 
To pave new paths for CQI, future research on interventions that convert individual educational improvement approaches to collective endeavors would be particularly valuable. In addition, case studies on best practices relating to the 5 themes identified in this study would provide levers to initiate organizational change and development for CQI.

결론
Conclusions

의과대학의 3대 사명인 교육, 연구, 임상 서비스 중 교육은 전통이 지배적이기 때문에 질적 개선과 변화가 필요하다는 인식이 가장 느린 경우가 많습니다. 의과대학에서 양질의 문화를 육성하기 위한 노력은 이러한 현 상황을 타개하는 데 도움이 될 것입니다. 

Of a medical school’s 3 missions—education, research, and clinical services—education is often the slowest to recognize that quality improvement and change are necessary, with traditions being dominant. Efforts to nurture the quality cultures in medical schools will help to unfreeze this status quo.


Acad Med. 2020 Dec;95(12):1913-1920. doi: 10.1097/ACM.0000000000003406.

Shaping a Culture for Continuous Quality Improvement in Undergraduate Medical Education

Affiliations collapse

1G.W.G. Bendermacher is educational policy advisor, Institute for Education, and a PhD candidate, School of Health Professions Education, Faculty of Health, Medicine and Life Sciences, Maastricht University, the Netherlands; ORCID: https://orcid.org/0000-0002-7804-4594.

2W.S. De Grave is an educational psychologist, School of Health Professions Education, Faculty of Health, Medicine and Life Sciences, Maastricht University, the Netherlands.

3I.H.A.P. Wolfhagen is associate professor, Department of Educational Development and Research, and deputy director, Institute for Education, School of Health Professions Education, Faculty of Health, Medicine and Life Sciences, Maastricht University, the Netherlands.

4D.H.J.M. Dolmans is professor of innovative learning arrangements, Department of Educational Research and Development, School of Health Professions Education, Faculty of Health, Medicine and Life Sciences, Maastricht University, the Netherlands; ORCID: https://orcid.org/0000-0002-4802-1156.

5M.G.A. oude Egbrink is professor of implementation of educational innovations, Department of Physiology, and scientific director, Institute for Education, School of Health Professions Education, Faculty of Health, Medicine and Life Sciences, Maastricht University, the Netherlands; ORCID: https://orcid.org/0000-0002-5530-6598.

PMID: 32287081

PMCID: PMC7678663

DOI: 10.1097/ACM.0000000000003406

Free PMC article

Abstract

Purpose: This study sought to identify key features of an organizational quality culture and explore how these features contribute to continuous quality improvement of undergraduate medical education.

Method: Between July and December 2018, researchers from Maastricht University in the Netherlands conducted a multicenter focus group study among 6 education quality advisory committees. Participants were 22 faculty and 18 student representatives affiliated with 6 medical schools in the Netherlands. The group interviews focused on quality culture characteristics in relation to optimizing educational development, implementation, evaluation, and (further) improvement. Template analysis, a stepwise type of thematic analysis, was applied to analyze the data.

Results: Five main themes resembling quality culture constituents to continuous educational improvement were identified: (1) fostering an open systems perspective, (2) involving stakeholders in educational (re)design, (3) valuing teaching and learning, (4) navigating between ownership and accountability, and (5) building on integrative leadership to overcome tensions inherent in the first 4 themes. A supportive communication climate (which can be fueled by the organization's leaders) contributes to and is integrated within the first 4 themes.

Conclusions: The results call for a shift away from static quality management approaches with an emphasis on control and accountability toward more flexible, development-oriented approaches focusing on the 5 themes of a culture for continuous quality improvement. The study provides new insights in the link between theory and practice of continuous quality improvement. Specifically, in addition to quality management systems and structures, faculty's professional autonomy, collaboration with peers and students, and the valuing of teaching and learning need to be amplified.

"'더 나은 것'이 무엇인지 알려주세요!": 의과대학생은 피드백을 자기조절학습의 렌즈를 통해 어떻게 경험하는가 (BMC Med Educ, 2023)
“Tell me what is ‘better’!” How medical students experience feedback, through the lens of self‑regulatory learning
Muirne Spooner1*, James Larkin2, Siaw Cheok Liew3, Mohamed Hasif Jaafar3, Samuel McConkey4 and Teresa Pawlikowska1 

 

배경
Background

피드백은 혁신적인 변화를 통해 학습자를 지원할 수 있는 잠재력을 가지고 있습니다[1, 2]. 효과적일 경우 학업 성취도를 향상하고 학습을 촉진하는 데 가장 영향력 있는 요소 중 하나입니다[3]. 그러나 피드백 및 피드백 프로세스는 다양하게 해석되고[4], 제대로 실행되지 않는 등 발달 계획에 따른 명시적인 메시지가 전달되지 않습니다[5]. 고등교육 학습자들은 피드백을 중요하게 생각하지만, 피드백에 대한 불만을 지속적으로 전달하고[6], 피드백을 사용하지 않는 경우도 많습니다[7,8,9,10].
Feedback has potential to support learners through transformational change [1, 2]. When effective, it is one of the most influential factors in improving academic achievement and promoting learning [3]. However, feedback and feedback processes are variably interpreted [4] and poorly executed, i.e. explicit messaging with developmental planning does not occur [5]. While learners in higher education value feedback, they also consistently communicate dissatisfaction with feedback [6], and frequently do not use it [7,8,9,10].

문헌에서는 전통적으로 교사 중심의 단방향 피드백이 보고되어 왔으며, 이는 보건 전문 교육에서 재개념화되었습니다[11, 12]. 이제 학습자는 자신의 개발 계획에 협력하는 능동적인 참여자로 간주됩니다. 피드백 수용에 대한 이러한 재조명은 학습자의 능동적 수용의 속성을 성공의 기본으로 확인했습니다: SAGE(자기 평가, 평가 리터러시, 목표 설정 및 자기 조절, 참여 및 동기 부여). 피드백 리터러시 "정보를 이해하고 이를 업무 또는 학습 전략을 개선하는 데 사용하는 데 필요한 이해, 역량 및 성향"으로 정의되며, 학습자가 피드백을 이해하고 실행하는 데 참여하고 유능한 사람으로 자리매김합니다[13]. 
Literature has traditionally reported teacher-centred unidirectional feedback, which has been reconceptualised in health professional education [11, 12]. Learners are now considered active participants who collaborate in their developmental planning. This re-focus on feedback uptake has identified attributes of learner proactive recipience as fundamental to its success: SAGE (self-assessment, assessment literacy, goal-setting and self-regulation, engagement and motivation). Feedback literacy, defined as “the understandings, capacities and dispositions needed to make sense of information and use it to enhance work or learning strategies”, positions the learner as engaged and competent in making sense of, and implementing feedback [13].

자기조절 학습(SRL) 이론은 심리학, 교육학, 사회학에서 유래한 잘 정립된 이론입니다. 이는 목표 설정, 전략 선택, 비용과 효과의 균형, 행동 모니터링, 적응 및 수정이 특징인 과제에 참여하는 스타일입니다. 인지적, 메타인지적, 행동적, 동기적, 정서적 영향과 함께 학습에 대한 총체적인 이해를 제안합니다. SRL의 주요 측면에는 기존의 지식과 신념을 활용하고 그에 따라 재조정하기 위해 결과를 모니터링하는 능동적 학습자가 포함됩니다 [14]. 따라서 학습자가 피드백과 같은 학습 정보를 어떻게 받아들이는지 이해하는 데 유용한 모델입니다.
Self-regulatory learning (SRL) theory is a well-established theory with origins in psychology, education and sociology. It is a style of engaging with tasks which is characterised by setting goals, selecting strategies, balancing cost and effect, monitoring, adapting and amending behaviours. It proposes a holistic understanding of learning, with cognitive, metacognitive, behavioural, motivational and emotional influences. Key aspects of SRL include active learners who draw on existing knowledge and beliefs and monitor outcomes to recalibrate accordingly [14]. It is therefore a helpful model to understand how learners take up learning information such as feedback.

다양한 SRL 모델이 존재하며, 본 연구에서는 세 가지 주요 모델에서 강조되는 공통된 개념을 사용합니다[15,16,17].

  • 첫째, SRL은 맥락에 중점을 둔 상황 중심 학습 접근 방식을 취합니다. 이는 피드백 반응이 발생하는 맥락과 분리될 수 없다는 기존의 연구 결과와 일치합니다[18].
  • 둘째, 자기조절은 목표 지향성을 특징으로 하며, 현대 교육은 학습 이득을 위한 변화에 영향을 미치는 피드백에 중점을 둡니다. "루프 닫기"는 적극적인 피드백을 통해 학습자의 현재 수준에서 더 높은 수준으로 이동하는 것을 의미합니다[19]. '변화를 위한 코칭'은 피드백 대화의 R2C2 모델과 관련이 있으며, 코칭은 합의된 개발 계획을 도출하는 협력적인 양방향 피드백 상호 작용의 일부입니다[20].
  • SRL은 학습 정보에 대한 학습자의 행동 변화가 학습자의 동기 및 인지와 관련이 있다는 점을 강조합니다. 피드백 반응은 종종 성장 마인드를 가진 동기 부여와 이를 실행하기 위한 인지 능력 적용과 연결됩니다[21].
  • 또 다른 주요 특징은 자기조절 학습자가 학습 전략을 지속적으로 점검하고, 재조정하고, 조정하는 재귀적 특성입니다.
  • 마지막으로, 반복할 때마다 다른 맥락에 따라 변화하는 역동적인 과정입니다. 

A number of SRL models exist; this study will employ the shared concepts emphasised across three seminal models [15,16,17].

  • Firstly, SRL takes a situated learning approach with an emphasis on context. This aligns with established findings that feedback responses cannot be separated from the context in which they occur [18].
  • Secondly, self-regulation is characterised by goal-directedness, with contemporary teaching focussing on feedback that effects change for learning gain. “Closing the loop” describes moving from the learner’s current standard to a higher level via actively implementing feedback [19]. “Coaching for change” relates to the R2C2 model of feedback conversations, whereby coaching is part of a collaborative bi-directional feedback interaction resulting in an agreed developmental plan [20].
  • SRL emphasises that learner’s behavioural changes in response to learning information relate to their motivations and cognition. Feedback responses are often linked to motivation with a growth mind-set and applying cognitive ability to implement it [21].
  • Another key feature is its recursive nature: the self-regulated learner constantly checks, re-calibrates and tweaks learning strategies.
  • Finally, it is a dynamic process which alters with each iteration and with differing contexts.

능동적으로 피드백을 받는 학습자는 학습자의 참여와 피드백 수용에 필요한 요소에 대한 이해를 발전시켜 왔습니다. 피드백 리터러시는 이를 바탕으로 학습자가 피드백을 사용하는 행위에서 중심을 잡을 수 있도록 합니다. 자기조절학습(SRL) 이론은 주어진 피드백 상호작용에서 학습자에게 나타나는 역동적인 전략을 탐구할 수 있는 또 다른 렌즈를 제공합니다. 이 이론은 이러한 개념의 측면을 하나의 프레임워크 아래에 통합합니다. SAGE 분류법(자기 평가, 평가 리터러시, 자기 조절, 참여 및 동기 부여)은 능동적인 피드백을 받기 위해 자기 효능감과 열정이 필요함을 나타내며, 이는 자기 신념과 동기가 학습 정보(예: 피드백)가 향후 학습에서 구현되는 방식에 영향을 미친다는 SRL의 주장과 유사합니다. 피드백 리터러시에서 '판단하기'와 '감정 관리하기'의 개념은 각각 메타인지 인식 및 정서 조절의 SRL 측면과 겹칩니다. 따라서 SRL은 이러한 모든 요소가 전체론적 접근 방식과 어떻게 상호 작용하는지 고려하는 중요한 프레임워크로 유용할 수 있습니다.
Proactive recipience has developed understanding of the factors required for learner engagement and uptake of feedback. Feedback literacy builds on this in centring the learner in the act of using feedback. Self-regulated learning (SRL) theory offers another lens to delve in to the dynamic strategies that play out in the learner in any given feedback interaction. It brings together aspects of these concepts under one framework. The SAGE taxonomy (self-assessment, assessment literacy, self-regulation, engagement and motivation) indicates self-efficacy and enthusiasm are needed to for proactive feedback recipience, which parallels SRL’s assertion that self-belief and motivation influence how learning information (such as feedback) is implemented in future learning. The concepts of “making judgments” and “managing emotions” in feedback literacy overlap with SRL aspects of metacognitive awareness and affective regulation, respectively. SRL may be useful therefore as an overarching framework to consider how all these factors interact with a holistic approach.

SRL의 렌즈를 적용하여 자기조절 활동이 학습 개선을 위한 피드백 통합과 어떻게 상호작용하는지 살펴보고자 합니다. 이를 위해 정서적, 인지적, 행동적, 상황적 요인 간에 상호 작용하는 하위 프로세스를 조사하고자 합니다. 이를 통해 피드백이 SRL을 활성화하는지 여부와 방법, 그리고 어느 하위 영역에서 피드백이 상호 작용하는지를 고려할 수 있습니다. 이것이 바로 우리의 연구 질문입니다: 
By applying the lens of SRL, we aim to explore how self-regulatory activity interacts with incorporating feedback for improved learning. We aim to examine sub-processes that interact between affective, cognitive, behavioural and contextual factors. It allows us to consider the feedback interaction in terms of if and how it activates SRL, and at which sub-areas. Hence our research question:

학습자의 피드백 경험은 학습의 자기 조절과 어떤 관련이 있나요?
In what ways do learners’ experiences of feedback relate to self-regulation of learning?

방법
Methods

연구 설계 및 방법론적 방향
Study design and methodological orientation

이 연구는 피드백을 참여자, 상황 및 환경적 요인에 의해 구성되는 사회적 행위로 개념화하는 상대주의적 존재론[22]에서 접근했습니다.

  • 피드백은 개인과 특정 상황(예: 성적, 대면 평가, 서면 보고서)에 따라 의미가 달라지는 역동적인 개념으로 간주합니다.

피드백에 관한 연구자의 인식론사회구성주의적 관점으로,

  • 피드백이 의미하는 바는 피드백 활동에 참여하는 사람들의 가치와 신념의 표현이며 특정한 문화적, 사회적 맥락에 내재되어 있습니다.

우리의 방법론적 접근은 의대생들의 생생한 피드백 경험을 해석하는 데 있어 질적 방법을 사용한 탐색적 접근입니다.

  • 이 연구는 질적 연구 보고를 위한 통합 기준(COREQ)에 따라 보고되었습니다(부록 B)[23]. 

This study was approached from a relativist ontology [22], where feedback is conceptualised as a social act, constructed by participants, circumstance and environmental factors.

  • We consider feedback a dynamic concept which changes meaning depending on the individual and their particular context, e.g. grades, face-to-face appraisals, written reports.

The researcher epistemology regarding feedback is socio-constructivist;

  • what feedback means is an expression of the values and beliefs of the participants in the feedback activity and are embedded in the specific cultural and social context.

Our methodological approach is exploratory using qualitative methods in interpreting the lived experience of feedback for medical students.

  • This study is reported in accordance with the consolidated criteria for reporting qualitative research (COREQ) (Appendix B) [23].

참여자
Participants

이 연구는 아일랜드 왕립 외과의과대학(RCSI) 의과대학 및 보건과학대학의 의대 4학년 학생을 대상으로 수행되었습니다. 아일랜드, 바레인, 말레이시아의 3개 캠퍼스에서 학생들을 초대하여 참여하게 했습니다. 모든 학생들은 연구 배경과 연구 질문을 설명하는 프레젠테이션을 통해 연구에 대해 소개받았습니다. 각 연구 연도에 대한 평가 및 피드백 정보는 제공되었지만 피드백 사용 방법에 대한 직접적인 교육은 제공되지 않았습니다. 
This study was undertaken with final-year medical students in the RCSI (Royal College of Surgeons in Ireland) University of Medicine and Health Sciences. Student participants from the three campuses in Ireland, Bahrain and Malaysia were invited to participate. All students were introduced to the study with a presentation explaining background to the study and research questions. They are provided with assessment and feedback information for each year of study, but not provided with directed training on how to use feedback.

연구자
Researchers

MS는 의대 졸업반 학생들과 정기적으로 피드백을 주고받는 선임 강사입니다. TP는 보건 전문직 교육 센터의 명예 교수입니다. MS와 TP는 피드백 연구에 참여하고 있으며 피드백 반응에 대한 체계적인 리뷰를 발표했습니다. JL과 MHJ는 학부 교육에 관여하지 않는 기관의 연구원입니다. 일부 저자(SCL, SM, TP, MS, TP)는 교육에 관여하고 있지만, 각 기관의 면접관(JL-아일랜드, MS-바레인, MHJ-말레이시아)은 대인관계 반사성의 원칙과 참가자에게 영향을 미칠 수 있는 힘의 역학을 인식하여 학생 평가 및 학습에 관여하지 않았습니다[24]. 팀 토론을 통해 다양한 내러티브가 해석에 반영될 수 있었습니다 [25, 26]. 
MS is a senior lecturer who regularly engages in feedback with the final-year medical students. TP is professor emerita of the Health Professions Education Centre. MS and TP are involved in feedback research and have published a systematic review on feedback responses. JL and MHJ are researchers at the institution who do not have a role in undergraduate education. Although some authors (SCL, SM, TP, MS, TP) are involved in education, the interviewer at each site (JL-Ireland, MS- Bahrain, MHJ- Malaysia) was not involved in student assessment and learning, in recognition of principles of interpersonal reflexivity and acknowledging the power dynamics that could influence participants [24]. Team discussions facilitated diverse narratives being captured in our interpretation [25, 26].

연구 참여자 모집 및 데이터 수집
Recruitment and data collection

우리 기관은 다국적 멀티캠퍼스 의과대학으로, 101개 국적의 학생들로 구성된 다양한 학생 집단을 보유하고 있습니다[27]. 모든 사이트에서 윤리 승인을 요청하고 받았습니다. 학생들의 피드백 경험을 조사하기 위해 개별 반구조화 인터뷰를 실시했습니다. 인터뷰 가이드를 설계하기 위해 SRL의 이론적 틀을 사용했습니다(부록 A). 각 사이트에서 파일럿 인터뷰를 실시하고 이를 녹음했습니다. MS는 이를 듣고 녹취록을 검토한 후 TP 및 JL과 함께 인터뷰 프로토콜을 다듬었습니다. 각 인터뷰를 시작할 때 참가자들에게 피드백 형식의 예와 광범위한 정의가 제공되었습니다. 참가자들은 피드백을 사용하는 이유와 방법에 대한 예를 포함하여 자신의 경험을 설명하도록 권장되었습니다. 특정 피드백 이벤트를 지정하지 않았기 때문에 참가자들은 어떤 경험에 대해서도 자유롭게 이야기할 수 있었습니다. 국적, 성별, 근무지에 따라 최대한의 다양성을 확보하기 위해 표본을 추출했습니다. 데이터 분석은 5번의 인터뷰 후 시작되었으며, 정보력이 충분하다고 판단될 때까지 반복적으로 진행되었습니다[28]. 
Our institution is a transnational, multi-campus medical school and our student population is diverse, with students from 101 nationalities [27]. Ethics approval was sought and received at all sites. We conducted individual semi-structured interviews to explore students’ feedback experiences. We used the theoretical framework of SRL to design the interview guide (Appendix A). We conducted pilot interviews at each site, which were recorded. MS listened to these and reviewed the transcripts, then refined the interview protocol with TP and JL. On commencing each interview, participants were provided with examples of feedback formats and a broad definition. Participants were encouraged to describe their experiences, including examples of why and how they use feedback. We did not specify particular feedback events, so participants were free to discuss any experiences. We sampled for maximal diversity for nationality, gender and site of study. Data analysis commenced after five interviews and continued iteratively until it was considered that information power was sufficient [28].

데이터 처리
Data processing

아일랜드의 인터뷰는 대면 인터뷰로 진행되었고, 바레인과 페르다나의 인터뷰는 온라인으로 진행되었습니다. 모든 인터뷰는 처음에 전사 소프트웨어(Otter.ai)를 사용하여 녹음하고 전사했습니다. 필사본은 오디오 녹음과 비교하여 정확성을 위해 편집되었습니다. 모든 식별 가능한 정보는 코딩 전에 익명으로 처리되었습니다. 이후 원본 녹취록은 파기되었습니다.
The interviews in Ireland were done in person, those in Bahrain and Perdana were conducted online. All interviews were recorded and transcribed initially using transcription software (Otter.ai). Transcriptions were edited for accuracy, by comparing with audio recordings. All identifiable information was anonymised prior to any coding. Original recordings were subsequently destroyed.

데이터 분석
Data analysis

주제별 분석에 대한 체계적인 접근법으로 템플릿 분석을 선택했습니다. 이 방법은 우리처럼 대규모 데이터 세트를 관리할 때 유용합니다. 템플릿 분석은 높은 수준의 구조를 가진 계층적 코딩의 사용을 강조합니다. 또한 선험적 코드, 즉 현재 데이터를 검토하기 전에 개발된 코드를 사용할 수 있습니다[29]. 이는 피드백 반응에 대한 이전의 체계적인 검토 결과[4]를 통해 이 분석에 정보를 제공하는 데 용이했습니다. 우리는 6단계 프로세스를 따랐습니다 [30]:
We chose template analysis as a systematic approach to thematic analysis. It is useful when managing large data sets such as ours. It emphasises use of hierarchical coding with high level of structure. It also allows for a priori codes, codes developed before examining the current data [29]. This facilitated informing this analysis with findings from a previous systematic review of feedback responses [4]. We followed the six-stage process [30]:

1. 데이터에 익숙해지기
1.
Familiarisation with data

MS와 TP가 녹취록을 읽고 다시 읽었습니다.
MS and TP read and re- read the transcripts.

2. 예비 코딩
2.
Preliminary coding

MS는 자기조절 학습 이론의 개념에 부합하는 피드백 응답을 바탕으로 선험적 주제를 제안했습니다. 그런 다음 MS는 각 캠퍼스 현장을 대표하는 다양한 국적과 성별의 학생을 대상으로 귀납적으로 새로운 코드를 선별했습니다. 한 줄 한 줄 코딩하는 대신, 각 녹취록을 읽고 다시 읽으면서 연구 질문과 관련된 상위 주제와 하위 주제를 파악했습니다.
A priori themes were suggested by MS based on feedback responses aligned to concepts of self-regulatory learning theory. MS then inductively screened for new codes initially on a selection of interviews representing each campus site, students of varying nationality and gender. Rather than coding line-by-line, each transcript was read and re-read to identify high-level themes and sub-themes relevant to the research questions.

3. 클러스터링
3.
Clustering

새로 식별된 주제는 시각적 매핑을 통해 기존 주제와 클러스터링했습니다. 분석에 도움이 되지 않는다고 판단되는 선험적 코드는 제거했습니다.
Newly identified themes were clustered with existing ones via visual mapping. A priori codes which were agreed to be unhelpful in the analysis were removed.

4. 초기 템플릿 생성
4.
Producing an initial template

상위 테마(MS)를 클러스터링하여 템플릿 초안을 개발했습니다.
A draft template was developed from the clustering of high-level themes (MS).

5.템플릿 적용 및 발전
5.
Applying and Developing the template

추가적인 반복 분석과 토론(MS, TP)이 이어지면서 템플릿은 주요 주제와 하위 수준 코드의 최종 합의된 계층 구조로 수정되었습니다(MS).
As further iterative analysis and discussion ensued (MS, TP), the template was revised to a final agreed hierarchy of major themes and lower-level codes (MS).

6. 최종 해석
6.
Final Interpretation

주요 주제는 합의(MS, TP)를 통해 합의되었으며 최종 템플릿은 세 가지 중요한 영역으로 구성되었습니다. 응답자 검증을 위해 분석 결과를 학생들에게 피드백했으며, 큰 변경은 필요하지 않았습니다(MS)[31].
Major themes were agreed by consensus (MS, TP) and the final template consisted of three overarching domains. We fed back our analysis to students for respondent validation, and no significant changes were required (MS) [31].

템플릿 진화
Template evolution

표 1에는 템플릿의 진화에 대한 자세한 정보가 나와 있습니다. 

  • 초기 선험적 코드는 피드백 응답에 대한 이전의 체계적 검토에서 도출되었습니다. 
  • 템플릿의 두 번째 반복은 세 가지 중요한 영역, 즉 핀트리치 SRL 모델에 특정한 피드백에 의해 활성화되는 SRL 프로세스로 구성되었습니다[15]. MS와 TP는 이 템플릿을 사용하여 대본을 코딩하여 테마를 평가하고, 새 테마를 추가하고, 기존 테마를 삭제 또는 수정하고, 테마 및 하위 테마의 계층 구조를 다시 정렬했습니다. 이를 통해 
  • 세 번째 템플릿 반복을 개발하게 되었으며, 구체적으로 식별된 SRL 응답의 하위 테마가 발생하는 분류로서 지원적 피드백 경험과 비지원적 피드백 경험에 명확히 초점을 맞추게 되었습니다. 
  • 최종 반복에서는 경험 유형(지원적 및 비지원적)과 SRL 반응을 이러한 변조의 조정 요소로 통합하여 학습 변조의 세 가지 중요한 영역을 적용합니다.

Table 1 includes more information on evolution of the template.

  • Initial a priori codes were derived from a previous systematic review on feedback responses.
  • The second iteration of the template consisted of 3 overarching domains: the SRL processes activated by feedback specific to the Pintrich model of SRL [15]. MS and TP used the template to code transcripts to evaluate for themes, add new themes, delete or modify existing themes and re-order the hierarchy of themes and sub-themes..
  • This led to the development of the third template iteration, with a clarified focus on supportive and unsupportive feedback experiences as classifications under which sub-themes of the specifically identified SRL responses occurred.
  • The final iteration applies three overarching domains of modulation of learning from integrating the experience type (supportive and unsupportive) and SRL responses as moderating factors of these modulations.

결과
Results

57명의 학생 인터뷰를 실시했습니다(표 2의 인구 통계). 
Fifty-seven student interviews were conducted (demographics in Table 2).

메타인지 지식과 신념의 조절
Modulation of metacognitive knowledge and beliefs

SRL은 역량 달성을 위한 인지적, 정서적, 행동적 과정의 변조를 설명합니다. 학습자는 피드백이 학습에 대해 생각하는 방식에 영향을 미치는 여러 가지 방법을 설명합니다(인지적 변조). 학습자는 피드백을 사용하여 자기 평가하고 동료와 비교하여 자신의 능력을 파악합니다. 때때로 자기 평가와 피드백이 일치하지 않을 때 긴장이 발생하기도 하지만, 학습자들은 대부분 반성 후 좋은 의도의 비평을 받아들인다고 말합니다: 
SRL describes modulation of cognitive, affective and behavioural processes to achieve competence. Learners describe a number of ways in which feedback affects how they think about learning (cognitive modulation). They use feedback to self-evaluate and locate their ability against their peers. Occasional tension arose when self-evaluation did not align with feedback; however learners mostly indicate accepting well-intended critique after reflection:

"이 선생님은 우리를 도우려는 좋은 선생님이라는 것을 알았기 때문에... 우리가 얼마나 못했는지에 대해 지적을 해도 해를 끼치려는 것이 아니라는 것을 알기 때문에 받아들입니다. 그들은 우리를 도우려는 것뿐입니다. 그래서 우리는 그것을 받아들일 것입니다."(S10).
“We knew that this teacher is a nice teacher who's trying to help us…So if they made a comment about how bad we were, we accept that because we know they're not trying to be harmful. They're just trying to help us. So we will accept that” (S10).

피드백 상호작용에서 경멸적인 언어와 인격에 대한 공격이 사용되었을 때 자기효능감에 부정적인 영향을 미쳤습니다. 이들은 그 후 오랜 기간 동안 "굴욕감"과 "황폐함"을 느꼈다고 보고했습니다. 이들은 이러한 피드백과 피드백이 자신의 능력에 미치는 영향에 대해 반추합니다. 
Feedback interactions negatively affected self-efficacy when derogatory language and attacks on their character were employed. They report feeling “humiliated” and “devastated” for long periods afterwards. They ruminate on this feedback and its implications of their ability.

피드백은 학습 자체와 관련된 학습자의 신념을 조절합니다. 참가자들은 긍정적인 피드백을 받은 후 특정 활동에 더 깊이 참여하게 되었다고 말합니다. 일반적으로 학습에 대한 열정이 새로워지고 향후 활동에서 적극적으로 피드백을 구하게 되었다고 설명합니다. 학습자는 피드백이 적대적일 때 학습 가치를 하향 조정했습니다. 이들은 피드백을 '무의미한 것'으로 묘사하고 학습을 '포기'하는 것에 대해 이야기합니다. 자세한 예는 표 3에 나와 있습니다.
Feedback modulates learners’ beliefs related to learning itself. Participants describe deeper engagement with a particular activity following a positive feedback encounter. They describe a renewed enthusiasm for learning in general and active feedback-seeking in future activities. Learners adjusted learning value downwards when feedback encounters were hostile. They describe feedback as “pointless” and talk about “walking away” from learning. Further examples are provided in Table 3.

학습 목표의 조절
Modulation of learning goals

SRL 이론은 학습에 있어 동기 부여와 효과적인 영향을 인정합니다. 연구 결과에 따르면 피드백은 학습 목표와 관련하여 이러한 요소와 상호작용하는 것으로 나타났습니다. 이는 크게 두 가지 주제로 나뉩니다: 동기 부여와 노력 투자, 그리고 감정 보호/스트레스 요인 회피입니다.
SRL theory acknowledges motivational and effective influences in learning. Our findings indicated feedback interacts with these factors in relation to learning goals. These came under two main themes: Motivation and effort investment and Protecting Emotions/Avoiding Stressors.

동기 부여 및 노력 투자
Motivation and effort investment

지원적인 피드백 경험은 특히 슈퍼바이저가 있는 곳에서 참가자들의 열정을 불러일으켰습니다. 
Supportive feedback experiences inspired enthusiasm in participants, particularly where the supervisor was.

"여러분이 발전하는 모습을 보고 싶어 하고, 여러분이 한 걸음 더 나아갈 수 있도록 열정적으로 도와주니 정말 변화하고 싶다는 동기가 생겼어요."(S8).
“really engaged in wanting to see you improve and is passionate about, you know, helping you make those extra steps, it really motivates me to change” (S8).


참가자들은 학습 활동에 투자하는 시간이나 노력을 늘림으로써 이를 활용했습니다. 목표를 재조정하고, 학습 일정을 변경하고, 학습 활동을 추가했습니다. 이는 성과가 최선이 아니더라도 피드백 경험이 건설적이었다면 더 많은 계획과 학습에 투자했을 때에도 일어납니다. 참가자들은 일반적으로 노력 투자에 부정적인 영향을 미치는 형식적인 피드백을 경험합니다:
Participants harnessed this by increasing time or effort on the learning activity. They re-calibrate targets, change study schedule, and add study activities. This happens even when their performance was sub-optimal: as long as the feedback experience was constructive, they invested more planning and studying. Participants commonly experience perfunctory feedback which negatively affects effort investment:

"(피드백은) 예상된 것이기 때문에 그냥 비평만 하는 것이기 때문에 피드백 세션을 마치고 나면 '내가 왜 참석했을까? 유용한 건 하나도 없었는데"... "피드백을 위한 피드백"(S8)이라는 생각을 하게 됩니다. 
“(they are) just sort of providing critique, because it's expected, so I leave a feedback session being like, “why did I even bother showing up? Like, nothing useful happened there”…you know, feedback for the sake of giving feedback” (S8).

적대적인 경험을 한 학생은 의욕을 잃은 반면, "부끄럽고 슬프지만 기억할 거예요... 시험에서 실수하는 것보다 수업 중에 부끄러워하는 것이 더 낫죠"(S48)라고 주장하는 학생도 있습니다.
With hostile experiences, some students are demotivated, while others argue that “I'll feel ashamed and sad…But I will remember it…so better ashamed during the class, rather than you make mistakes in the exam” (S48).

감정 보호/스트레스 요인 피하기
Protecting emotions/avoiding stressors

학습자는 위협을 받는다고 느끼는 피드백을 피합니다. 이는 학습자가 "굴욕적", "당황스럽다", "공격적"이라고 묘사하는 이전 경험에서 비롯됩니다. 학습자는 그 상황에서 벗어나는 데 집착하게 됩니다. 이는 향후 학습에 영향을 미치며, 일부는 반복되는 사건을 피하기 위해 노력합니다. 한 참가자는 "그가 나를 많이 괴롭혔다"고 느꼈던 피드백 상황을 설명하며 눈물을 흘렸고, "그 경험 때문에 컨설턴트와 이야기하는 것이 두렵다"고 말하며 더 이상 적극적으로 피드백을 구하지 않는다고 말했습니다(S11). 이는 또한 불출석으로 나타나기도 했는데, 특정 상사나 활동(예: 병동에서 발표하는 것)을 피하는 것으로 나타났습니다. 자세한 예는 표 4에 나와 있습니다. 
Learners avoid feedback where they feel threatened. This stems from prior experiences they describe as “humiliating”, “embarrassing” and “aggressive”. They becomes fixated on escaping. This colours future learning where some go out of their way to avoid a repeat incident. One participant broke down in tears describing a feedback situation where they felt “he was picking on me a lot” and goes on to say they no longer actively seek feedback, “that’s why I'm scared to talk to the consultants because of that experience” (S11). This also manifested as non-attendance; avoiding specific supervisors or activities, e.g. presenting on wards. Further examples are provided in Table 4.

학습 전략의 조절
Modulation of learning strategies

SRL은 학습자가 자신의 상황 내에서 자신의 진행 상황을 모니터링하고 그에 따라 적응할 것을 요구합니다. 학습자들은 피드백과 관련하여 학습 설계 방식이 영향을 미친 두 가지 주요 영역을 설명합니다. 이는 크게 두 가지 주제로 분류됩니다: 전략 선택과 전략 조정.
SRL requires the learner to monitor their progress within their context and adapt accordingly. In relation to feedback, our learners describe two key areas where how they design their learning was affected. These are broadly categorised under two themes: Selecting strategies and adapting strategies.

전략 선택하기
Selecting strategies

학습자들은 피드백을 학습 전략으로 전환하는 데 어려움을 표했습니다. 그들은 명확한 '다음 단계'를 제시하는 피드백을 선호했습니다. 그들은 다음과 같이 말합니다: 
Learners expressed difficulty translating feedback in to learning strategies. They favoured feedback which gave clear “next steps”. They comment that:

"무엇을 해야 할지는 알지만 어떻게 해야 할지는 모르겠다"(S50).
“you'll know what you need to do but you don't know how” (S50).

"좋은" 피드백은 감독자가 단순히 어떤 작업이 필요한지보다는 어떻게 변화할지를 제안하는 것을 의미했습니다. 한 학습자는 그 피드백을 이렇게 설명합니다: 
“Good” feedback meant when supervisors suggested how to change rather than just what needed work. A learner explains that feedback:

"중요한 점에 집중할 수 있도록 도와주셨어요... 그런 조언을 해주지 않았다면 저는 실제로 병동에 가지 않았을 것입니다. 피드백은 제가 개선하기 위해 무엇을 해야 하는지 정확하게 알려주었기 때문에 매우 긍정적이었습니다."(S4). 
“helped me to concentrate on important points…And if he didn't give me that advice, I wouldn't actually go to the wards. And so that was very positive because it really directed me towards exactly what I need to do to improve” (S4).

이들은 발달 영역이 확인된 피드백을 어떻게 다룰 것인지에 대한 명확한 방향이 없으면 이를 활용할 전략을 선택하기가 어렵다는 것을 알게 됩니다. 이 중 일부는 모호한 의견에서 비롯되었습니다: 
They find it challenging to select strategies to use feedback which identified developmental areas, without explicit direction in how to address these. Some of this came from vague comments:

"모든 것이 괜찮다"는 식의 피드백을 주면 어떻게 해야 하나요?" (S45).
“if you give me feedback, like "everything's Okay", what am I gonna do with it?” (S45).

이들은 명확성을 확보하려는 시도가 실패하여 이 전략을 포기하게 되었다고 설명합니다: 
They describe unsuccessful attempts to gain clarity, lead to abandoning this strategy:

"저는 그냥 질문을 그만뒀어요... 저는 중고등학교 때 선생님들과 함께 살면서 서로가 같은 생각을 할 때까지 모든 것을 물어봤어요... 여기서는 기본적인 질문을 해도 제대로 된 대답을 들을 수 없으니까요... 그래서 그냥 질문을 그만뒀는데, 끔찍하죠. 지금은 많은 것을 이해하지 못하니까요. 물어볼 사람이 없어요" S26. 
“I just stopped asking questions …I would live with the teachers in secondary school…and ask everything until we both were on the same page… Here, you can ask a basic question and you don’t get a proper response… So I just stopped asking, which is awful. Because now I don't understand lots of things. I have no one to ask” S26.

적응 전략
Adapting strategies

저희는 이를 행동 전략, 인지 전략, 감정 관리를 위한 분리라는 세 가지 주제로 나누었습니다. 
We have divided these in to three themes: behavioural strategies, cognitive strategies and disengagement to manage emotions.

행동 전략
Behavioural strategies

참가자들은 신체검사의 기법 변경, 병력 청취 시 질문 추가, 구조와 우선순위를 조정한 사례 발표 등 과제별 행동에 피드백을 통합하는 방법에 대한 다양한 사례를 제공했습니다. 
Participants provided many examples of how they incorporated feedback in to task-specific actions, e.g. changing technique in physical examination, adding questions in history-taking, presenting cases with tweaks to structure and prioritisation.

인지 전략
Cognitive strategies

피드백은 인지 학습 전략에 더 다양한 영향을 미쳤습니다. 참가자들은 약리학 지식을 위한 특정 앱을 사용하거나, 프레젠테이션을 서면 연습에서 구두 연습으로 학습 방향을 바꾸는 등 과제 또는 활동별 조정을 쉽게 실행했습니다.
Feedback had more variable influences on cognitive learning strategies. Participants readily implemented task or activity-specific tweaks, e.g. using a specific app for pharmacology knowledge, redirecting study from written to verbal practice of presentations.

학습 전략에 대한 보다 정교한 점검이 제안되었을 때 학생들의 반응은 엇갈렸습니다. 어떤 학생들은 기꺼이 적응하겠다는 반응을 보였고, 어떤 학생들은 신뢰할 수 있는 전략을 바꾸는 것이 불편하다고 말했습니다;
When more elaborate overhaul of their study strategy was suggested, responses were mixed. Some were open to adapt; others were uncomfortable changing reliable strategies, saying;

"이 시점에서 제 공부 습관이 꽤 굳어졌어요...., 지금까지는 잘 해왔어요, 누군가 저에게 모든 것을 완전히 바꾸라고 하면 조언에 감사할 뿐 실제로 실행하지는 않을 거예요"(S4).
“I've found my studying habits to be pretty set in stone at this point…., they've done me well, up until now, if somebody told me to completely revamp everything, I would thank them for their advice and not actually implement it” (S4).

학습자는 후속 학습 전략에 통합할 피드백 요소를 정기적으로 골라 선택합니다:
Learners regularly pick and choose which elements of feedback to incorporate in subsequent learning tactics:

"피드백이 좋은지, 나쁜지, 사용할 만한 가치가 있는지 등 피드백을 필터링하는 것이 중요합니다."(S1).
“it's a lot about filtering the stuff that comes, whether it's good, whether it's bad, you know, if there's something worth using, to go with it” (S1).

참여도 저하
Disengagement

학습자는 피드백이 적대적인 것으로 인식될 때 참여를 중단합니다. 이러한 상황은 빈번하게 발생하며 경멸적인 언어, 부정적인 비언어적 커뮤니케이션(눈동자 굴리기, 한숨, 훈계하는 어조), 끼어들기 및 고함, 인격이나 의사로서의 능력에 대한 언급 등 여러 가지 형태로 나타납니다. 피드백 메시지가 잠재적으로 유용할 수도 있지만, 일부는 의식적으로 무시하기도 합니다:
Learners disengage when feedback is perceived as hostile. Such encounters are frequent and have several manifestations—derogatory language, negative non-verbal communication (eye-rolling, sighing, admonishing tone), interrupting and shouting, making comments about character or ability to be a doctor. While the feedback message may be potentially useful, some consciously walk away:

"상대방이 한 말이 사실이든 아니든, 내가 불편하게 느낀 방식으로 표현했고 마음에 들지 않는다면 그냥 "그거 알아? 아니, 고마워요"라고 말할 것입니다. 프로페셔널하게 대하지 않을 거라면 "안녕히 가세요"라고 말하죠.
“Whether what they said is true or not, if they phrased it in a way that I felt uncomfortable with, and I didn't appreciate, I will just be like "do you know what? Nah, thanks". If you're not going to be professional then, “Bye””S30.

다른 사람들은 그 순간에 부담을 느낍니다:
Others feel overwhelmed in the moment:

"나는 문을 닫았다. 어린아이가 된 기분이에요, 정말 싫어요."(S2).
“I shut down. I feel like a small child, I hate it” (S2).

이러한 피드백을 사용하지 않거나 사용할 수 없다고 생각하지만, 많은 학습자가 피드백에 많은 시간을 할애합니다: 
While they feel they will not or cannot use this feedback, many learners spend a lot of time with it:

"나는 그것에 대해 숙고할 것이다... 나는 그것에 대해 꽤 오랫동안 생각할 것이다. .... 만약 그것이 매우 가혹하다면, 그것은 영원히 남아있을 것이다"(S49).
“I'll be like pondering over it…I'll be thinking about it quite some time ….if it's very very harsh, then it will remain forever” (S49).

더 많은 예가 표 5에 나와 있습니다. 
Further examples are provided in Table 5.

토론
Discussion

이 연구는 의대생들의 피드백 경험이 학습의 자기조절과 어떤 관련이 있는지 살펴보고자 했습니다. 이는 확립된 이론적 틀에 기반하여 미래 학습의 관점에서 피드백 반응을 고려했다는 점에서 문헌에 중요한 추가 사항입니다. 다양한 학생 집단을 대상으로 세 곳의 해외 사이트에서 연구를 수행함으로써 연구 결과의 신뢰성이 더욱 강화되었습니다. 
This study sought to explore how medical students’ experiences of feedback relate to self-regulation of learning. This is an important addition to the literature as it considers feedback responses from the perspective of future learning, underpinned by an established theoretical framework. The trustworthiness of our findings are strengthened by having conducted this research across three international sites with diverse student populations.

연구 결과에 따르면 의대생들은 피드백을 자신과 함께 하는 것이 아니라 자신에게 일어나는 것으로 인식하는 경우가 많습니다. 최근의 피드백 연구와 대조적으로, 학생들은 향후 학습에 피드백을 포함시키기 위해 평가자가 주도하는 교육을 기대하는 경우가 많다는 것을 보여줍니다. 학습자가 자기 조절을 위해 피드백을 사용하면 메타인지적 학습 전략과 행동적 학습 전략이 모두 변화하지만, 이는 특히 피드백이 감정에 미치는 영향에 따라 맥락화됩니다.
Our findings indicate that medical students often perceive feedback as something that happens to them, rather than with them. Contrasting with contemporary feedback research, we show that they often expect rater-led instruction in order to embed it in future learning. When learners use feedback to self-regulate, they change both metacognitive and behavioural learning strategies, but this is contextualised particularly by how feedback affects their emotions.

학습자들은 좋은 피드백을 명시적인 지시라고 설명했습니다. 이는 '어떻게' 또는 '무엇을' 하라는 지시를 받으면 쉽게 변화하는 과제별 피드백의 사례에서 더욱 강조됩니다. 최근의 피드백 모델은 공동 구성에 중점을 둔 학습자 주도 계획[20, 32]에 초점을 맞추고 있는데[33], 이는 참가자들이 피드백을 개념화하는 방식과 상충됩니다. 졸업후교육 연구에서 비롯된 모델은 의대생의 자기조절 능력을 과대평가한 것일 수 있습니다. 주니어 학습자는 학습을 탐색하는 데 더 많은 지원이 필요하며, '노력형'과 같이 무엇을 해야 하는지 알려줘야 하는 특정 학습자 '유형'이 존재할 수 있다는 것은 이미 밝혀진 사실입니다[34]. 따라서 교육자들은 학부 맥락에서 보다 명확한 지원과 점진적인 책임 분담을 제공하기 위해 대학원 모델을 수정해야 할 수도 있다는 점을 고려할 것을 권장합니다. 
Learners described good feedback as explicit direction. This was further emphasised by examples of enacted task-specific feedback- they readily changed when told “how” or what”. Recent feedback models focus on learner-led planning [20, 32], with a focus on co-construction [33]; this is at odds with how our participants conceptualise it. It may be that models originating from postgraduate research overestimate the self-regulatory abilities of medical students. It is established that junior learners need more support in navigating learning and certain learner “types” can exist such as “effortful”, who need to be told what to do [34]. Therefore, we encourage educators to consider that postgraduate models may require modification to provide this more explicit support and gradual sharing of responsibility in undergraduate contexts.

피드백을 해독하는 데 있어 학습자의 한계나 피드백 리터러시 개념은 새로운 것이 아닙니다[13, 35]. 이 연구는 학습자가 피드백에 의해 어떻게 도전을 받는지에 대해 알려진 것을 확장합니다. 의대생은 피드백의 의미와 실행 방법을 이해하는 데 있어 적극적인 참여자라기보다는 정보의 수신자로서의 위치에 있습니다. 많은 연구에서 높은 수준의 자기조절 능력을 가진 학습자조차도 독립적으로 학습 전략을 개발하는 데 어려움을 겪는다는 사실을 뒷받침합니다[36, 37]. 학습자는 학습이 진행됨에 따라 학습을 통제해야 할 필요성을 인식하지만[38], 이러한 과정을 안내하는 것이 매우 중요해 보입니다.
Neither the concept of learners’ limitations in decoding feedback, nor feedback literacy are new [13, 35]. This study extends what is known about how learners are challenged by feedback. Medical students position themselves as recipients of information rather than active participants in making sense of what feedback means and how to enact it. Much work supports that even learners with high levels of self-regulation struggle to develop their learning strategies independently [36, 37]. Learners identify a need to take control of learning as they progress [38] but guidance through this progression appears critical.

학습자가 피드백을 중요하게 생각하더라도 피드백을 사용하지 않을 수도 있습니다. 학습 습관과 같은 인지적 전략은 뿌리 깊게 박혀 있어, 이를 바꿀 가치가 있다고 인정하면서도 변화를 고려하는 참가자는 소수에 불과했습니다. 학습에 대한 학생들의 신념은 비교적 안정적이며 피드백과 상호작용하는 방식에 영향을 미칩니다[16, 39]. 두 번째 수준의 경험은 의대생이 사전 학습에서 관행을 가져와서 더 절대주의적인 입장을 조장할 수 있습니다[40][41]. 학습 전략에 영향을 미치는 데 있어 피드백보다 기존의 신념이 더 중요할 수 있습니다.
Even when learners value feedback, they may choose not to use it. Cognitive strategies such as study habits were ingrained: only a few participants would consider changing them, even while acknowledging this could be worthwhile. Students’ beliefs about learning are relatively stable and influence how they interact with feedback [16, 39]. Second- level experiences potentially foster more absolutist stances [40] with medical students importing practices from prior learning [41]. It is possible that their pre-existing beliefs out-weigh feedback in affecting their learning strategies.

지지적이지 않은 경험은 학생에게 갈등을 야기합니다. 학습자는 인지적으로는 피드백이 잠재적으로 유용하다고 판단할 수 있지만, 적대적으로 전달되는 피드백 때문에 참여를 중단하거나 감정적인 압도감으로 인해 학습에 집중할 수 없게 됩니다. Sargeant는 또한 부정적인 감정이 피드백을 받아들이는 데 장애가 된다는 사실을 확인했습니다[42]. 연구 결과에 따르면 부정적인 감정이 학생의 피드백 반응을 유도하는 데 있어 다른 고려 사항보다 더 중요하다고 합니다. 이전 연구에 따르면 학습자[43]는 관찰된 능력이 학습자의 자아 개념과 상충할 때 긴장을 경험한다고 합니다[43, 44]. 우리의 연구 결과는 이러한 선행 연구 결과와 다릅니다. 개인의 약점을 강조하는 피드백은 학습자를 실망시키지만, 적대적인 피드백은 감정적으로 과부하가 걸리고 학습을 방해한다는 점에서 이와는 구별됩니다. 우리는 부정적인 감정이 "나쁘다"는 것이 아니라 학습에 외재적 동기 부여가 잠재적인 역할을 한다는 것을 인정합니다[45, 46]. 우리는 감정이 문제가 아니라 심리적 안전의 부족이 문제라고 제안합니다 [47]. 학생들이 SRL에 참여하는 능력은 긍정적인 감독자 관계의 영향을 받으며[48], '불확실한' 학습자는 감독자 관계와 안전한 환경에 따라 효율적으로 자기 조절을 할 수 있습니다[34]. 피드백에서 감정의 담론을 성찰적 실천으로 활용하고, 슈퍼바이저가 동맹이 되어 성찰을 촉진하면 수용과 건설적인 사용으로 재구성할 수 있습니다[20, 49].
Unsupportive experiences create a conflict for students. While the learner may judge cognitively that the feedback has potential use, they either choose to disengage because of the hostile delivery, or they are incapable of engaging, because the emotional overwhelm distracts them from learning. Sargeant also identified that negative emotions are a barrier to accepting feedback [42]. Our findings suggest that negative emotions outweigh other considerations in guiding students’ feedback response. Previous work reports learners [43] experience tension when observed ability conflicts with the learner’s self-concept [43, 44]. Our findings are distinct from these prior findings. Feedback that highlights personal weaknesses disappoints but feedback that is hostile is distinct from this: it emotionally overloads and derails learning. We do not suggest that negative emotions are “bad”; we acknowledge their potential role in extrinsic motivation of learning [45, 46]. We propose that the emotions are not the problem, the lack of psychological safety is [47]. Students’ ability to engage with SRL is influenced by positive supervisor relationships [48], and the “uncertain” learner depends on supervisor relationships and a safe environment to self-regulate efficiently [34]. Employing the discourse of emotion in feedback as reflective practice, whereby supervisors are allies and facilitate reflection can reframe towards acceptance and constructive use [20, 49].

효율적인 자기 조절자는 피드백을 더 많이 활용합니다[2, 50]. 우리의 연구 결과는 '사용'이 모든 SRL 반응을 반영하지 않을 수 있음을 시사합니다. 사용은 종종 외부에서 관찰할 수 있는 결과인 반면, SRL은 학습자의 내부 규제 과정과 그에 따른 관찰 가능한 변화를 모두 반영합니다 [2]. 학습자들은 '사용'이라고 표현하기는 부적절하지만, 학습을 크게 변화시킨 내부 반응을 설명합니다. 첫째, '사용'은 선형적이지 않았습니다. 동일한 학습자가 다른 맥락에서 피드백을 사용하거나 거부하는 것을 설명합니다. 둘째, 학습자는 사용하지 않는 피드백에 많은 시간을 소비합니다. 학습자들은 이러한 피드백에 대해 반추합니다. 일부는 자신감과 동기를 잃고, 일부는 피드백 및 관련 학습 과제를 평가 절하합니다. 학습자는 이러한 반응을 향후 학습에 반영합니다. 이러한 눈에 보이지 않는 반응은 학습 가치와 후속 행동에 큰 영향을 미칩니다.
Efficient self-regulators make more use of feedback [2, 50]. Our findings suggest that “use” may not reflect all SRL responses. Use is often an externally observable outcome, whereas SRL reflects both the learner’s internal regulatory processes and the subsequent observable changes [2]. Our learners describe internal responses which are inadequately represented by “use” but significantly changed their learning. Firstly, “use” was not linear- the same learner describes using feedback and rejecting it in different contexts. Secondly, learners spend a lot of time with feedback that they do not use. They ruminate on this feedback.. Some lose confidence and motivation; some devalue feedback and the associated learning task. They take these responses with them to their future learning. These invisible responses have significantly impacted their learning values and subsequent actions.

교육자를 위한 시사점
Implications for educators

학습자는 피드백에 기반한 전략 선택에 어려움을 겪습니다.
Learners struggle to select strategies based on feedback

학습자는 피드백을 실질적인 전략으로 전환하는 데 어려움을 겪습니다. 의과대학 학부생들은 교사가 단계별 실행 계획을 명시적으로 제시할 것이라는 기대가 있으며, 공동 개발에서 학습자의 역할에 대한 고려는 거의 없습니다. 교육자는 학습자가 전략에 대해 점진적으로 더 많은 소유권을 갖도록 장려하고, 교사는 지도가 필요한 부분을 입력함으로써 학습자의 자기 조절력을 키워야 합니다. 이전 연구에 따르면 문제 기반 학습(PBL) 맥락에서의 퍼실리테이션은 학습자가 학습 계획을 세우는 데 도움이 됩니다[51]. 교육자는 성찰적 피드백 대화를 사용함으로써 학습자의 자율성을 높이고 피드백을 통한 자기 조절에 참여할 수 있습니다[52].
Learners find it hard to translate feedback into tangible strategies. There is an expectation in undergraduate medical students that the teacher will provide an explicit step-by-step action plan, and little consideration for learner role in co-development. Educators should foster learners’ self-regulation through encouraging incrementally more ownership of strategies, with teachers inputting where guidance is needed. Previous work indicates that facilitation in a problem-based learning (PBL) context helps learners to generate learning plans [51]. By employing reflective feedback conversations, educators can increase learner agency and engagement in self-regulation with feedback [52].

학습자는 피드백을 실행하는 데 장애가 되는 안정적인 학습 신념을 가지고 있을 수 있습니다.
Learners may have stable learning beliefs that are a barrier to enacting feedback.

학습자가 학습에 대한 기존의 신념을 가지고 있는 경우, 피드백을 통해 조언되는 변화와 충돌할 수 있습니다. 교육자 및 동료와의 성찰 대화 및 자기 평가는 학습자가 피드백에서 자신의 역할을 이해하고 인식하는 데 도움이 되며[53], 잠재적으로 개발을 제한하는 선입견에 도전할 수 있습니다. 
When learners have pre-existing beliefs about learning, they may be in conflict with changes advised via feedback. Facilitated reflection conversations with educators and peer and self-assessment help learners understand and appreciate their role in feedback [53]; potentially challenging preconceived notions that limit development.

학습자가 안전하지 않다고 느낄 때 피드백은 듣지 않습니다.
Feedback goes unheard when learners feel unsafe

첫째, 피드백 상호 작용에서 빈번하게 적대감으로 인해 감정적 반응이 흔하게 나타나며, 이는 이후의 태도, 참여도 및 학습 가치에 영향을 미칩니다. 학습자는 괴로울 때 피드백 메시지를 처리하기보다는 감정을 관리하는 데 에너지를 쏟습니다. 피드백 리터러시는 학습자가 비평을 처리할 때 자신의 감정을 관리할 것을 요구하지만, 이는 노골적인 적대감을 처리하는 것과는 구별됩니다. 피드백 리터러시를 갖춘 교육자는 취약성을 인식하고 학생의 감정이 존중되는 안전한 환경을 조성하여 피드백 메시지를 들을 수 있도록 합니다.
Firstly, emotional reactions are commonplace due to frequent hostility in feedback interactions, and influence subsequent attitude, engagement and learning values. Learners direct their energy to managing emotions rather than processing feedback messages, when they are distressed. While feedback literacy requires the learner to manage their emotions in navigating critique, this is distinct to handling overt hostility. Feedback-literate educators will recognise vulnerability and create a safe environment where students’ emotions are respected, allowing feedback messages to be heard.

시행하지 않는다고 해서 영향이 없는 것은 아닙니다
Not implemented does not mean no impact

피드백을 사용하지 않는 것을 무시하는 것과 혼동해서는 안 됩니다. 학습자는 종종 피드백을 숙고하는 데 많은 시간을 소비합니다. 무반응은 때로는 자신감의 '파괴'로 인한 마비, 때로는 학습 및 학습 목표에 대한 평가 절하를 나타냅니다. 교육자는 학습자가 무관심하다고 가정하기보다는 학습자가 피드백을 적용하지 않게 된 원인을 파악하기 위해 학습자와 함께 세심한 탐색을 수행할 것을 권장합니다. 이는 '효과적인' 피드백을 평가하는 방법에도 영향을 미치며, 학습자와 학습자의 성과에서 관찰 가능한 변화와 더불어 성찰을 촉진하는 데 초점을 맞춰야 합니다. 피드백 활동에서 SRL을 지원하기 위한 주요 결과와 제안된 접근 방식에 대한 요약은 표 6에 나와 있습니다.
Not using feedback should not be confused with ignoring it. Learners often spend a lot of time pondering feedback. They indicate inaction is sometimes paralysis from “destroyed” confidence, sometimes devaluation of learning and learning goals. Rather than assuming indifference, we encourage educators to undertake sensitive exploration with learners to identify what may have led them to not apply feedback. This also has implications for how we evaluate “effective” feedback; a focus on facilitated reflection should be considered in addition to observable changes in the learner and their performance. A summary of our key findings and suggested approaches to supporting SRL in feedback activities is provided in Table 6.

강점 및 한계
Strengths and limitations

이 연구는 학습의 이론적 틀 안에서 피드백 반응을 해석하는 데 있어 독특합니다. 이를 통해 이러한 반응을 학습 행동과 연계할 뿐만 아니라 학습의 인지적, 메타인지적, 효과적, 행동적 측면을 인정합니다. 템플릿을 여러 번 반복하여 개발함으로써 주제 간의 상호 연결과 상호작용에 대한 풍부한 이해를 촉진했습니다. 국제적인 연구팀이 참여하여 다양한 맥락에서 해석할 수 있었습니다. 다양한 배경과 경험을 가진 다양한 국적의 학생들을 대상으로 한 이 다중 사이트 연구는 연구 결과의 적용 가능성을 높였습니다. 우리는 피드백에 대한 반응이 복잡하고 고도로 맥락화되어 있으며, 대학원생과 다른 학습자의 연구 결과가 그러한 맥락의 영향을 받을 수 있음을 인정합니다. 우리의 연구는 복잡한 분석을 가능하게 하는 풍부한 내러티브 설명을 이끌어냈지만, 학습자의 내적 처리를 탐구하는 데는 성찰 일기와 같은 대체 데이터 수집 방법이 도움이 될 수 있음을 인식하고 있습니다.
This study is unique in interpreting feedback responses within a theoretical framework of learning. In doing so, it aligns these responses not just with learning actions, but acknowledges the cognitive, metacognitive, effective and behavioural aspects to learning. The development of multiple iterations of the template facilitated rich understanding of interconnections and interactions between themes. Having an international research team brought a diversity of context to the interpretation. This multi-site study of students from a broad range of nationalities, with varied backgrounds and experiences, improves the likelihood of transferability of our findings. We recognise that feedback responses are complex and highly contextualised; we acknowledge that findings in postgraduate populations and other learners may be affected by such contexts. While our work elicited rich, narrative descriptions that allowed a complex analysis, we recognise that exploring learners’ internal processing may benefit from alternative data collection methods such as reflective diaries.

결론
Conclusions

SRL의 렌즈를 통해 피드백 반응을 탐구함으로써 학습자가 피드백 처리를 통해 어떻게 움직이는지 더 잘 이해하는 데 기여할 수 있습니다. 피드백은 상황에 따라 학습 목표에 긍정적으로 또는 부정적으로 영향을 미칠 수 있습니다. 의대생은 과제별 피드백을 기꺼이 받아들이지만, 현대 피드백의 정의에서 이해되는 공동 구성보다는 단방향 교육을 계속 기대합니다. 피드백이 적대적인 경험일 경우, 학습은 고통으로 인해 탈선하기 때문에 그 잠재력을 잃게 됩니다. 피드백을 사용하지 않는 것을 무관심과 혼동하는 것은 주의해야 합니다. 피드백을 사용하지 않는 것은 향후 학습에 중대한 영향을 미치는 장기간의 내부 처리의 신호일 수 있습니다.
By exploring feedback responses through the lens of SRL, we contribute to a better understanding of how learners move through feedback processing. Feedback can influence learning goals positively and negatively, depending on context. Medical students willingly adapt task-specific feedback, but continue to expect unidirectional instruction rather than the co-construction understood in modern feedback definitions. When feedback is a hostile experience, its potential is lost, as learning is derailed by distress. Conflation of non-use with indifference is cautioned; disengagement may be a sign of prolonged internal processing with significant impact on future learning.


BMC Med Educ. 2023 Nov 22;23(1):895. doi: 10.1186/s12909-023-04842-9.

"Tell me what is 'better'!" How medical students experience feedback, through the lens of self-regulatory learning

Affiliations collapse

Affiliations

1Health Professions Education Centre, Royal College of Surgeons in Ireland University of Medicine and Health Sciences, Dublin, Ireland. mspooner@rcsi.ie.

2Department of General Practice, Royal College of Surgeons in Ireland University of Medicine and Health Sciences, Dublin, Ireland.

3Department of General Practice, Perdana University Serdang, Selangor, Malaysia.

4Department of International Health & Tropical Medicine, Royal College of Surgeons in Ireland University of Medicine and Health Sciences, Dublin, Ireland.

5Health Professions Education Centre, Royal College of Surgeons in Ireland University of Medicine and Health Sciences, Dublin, Ireland.

PMID: 37993832

PMCID: PMC10666439

DOI: 10.1186/s12909-023-04842-9

Free PMC article

Abstract

Introduction: While feedback aims to support learning, students frequently struggle to use it. In studying feedback responses there is a gap in explaining them in relation to learning theory. This study explores how feedback experiences influence medical students' self-regulation of learning.

Methods: Final-year medical students across three campuses (Ireland, Bahrain and Malaysia) were invited to share experiences of feedback in individual semi-structured interviews. The data were thematically analysed and explored through the lens of self-regulatory learning theory (SRL).

Results: Feedback interacts with learners' knowledge and beliefs about themselves and about learning. They use feedback to change both their cognitive and behavioural learning strategies, but how they choose which feedback to implement is complex. They struggle to generate learning strategies and expect teachers to make sense of the "how" in addition to the "what"" in planning future learning. Even when not actioned, learners spend time with feedback and it influences future learning.

Conclusion: By exploring our findings through the lens of self-regulation learning, we advance conceptual understanding of feedback responses. Learners' ability to generate "next steps" may be overestimated. When feedback causes negative emotions, energy is diverted from learning to processing distress. Perceived non-implementation of feedback should not be confused with ignoring it; feedback that is not actioned often impacts learning.

Keywords: Emotions; Feedback engagement; Feedback response; Feedback use; Medical students; Negative feedback; Self-regulatory learning.

중국의 2021년 의료전문직 면허법 수정: 코멘터리(Health Syst Reform. 2022)
Amending the Law for Licensing Medical Practitioners of China in 2021: A Commentary
Chengxiang Tanga,b, Jiayi Jiangb,c, Yuanyuan Gua, and Gordon Liub

 

 

소개
Introduction

다양한 의료 서비스 제공자 중 의사는 의료 시장에서 가장 중요한 부분을 차지하고 있으며, 의사의 자질은 의료 시스템에서 결정적인 역할을 합니다. 의사 면허 시험은 의사의 질과 그에 따른 의료 서비스를 보장하는 데 매우 중요한 의미를 갖습니다. 1950년대 이후 중국 의료 시스템의 주요 임무는 절박한 의료 인력 부족 문제를 해결하고 증가하는 의료 수요를 충족하는 것이었습니다.1 이 문제를 해결하기 위해 중국은 의사를 양성하기 위해 다단계 의학교육 시스템을 도입했고, 그 결과 일부 현직 의사들은 3차 학위 중심의 의학교육을 받지 못했습니다.2  
Among the various health service providers, physicians are the most important part of the health care market, and their quality plays a determining role in the health system. The licensing examination for medical practitioners is of great significance to guarantee the quality of physicians and consequent health care services. Since the 1950s, the primary job of the health care system in China is to address a desperate shortage in the health workforce and satisfy the increasing demand for health care.1 To address this problem, China adopted a multi-tiered medical education system to train doctors, which resulted in a part of the practicing physicians receiving no tertiary degree-oriented medical education.2

의학교육 개혁의 발전과 함께 중국은 1999년에 처음으로 개업의에 관한 법률을 제정하여 중등직업학위(SVD)을 가진 '의대생'만이 의사 자격시험에 응시하여 개업의가 될 수 있도록 허용했습니다. 2002년 기준으로 중국에서는 개업 의사의 41%가 SVD만 가지고 있었습니다. 
With the development of medical education reform, China enacted its first Law for Practicing Medical Practitioners since 1999, which allowed “medical students” with only a secondary vocational diploma (SVD) to take the medical examination and therefore become practicing physicians. As of 2002, 41% of practicing physicians only had an SVD in China.

2021년 1월, 전국인민대표대회 상무위원회는 의사의 질과 의료 시장의 형평성을 개선하기 위해 1999년 시행 이후 처음으로 '의료인 면허법'을 개정했습니다. 주요 개정 내용은 다음과 같습니다: 

  • (1) 전문직 규제에 관한 다른 법률과의 일관성을 유지하기 위해 현행 "개업의사"라는 명칭을 "의사"로 단순 대체합니다.
  • (2) 개정안은 의사의 의무와 의무, 의사의 법적 권리 및 보수를 더욱 명확히 규정합니다. 개정안은 또한 의료 시설과 시장을 교란하는 행위에 대한 형사 책임을 규정하고 있습니다.
  • (3) 면허시험의 최소 학력 수준을 종전의 중등 전문학사 학위에서 개정안에서는 전문학사 또는 전문대졸 수준으로 재설정합니다. 또한 개정안은 시험 등록에 관한 사항을 개선합니다.
  • (4) 1차 의료 제공자, 특히 일반의에 대한 교육이 강화됩니다.
  • (5) 개정안에는 SARS-COV-19의 예방 및 통제 전략에 대한 교훈과 경험이 포함됩니다.

In January 2021, the Committee meeting of the National People’s Congress amended the “The Law for Licensing Medical Practitioner” for the first time since its implementation in 1999, aiming to improve the quality of physicians and equity in the health care market. The major amendments include:

  • (1) In order to be consistent with other legislations on professional regulation, the current title “Practicing Physician” is simply replaced by “Physician.”
  • (2) The amendments further clarify the physicians’ duties and obligations, as well as the legal rights and remuneration of physicians. The amendments also stipulate the criminal liability for disrupting health care facility and the market.
  • (3) The minimum education-level of the licensing examination is re-set to a vocational diploma or junior college level in the amendment, rather than a secondary vocational diploma previously. In addition, the amendments make improvements on the registration of the examination.
  • (4) The training of primary care providers, in particular the general practitioners, will be strengthened.
  • (5) The amendments include the lessons and experiences of the prevention and controlling strategy of SARS-COV-19.

우리는 의사 자격 면허 시험의 최소 교육 수준을 학사 학위로 설정해야한다고 생각하는 것을 제외하고는 이러한 개정안을지지합니다. 다음 섹션에서는 먼저 중국 보건통계연감의 데이터를 기반으로 중국 내 의사들의 교육 수준 구조와 분포를 제시합니다. 그런 다음 의사 자격 시험의 문턱을 학사 수준으로 설정해야 하는 여러 가지 이유를 설명합니다.3
We endorse these amendments except that we believe the minimum education-level of the licensing examination of physicians’ qualifications should be set to a bachelor degree. In the following sections, we first present the structure and distribution of practicing physicians’ educational level in China based on the data from the Health Statistical Yearbook of China. We then give a number of reasons to explain why the threshold for the physician qualification examination should be set at the bachelor level.3

의사 의학교육의 구조와 분포
The Structure and Distribution of Physicians’ Medical Education

다단계 의학교육 체계에서 비롯된 현재의 의학교육 의사 구성은 보건의료체계의 질적 발전을 저해할 것입니다. 그림 1은 2002년부터 2019년까지 중국 내 의사 교육 분포의 연도별 변화를 보여줍니다. 일반적으로 학사 학위 이상의 의사의 비율은 수년에 걸쳐 증가해 왔으며, 전문학사 및 SVD 이하의 의사의 비율은 감소했지만, 이러한 '부적절한' 의학교육 파이프라인은 여전히 교육 시스템에서 사소하다고 볼 수 없는 요소로 남아있었습니다. 2019년에도 전문대 졸업장 이하의 학력을 가진 개업 의사의 비율은 여전히 34%에 달했습니다. 
The current physician mix in medical education originating from a multi-tiered medical education system will inhibit quality development of a health care system. Figure 1 illustrates the annual changes of physicians’ educational distribution in China from 2002 to 2019. In general, the proportion of physicians with a bachelor degrees or above had been increasing over the years, and the share of physicians with vocational diploma and SVD or below has decreased, even though this “inappropriate” pipeline of medical education remained as a non-trivial component of the training system. There were still 34% of practicing physicians with vocational college diplomas or below in 2019.

중국에서는 의대 입학 경쟁이 치열하며, 의대 입학은 국가 대학입학자격시험(NCEE) 점수에 따라 결정됩니다. 중국 교육부는 의학 학사 프로그램의 입학 점수를 전문 의과대학의 전문학사 프로그램보다 훨씬 높게 책정하고 있습니다. 이 두 프로그램 간의 학업 성취도로 표시되는 입학 학생의 질은 크게 다릅니다. 2018년 학부 입학률은 43.3%로, 이는 단기 의과 대학에 입학한 학생들이 NCEE를 응시한 상위 43.3% 학생들보다 낮은 순위를 차지했음을 의미합니다.4 
Admission to medical school is highly competitive in China and is based on the students’ scores in the national college entrance examination (NCEE). The National Education Department sets the enrollment scores for medical bachelor programs much higher than vocational diploma programs at junior medical colleges. The quality of admitted students, indicated by their academic performance, between these two programs are significantly different. In 2018, the undergraduate admission rate was 43.3%, which implies that those students admitted into junior medical colleges were ranked below the top 43.3% students taking the NCEE.4

또한 두 의학 프로그램 간에는 교육의 질과 자원에 현저한 차이가 있습니다. 의과 대학 3학년 학생은 3년 동안만 공부해야 하며 실습은 매우 제한적입니다. 특히 이러한 학생들은 고등학교 과정을 통해 학문적 기초가 탄탄하지 않을 수 있다는 점을 고려할 때 의료 교육이 불충분할 수 있습니다. 반면, 의학 학사 프로그램의 학생은 일반적으로 실험실 및 임상 환경에서 보다 엄격한 교육을 포함하여 약 5년의 의학 교육을 받아야 하며, 학위 요건의 일부로 논문을 작성해야 합니다.
There are also remarkable differences in the teaching quality and resources between two streams of medical programs. A junior medical college student needs to take only 3 years of study, with very limited practical training. This may result in insufficient medical training, especially given the fact that these students may not have solid academic foundations through their high school study. By contrast, a student in a medical bachelor program usually needs to take around 5 years of medical training, including more rigorous training in laboratory and clinical settings, and needs to complete a thesis as the partial fulfillment of the requirements for the degree.

의대생들의 학력 이질성이 높기 때문에 중국 내 개업 의사의 질과 분포에 상당한 이질성이 존재합니다. 예를 들어, 중국에서는 개업 의사의 질과 양이 도시와 농촌 간에 불균등하게 분포되어 있으며,2,5,6 고급 의사(학사 학위 이상)는 도시 지역에 집중되어 있습니다. 표 1은 중국 도시와 농촌의 교육 수준과 의사의 직업 선택(보조 의사 포함) 간의 관계를 보여줍니다.

  • 도시 지역에서는 2019년 병원에서 근무하는 의사의 27%, 지역 보건소에서 근무하는 의사의 48.6%가 전문학사 이하의 학력을 가지고 있었습니다.
  • 농촌 지역에서는 향진 병원에서 근무하는 의사의 75.8%, 지역 보건 센터에서 근무하는 의사의 97%가 전문학사 이하의 학력을 가지고 있었습니다.

학사 학위 이상을 소지한 개원의는 도시 병원에 더 집중되어 있었습니다. 따라서 농촌 지역 주민들은 도시 지역 주민들과 동등하게 양질의 서비스를 제공받지 못할 수 있습니다. 
The high heterogeneity in educational background of medical students has led to significant heterogeneity in quality and distribution of practicing physicians in China. For example, the quality and quantity of practicing physicians are unequally distributed between urban and rural areas in China,2,5,6 and high-quality physicians (with a bachelor degree or above) are concentrated in urban areas. Table 1 shows the relationship between education levels and physicians’ job choices (including assistant physicians) between urban and rural areas in China.

  • In urban areas, 27% of the practicing physicians at hospitals and 48.6% at community health centers had vocational diplomas or below in 2019.
  • In rural areas, 75.8% of the practicing physicians at township hospitals and 97% at community health centers had vocational diplomas or below.

Practicing physicians with a bachelor degree or above were more concentrated in urban hospitals. Therefore, residents in rural areas may not have equal access to quality services as those in urban areas.

학사 수준의 일차 의료 교육 통합
Unifying Primary Medical Education at Bachelor-level

중국 전국인민대표대회 상무위원회는 의사 자격 시험 응시 자격을 중등 교육에서 비학사 고등 교육으로 상향 조정하는 의사법 개정안을 통과시켰습니다. 그러나 의료 기준 설정에 있어 이러한 점진적 접근 방식은 장기적으로 실수를 누적시키고 경제적, 보건적 비용을 증가시킬 뿐입니다. 현재 의료 서비스의 복잡성과 의학교육 및 기술의 급속한 발전을 고려할 때, 우리는 가능한 한 빨리 의학교육을 학사 수준으로 통합하는 것이 합리적이라고 강력히 주장합니다. 다음 섹션에서 그 이유를 설명합니다.
The review conference of China’s Congress amended the Law for Practicing Physicians to upgrade the eligibility for the examination of physician qualification from secondary education to non-bachelor tertiary education. However, this incremental approach in a medical standards setting will only accumulate long-term mistakes, and aggregate economic and health costs. Given the complexity of health care at present and rapid development in medical education and technology, we strongly argue that it is reasonable to unify medical education at a bachelor level as soon as possible. We present our reasons in the following sections.

전 세계 의사를 위한 입문 수준 기준
Entry-level Standards for Physicians around the World

국가마다 의학교육과 훈련에 큰 차이가 있지만, 초급 의학교육의 최소 기준은 학사 학위 교육을 요구합니다.

  • 호주, 필리핀, 미국 등 일부 국가에서는 의대생이 의학전문대학원 수준의 의학 프로그램에 입학해야 면허 시험에 응시할 수 있습니다.7
  • 미국의 경우 미국 의사 면허 시험(USMLE)에 응시하려면 학사 학위(4년)와 의과대학 4년 졸업 학위가 필요합니다.8 공인된 레지던트 프로그램(3~7년)을 마친 후 의료 행위가 허용됩니다. 따라서 미국에서 주치의가 되기 위해서는 약 11년, 외과의사가 되기 위해서는 13년 이상의 수련 기간이 소요됩니다.

Although there are large variations in medical education and training across countries, the minimum standards for entry-level medical education require bachelor-degree training.

  • Some countries, such as Australia, Philippines, and the USA, even require medical student to enter the graduate-level medical program before they can attend the licensing examinations.7 
  • In the US, in order to be eligible for the United States Medical Licensing Examination (USMLE), candidates require a bachelor degree (4 years), and a four-year graduate degree from medical schools.8 Medical practice is allowed after completing an accredited residency program (3–7 years). Therefore, it takes around 11 years of training to become a primary care physician and over 13 years to become a surgeon in the United States.

개발도상국인 태국의 경우 반세기 전에 학사 수준의 교육으로 입문 의학교육이 시작되었습니다.9

  • 태국의 학부 의학교육(입문 의학교육)은 6년이 걸리며 보통 1학년은 기초 교육, 2~3학년은 기초 의학 교육, 4~6학년은 임상 교육으로 3단계로 나뉩니다.9,10
  • 태국보다 선진국인 중국이 왜 이런 이상한 의학교육 시스템을 유지하면서 교육 기간과 재능이 다른 모든 졸업생을 의사가 될 수 있도록 하는지 의문이 들 수밖에 없습니다. 의료 인력 부족과 인구 건강 개선의 시급한 필요성이 이러한 의료 교육 및 훈련 시스템을 개발하게 된 초기 이유였지만, 오늘날 중국이 직면하고 있는 주요 문제는 더 이상 아닙니다.

In Thailand, a developing country, the entry-level medical education was set at bachelor-level training a half-century ago.9 

  • The Thai undergraduate medical program (entry-level medical education) takes 6 years to finish and is usually divided into three stages: the first year for basic education, the 2nd-3rd years for basic medical education, and the 4th-6th years for clinical training.9,10 
  • This raises a question why China, a country more developed than Thailand, would keep such an odd medical education system and allow all graduates with different training periods and different talents to become doctors. The shortage of health workforce and urgent need to improve population health were the initial reasons for developing this system of medical education and training, but they are no longer the main issues China is facing today.

의대생 공급 과잉
Oversupply of Medical Students

중국은 세계에서 가장 큰 규모의 의료 인력 교육 시스템을 갖추고 있습니다. 2014년부터 2018년까지 의과대학 통계에 따르면 중국의 의대 졸업생 수는 총 560만 명 증가했지만, 의료 인력 규모는 210만 명 증가에 그쳤습니다. 일정 비율의 인력 감소가 있더라도 전체 시장에서 의대 졸업생의 공급과 고용 사이의 큰 격차를 설명하기는 여전히 어렵습니다. 한 가지 가능한 설명은 일부 저학력 의대생이 보건 분야에 진출하지 않아 의대 졸업생 공급 과잉을 초래했다는 것입니다. 
China has the largest education system for health workforce in the world. According to the statistics of medical schools from 2014 to 2018, the number of medical graduates in China increased by 5.6 million in total, but the size of the health workforce increased by only 2.1 million. Even though there is a certain proportion of workforce attrition, it is still difficult to explain the huge gap of the whole market between supply and employment of medical graduates. One possible explanation is that some less-educated medical students did not enter into the health sector and created an oversupply of medical graduates.

또한, 매년 전문 의과대학 및 중등 의과대학에 등록하는 의대생 수는 의학 학사 이상 프로그램에 등록하는 학생 수보다 훨씬 더 많았습니다.

  • 예를 들어, 2018년에 고등 의과대학에 등록한 의대생 수는 85만 5,000명이었으며, 이 중 61만 1,000명이 전문대학 학사 학위 프로그램에 등록했습니다. 또한 전문학사 학위 프로그램에 등록한 학생은 39만 명에 달했습니다.

이러한 공급 패턴은 의학교육의 질에 영향을 미칠 뿐만 아니라 보건 인력 수급의 불균형을 심화시켰습니다.11 중등 직업 교육 및 직업 교육 졸업생의 과잉 공급은 전문과목 혼합의 왜곡(잘 훈련된 일차 진료 의사의 부족)과 의사 시장에서 의대 졸업생 간의 치열한 경쟁을 심화시키는 결과를 초래했습니다. 
Moreover, the number of medical students enrolled each year at junior medical colleges and secondary schools was much higher than that at medical bachelor programs or above.

  • For example, in 2018, the number of medical students enrolled in tertiary medical schools was 855,000, of which 611,000 were enrolled in vocational college diploma programs. In addition, there were 390,000 students enrolled in SVD programs.

This supply pattern not only affected the quality of medical education but also led to an increasing imbalance in the supply and demand of health workforce.11 The oversupply of graduates with secondary vocational education and vocational education further resulted in the distortion of specialty mix (inadequate numbers of well-trained primary care physicians) and increased intensive competition among medical graduates in physicians’ market.

같은 규모로 볼 때 중국의 개업 의사 수는 더 이상 다른 비교 대상 국가보다 적다고 볼 근거가 없습니다. 1인당 의료비 지출과 인구 10,000명당 의사 수 사이에는 양의 상관관계가 있는 것으로 알려져 있는데, 이는 의료비 지출이 의료 시스템에서 고용하는 직원 수와 관련이 있다는 사실을 반영하는 것입니다. WHO 데이터를 기반으로 국제 비교 관점에서 중국의 의사 수를 조사했습니다.12 그림 2에 따르면 중국의 의사 공급은 모든 국가 중에서 상대적으로 적절한 수준입니다. 
We have reasons to believe that, on the same scale, the stock of practicing physicians in China is no longer less than that in any other comparable countries. There is a known positive association between health expenditure per capita and the number of medical doctors per 10,000 people, which reflects the fact that how much you spend is related to how many employees the system hires. We examined the number of physicians in China from the perspective of international comparison based on the WHO data.12 Figure 2 suggests that the provision of practicing physician in China is relatively adequate among all nations.

다단계 의료 교육 시스템이 의료 시장에 미치는 장기적 비용
The Long-term Costs of Multi-tiered Medical Education System on Healthcare Market

2019년 Hsieh & Tang의 논문에 따르면, 의사들의 학력 차이가 현저한 의료 시장은 환자 입장에서 더 나은 서비스를 제공할 수 있는 양질의 의사를 찾는 데 추가적인 탐색 및 정보 비용을 부과합니다.2 학력이 높은 의사는 도시 지역에서 진료하는 경향이 있기 때문에 대형 병원에 고용될 가능성도 높습니다. 이로 인해 농촌 지역과 1차 의료 기관의 인적 자원이 악화되고 있습니다. 1970년대에 아켈로프가 개발한 모델에 따르면13 의사의 능력은 관찰할 수 없으며, 이는 의사의 보수가 그들이 제공하는 서비스의 질에 따라 달라지지 않는다는 것을 의미합니다. 규제 시장regulated market에서는 모든 의사에게 평균 수준의 임금을 지급하므로 우수한 의대생이 더 나은 보수를 기대하기 때문에 의료 시장에 진입할 동기가 부족합니다. 또한 의사들은 추가 의료 교육을 받거나 인적 자본에 투자할 인센티브가 적습니다. 장기적으로 다단계 의학교육 시스템은 의료 시장의 형평성과 효율성 측면에서 두 가지 해악을 초래할 것입니다. 
According to Hsieh & Tang’s paper in 2019, the health care market in which physicians have remarkable differences in their education, imposes additional searching and information costs to the seeking of quality doctors who can provide better services from patients’ perspective.2 As doctors with higher educational background tend to practice in urban areas, they are also more likely to be employed by larger hospitals. This has caused a deterioration of human resources in rural areas and at primary health care facilities. Based on the model developed by Akerlof in the 1970s,13 physicians’ ability cannot be observed, which means physicians’ remuneration does not depend on the quality of services they provide. The regulated market pays all the doctors an average level of wage, so there is a lack of motivation for high-quality medical students to enter the health care market since they would expect better payments. Doctors would also have less incentives to receive further medical education and to invest in their human capital. In the long run, a multi-tiered medical education system will contribute to twofold harms in both equity and efficiency of the health care market.

법 개정 및 시행의 어려움
Difficulties in Amending and Implementing the Law

일반적으로 법률과 사법 제도는 그 시행을 위해 비교적 안정적인 기대치를 필요로 합니다. 안정성을 유지하기 위해서는 일단 의사에 대한 법률이 시행된 후 사회적 상황이 크게 변화하기 전에 자의적으로 폐지하거나 개정할 수 없습니다. 따라서 법 개정은 법안 제출부터 법안 심사, 의결 및 채택, 공포 및 시행에 이르기까지 엄격한 절차를 거쳐야 하므로 시간이 걸립니다. 예를 들어, 중화인민공화국 의사 개업에 관한 법률은 1998년에 국회 위원회에서 통과되었지만 23년 후인 2021년에 첫 번째 개정이 이루어졌습니다. 이 기간 동안 중국의 경제 발전은 눈부신 성과를 거두어 지난 20년 동안 GDP가 7조 9,700억 위안에서 82조 7,122억 위안으로 약 937% 증가했으며,14 이에 따라 의료 수요와 의료 시장에도 큰 변화가 일어났습니다. 법 개정 과정이 더디더라도 의학교육 체계 정상화를 위한 법 개정이 시급한 상황입니다. 
Generally speaking, legislation and the judicial system require a relatively stable set of expectations of the law for its implementation. In order to maintain stability, it cannot be abrogated or amended discretionarily before the social contexts change significantly once the law for physicians goes into effect. Therefore, any amendment of the law must follow a rigorous procedure, from submission to the consideration of the draft law, then to the vote and adoption, and to the promulgation and implementation of the law, which will take time. For example, the law for practicing physicians of the People’s Republic of China was passed by the Committee of the Congress in 1998, while its first amendment came twenty-three years later in 2021. The economic development of China has made extraordinary achievements during this period, with its GDP growing from 7.97 trillion yuan to 82.7122 trillion yuan in the last two decades and an increase of about 937%,14 and thus significant changes have taken place in health demands and health care market. There is an urgency to amend the law to normalize the medical education system even though the process of amendment will be slow.

2021년 개정안 제정 및 시행을 위한 장애물도 존재합니다. 2019년 중국 보건 통계 연감의 데이터에 따르면15 현재 전문 학사 학위 대학에 재학 중인 의대생은 100만 명이 넘습니다. 이러한 의대생의 경력 개발과 전환, 교수진 및 관련 인력에 미치는 영향을 신중하게 고려해야 하며, 그렇지 않으면 모든 개혁이 그들의 이익에 대한 비용으로 간주되어 저항을 받게 될 것입니다. 
There are also obstacles for the enactment and implementation of the 2021 amendment. According to data from the Health Statistical Yearbook of China in 2019,15 there are over one million medical students currently enrolled in vocational diploma colleges. The career development and transition for these medical students and the impact on their faculties and related personnel need to be carefully considered, otherwise any reforms will be seen as costs of their interests and thus resisted.

결론
Conclusion

우리는 중국 의회가 오랜만에 마침내 최근 의사법 개정을 추진한 것을 기쁘게 생각합니다. 그러나 우리는 중국의 의사 자격 시험에 대한 교육 자격이 여전히 국제 표준에 훨씬 못 미친다고 주장합니다. 중앙은행이 과도한 유동성을 흡수할 수 있기 때문에 통화(자본) 완화는 쉬운 반면, 이미 의료 시장에서 형성된 의사들의 인적 자본을 되돌리기는 거의 불가능합니다. 따라서 우리는 중국이 최근 개정안과 같이 의사 자격 시험의 문턱을 전문대 수준이 아닌 학사 수준으로 설정하는 것을 의미하는 초급 의학교육 기준을 개선할 것을 강력히 권고합니다. 이는 의사의 자질을 향상시킬 뿐만 아니라 중국 의료 시장의 형평성과 효율성 문제를 해결하는 데 도움이 될 것입니다.

We are delighted to see that the Congress in China finally promoted the recent amendments on the Legislation of Practicing Physician after a long time. However, we argue that the educational eligibility for examination of the physicians’ qualification in China is still far below the international standard. It is easy to do monetary (capital) easing because the central bank can soak up excessive liquidity, while it is hardly possible to take back human capital of physicians already formed in a health care market. Therefore, we highly recommend that China improve the entry-level medical education standards, which implies setting the threshold at bachelor level for the examination of physician’s qualification, instead of junior college level as in the recent amendment. This will not only improve the quality of the physicians but also will help to address equity and efficiency issues in the health care market in China.


Health Syst Reform. 2022 Jan 1;8(1):e2048438. doi: 10.1080/23288604.2022.2048438.

Amending the Law for Licensing Medical Practitioners of China in 2021: A Commentary

Affiliations

1Macquarie University Centre for the Health Economy, Macquarie Business School, Macquarie University, NSW, Australia.

2Institute for Global Health and Development, Peking University, Beijing, Haidian, China.

3The Second Affiliated Hospital of Guangzhou Medical University, Guangzhou, Guangdong, China.

PMID: 35416752

DOI: 10.1080/23288604.2022.2048438

Abstract

The Law for Licensing Medical Practitioners of the People's Republic of China, enacted in 1999, was amended in 2021. This commentary reviews the key points of the amendment and raises doubts as to one of its points. Specifically, we argue that the minimum education level required to take the physicians' licensing examination should be set to completion of a bachelor degree, instead of a vocational diploma or junior college graduation as in the 2021 amendment. China adopted a system of multi-tiered medical education more than 70 years ago. This policy has resulted in a threshold of entry-level medical education far below the global standards. The highly heterogeneous education background of physicians in China has led to low standards of practicing physicians, which in turn have significantly negative impacts on the health care market. We illustrate changes over time in the educational distribution and regional distribution of practicing physicians in China, and present reasons to improve entry-level educational standards, by setting the physician licensing threshold at an appropriate level. This will not only improve the overall quality of physicians but will also help address equity and efficiency issues in the health care market.

Keywords: China; licensing; medical education; physician supply.

인도네시아의 의사국가면허시험: 학생, 교수, 대학의 관점(© 2018 The University of Leeds and Rachmadya Nur Hidayah)
Impact of the national medical licensing examination in Indonesia: perspectives from students, teachers, and medical schools. 
Rachmadya Nur Hidayah

3.3 국가 라이선스 시험의 역사
3.3 The history of national licensing examination

이 섹션에서는 북미에서 시작하여 유럽과 아시아로 확장된 NLE의 기원과 발전 과정을 설명합니다. 이 섹션은 인도네시아의 국가면허시험의 배경과 현재 논의 중인 분야를 포함하여 인도네시아의 국가면허시험의 역사를 소개하는 것으로 마무리합니다.
This section will describe the origins and development of the NLE started and developed; starting from North America and extending through Europe and Asia. The section concludes by presenting the history of NLE in Indonesia, including its background and areas of current debate.

북미의 국가 시험
National examination in North America

미국과 캐나다는 의대 졸업생을 대상으로 국가 시험을 최초로 시행한 국가 중 하나입니다. 미국 의사 면허 시험(USMLE)은 남북전쟁(1861~1865년) 이후 의료 종사자에 대한 규제에서 파생되었습니다. 이 시험은 의사들 간의 높은 역량 편차를 줄이기 위한 목적으로 수십 년 동안 시행되었습니다. 미국국립시험위원회(NBME®)는 1915년 미국에서 국가 시험 시스템을 관리하기 위해 설립되었습니다(Melnick et al., 2002). 시험의 구조는 그 이후로 발전해 왔습니다. 

  • 첫 번째 시험 형식(1916년)은 환자 사례, 구두(구술) 시험, 필기시험을 이용한 복잡한 병상 시험이었습니다.
  • 필기 시험은 1922년 주관식 문제로 시작하여 선택형 문제로 발전했으며, 이후 1980년대에는 객관식 문제(MCQ)로 형식이 변경되었습니다(Melnick et al., 2002).
  • NBME는 1999년에 임상 술기 시험을 승인하고 2004년에 2단계 임상 술기 평가(CSA)를 시행했습니다. 이러한 결정은 주로 장시간의 구술 시험이 신뢰성이 떨어진다는 비판을 받은 후 임상 술기 평가의 필요성에 의해 주도되었습니다.

The United States of America and Canada were among the first countries that conducted national examinations for their medical graduates. The United States Medical Licensing Examination (USMLE) was derived from regulatory entry for medical practitioners after the Civil War (1861-1865). It served the purpose of reducing the high variation of competence amongst practitioners and was implemented for several decades. The National Board of Medical Examiner (NBME®) was founded in 1915, to administer a national examination system in the United States of America (Melnick et al., 2002). The structure of the examination has evolved since then.

  • The first structure of the format (1916) was a complex bedside examination using patient cases, oral (viva) examinations, and written examinations.
  • Written examinations started with essay questions in 1922 and evolved to selected-response questions and later, in the 1980s, the format of USMLE was changed to multiple-choice questions (MCQ) (Melnick et al., 2002).
  • The NBME approved the clinical skills examination in 1999 and implemented the Step 2 Clinical Skills Assessment (CSA) in 2004. This decision was mainly driven by the need to assess clinical skills after the long case oral examination was criticised for poor reliability.

캐나다 의료 위원회(MCC)는 캐나다에서 의사가 진료할 수 있도록 면허를 부여하는 기관입니다. 자격을 갖춘 후보자를 결정할 때 MCC는 의대생의 학부 과정 중과 종료 시점에 평가 절차를 사용합니다. 

  • 1970년까지 MCC는 전통적인 에세이 및 구술 시험을 사용했습니다.
  • 1980년, MCC 면허증 개발 과정이 완료되자 이 시험은 캐나다의 모든 지역에 적용되었습니다(Dauphinee, 1981).
  • 몇 년 후, MCC는 면허 시험으로서의 목표를 검토한 결과 필기 시험으로는 평가할 수 없는 의대 졸업생의 필수 역량이 있다는 결론에 도달했습니다. 여기에는 병력 청취, 신체 검사, 의사소통 능력 등이 포함되었습니다.
  • 그 후 MCC는 1980년대 후반에 임상 기술 평가를 위한 파일럿 연구를 실시하기로 결정했습니다.
  • 1992년에는 객관적 구조화 임상시험(OSCE)이 면허 시험의 일부가 되었습니다(Reznick et al., 1993).

The Medical Council of Canada (MCC) acts as the authority to grant licentiate for physicians to practice in Canada. In determining eligible candidates, MCC uses assessment procedures during and at the end of medical students’ undergraduate programmes.

  • Until 1970, MCC used traditional essay and oral examinations.
  • In 1980, when the development process of MCC licentiate was finished, the examination was applied to all regions of Canada (Dauphinee, 1981).
  • After a few years of this assessment, MCC reviewed its objectives as a licensing examination and came to the conclusion that there were essential competences for medical graduates that could not be assessed using written examination. These included: history taking, physical examination, and communication skills.
  • MCC then decided to conduct a pilot study for clinical skills assessment in the late 1980s.
  • In 1992, the Objective Structured Clinical Examination (OSCE) became part of the licensing examination (Reznick et al., 1993).

USMLE와 MCCQE는 정기적인 평가와 연구를 바탕으로 잘 정립된 제도로 자리 잡을 수 있는 시간을 가졌습니다. 시험 관리자(NBME 및 MCC)가 수행한 연구는 대부분 시험의 심리측정 측면에 초점을 맞추었습니다. 그러나 지난 10년 동안 NLE가 대학원 학업, 임상 성과 및 환자 치료에 미치는 영향에 대한 연구가 더 많이 진행되었습니다. 이에 대해서는 이 장의 뒷부분에서 설명합니다. 
Both USMLE and MCCQE have had the time to become well-established systems based on regular evaluation and research. Studies conducted by test administrators (NBME and MCC) mostly focussed on the psychometric aspects of the test. However, in the last decade, there has been more research on the consequences of the NLE on postgraduate study, clinical performance, and patient care. This will be discussed later in this chapter.

유럽의 국가 시험
National examination in Europe

대서양 횡단 시험에 비해 유럽에서는 지난 10년 동안 국가 또는 유럽 면허 시험에 대해 더 많은 논쟁이 있었습니다. 영국을 포함한 유럽 국가들 사이에서는 국가 또는 대규모 시험 도입 문제에 대한 논의가 계속되고 있습니다. 유럽 국가들은 유럽연합 회원국의 의대 졸업생이 유럽연합(EU) 내에서 활동할 수 있도록 인정했기 때문에 표준화된 의료 교육 및 실습의 질을 확보해야 할 공동의 책임이 있었습니다(Gorsira, 2009). 이 글에서 Gorsira는 제안된 유럽 NLE에 대한 반대 의견을 국가 간 이해, 신뢰, 협력과 같은 주요 이슈와 함께 설명했습니다. 유럽 국가들은 의학교육 시스템이 다양하기 때문에 유럽에서 기대하는 의사 표준을 달성하는 것에 대한 우려가 있었습니다. 그러나 Gorsira(2009)가 지적했듯이, NLE의 잠재적 이점과 함정은 논쟁의 여지를 남겼습니다. 그녀는 유럽 NLE의 즉각적인 시행이 환자의 안전을 보장하지 못할 뿐만 아니라 의학교육에도 해를 끼칠 수 있다고 결론지었습니다(Gorsira, 2009). 의사를 위한 유럽 표준에 동의하는 것은 브렉시트가 시행되면 영국을 포함한 비유럽 졸업생들과 어떻게 조화를 이룰 것인지에 대한 문제를 부각시킵니다.
Compared to their transatlantic counterparts, there had been wider debate about the national or European licensing examination during the last decade. There is on-going discussion amongst European countries, including the United Kingdom, about the issue of establishing national or large-scale examinations. Since European countries recognised medical graduates from the European Union members to practice within the European Union (EU), there was a shared responsibility to have a standardised quality of medical education and practice (Gorsira, 2009). In her article, Gorsira described the opposing views in response to a proposed European NLE, with key issues such as understanding, trust, and collaboration between countries. European countries varied in their medical education system, thus there was concern about achieving the expected standard of doctors in Europe. However, as Gorsira (2009) pointed out, the potential benefits and pitfalls of the NLE left the debate open. She concluded that immediate implementation of an European NLE would not guarantee patient
safety and would also cause harm to medical education (Gorsira, 2009). Agreeing European standards for doctors highlights the issue of how they align with non-European graduates, which will include the UK when Brexit is implemented.

네덜란드와 같이 의학교육에 대한 엄격한 인증과 동질적인 커리큘럼을 갖춘 일부 유럽 국가에서는 국가시험을 우선순위로 여기지 않는다고 말한 van der Vleuten(2013)에 의해 이 논쟁은 더욱 복잡해졌습니다. 일부 학교에서는 이미 커리큘럼의 비교 가능성을 보장하기 위해 집단 진도 테스트를 시행하고 있었습니다(Schuwirth 외., 2010). GMC의 '내일의 의사'를 기반으로 의과대학의 커리큘럼 설계와 실행에 더 큰 자유가 있는 영국의 경우, 의대 졸업생의 역량 비교 가능성에 초점을 맞춰 국가시험이 논의되었습니다(McCrorie and Boursicot, 2009). 객관성, 일관성, 품질 보증, 환자 안전 등의 논거를 고려한 GMC는 최근 국가 면허 시험에 대한 지지를 발표했습니다. 국가 시험을 제안하는 또 다른 이유는 졸업후 교육에 입학하는 학생의 기준을 설정하기 위해서입니다. 졸업후 교육에서 투명한 정량적 선발 메커니즘을 개발해야 한다는 문제도 영국에서 국가 면허 시험의 필요성을 제기했습니다. 미국과 달리 영국의 대학원 교육 선발은 국가시험(예: USMLE)의 순위를 사용하지 않기 때문에 동일한 평가 프로그램에서 해외 의대 졸업생과 영국 졸업생을 비교할 수 없었습니다(Gorelov, 2010).
This debate was further complicated by van der Vleuten (2013), who stated that some European countries that had strict accreditation of medical education and homogenous curricula, e.g. the Netherlands, did not see a national examination as a priority. Some schools already had collective progress testing to ensure the comparability of their curriculum (Schuwirth et al., 2010). In the UK, where there is greater freedom to design and implement medical schools’ curriculum based on the GMC’s Tomorrow’s Doctors, the national examination had been discussed following the focus on comparability of medical graduates’ competences (McCrorie and Boursicot, 2009). Considering the arguments of objectivity, consistency, quality assurance, and patient safety, the GMC recently announced its support for a national licensing examination. Other reasons for proposing national examinations would be to set the standard for students entering postgraduate education. The concern to develop a transparent quantitative mechanism of selection in postgraduate training also raised the need for national licensing examinations in the UK. Unlike the US, postgraduate training selection in the UK does not use the ranks in a national examination (such as USMLE), therefore it could not compare international medical graduates and UK graduates in the same assessment programme (Gorelov, 2010).

다른 유럽 국가에서는 NLE에 대해 보다 긍정적인 접근 방식을 취했습니다. 스위스는 2013년에 국가 면허 시험을 도입했습니다. 연방 면허 시험(FLE)은 학부 의학교육이 끝날 때 지식과 기술을 평가하여 품질 보증을 위한 수단으로 개발되었습니다. 스위스는 자국의 의료 및 의학교육의 높은 수준을 유지하기 위해 FLE를 도입했습니다. 기대되는 질은 졸업생의 역량 수준으로 설명되었습니다. 2010/2011년에 파일럿 시험을 실시한 후, 중앙에서 관리하고 지방에서 시행하는 시험(MCQ 필기시험과 OSCE로 구성)이 실시되었습니다(Guttormsen 외., 2013). 국가 시험으로 OSCE를 도입한 목적은 응용 임상 지식과 실제 임상 기술을 평가하여 졸업생의 수준 높은 수준을 보장하기 위한 것이었습니다.
Other European countres took a more positive approach to NLEs. Switzerland introduced a national licensing examination in 2013. The federal licencing examination (FLE) was developed as a means of quality assurance by assessing knowledge and skills at the end of undergraduate medical education. The reason the FLE was introduced was that Switzerland wanted to maintain the high quality of health care and medical education in their country. The expected quality was described as the level of competence of graduates. After performing a pilot in 2010/2011, the examination (which is centrally-managed and locally administered) was conducted, comprising MCQ written examinations and OSCEs (Guttormsen et al., 2013). The aim of establishing an OSCE as a national examination was to assess applied clinical knowledge and practical clinical skills to ensure a high-quality standard of graduates.

앞서 언급했듯이 EU 국가 내에서 의료 전문가의 이동성은 장점이자 단점으로 여겨져 왔습니다. 예를 들어, 영국에서는 해외 졸업생들이 의사 부족 문제를 해결하는 데 도움이 되었지만 EU 국가 간 교육 격차로 인해 EU에서 교육받은 의사 수가 증가하면서 우려가 제기되었습니다. 2015년 GMC는 2022년까지 영국 내에서 진료하려는 국내, 유럽 및 해외 의사를 대상으로 NLE의 한 형태인 의료 면허 평가(MLA)를 구축하는 프로젝트를 시작했습니다(Gulland, 2015; Archer 외., 2016a; Archer 외., 2016b). MLA는 국제 졸업생을 대상으로 하는 현재의 전문 및 언어 평가 위원회(PLAB) 시험을 대체할 것입니다. PLAB 시험은 임상 실무에서 영어에 대한 이해와 맥락을 테스트합니다. 영국에서 MLA 시범 프로젝트가 아직 진행 중이지만, 다양성에 대한 의문과 NLE가 의과대학의 현재 평가에 잘 맞을지에 대한 의문은 여전히 남아 있습니다(Archer 외., 2016a; Archer 외., 2016b; Stephenson, 2016). NLE를 설계하고 전달하는 '방법'에 대한 이러한 문제는 NLE를 도입하는 국가에서 흔히 발견되며, 이를 자세히 고려하면 그 결과의 잠재적 이점과 단점을 강조할 수 있습니다.
As mentioned earlier, the mobility of healthcare professionals within the EU countries has been seen as both a benefit and drawback. For example, in the UK although international graduates have helped to address the shortage of doctors the difference in training across the EU countries raised concerns when the number of EU-trained doctors increased. In 2015 the GMC initiated a project to establish by 2022 a medical licensing assessment (MLA), a form of NLE, for home, Europe, and international doctors intending to practice within the UK (Gulland, 2015; Archer et al., 2016a; Archer et al., 2016b). The MLA will replace the current Professional and Linguistic Assessment Board (PLAB) examination which is aimed at international graduates. The PLAB examination tests the understanding and context of English in clinial practice. While the pilot project for the MLA in the UK is still ongoing, the questions about diversity and whether the NLE would sit well within medical schools’ current assessment remains (Archer et al., 2016a; Archer et al., 2016b; Stephenson, 2016). This problem of “how” in designing and determining the delivery of NLE is commonly found in countries introducing the NLE; considering this in detail highlights the potential benefits and drawbacks of its consequences.

중동 및 아시아의 국가 시험
National examinations in Middle East and Asia

많은 전문가들은 교육과정의 다양성이 높은 곳에서 NLE가 하나의 옵션이 될 수 있음을 인식하고 있습니다. Van der Vleuten(2013)은 한 국가 또는 지역의 교육 프로그램과 평생교육의 다양성이 NLE의 필요성을 강화한다고 제안했습니다. 대부분의 아시아 국가에서 의과대학은 여전히 커리큘럼과 함께 일하기 위한 '최선의 방법'을 개발하고 있습니다. 학교는 교육 전문가와 협력하여 프로그램과 교육 전략을 개발하여 혁신하고 있습니다. 그들은 평가 시스템과 함께 주기적으로 커리큘럼을 평가하고 변경하여 국가 또는 국제적 요구에 맞게 조정합니다(Telmesani 외., 2011; Lin 외., 2013). 
Many experts recognised that NLEs could be an option where there is a high diversity in curriculum implementation. Van der Vleuten (2013) suggested that the diversity of training programs and continuing education in a country or region strengthens the need for NLE. In most Asian countries, medical schools are still developing their ‘best way’ to work with the curriculum. Schools work with educational experts to innovate, developing their programme and educational strategies. They evaluated and changed their curriculum periodically, along with the assessment system, to suit national or international needs (Telmesani et al., 2011; Lin et al., 2013).

중동에서 사우디아라비아는 졸업생의 질을 보장하기 위해 역량 기반 커리큘럼과 NLE를 구축하려고 시도한 국가 중 하나입니다. 이러한 결정은 사우디 아라비아의 의학교육에 대한 변화로 인해 이루어졌습니다. 여기에는 다음이 포함됩니다: 

  • 1) 의과대학의 수 증가와 각 의과대학이 채택한 다양한 커리큘럼 및 평가 시스템,
  • 2) 사우디아라비아에서 진료하기를 원하는 다른 나라 졸업생의 증가,
  • 3) 해외에서 의학을 공부하는 사우디 원주민의 증가(Bajammal 외., 2008).

In the Middle East, Saudi Arabia was the one of the countries to attempt to establish a competence-based curriculum and NLEs to ensure the quality of their graduates. This decision was driven by changes to medical education in Saudi Arabia. These included:

  • 1) The increasing number of medical schools, and the different curricula and assessment systems they adopted;
  • 2) Increasing numbers of graduates from other countries who wanted to practice in Saudi Arabia; and
  • 3) The increasing number of Saudi natives who pursued their medical study abroad (Bajammal et al., 2008).

비슷한 이유로 아시아에서는 한국이 2008년에 NLE와 OSCE를 최초로 시범 운영한 국가 중 하나였으며, 대만과 인도네시아가 그 뒤를 이었습니다

  • 한국은 2008년부터 표준화된 환자를 대상으로 한 임상 술기 평가와 마네킹을 이용한 OSCE를 시작했습니다. 한국 국가 OSCE는 임상 교육을 개선하는 것을 목표로 했습니다. 2010년부터는 임상 술기 시험 센터에서 3개월에 걸쳐 12개 스테이션으로 구성된 OSCE를 시행하고 있습니다. OSCE는 표준화된 환자(SP) 평가자와의 환자 대면 상황을 기반으로 한 6개의 스테이션과 의료진 평가자와의 시술 술기를 기반으로 한 6개의 스테이션으로 구성됩니다(Park, 2008). SP 평가자를 활용하고 장기간에 걸쳐 시행되기 때문에 시험의 공정성 및 타당성, 시험 정보 공유/공개와 관련된 여러 가지 문제에 직면했습니다. 
  • 대만에서 NLE는 필기 시험으로 시작되었습니다. 2008년 말, 대만 당국은 필기 면허 시험의 전제 조건으로 국가 OSCE를 발표했습니다. 2011년과 2013년에 대규모 파일럿 OSCE가 실시되었고, 이후 본격적인 OSCE가 시행되었습니다(Lin et al., 2013). 
  • 일본과 같은 다른 국가에서는 의대 졸업반 학생을 대상으로 NLE에 대한 필기 평가만 계속 요구하고 있습니다(Kozu, 2006; Suzuki et al., 2008).

For similar reasons, in Asia, South Korea was one of the first countries to pilot their NLE and its OSCE in 2008, followed by Taiwan and Indonesia.

  • South Korea started clinical skills assessment in 2008, having a clinical performance examination with standardized patients and an OSCE using manikins. The South Korean national OSCE aimed to improve clinical education. Since 2010, it has been carried out as a 12-station OSCE and administered over the course of three months in clinical skill test centres. The OSCE consists of 6-stations based on a patient encounter with standardised patient (SP) raters and 6-stations based on procedural skills with medical faculty raters (Park, 2008). It faced several challenges related to test fairness and validity of the exam, since it used SP raters and was administered over a long period, which enabled information sharing/ disclosure of exam information.
  • In Taiwan, NLE started as a written examination. Later in 2008, Taiwanese authorities announced the national OSCE as a prerequisite for taking the written licencing examination. Large-scale pilot OSCEs were held in 2011 and 2013 before the high-stake OSCE was implemented (Lin et al., 2013).
  • Other countries, such as Japan, continue to require only written assessment for the NLE for final year medical students (Kozu, 2006; Suzuki et al., 2008).

동남아시아에서는 10개국 중 4개국만이 NLE를 시행하고 있으며, 각 국가마다 목적과 대상이 다릅니다.

  • 태국, 필리핀, 인도네시아, 말레이시아는 MCQ 또는 수정된 에세이 질문(MEQ) 형식을 사용하여 지식 평가를 실시합니다.
  • 말레이시아는 해외 졸업생만 평가하고 나머지 3개 국가는 국내 및 해외 졸업생을 평가합니다.
  • 필리핀을 제외한 나머지 3개 국가는 OSCE 형식을 사용하여 임상 기술을 평가합니다.
  • 베트남과 라오스는 NLE를 개발 중이며
  • 브루나이, 싱가포르, 캄보디아, 미얀마는 NLE가 없습니다.

동남아시아에서 NLE에 대한 논의는 이 지역의 다른 국가에서 의술을 펼칠 수 있도록 의료 전문가의 자유로운 이동을 장려하는 아세안1 경제 공동체(AEC)에도 과제를 안겨주고 있습니다(Kittrakulrat 외., 2014). 
In South East Asia, only four out of ten countries have implemented NLEs and each have different purposes/ targets.

  • Thailand, Phillipines, Indonesia, and Malaysia, have knowledge assessment using the MCQ or modified essay questions (MEQ) formats.
  • Malaysia assesses international graduates only, while the other three assess home and international graduates.
  • Aside from the Phillipines, the other three countries assess clinical skills using OSCE formats.
  • Vietnam and Lao are in the process of developing NLEs, while
  • Brunei, Singapore, Cambodia, and Myanmar do not have one.

The discussion of NLEs in South East Asia also brings challenges to the ASEAN1 Economic Community (AEC) which promotes for the free movement of medical professions to practice medicine in another country in this region (Kittrakulrat et al., 2014).

인도네시아 국가 시험
National examination in Indonesia

인도네시아에서 NLE의 발전은 21세기 초 고품질 의료 전문가에 대한 필요성 증가에 뿌리를 두고 있습니다. 2007년 보건부 보고서에 따르면, 지역사회의 의료 접근성은 향상되었지만 의료 서비스 결과는 약간만 개선되었습니다. 2010년 세계보건기구(WHO) 보고서에 따르면 인도네시아의 의사 밀도는 인구 1,000명당 0.15명으로 예상 표준 비율에 미치지 못했습니다. 또한 도시와 농촌 지역에 의료 전문가가 고르지 않게 분포되어 있었습니다. 2006년에 의사의 17%만이 의료 서비스가 부족한 지역에서 근무한 반면, 83%는 인구 밀도가 높은 지역에서 근무했습니다(WHOSEARO, 2011). WHO와 인도네시아 정부는 네 가지 전략을 강조하여 보건 인적 자원을 개발하고 역량을 강화하는 것을 목표로 삼았습니다: 1) 계획 강화, 2) 공급/생산 증가, 3) 관리(유통 및 활용) 개선, 4) 품질 감독 및 관리 강화입니다(WHOSEARO, 2011).
The development of the NLE in Indonesia was rooted in the increasing need for high quality health care professionals at the beginning of 21st century. According to the report from the Ministry of Health in 2007, whilst communities had better access to health care, there were only slight improvements in health care outcomes. According to a World Health Organisation (WHO) report in 2010, Indonesia had a physician density of 0.15 per 1,000 population, which was less than the expected standard ratio. Moreover, there was uneven distribution of healthcare professionals in urban and rural areas. In 2006, only 17% of physicians worked in underserved areas, while 83% worked in highly populated areas (WHOSEARO, 2011). WHO and the Indonesian Government aimed to develop and empower human resources for health by emphasizing four strategies: 1) strengthening planning, 2) increasing supply/ production, 3) improving management (distribution and utilization), and 4) strengthening supervision and control of quality (WHOSEARO, 2011).

이러한 틀 안에서 정부는 이러한 목표를 달성하기 위해 설계된 보건의료 및 보건 전문직 교육 정책을 지속적으로 시행했습니다. 몇 년 전부터 정부가 보건 전문직 교육 법안을 제정하면서 변화가 시작되었습니다: 2003년에는 국가 교육 시스템 법안, 2004년에는 의료 실무 법안이 제정되었습니다. 이 법안들은 2006년에 인도네시아 의료 위원회의 설립을 촉구했습니다. 이 법안은 또한 교육부가 학부 의학교육 커리큘럼을 개선하는 데 촉매제 역할을 했습니다. 역량 기반 커리큘럼이 시행되었고, 커리큘럼의 참고 자료로 인도네시아 의사 역량 표준(인도네시아 의사 역량 표준 - SKDI)이 만들어졌습니다. 
Within this framework, the Government continued to implement policies in health care and health professions education designed to achieve these aims. Changes had begun a few years before, when the government established health profession education bills: The National Education System Bill in 2003 and Medical Practice Bill in 2004. The Bills urged the establishment of the Indonesian Medical Council in 2006. The Bills also acted as a catalyst for the Ministry of Education to improve the undergraduate medical education curriculum. Competence-based curricula were implemented and the Standard of Competence for Indonesian Medical Doctors (Standar Kompetensi Dokter Indonesia – SKDI) created as a reference for curricula.

역량 기반 커리큘럼 구현은 세계은행이 후원하는 교육부의 보건 전문가 교육 품질 프로젝트의 감독하에 진행되었습니다. NLE 설립에 앞서 인도네시아 의과대학 간 벤치마킹 테스트가 진행되었습니다. 자바섬(본섬)과 수마트라섬(외딴 지역)의 공립대학을 대상으로 한 벤치마킹 테스트 결과, 인도네시아 의과대학의 질에 차이가 있는 것으로 나타났습니다(Agustian and Panigoro, 2005). 위원회가 각 학교를 지속적으로 방문한 결과, 의학교육의 질을 보장하기 위해 의과대학의 '역량과 능력'을 개선할 필요가 있다는 사실이 밝혀졌습니다. '역량과 역량'이라는 용어는 자원에만 국한된 것이 아니라 교육 기관 내부의 학습 과정도 포함했습니다. 
The competence-based curriculum implementation was conducted under the supervision of the Ministry of Education’s Health Professionals Education Quality project sponsored by the World Bank. Prior to the establishment of the NLE, a series of benchmarking tests among medical schools in Indonesia took place. A benchmarking test between a public university in Java (the main island) and in Sumatera (a more remote area) shows that there were gaps among medical schools’ quality in Indonesia (Agustian and Panigoro, 2005). A continuous visit to each school by the committee revealed the need to improve the ‘capacity and capability’ of medical schools to ensure the quality of medical education in the institution. The term ‘capacity and capability’ was not only limited to resources but also included the learning process inside the institution.

세계보건기구에 따르면 2008년 인도네시아 의과대학을 졸업한 의사는 4325명이었습니다(WHO, 2011). 2013년에는 이 숫자가 거의 두 배로 증가하여 7047명이 졸업했습니다. 2008년 이후 20개 이상의 의과대학이 새로 설립되어 인도네시아의 의대생 수가 크게 증가했습니다. 일부 신설 학교는 기존 학교보다 더 많은 학생을 수용하기도 했는데, 예를 들어 C-인증을 받은 한 신설 학교는 연간 400명의 신입생을 수용했습니다(HPEQ, 2013). 2013년 이전에는 의과대학에 대한 학생 정원 규제가 없었기 때문에 이런 일이 가능했습니다. 각 대학(사립 또는 공립)의 내부 정책에 따랐을 뿐입니다. 현재 인도네시아의 의과대학은 매년 약 7,000~8,000명의 졸업생을 배출하고 있습니다. 인도네시아의 의료 수요를 충족하기 위해 이 숫자는 앞으로 크게 증가할 수 있습니다. 이처럼 의사 수가 크게 증가함에 따라 의학교육의 질을 보장하는 데 어려움이 있습니다. 
According to WHO, in 2008, 4325 doctors graduated from medical schools in Indonesia (WHOSEARO, 2011). In 2013, this number almost doubled, with 7047 graduates. Since 2008 more than 20 new medical schools were established, which significantly increased the number of medical students in Indonesia. Some new schools even accepted more students than the established schools; for example a new and C-accredited school accepted 400 new students per year (HPEQ, 2013). This was possible because, before 2013, there was no regulation of student quota for medical schools. It was only based on each university’s (private or public) internal policy. Nowadays, medical schools in Indonesia produce roughly around 7,000-8,000 graduates per year. This number could increase in the future significantly to meet health care needs in Indonesia. Such a significant increase in the number of medical doctors creates a challenge in assuring the quality of their medical education.

이에 보건부는 인도네시아 의대 졸업생들이 SKDI의 역량을 기반으로 특정 기준을 충족할 수 있도록 NLE를 설립하여 그 질을 높이기로 결정했습니다. 이 시험은 의과대학 내 역량 강화 및 개선을 유도하기 위한 목적도 있었습니다. 인도네시아 보건부와 인도네시아 의사회가 공동 주관하는 위원회에서 관리하는 NLE는 2007년에 설립되었습니다. 시험은 MCQ를 이용한 지식 평가로 시작되었습니다. 임상 술기 역량에 대한 논의가 시작되기 전까지는 지식 평가로 졸업생의 임상 분야 역량을 평가하는 것으로 충분하다고 여겨졌습니다. 2011년, 의사면허시험을 주관하는 인도네시아 국가역량시험 공동위원회(Komite Bersama Uji Kompetensi Dokter Indonesia - KBUKDI)는 MCQ로 평가할 수 없는 임상 술기를 평가하기 위해 OSCE를 개발하기로 결정했습니다(의사면허시험 공동위원회, 2013a). OSCE 시행 준비 과정은 다음과 같이 구분되었습니다:

  • 1) 청사진 설계,
  • 2) 문항 은행 및 지침 개발,
  • 3) 시험 속성 구성(도구, 인쇄된 루브릭, 컴퓨터 기반 채점),
  • 4) 2011-2012년 연 4회 시범 실시,
  • 5) 시범 실시 평가,
  • 6) 2013년 시행, 처음에는 두 번의 시험 기간에 형성 평가로, 다음 시험 기간에 종합 평가로 실시. 

The MoHER then decided to lever the quality of Indonesian medical graduates to meet certain standards, based on competences in SKDI, by establishing a NLE. This examination was also intended to drive improvement or capacity building within medical schools. Managed by a committee coordinated by the MoHER and the Indonesian Medical Council, a NLE was established in 2007. The examination started with an assessment of knowledge using MCQ. Until the discussion of clinical skills competence came up, it was considered sufficient to assess graduate competence in the clinical area by assessing their knowledge. In 2011, the Joint Committee of Indonesia National Competency Examination (Komite Bersama Uji Kompetensi Dokter Indonesia – KBUKDI), who act as an executive for the licensure, decided to develop an OSCE to assess clinical skills which could not be assessed using MCQ (Joint Committee on Medical Doctor Licensing Examination, 2013a). The process of preparing OSCE implementation was divided into:

  • 1) Designing the blueprint;
  • 2) Developing an item bank and guidelines;
  • 3) Organizing exam attributes (tools, printed rubrics, computer-based scoring);
  • 4) Piloting four times a year within 2011-2012;
  • 5) Evaluation of pilots;
  • 6) Implementation in 2013, initially as a formative assessment in two examination periods and summative in the next ones.

OSCE는 15분 분량의 12개 스테이션으로 구성되었습니다12개 스테이션은 12개 신체 시스템을 대표하며, 2012년 SKDI를 청사진으로 삼았습니다. 각 스테이션에서는 외래 진료실, 응급실, 수술실, 수술실로 설정된 공간에서 시뮬레이션된 임상 시나리오를 사용했습니다. 표준화된 환자 발생 사례와 마네킹을 이용한 시뮬레이션이 있었습니다. 수험생들은 로테이션을 위해 부저 소리로 안내를 받았습니다. 시험관들은 루브릭으로 학생들을 평가하고, 해당 스테이션의 케이스에 대한 임상 정보에 대한 가이드라인을 제공했습니다.
The OSCE comprised of twelve 15-minute stations. The twelve stations represented 12 body systems, referring to the 2012 SKDI as the blueprint. The stations used simulated clinical scenarios in rooms set as outpatient clinics, emergency room, and operation/ surgical room. There were standardised patient encounter cases as well as simulation using manikins. Examinees were guided by buzzer sounds for the rotation. Examiners assessed students with rubrics; provided with guidelines for clinical information regarding the case in the particular station.

2011년 8월부터 6차례의 시범 운영이 실시되었으며, 초기에는 1개 의과대학이 참여하여 2012년 말에는 44개 의과대학이 참여했습니다. 시험 센터에서 2단계(임상술기 평가)를 실시하는 미국과 달리, 인도네시아에서는 각 의과대학이 해당 시험 기간에 의대 졸업생을 배출한 경우 시험 센터가 되어야 합니다. 즉, 의과대학은 시험에 필요한 시험관, 직원, 시설 및 자원을 갖추어야 합니다. 시험 시행에 필요한 자원은 졸업생 수에 맞게 충분해야 합니다. 
Six pilots were conducted from August 2011, involving one medical school at the beginning to 44 medical schools at the end of 2012. Unlike in the US where the Step 2 (the clinical skills assessment) is conducted in test centres; in Indonesia, each medical school must be a test centre if they had medical graduates in that current period of examination. This means that medical schools must have the examiners, staff, facilities, and resources needed for the examination. The resources needed to deliver the examination should be sufficient to suit the number of graduates.

NLE의 일부로 OSCE를 시행하는 것은 2013년 고등교육부 고등교육국장의 법령에 명시되어 있습니다. 이 법령에 따르면 NLE는 컴퓨터 기반 MCQ와 OSCE로 구성되며, NLE는 학부 교육이 끝날 때 졸업 시험의 역할을 합니다. 처음 두 차례(2013년 2월과 5월)에 걸쳐 실시된 OSCE는 교육적 목적의 평가였습니다. 2013년 8월부터 OSCE는 필기 시험과 함께 종합적인 목적으로 사용되었습니다. 의대생은 두 시험을 모두 통과해야 의과대학을 졸업할 수 있습니다. 시험에 합격한 학생은 인도네시아 의학위원회로부터 역량 인증서를 받고 의과대학을 졸업할 수 있습니다. 이 증명서는 인도네시아 보건부로부터 의사 면허를 취득하는 데 필요합니다. 시험에 불합격한 학생은 재시험에 응시해야 하며, 의과대학은 이들을 위한 재교육 프로그램을 제공해야 합니다. 2014년 1월부터 보건부 고등교육국장은 의과대학의 NLE 합격률과 인증을 규제하는 법령을 제정하여 다음 학년도 신입생 최대 정원을 결정했습니다. 이 법령은 전임상 및 임상 교육 단계의 교사와 학생 비율의 균형을 맞추기 위한 것이었습니다. 이 법령은 일부 의과대학의 행태로 인해 촉발되었습니다. 예를 들어, C-인증을 받은 한 학교는 교사가 100명 미만인데도 연간 400명의 학생을 수용했습니다(HPEQ, 2013).  
The implementation of the OSCE as part of the NLE was described in the 2013 decree by Higher Education General Director of the MoHER. It stated that the NLE consists of computer-based MCQ and an OSCE; and the NLE serves as an exit exam at the end of undergraduate education. In the first two periods of the OSCE as the NLE (February and May 2013), the assessment was for formative purposes. Starting in August 2013, the OSCE served summative purposes, alongside the written examination. Medical students must pass both
examinations before they can graduate from medical school. Students who pass the examination gain a certificate of competence from the Indonesian Medical Council and graduate from medical schools. This certificate is required for a licence of practice from the MoH. Students who fail the examination must retake the examination and medical schools must provide remediation programmes for them. Starting in January 2014, the Higher Education General Director under the MoHER established a decree that regulates the passing rate of medical schools in NLE and their accreditation to determine the maximum quota for new students in the next academic year. This decree was meant to balance the ratio of teachers and students in preclinical and clinical phases of education. This decree was precipitated by the behaviour of some medical schools. For example, a C-accredited school accepted 400 students per year when they had less than 100 teachers (HPEQ, 2013).

이로 인해 합격률이 낮고 인증 수준이 낮은 의과대학들 사이에서 우려의 목소리가 높았습니다. A 인증 의과대학은 NLE 합격률이 90% 이상인 경우 최대 250명의 학생을 수용할 수 있었습니다. 반면, C 인증 의과대학은 NLE 합격률이 90% 이상인 경우 100명, 50% 미만인 경우 50명의 학생만 수용할 수 있었습니다. 이 규칙을 위반하는 의과대학(또는 대학)에 대해서는 보건복지부가 제재를 가하고 있습니다. 학생들의 등록금이 주 수입원인 사립학교의 경우, 이는 심각한 문제를 야기할 수 있습니다. 
This caused worries among medical schools that had lower passing rates and low levels of accreditation. The A-accredited medical schools could have a maximum of 250 students if they had a 90%+ passing rate in the NLE. Meanwhile, the C-accredited schools could only accept 100 students if they had a 90%+ passing rate in the NLE, and 50 students if they had less than 50%. There are sanctions from the MoHER for medical schools (or universities) that violate this rule. For private schools, whose main income is student’s tuition fees, this might raise significant problems.

인도네시아에서 NLE를 도입하고 그 일환으로 OSCE를 시행하는 것은 NLE를 시행한 다른 국가들의 경우와 마찬가지로 의학교육에 상당한 영향을 미칠 것으로 보입니다.
In Indonesia, the introduction of the NLE and the implementation of the OSCE as part of it, are likely to generate a significant impact on medical education, as has been the case for other countries that have implemented the NLE.

3.4 NLE의 결과: 현재의 논쟁
3.4 The consequences of the NLE: current debate

Kane(2014)이 제안한 평가의 타당성에는 결과 영역이 포함됩니다. 즉, 시험 점수의 해석을 뒷받침하는 증거가 있어야 하며, 평가의 결과에 대한 증거가 있어야 합니다. 평가의 타당성 정도는 개입으로서의 영향에 대한 증거를 포함하여 증거가 얼마나 강력한지에 따라 달라집니다(Kane, 2014). 면허 시험은 실무에 필요한 지식, 기술, 판단력을 갖춘 응시자만이 시험에 합격할 수 있도록 함으로써 대중을 보호하는 역할을 합니다. 시험 점수가 향후 업무 수행 능력과 상관관계가 있다고 가정하면 시험 점수가 낮은 수험생이 공공에 위협이 될 수 있다고 생각할 수 있습니다. 그러나 시험 점수가 높다고 해서 반드시 좋은 실무자가 되는 것은 아닙니다. NLE의 타당성은 시험 점수에만 의존하는 것이 아니라 이해관계자에게 미치는 영향도 고려해야 합니다. 
The validity of assessment, as proposed by Kane (2014), includes the consequences domain: there should be evidence that supports the interpretation of test scores; meaning there must be evidence of the consequences of the assessment. The degree of any assessment’s validity depends on how strong is the evidence, including the evidence of its impact as an intervention (Kane, 2014). The licensing examination works as a protection to the public by ensuring that only candidates who have the necessary knowledge, skills, and judgement for practice, pass the test. It could be assumed that the test score correlates with future performance, so that students with low test scores could pose a threat to public. However, it does not necessarily mean that those who have higher test
scores will be good practitioners. The validity of the NLE does not solely rely on test scores, but also its consequences for stakeholders.

다우닝의 프레임워크를 사용하여 체계적 문헌고찰을 수행한 Archer 등(2016)이 설명한 바와 같이, NLE의 결과는 참가자, 의과대학, 규제기관, 정책 입안자 또는 더 넓은 사회에 미칠 수 있으며, 의도적이거나 의도하지 않았거나, 유익하거나 해로울 수 있습니다(Archer 등, 2016a). NLE의 영향은 의료 시스템에만 국한되지 않고 의학교육 시스템에도 영향을 미친다는 점에 유의하는 것이 중요합니다. NLE의 결과에 대한 몇 가지 연구가 있었지만 이 분야에 대한 지식은 제한적입니다. Archer 등이 GMC(2016)를 대상으로 실시한 체계적 문헌고찰에서는 수험생의 과거 및 미래 성과, 환자 결과 및 불만과의 관계, 국내 졸업생과 해외 졸업생 간의 성과 차이 등 세 가지 영역의 결과를 조사했습니다. 
As described by Archer, et al. (2016), who used Downing’s framework to conduct a systematic review, the consequences of NLEs may fall on participants, medical schools, regulators, policy makers, or wider society; and they can be intended or unintended, beneficial or harmful (Archer et al., 2016a). It is important to note that the impact of NLEs will not be limited to the healthcare system, but also to the medical education system. There have been some studies of the consequence of NLEs but knowledge in this area is limited. The systematic review conducted by Archer et al. for the GMC (2016) looked into three areas of consequences: prior and future performance by examinees, relationship to patient outcomes and complaints, and variation in performance between home and international graduates.

대부분의 연구에 따르면 학교 평가에서 우수한 학생은 NLE에서도 우수한 성적을 거둘 수 있으며(Hecker and Violato, 2008), NLE 결과는 대학원 평가에서 더 나은 성과를 예측하는 것으로 나타났습니다(Thundiyil 외., 2010; Miller 외., 2014; Yousem 외., 2016). 그러나 Archer 등이 지적했듯이 의과대학의 의학교육에 대한 다른 접근 방식이 결과에 영향을 미칠 수 있습니다(Archer 등, 2016a). 그의 검토에 따르면 NLE의 결과로 환자 예후가 개선되었다는 증거가 부족하다고 합니다. NLE의 개입이 더 나은 환자 치료로 이어질 수 있다는 명확한 증거는 없습니다. 연구에 따르면 NLE의 성과와 환자의 불만 비율 사이에는 상관관계가 있는 것으로 나타났습니다(Tamblyn 외., 2007). 이는 인과관계를 설명하는 것이 아니라 환자 치료에 대한 NLE의 예측 가치가 있음을 보여줄 뿐이었습니다. 그러나 Archer의 검토에서는 이러한 연구가 NLE를 지지하는 강력한 논거를 제공한다는 점을 인정했습니다(Archer et al., 2016a). 
Most of the studies found that students who excelled in schools’ assessment would do well in NLEs (Hecker and Violato, 2008) and the NLE results predicted better performance in postgraduate assessment (Thundiyil et al., 2010; Miller et al., 2014; Yousem et al., 2016). However, as Archer et al. pointed out, the different approach to medical education in the medical schools might affect the results (Archer et al., 2016a). His review also revealed that there is the lack of evidence for the improvement of patient outcome as an NLE consequence. There is no clear evidence that the intervention of NLEs could lead to better patient care. The studies showed there was a correlation between performance in the NLE and rate of complaints made by patients (Tamblyn et al., 2007). This did not explain the causation; it only showed that there is a predictive value of the NLE on patient care. However, it was acknowledged in Archer’s review that these studies provided a strong argument in favour of NLEs (Archer et al., 2016a).

NLE의 타당성에 기여하는 NLE의 영향은 환자 치료와 의사의 임상 성과 영역에만 국한되지 않습니다. NLE가 교육에 미치는 영향도 중요하지만, 이 영역에 대한 증거는 매우 제한적입니다. 대부분의 연구는 임상 술기 평가의 NLE 구성 요소로 인한 임상 술기 커리큘럼 및 평가의 변화를 설명했습니다. 미국에서는 USMLE의 2단계 CSA가 임상 술기 교육의 변화를 주도했습니다. 의학 커리큘럼, 특히 자체 임상 술기 평가에 미치는 영향은 많은 학교가 의학교육에서 임상 술기의 중요성을 바라보는 시각을 바꾼 것으로 나타났습니다(Hauer 외, 2005; Hauer 외, 2006). 대부분의 학교는 의사소통 능력에 중점을 두고 종합적인 임상 술기 평가를 실시합니다(Hauer et al., 2005). Archer 등(2016)은 미국과 캐나다와 같은 기존 시스템에서는 임상 술기의 중요성이 부각되면서 의과대학의 임상 술기 교육에 집중하여 전국적으로 덜 자주 가르치는 특정 술기에 대한 필요성을 해결하고 있다고 강조했습니다. 
The impact of the NLE, which contributes to its validity, is not limited to the area of patient care and clinical performance of a doctor. NLEs’ consequences on education are also important, however, the evidence in this area is very limited. Most of the studies described changes in clinical skills curricula and assessment as a result of the NLEs’ component of clinical skills assessment. In the US, the Step 2 CSA of USMLE drove changes in clinical skills education. The impact on medical curricula, especially in-house clinical skills assessments, showed that many schools changed how they viewed the importance of clinical skills in medical education (Hauer et al., 2005; Hauer et al., 2006). Most schools conduct comprehensive clinical skills assessment with an emphasis on communication skills (Hauer et al., 2005). Archer et al. (2016) highlighted that in the established system, like the USA and Canada, the emerging importance of clinical skills was used to focus medical schools’ clinical skills teaching to address the need for specific skills which were less frequently taught nationwide.

의학교육의 변화가 비교적 최근이고 OSCE가 비교적 새로운 아시아 국가에서는 NLE의 일부로 도입하는 것이 어려운 도전이 될 수 있습니다. 대만의 경우, Lin 등(2013)이 설명한 바와 같이 임상시험의 난이도가 높아지면서 임상술기 평가의 사용이 증가하고 병원 내 임상술기 교육 시설이 개선되었습니다. 이 연구진은 설문지를 통해 OSCE 프로그램이 활성화된 교육 병원을 조사하여 OSCE 시행과 그 구성 요소에 대한 정보를 얻었습니다. 그 결과 교육 및 시험실, 모의 환자(SP), 임상 술기 평가를 위한 케이스 개발 수가 모두 증가했다는 사실을 발견했습니다. 그러나 교육이나 평가에 사용되는 병원 공간, 직원, SP 등의 한계도 확인했으며, 시험 시행에 필요한 자원이 충분한지에 대한 우려도 제기했습니다. 이러한 문제에도 불구하고 이 연구는 의료 수련 기관에서 NLE에 대한 강력한 지지를 나타냈습니다(Lin et al., 2013). 마찬가지로 한국에서도 OSCE 도입으로 임상술기 교육 커리큘럼, 평가, 시설 등이 개선되었다는 연구 결과가 있었습니다(Kim, 2010; Park, 2012; Ahn, 2014). 
In Asian countries, where changes in medical education are more recent and the OSCE is relatively new, its introduction as part of the NLE can be a daunting challenge. For Taiwan, as explained by Lin et al. (2013), the high stakes clinical examination drove the increasing use of clinical skills assessments and the improvement of clinical skills teaching facilities in hospitals. They investigated teaching hospitals with active OSCE programs using questionnaires to gain information about OSCE implementation and its components. They found that the number of rooms for training and examination, simulated patients (SP), and case development for clinical skills assessment all increased. However, they also identified limitations: hospital spaces used for teaching or assessment, staff, and SPs, raising the concern of whether there were sufficient resources to establish the examination. Despite these issues, the study indicated strong support from medical training institutes toward a NLE (Lin et al., 2013). Similarly, studies in South Korea also indicated that the introduction of OSCE drove improvement in clinical skills teaching curricula, assessment, and facilities (Kim, 2010; Park, 2012; Ahn, 2014).

문헌에 나타난 NLE의 긍정적인 결과와 부정적인 결과로 요약되는 이러한 상반된 의견은 아래 표에 요약되어 있습니다: These contrasting opinions, summarised as positive and negative consequences of the NLE from the literature are summarised in the table below:

지난 10년 동안 추가 연구가 수행되었는데, 대부분 의학교육과 의료 시스템이 인도네시아와 같은 개발도상국과 다른 선진국에서 데이터를 가져왔습니다. Archer 등(2016)이 GMC에 대한 검토에서 언급했듯이, 곧 도입될 영국의 MLA는 영국과 유사한 특성을 공유하는 다른 국가의 NLE(인간개발지수가 높고 의학교육 및 보건의료 시스템이 유사한 선진국)와 비교할 수 있습니다. 이를 통해 NLE를 둘러싼 담론에서 개발도상국에서의 실행 및 영향과 관련된 격차가 있음을 확인할 수 있습니다. 
Further studies have been conducted in the last decade, most of which draw their data from developed countries, where both medical education and the health care system differs from those in developing countries such as Indonesia. As Archer, et al. (2016) stated in his review for the GMC, the upcoming MLA in the UK could be compared with NLEs in other countries sharing similar characteristics with the UK: highly developed countries with a high human development index, similar systems of medical education and health care. This confirms gaps in the discourse surrounding the NLE to do with its implementation and impact in developing countries.

인도네시아에서는 2007년부터 SKDI를 '표준'으로 도입하고 NLE를 통해 커리큘럼을 변화시켜 역량 기반 커리큘럼으로 이끌었습니다. 이러한 혁신에 대한 연구는 제한적이며 대부분의 문헌은 인도네시아 문화와 이해관계자의 고유한 특성을 다루지 않았습니다. 국가 위원회에서 수행한 연구는 시험의 타당도와 신뢰성 요소에 초점을 맞추었습니다.
In Indonesia, the introduction of SKDI as the “standard” and the NLE drove curriculum changes from 2007 leading to the competence-based curriculum. There is limited research on these innovations and most of the literature has not covered the unique characteristics of Indonesian culture and stakeholders. The studies carried out by the national committee focussed on the validity and reliability component of the examination.

인도네시아의 의학교육과 의료 시스템 이해당사자들에게 NLE가 미친 영향에 대해서는 알려진 바가 거의 없습니다. 소규모 연구에 따르면 NLE가 학생 학습의 질과 학생의 메타인지 조절에 영향을 미쳤다고 합니다(Firmansyah 외., 2015). 그러나 교사는 교육의 예상 결과를 학습 목표로 해석하여 학생들에게 전달해야 하므로 NLE는 교사의 수업과 평가를 수정할 수 있는 잠재력을 가지고 있습니다. 마찬가지로, 이는 학생들이 시험과 관련된 방식에 영향을 미치고 의과대학이 정책 및 교육 관행에 필요한 변화를 파악하도록 유도할 수 있습니다. 그러나 NLE를 경험한 사람들에게 이러한 영향이 구체적으로 어떤 영향을 미쳤는지에 대해서는 알려진 바가 거의 없습니다. 따라서 인도네시아의 매우 다양한 의과대학 시스템에서 NLE가 학생의 학습, 교사의 개발, 의과대학의 정책에 어떤 영향을 미쳤는지 이해하는 것이 중요합니다. 
Little is known about the consequences of the NLE on medical education in Indonesia and the stakeholders in the health care system. A small scale study proposed that the NLE affected the quality of student learning and students’ metacognitive regulation (Firmansyah et al., 2015). However, as teachers have to interpret the expected outcome of education into learning objectives and deliver it to students the NLE has the potential to modify their teaching and assessment. Similarly, this would affect how students relate to the examination and lead medical schools to identify changes needed in their policy and educational practice. However, very little is known about the details of this impact on those who experienced the NLE. It is, therefore, important to understand how the NLE affected students’ learning, teachers’ development, and medical schools’ policy in the very diverse system of medical schools in Indonesia.

  • 따라서 이 연구는 인도네시아의 문화와 이해관계자 및 그들의 특성이 NLE 시행의 결과에 어떤 영향을 미칠 수 있는지 인식하면서 인도네시아에서 NLE의 영향을 이해하는 데 중점을 두었습니다.
    Consequently, this study focussed on understanding the impact of the NLE in Indonesia, recognising how the culture and the stakeholders and their characteristics might affect the consequences of implementing the NLE.

 


 

베트남의 의사면허시험에 대한 코멘터리(MedEdPublish, 2023)
A commentary on the National Medical Licensing Examination in Vietnam: why, what, who and how [version 2;
peer review: 2 approved] 
Thuy Minh Ha

소개
Introduction

베트남의 의과대학 수는 사회적 수요 증가와 경제 발전의 결과로 1997년 이후 두 배로 증가했습니다(세계보건기구, 2021). 공립과 사립 모두 29개의 의과대학이 있으며, 매년 8천 명 이상의 학생이 졸업하는 것으로 추산됩니다(Duong et al., 2021). 보다 엄격한 품질 보증 통로가 필요하기 때문에 새로 졸업한 의사의 역량에 대한 우려가 있습니다. 현재 의료 교육에 대한 구체적인 인증 기준은 존재하지 않습니다(Ha & Siddiqui, 2022). 의료 면허 시험은 아직 전국적으로 의대 졸업생에게 의무화되어 있지 않습니다. 의료 행위 면허는 현재 의사의 역량에 대한 평가보다는 과거 진료 기록에 대한 검토를 기반으로 합니다.

  • 의료 행위 면허 신청자는 의사로서 최소 18개월의 실무 경력이 있어야 하며, 이 기간에 대한 서면 증명서를 제출해야 합니다(정부, 2022). 2024년 이전에는 면허를 갱신할 필요 없이 영구적인 것으로 간주됩니다(정부, 2022).

The number of medical schools in Vietnam has doubled since 1997 as a result of growing societal demands and economic development (Word Health Organization, 2021). There are 29 medical schools in both public and private sectors, with an estimated more than eight thousand students graduating yearly (Duong et al., 2021). There is concern about the competency of the newly graduated physicians due to the need for more rigorous quality assurance corridors. As of today, specific accreditation standards do not exist for medical training (Ha & Siddiqui, 2022). Medical licensure exam is not yet required for medical graduates throughout the country. Medical practice licenses are currently based on the review of historical practice documents rather than an evaluation of physicians' competency.

  • An applicant for a medical practice license must have a minimum of 18 months of practice as a physician, with a written certification of this period (Government, 2022). Before 2024, the license is considered permanent, with no requirement for renewal (Government, 2022).

이는 각 의과대학이 졸업생 결과에 대한 책임이 있다는 것을 의미하며, 그 결과 의료 교육의 질에 대한 주요 우려가 제기되고 있습니다. 이론 강의가 많은 전통적인 학문 기반 접근 방식은 대부분의 의과대학에서 주로 사용되는 교육 방식입니다(Fan et al., 2012). 의학 교육에 대한 커리큘럼 프레임워크 요건은 2012년에 발표되어 업데이트를 촉구했습니다(교육훈련부, 2012). 2015년에 일반의에 대한 졸업역량이 승인되었지만, 졸업역량이 교육과정에 어떻게 통합되는지에 대해서는 더 많은 데이터가 필요합니다(보건부, 2015). 다양한 의학 커리큘럼이 제공되며 시험 및 대학원 연구 논문과 관련된 졸업 요건도 다양합니다. 지난 몇 년 동안 다양한 의학 교육 포럼에서 국가 의사 면허 시험(NMLE)에 대한 전망이 자주 논의되었습니다. NMLE는 의사가 임상 진료에 들어가기 전에 최소한의 역량을 갖추도록 하여 환자 치료 결과를 개선하는 데 중요한 역할을 할 수 있습니다. 나아가 커리큘럼 개혁의 원동력이 되어 의학교육 시스템을 개선할 수 있을 것으로 기대됩니다. 이 백서에서는 베트남에 NMLE 도입이 필요한 이유, 설립 과정에서 고려해야 할 사항, 책임 기관, 교육자 및 정책 입안자에게 좋은 사례를 배우고 활용할 수 있는 방법에 대해 간략하게 설명합니다. 
It implies that each medical school is responsible for the graduate outcome, resulting in the major concern about medical training quality. A traditional discipline-based approach with heavy theoretical lectures is the primary method of teaching in most medical schools (Fan et al., 2012). The curriculum framework requirement for medical training was issued in 2012 and urged to be updated (Ministry of Education and Training, 2012). Graduated competencies were endorsed for general practitioners in 2015, however, more data is needed regarding how they are incorporated into the curriculum (Ministry of Health, 2015). A wide variety of medical curricula are available, as well as different graduate requirements pertaining to exams and graduate research theses. Over the past few years, the prospects for the National Medical Licensing Exam (NMLE) have been frequently discussed in a variety of medical education forums. An NMLE could play an important role in ensuring that physicians possess a minimum level of competency prior to entering clinical practice, thereby improving patient outcomes. Furthermore, it is expected to be a driver of curriculum reform, which will subsequently improve the medical education system. This paper discusses briefly why the introduction of NMLE is necessary for Vietnam, what should be considered during establishing process, who could be the responsible organization, and how good practices can be learned and used as personal recommendations for educators and policymakers.

베트남에 NMLE를 설립해야 하는 이유는 무엇인가요?
Why is it necessary to establish NMLE in Vietnam?

동남아시아국가연합(ASEAN) 회원국 중 하나인 베트남은 2009년에 의료 전문가가 필요한 전문 의료 교육을 이수하고 자국에서 전문 의료 자격을 취득한 경우 상대국에서 진료할 수 있도록 하는 상호인정협정(MRA)을 체결했습니다(ASEAN, 2009). 이 협정의 회원국들은 의료 전문가의 이주에 대한 규정을 마련해야 했습니다. 그 결과 태국, 필리핀, 인도네시아, 말레이시아, 라오스 등 아세안 국가 10개국 중 5곳이 NMLE를 조직했습니다(Kittrakulrat 외, 2014; Nur Hidayah 외, 2020; Sonoda 외, 2017). 주변 국가에서 얻은 경험과 사례는 크게 달랐습니다. 대상 인구(외국인 의사 대 국내 졸업생), 언어의 다양성(현지어 대 부분 영어), 의료 전문가가 역내 어느 국가에서든 자유롭게 의료 활동을 할 수 있도록 하는 데 어려움을 주는 문제 등 NMLE 제도에 대한 몇 가지 우려 사항이 확인되었습니다(키트라쿨랏 등, 2014). 그러나 이는 세계보건기구(WHO)의 보편적 의료보장 전략에 따라 아세안에서 비슷한 수준의 환자 치료 및 의학교육을 달성하기 위한 첫걸음으로 여겨지고 있습니다(누르 히다야 외., 2020). MRA 목표가 달성되면 역내 고도로 숙련된 의사의 고용 가능성이 크게 개선될 것입니다. 외국인 의사와 해외에서 수련을 받은 의사들이 더 자유롭게 입국할 수 있게 되어 의료 종사자 수가 증가하여 베트남의 의료 전문가 부족 문제가 개선될 것입니다.
As one of the Association of Southeast Asian Nations (ASEAN) members, Vietnam signed a mutual recognition agreement (MRA) in 2009 that allowed healthcare professionals to practice in a host country if they had completed the required professional medical training and received the professional medical qualification in their homeland (ASEAN, 2009). Members of this agreement were required to establish regulations for the migration of healthcare professionals. As a result, five out of 10 ASEAN countries have organized NMLEs, including Thailand, Philippines, Indonesia, Malaysia, and Laos (Kittrakulrat et al., 2014; Nur Hidayah et al., 2020; Sonoda et al., 2017). Experiences and practices gained from neighboring countries differed greatly. Several concerns about the NMLE system were identified including the targeted populations (foreign doctors versus domestic graduates); language variation (local language versus partial English); which poses challenges to the free flow of medical professionals to practice medicine in any of the region's countries (Kittrakulrat et al., 2014). It is, however, believed to be the first step towards achieving comparable standards of patient care and medical education in ASEAN, in accordance with the World Health Organization (WHO) strategy for universal health coverage (Nur Hidayah et al., 2020). The employability for highly skilled physicians across the region will be significantly improved if the MRA goal is achieved. Foreign doctors and overseas trained doctors will be able to enter the country more freely, resulting in an increased number of medical practitioners, thus improving the shortage of healthcare professionals in Vietnam.

베트남에 NMLE가 설립되기 위해서는 여러 가지 과제가 있습니다. 

  • 베트남에서 가장 주목할 만한 문제 중 하나는 의과대학마다 준비 수준이 다르다는 점입니다
    • 많은 기관이 역량 기반 의학교육으로의 개혁적 변화를 수용하고 있는 반면, 
    • 다른 기관은 이러한 변화를 채택하는 데 필요한 문헌, 증거 또는 동기가 부족하여 여전히 상당한 결함에 직면해 있습니다(Tran et al., 2022). 
  • 자원 소비, 불충분한 타당성 및 신뢰성, NMLE와 환자 치료 간의 직접적인 연결 부족 등 NMLE와 관련된 수많은 과제가 있지만여전히 이점이 위험보다 큽니다(Nur Hidayah 외, 2020)(Wang, 2022). 
  • 최근 주변국의 경험에 따르면 NMLE는 정책 변화 동기 부여, 커리큘럼 및 평가 관행 재검토, 학습 자원 및 시설 개선, 의사 역량 향상을 위한 교육 네트워크 간 협력 강화 등 긍정적인 영향을 미쳤습니다(Nur Hidayah 외, 2020).

For the NMLE to be established in Vietnam, there are a number of challenges.

  • It is obvious that one of the most noteworthy issues in Vietnam could be the varying levels of readiness among different medical universities.
    • A number of institutions are embracing a reformative shift towards competency-based medical education,
    • while others still face substantial deficiencies, lacking the necessary literature, evidence, or motivation to adopt such a change (Tran et al., 2022).
  • Although there are numerous challenges associated with the NMLE, such as resource consumption, insufficient validity, and reliability, and a lack of direct connection between the NMLE and patient care, the benefits still outweigh the risks (Nur Hidayah et al., 2020) (Wang, 2022).
  • According to recent experiences from neighboring countries, NMLEs have had positive impacts, such as motivating policy changes, revisiting curriculums and assessment practices, enhancing learning resources and facilities, and strengthening collaboration between educational networks in order to improve physician competence (Nur Hidayah et al., 2020).

NMLE는 의대생에게 장벽을 만들고 자격을 갖춘 의사의 수를 제한하는 단점이 있을 수 있습니다. 반면, 질 관리 측면에서 NMLE는 국가별 의과대학과 지역 내 잠재력 수준을 벤치마킹하는 데 사용할 수 있는 도구가 될 수 있으며, 이를 통해 의과대학이 커리큘럼을 개선하고 질 향상을 위해 지속적으로 노력하도록 장려할 수 있습니다(Swanson & Roberts, 2016). 
NMLE may have the disadvantage of creating a barrier for medical students and limiting the number of qualified physicians. On the other hand, when it comes to quality assurance, NMLE can be a tool that can be used to benchmark medical schools across countries and levels of potential across the region, thereby encouraging them to improve their curricula and continuously strive for quality improvement (Swanson & Roberts, 2016).

설립을 위해 고려해야 할 사항은 무엇인가요?
What should be considered for the establishment?

우선, NMLE 설립을 위한 전략 계획은 국가 법률 체계와 교육 시스템을 준수해야 합니다. 베트남의 의학박사 프로그램은 교육훈련부와 보건부가 감독하는 6년제 학부 교육 프로그램입니다(Ha & Siddiqui, 2022). 건강 검진 및 치료에 관한 법률, 국가 자격 프레임워크, 보건 과학 전문가 교육에 관한 규정을 포함하되 이에 국한되지 않는 여러 규제 문서가 NMLE의 탄생에 영향을 받을 것입니다. 따라서 이해 관계자의 조기 참여와 효과적인 커뮤니케이션 계획이 포함 된 NMLE 도입을위한 포괄적 인 전략 계획은 프로젝트의 성공에 필수적입니다. 아세안 지역 내 의사 인력의 '공정한 교류'를 촉진하기 위해 다른 국가의 의료 자격 제도를 벤치마킹할 것을 강력히 권장합니다(키트라쿨랏 외., 2014).
In the first instance, the strategic plan for the establishment of NMLE must comply with the national legal framework and educational system. The medical doctoral program in Vietnam is a six-year undergraduate training program overseen by the Ministry of Education and Training and the Ministry of Health (Ha & Siddiqui, 2022). Several regulatory documents will be affected by the birth of NMLE, including but not limited to the Law on Medical Examination and Treatment, National Qualification Framework, Regulations on specialist training in the health sciences. Thus, a comprehensive strategic plan for the introduction of NMLE, which involves early involvement of the stakeholders, and an effective communication plan is essential to the project's success. A benchmarking with other countries' medical qualification systems is strongly recommended to facilitate a "fair exchange" of physician workforces within the ASEAN region (Kittrakulrat et al., 2014).

변화에 대한 저항은 NMLE 설립에 대한 대응으로 예측할 수 있습니다. 그럼에도 불구하고 기관의 혜택이 강조된다면 하향식 힘과 기관의 혜택이 균형을 이루게 되어 달성 가능성이 높아질 수 있습니다. 예를 들어, 성과가 낮은 교육기관은 NMLE 결과를 바탕으로 추적하여 개선을 위한 더 많은 지원을 제공함으로써 교육 성과를 개선할 수 있습니다(Saleem & Afzal, 2022). 2015년 현재 일반의에 대한 역량이 승인되었으며, 이는 청사진을 작성하여 NMLE 개발의 첫 출발점이 될 수 있습니다(Ministry of Health, 2015). 따라서 학교가 역량 기반 커리큘럼을 구현하도록 동기를 부여하여 다가오는 NMLE에 대응하여 더 높은 품질을 위해 노력합니다. 시험에 할당된 시간은 추가 시험이 될 것인지 아니면 학교의 기존 시험을 대체하여 교수진과 학생의 업무량을 줄일 것인지 고려해야합니다. 대부분의 의대생이 전문의 수련에 관심이 있다는 점을 고려할 때(Ngo, 2021), NMLE는 졸업 후 해당 과정의 입학 시험으로도 활용될 수 있습니다.  
Resistance to change is predictable in response to the NMLE establishment. Nevertheless, if the benefits of the institutions are emphasized, the top-down forces would be balanced with the institutions' benefits, which could increase achievement chance. As an example, low performing institution can be traced based on the NMLE results and offer more support for remediation, hence improving the educational outcomes (Saleem & Afzal, 2022). As of 2015, the competency for general practitioners has been endorsed, which can serve as the very first starting point for the development of the NMLE by creating the blueprints (Ministry of Health, 2015). Thus, it motivates schools to implement competency-based curricula, striving for higher quality in response to the upcoming NMLE. The time allocated for the exam should be considered if it will be an additional exam or if it will replace the existing tests from the schools, thus reducing workloads for faculty and students. Considering that most medical students are interested in specialist training (Ngo, 2021), NMLE can also serve as an entry test for these courses after graduation.

시험에 사용되는 언어도 고려해야 할 중요한 요소입니다. 10개국 중 7개국에서 시험의 일부 또는 전체에 영어가 사용되고 있습니다(Kittrakulrat 외., 2014). 필리핀 및 말레이시아 모델과 유사하게 영어를 시험에 어느 정도 포함시키는 것은 MRA에 대응하기 위한 옵션이 될 수 있습니다. 이 개념은 영어 능력에 대한 요구가 점점 더 인식되고 있는 베트남의 지속적인 입학 추세에서 더 많은 지지를 받고 있습니다(Nga, 2022). 그럼에도 불구하고 관련 과제는 신중한 고려가 필요하며, 전환 및 준비 계획이 잘 실행되는 것이 이상적입니다. 이러한 과제에는 교수진과 학생 모두 복잡한 의학 개념을 이해하는 데 방해가 될 수 있는 잠재적인 언어 장벽이 포함됩니다(Oducado 외., 2020). 또한, 특히 교육 과정에서 영어에 대한 노출이 제한되어 불이익을 받는 학생들에게는 형평성과 접근성에 대한 우려가 제기됩니다(Chur-Hansen 외., 1997). 또한 부정확하게 전달된 의학 용어는 오해를 불러일으킬 수 있으므로 문화적 민감성을 염두에 두어야 합니다. 이러한 전환이 의료 품질에 미치는 영향에 대한 명확한 이해를 얻으려면 의심할 여지없이 시간이 필요합니다(Price et al., 2018). 궁극적으로 의학교육의 주요 목표는 우수한 환자 치료를 제공할 수 있는 유능한 의사를 양성하는 것입니다. 의학 지식과 기술을 희생하면서 언어 능력에 지나치게 중점을 두면 졸업생이 제공하는 의료 서비스의 질이 저하될 수 있습니다. NMLE에 있어서는 획일적인 접근 방식은 불가능합니다. 이러한 고부담 시험의 경우,

  • (i) 해당 국가 내 의대 졸업생에게만 해당 시험을 통과해야만 면허를 부여할 것인지, 아니면
  • (ii) 해당 관할권 내에서 진료하기 위해 모든 예비 의사 또는 외국 의대 졸업생(IMG)이 시험을 통과해야 하는지 여부도 고려해야 합니다(Price et al., 2018). 두 번째 옵션은 이 활동을 위해 추가 절차가 필요할 수 있지만, NMLE는 의사가 어디서 교육을 받았는지에 관계없이 의사를 평가하기 위한 국가적 최소 기준을 나타냅니다. 

The language used in the examination is also an important factor to consider. In seven out of 10 countries, English is being used for examinations in part or in full (Kittrakulrat et al., 2014). The inclusion of English in this examination to some extent, similar to the Philippines and Malaysian models, could be an option for responding to the MRA. This notion finds further support in the ongoing admission trend in Vietnam, where the demand for English proficiency is increasingly recognized (Nga, 2022). Nevertheless, the associated challenges merit careful consideration, ideally accompanied by a well-executed transition and preparation plan. These challenges encompass the potential language barrier that could hinder the understanding of complex medical concepts by both faculty and students (Oducado et al., 2020). Additionally, it raises concerns about equity and accessibility, especially for students disadvantaged by limited exposure to English during their education (Chur‐Hansen et al., 1997). Furthermore, there's a need to be mindful of cultural sensitivity; inaccurately communicated medical terms might lead to misinterpretations. This transition undoubtedly requires time to yield a clear understanding of its impact on healthcare quality (Price et al., 2018). Ultimately, the primary objective of medical education is to foster capable doctors capable of delivering superior patient care. Should the emphasis shift excessively towards language proficiency at the expense of medical knowledge and skills, the quality of healthcare provided by graduates could be compromised. A one-size-fits-all approach is not possible when it comes to the NMLE. For an examination of this high stakes nature, it should also be considered whether:

  • (i) it is only applicable to graduating medical students within the country to pass the exam before being granted a license to practice, or
  • (ii) all prospective physicians from national jurisdictions or foreign medical graduates (IMGs) are required to pass the exam in order to practice within that jurisdiction (Price et al., 2018). While the second option may require an additional process for this activity, NMLE represents a national minimum standard for evaluating doctors regardless of where they were trained.

베트남의 책임 기관은 어디인가요?
Who could be the responsible organization for Vietnam?

일반적으로 보건부 산하 국가의료위원회, 의료위원회 또는 전문직 협회가 NMLE(전문 의료 규제)를 조직할 책임이 있습니다(Kittrakulrat 외., 2014). 2020년에는 WHO 및 베트남 보건부와 협력하여 베트남 국립의료위원회(VNMC)가 설립되었습니다(베트남 정부 총리, 2020). 이 결정에 따라 VNMC는 의료 전문가의 역량을 정의, 개발, 실행, 모니터링 및 평가할 책임이 있습니다. 베트남 의료위원회의 지도하에 2022년 9월에 전국 의료 전문가 네트워크(베트남 의료 교육 협회 - VAME)가 설립되었습니다. 이러한 발전은 NMLE 개발에 훌륭한 모멘텀을 제공합니다. 또한 여러 국가에서 이 과정을 통한 학습의 중요성을 인식하고 조직/전문기구를 인증 기관의 기반으로 삼고 세계 의학교육 인정 프로그램(세계의학교육연맹)의 인정을 받기 위한 노력을 기울이고 있습니다. 
Generally, the National Medical Council, Medical Board, or Professional Associations under the Ministry of Health are responsible for organizing the NMLE (Professional Medical Regulatory) (Kittrakulrat et al., 2014). In 2020, Vietnam National Medical Council (VNMC) was established in collaboration with WHO and the Vietnam Ministry of Health (Prime Minister of the Government, 2020). According to the decision, the VNMC is responsible for defining, developing, implementing, monitoring, and assessing health professionals' competency. A national network of medical professionals (Vietnam Association for Medical Education – VAME) has been established in September 2022 under the guidance of the Vietnam Medical Council. These developments provide excellent momentum for NMLE development. In addition, several countries have recognized the importance of learning from this process and have created efforts to make the organization / professional body the foundation for accreditation agencies as well as be recognized by the World Federation of Medical Education Recognition Program (Word Federation For Medical Education).

현재 실무에서 어떻게 배울 수 있나요? 
How could it be learned from the current practices?

임상 실무에 진출하고자 하는 의대 졸업생은 최소한의 요건으로 NMLE를 충족해야 할 수도 있습니다. 또한 지속적인 전문성 개발 활동(CME/CPD) 참여에 대한 평가를 바탕으로 5년마다 등록 갱신을 고려할 수도 있습니다. 이는 평생 의학 교육(CME/CPD) 시스템의 활성화에 기여하고 의료 전문가가 자신의 지식과 기술을 최신 상태로 유지할 수 있도록 보장합니다. 
Medical graduates who wish to enter clinical practice could also be required to meet the NMLE as a minimum requirement. Registration renewal can also be considered every five years based on an evaluation of their participation in continuing professional development activities (CME/CPD). This contributes to the promotion of the continuing medical education (CME/CPD) system and ensures that health professionals are up to date on their knowledge and skills.

로버트와 동료들이 제안한 바와 같이, 고부담 시험을 준비하기 위해 취할 수 있는 몇 가지 단계는 다음과 같다.

  • (i) 교육 목표 및 역량 표준의 청사진 마련,
  • (ii) 적절한 시험 형식 선택,
  • (iii) 적절한 신뢰성을 보장하기 위한 평가 전략 적용,
  • (iv) 적절한 표준 설정 및 의사 결정 절차 구현(Roberts et al., 2006).

이집트 의사 면허 시험 로드맵을 검토하면 (i) 시험 물류와 (ii) 시험 설정의 두 가지 주요 섹션으로 구성되어 있기 때문에 NMLE 설립 목적에 도움이 될 수 있습니다 (Nasser 외., 2021).

  • 시험 물류에는 시험위원회 구성, 시험 전제 조건, 입학 기준 및 수수료, 면허의 유효성 등이 포함됩니다.
  • 시험 설정에는 시험 구조, 표준 설정, 합격 점수 정책 및 재설정 정책이 포함됩니다(Nasser et al., 2021).

Several steps can be taken to prepare for high-stakes exams, as suggested by Roberts and colleagues:

  • (i) blueprinting educational objectives and competency standards,
  • (ii) selecting appropriate test formats,
  • (iii) applying assessment strategies to ensure adequate reliability, and
  • (iv) implementing appropriate standard-setting and decision-making procedures (Roberts et al., 2006).

A review of the Egyptian Medical License Exam Roadmap can also benefit for the purpose of NMLE establishment, as it consists of two main sections: (i) exam logistics and (ii) exam set up (Nasser et al., 2021).

  • The logistics of the exam include the composition of the exam committee, the prerequisites for the exam, the admission criteria and fees, and the validity of the license.
  • The exam set up encompasses the examination structure, the standard setting, the pass mark policy, and the reset policy (Nasser et al., 2021).

시험 위원회를 위해 다분야 팀이 제안되었으며 편리한 인프라, 물류 및 인적 자원과 함께 제안되었습니다. 시험 과정의 모든 단계에서 유지되어야 하는 시험의 투명성과 책임성에 대해 동일한 우려를 제기하는 여러 논문이 있습니다(Park & Yang, 2015)(Sloane & Kelly, 2003).

  • 시험 위원회부터 행정 직원에 이르기까지 관련된 모든 이해관계자들은 그들의 행동과 결정에 있어 최대한 투명하게 운영되어야 합니다.
  • 둘째, 견제와 균형의 강력한 시스템을 구축해야 합니다(Roberts et al., 2006).
    • 여기에는 부당한 영향력이나 조작을 방지하기 위해 심사 프로세스 내에서 다양한 기능을 분리하는 것이 포함됩니다.
    • 시험 기준을 설정하는 책임자가 문제를 선택해서는 안 됩니다. 
    • 여러 단계에서 내려진 결정을 교차 검증하고 검증할 수 있는 메커니즘이 마련되어야 합니다.
  • 또한 시험 위원회 구성을 다양화하는 것도 필수적입니다.
    • 다양한 분야와 배경을 가진 전문가를 포함하면 편견을 최소화하고 보다 포괄적인 시각을 키울 수 있습니다(Thompson, 2006).
  • 위원들이 공정성과 객관성을 유지할 수 있도록 엄격한 교육과 명확한 행동 강령을 시행해야 합니다.

A multidisciplinary team was proposed for the exam committee, along with convenient infrastructure, logistics, and human resources. There are multiple articles that raise the same concern regarding exam transparency and accountability that must be maintained at every stage of the examination process (Park & Yang, 2015) (Sloane & Kelly, 2003).

  • All stakeholders involved, from the exam committee to the administrative staff, should operate with the utmost transparency in their actions and decisions.
  • Secondly, a robust system of checks and balances should be established (Roberts et al., 2006).
    • This involves maintaining separation between various functions within the examination process to prevent undue influence or manipulation.
    • Those responsible for setting exam standards should not select questions.
    • There should be mechanisms in place to cross-verify and validate decisions made at different stages.
  • Furthermore, diversifying the exam committee composition remains essential.
    • Including professionals from a variety of disciplines and backgrounds can minimize bias and foster a more comprehensive perspective (Thompson, 2006).
  • Rigorous training and clear codes of conduct should be enforced for committee members to ensure they uphold fairness and objectivity.


새로운 고부담 시험을 시행하려면 신중한 계획이 필요합니다. 새로운 시험 결과를 인정하고 시험 전 졸업생을 처리하는 일정이 명확해야 합니다(Saleem & Afzal, 2022). 커뮤니케이션 전략은 학교와 학생 모두에게 정보를 제공해야 합니다. 피드백에 기반한 협업, 조정 및 콘텐츠 개선을 통한 지속적인 개선이 핵심입니다(Bajammal et al., 2008). 결론적으로, 새로운 고부담 시험으로의 성공적인 전환을 위해서는 세심한 계획과 투명한 커뮤니케이션이 필수적이며, 이는 의학교육과 의료의 질 향상에 도움이 됩니다. 
Rolling out a new high-stakes exam requires careful planning. The timeline for recognizing new results and dealing with pre-exam graduates should be clear (Saleem & Afzal, 2022). Communication strategies should inform both schools and students. Continuous improvement is key through collaboration, adjustments, and refining content based on feedback (Bajammal et al., 2008). In conclusion, meticulous planning and transparent communication are essential for a successful transition to a new high-stakes exam, benefiting medical education and healthcare quality.

결론
Conclusion

베트남에서 국가 의료 면허 시험이 도입되는 데에는 많은 어려움이 있을 수 있지만, 커리큘럼 변화에 긍정적인 영향을 미치고 지역 내 의료 인력의 이동성을 향상시킬 수 있을 것으로 기대됩니다. NMLE 도입은 네트워크 내 모든 이해관계자의 관심이 필요한 시급한 문제입니다. 면허 시스템의 여러 측면을 고려해야 하며 추가 연구가 필요합니다. 이 백서를 발간함으로써 저자는 규제 당국과 정책 입안자들이 국익을 위한 합의에 도달할 수 있도록 논의에 기여하고자 합니다.
The establishment of the National Medical Licensing Exam in Vietnam may present numerous challenges, but it is expected to positively impact curriculum changes and enhance the mobility of the medical workforce in the region. The introduction of an NMLE is a time-critical issue that requires the attention of all stakeholders within the network. Several aspects of the licensing system remain to be considered and further research is required. By publishing this paper, the author hopes to contribute to discussion among regulators and policymakers in order to reach a consensus for the best interests of the country.


Abstract

As a result of increasing societal demands and economic development, the number of medical schools in Vietnam has increased significantly over the past decade. In order to ensure physician competency, it is imperative that medical training meets a minimal threshold before entering clinical practice. The prospects of the National Medical Licensing Exam (NMLE) have been discussed extensively and are expected to be instrumental in influencing curriculum reform, thus enhancing the quality of medical education. This paper discusses briefly why NMLE is necessary for Vietnam, what should be considered when establishing it, who could be the responsible organization, and how good practices can be learned and used as personal recommendations for educators and policymakers.

베트남 의학교육의 혁신(BMJ Innov 2021)
Innovations in medical education in Vietnam
David B Duong ,1,2 Tom Phan ,3 Nguyen Quang Trung,4 Bao Ngoc Le,4 Hoa Mai Do,4 Hoang Minh Nguyen,4 Sang Hung Tang,4 Van-Anh Pham,5 Bao Khac Le,6 Linh Cu Le,7 Zarrin Siddiqui,7 Lisa A Cosimi,2,8 Todd Pollack9

 

 

소개
Introduction

현재 증거에 따르면 저소득 및 중저소득 국가(LMIC)에서는 진화하는 역학적 질병 패턴, 인구 통계 변화, 새로운 전염병 건강 위협, 기후 변화의 건강 영향 등 현재와 미래의 인구 요구를 충족하기 위해 의료 시스템 개선이 필요합니다.1 의학교육 개혁은 이러한 노력의 필수 요소이며, 양질의 의사 인력을 모집, 교육, 유지 및 분배하는 데 어려움을 겪고 있는 LMIC의 중요한 관심 분야입니다.2 3 2010년, 랜싯 위원회는 '의료 전문 교육을 혁신하는 새로운 세기'를 촉구하며 특히 LMIC의 필요성에 중점을 두었습니다.4 랜싯 보고서는 의료 교육을 개혁하기 위한 프레임워크와 권고 사항을 제공하고 필요한 정치적, 재정적, 리더십 약속을 포함하여 전 세계 의료 교육을 혁신하기 위한 행동 촉구입니다.4 이 야심찬 목표를 달성하려면 정책 입안자와 교육 지도자들이 의학교육의 설계 및 제공에 새롭고 혁신적인 접근법을 찾고 적용해야 합니다.5 6 
Current evidence suggests that, in low-income and middle-income countries (LMICs), healthcare system improvements are necessary to ensure healthcare services can meet current and future population needs, including evolving epidemiological disease patterns, shifting demographics, new infectious disease health threats and the health impacts of climate change.1 Medical education reform is an integral component of any such effort, and is an area of significant concern for LMICs, which struggle with recruiting, training, retaining and distributing a high-quality physician workforce.2 3 In 2010, a Lancet commission called for ‘a new century of transformative health professional education’, with a particular focus on the needs of LMICs.4 The Lancet report is a call to action to transform healthcare education worldwide by providing a framework and recommendations to reform healthcare education, including much needed political, financial and leadership commitments.4 Achieving this ambitious goal requires policymakers and educational leaders to find and apply novel and innovative approaches to the design and delivery of medical education.5 6

베트남의 의학교육
Medical education in Vietnam

동남아시아의 LMIC인 베트남의 의학교육 시스템(그림 1)은 최근까지 사회 대부분의 다른 부분과 환자의 건강 요구가 크게 발전했음에도 불구하고 거의 변하지 않았습니다.7 이전의 의학교육 개혁 노력은 산발적이고 특정 학과의 단일 의과대학에 국한되어 있었으며 포괄적이거나 광범위하지 않았습니다.8 1999년부터 베트남 전역의 의학교육자들은 학부 의학교육(UME)에 대한 지역사회 지향성을 촉진하기 위해 협력하여 의과대학 졸업생에게 기대되는 지식, 태도 및 기술의 형태로 학습 목표와 결과를 확인했습니다.9 10 이는 보건부(MOH)가 2015년에 일반 의사를 위한 최초의 표준 역량 세트를 발표할 수 있는 토대가 되었습니다. 그러나 그 실행은 제한적이었습니다.8 
Vietnam, an LMIC in Southeast Asia, has a medical education system (figure 1) which, until recently, had changed little despite substantial advancement in most other parts of society and in the health needs of patients.7 Previous medical education reform efforts have been sporadic and limited to single medical universities in specific departments, and not comprehensive or widespread.8 Starting in 1999, medical educators across Vietnam collaborated to promote a community orientation to undergraduate medical education (UME), and identified learning objectives and outcomes in the form of the knowledge, attitude and skills expected of a medical school graduate.9 10 This was the groundwork for the Ministry of Health (MOH) to issue the first set of standard competencies for general doctors in 2015. However, their implementation has been limited.8


현재 베트남에는 29개의 의과대학이 있으며, 각 학교당 연평균 400~600명의 의대생이 입학하고 있습니다.11 UME는 중등교육 이수 후 4~6년 과정으로 운영됩니다.7 대부분의 프로그램은 6년 과정으로, 초기 임상 노출을 최소화한 기초과학 2년과 나머지 4년 동안 임상 이론과 국립 및 지방 병원을 통한 병원 로테이션이라는 전통적인 형식으로 구성됩니다.7 UME를 수료하면 졸업생은 크게 두 가지 경로로 진로를 선택할 수 있습니다.

  • 첫 번째는 프랑스의 경쟁형 인턴 제도를 모델로 한 레지던트 트랙입니다.12 레지던트 트랙은 주요 학술 의료 센터에서 진행되며 전문과목과 하위 전문과목 프로그램이 모두 포함됩니다. 현재 베트남에서는 졸업생의 약 10%가 레지던트 트랙에 입학합니다.
  • 나머지 졸업생들은 다양한 병원에서 스스로 일자리를 찾아 18개월 동안 도제식 인턴십에 들어가며, 그 후 지방 보건부에 의사 면허를 등록할 수 있습니다.

인턴십을 마치고 의사 면허를 취득한 의사는 수련 분야에서 진료를 하거나 즉시 추가 전문의 수준('추옌 코아'(CK)) 교육(전문의 레벨 1 또는 CK I)을 받은 후 세부 전문의 교육(전문의 레벨 2 또는 CK II)을 더 받을 수 있습니다. CK I과 CK II 모두 완료하는 데 평균 2년이 걸립니다. 레지던트 과정을 마친 의사는 의사 면허를 취득한 후 진료를 하거나 바로 CK II 교육에 들어갈 수 있습니다. 현재 면허 취득 전 국가 시험은 없지만 가까운 시일 내에 시행할 계획이 수립되어 있습니다.7
Currently, there are 29 medical universities in Vietnam, with an average of 400–600 medical students matriculating per year at each school.11 UME is a programme of 4–6 years following the completion of secondary education.7 Most programmes are 6 years in duration and are organised in a traditional format of 2 years of basic science with minimal early clinical exposure, followed by clinical theory and hospital rotations through national and provincial hospitals in the remaining 4 years.7 On completion of UME, there are two major pathways for graduates.

  • The first is the residency track, modelled after the French competitive interne des hôpitaux system.12 These take place at major academic medical centres and include both specialty and subspecialty programmes. Currently, approximately 10% of graduates enter the residency track in Vietnam.
  • The remaining graduates find their own placement at various hospitals and enter apprenticeship-style internships for 18 months, after which they are eligible to register for a medical licence from a provincial department of health.

After completing internship and attaining a medical license, physicians can practice in the area of their training or immediately pursue further specialist-level ('chuyên khoa' (CK)) training (specialist level 1 or CK I) and then further subspecialist training (specialist level 2 or CK II). Both CK I and CK II take an average of 2 years to complete. Physicians completing residency are able to either practice after obtaining a medical licence or enter CK II training directly. Currently, there is no national examination prior to licensing, although a plan has been set for its establishment in the near future.7

현재 베트남의 정책 입안자들 사이에서는 비전염성 질병, 신종 전염병, 글로벌 기후 위기로 인한 보건 문제, 불평등 심화로 인해 베트남의 교육 시스템을 개혁하고 현대화할 필요성에 대한 광범위한 공감대가 형성되어 있습니다.10 13 14 특히 학부와 대학원 모두에서 의학교육의 커리큘럼 개혁이 우선순위에 놓여 있습니다.7 9 15 보건부는 국제 의료계와의 통합을 향한 비전과 함께 투자와 정책 변화를 통해 개혁 노력을 촉진했습니다.8 이러한 국가 차원의 노력은 교육자들이 혁신을 개발하고 공유할 수 있는 환경을 조성했습니다. 현재 호치민시 의약대학(UMP), 후에 의약대학, 타이빈 의약대학, 하이퐁 의약대학, 타이응우옌 의약대학 등 5개 공립 의과대학에서 UME 커리큘럼 개혁이 진행 중입니다. 현재 호치민에서는 대학원 의학교육(GME) 개혁이 진행 중입니다. 또한 베트남 의료 전문가의 질을 높이기 위한 노력의 일환으로 최근 비영리 사립 보건과학 대학인 빈대학교(VinUni)가 새롭게 출범했습니다.
There is currently wide consensus among policymakers in Vietnam on the need to reform and modernise the country’s educational system due to increasing incidence of non-communicable disease, emergent infectious diseases, health challenges arising from the global climate crisis and widening inequalities.10 13 14 In particular, curricular reform in medical education, both undergraduate and graduate, has been prioritised.7 9 15 The MOH has catalysed reform efforts through investments and policy changes with a vision towards integration with the international medical community.8 Such national-level commitment has created an enabling environment for educators to develop and share innovations. UME curriculum reform is currently under way at five public medical universities, including the University of Medicine and Pharmacy (UMP) at Ho Chi Minh City (HCMC), Hue UMP, Thai Binh UMP, Hai Phong UMP and Thai Nguyen UMP. Graduate medical education (GME) reform is currently under way in HCMC. Additionally, a new private not-for-profit health sciences college at VinUniversity (VinUni), has recently launched in efforts to increase the quality of healthcare professionals in Vietnam.

이 글에서는 베트남의 의료 교육 혁신의 두 가지 영역(커리큘럼과 기술)에 대해 설명하고, 파트너십과 정책 변화를 통해 이러한 혁신이 어떻게 가능했는지 살펴봅니다. 이러한 혁신은 베트남 내에서 새롭게 발생했거나 다른 국가의 관행을 베트남에 맞게 변형한 것이지만 베트남의 상황에 맞게 새로운 것으로 간주됩니다. 이러한 혁신은 베트남 의과대학이 직면한 문제를 해결합니다. 우리의 검토는 이 원고에 대한 저자들의 집단적 경험과 지식뿐만 아니라 출판된 문헌과 회색문헌을 기반으로 합니다.
In this article, we describe two areas of innovation in medical training in Vietnam (curriculum and technology) and we review how these innovations were enabled through partnerships and policy changes. These innovations are de novo (arising within Vietnam) or they are Vietnamese adaptations of practices from other countries but are considered new for the Vietnamese context. They address challenges experienced by medical universities in Vietnam. Our review is based on the published and grey literature, as well as the collective experience and knowledge of the authors on this manuscript.

커리큘럼 혁신
Curricular innovations

전 세계의 많은 교육기관과 마찬가지로 베트남의 교육기관도 인구 보건 수요를 충족할 수 있는 의사를 양성하기 위한 커리큘럼 개혁에 착수하고 있습니다. 5개의 공립 UMP가 UME 프로그램을 역량 기반 커리큘럼으로 전환하고 있습니다. 역량 기반 의학교육(CBME)은 성과에 초점을 맞추고 지식과 실습의 적용을 강조하며 학습자 중심주의를 촉진합니다.16-18 2015년 베트남 보건부는 베트남의 인구 보건 요구사항을 기반으로 표준 UME 역량을 만들었습니다.19 이러한 역량을 기반으로 새로운 커리큘럼을 개발하기 위해 UMP 교수진은 백워드 코스 설계 원칙을 사용하여 각 역량에 대한 하위 구성 요소를 정의하고, 이정표를 만들고, 적절한 평가 도구와 전략을 선택하고, 마지막으로 교육 활동과 교수법을 설계했습니다.20 UME 커리큘럼에서 이정표를 적용하는 것은 새로운 접근 방식입니다. 개발 마일스톤은 다양한 GME 프로그램에서 적용되어 왔지만, UME 맥락에서의 적용은 비교적 새롭고 전 세계적으로 다양합니다.21 22 마일스톤은 학습자가 역량을 향한 경로를 따라 특정 시점에 달성해야 하는 최소 기준을 정의하므로 학습자와 교수진 모두에게 투명성을 높여줍니다.23
Like many institutions around the world, those in Vietnam are embarking on curricular reforms aiming to train physicians better prepared to meet population health needs. Five public UMPs are transforming their UME programmes to a competency-based curriculum. Competency-based medical education (CBME) focuses on outcomes, emphasises application of knowledge and practice, and promotes greater learner-centredness.16–18 In 2015, the Vietnam MOH created standard UME competencies based on the population health needs of Vietnam.19 To develop the new curriculum based on these competencies, UMP faculty used the principles of backward course design, defining subcomponents for each competency, creating milestones, selecting appropriate assessment tools and strategies, and finally designing educational activities and teaching methods.20 The application of milestones in the UME curriculum is a novel approach. Developmental milestones have been applied in various GME programmes; however, its implementation in the UME context is relatively new and varies globally.21 22 Milestones increase transparency for both the learner and faculty as they define the minimum standard that learners need to accomplish at a point of time along their pathway towards competency.23

개혁된 커리큘럼은 기대되는 학습 결과의 달성에 건설적으로 연계된 쌍방향 교수 및 학습 활동이 특징입니다. 커리큘럼을 제공함에 있어 개혁 노력은 수동적인 강의 시간을 줄이고 능동적인 교육적 접근 방식을 구현하는 데 중점을 두었습니다. 수동적인 학습 접근 방식은 학생의 이해에 부정적인 영향을 미치고 문제 해결, 자기 주도적 학습 및 의료 전문가에게 필요한 기타 핵심 기술을 저해하는 것으로 나타났습니다.24-26 이는 대규모 학급 규모(UMP당 연간 400~600명의 의대생)와 낮은 교수 대 학생 비율로 인해 베트남 상황에서 특히 어려운 문제입니다. 이러한 문제를 극복하기 위해 교수진은 다른 환경에서 사용되는 교육적 접근 방식을 검토하고 베트남 강의실의 필요에 맞게 반복하여 적용했습니다. 소그룹 학습이 바람직하지만, 학급 규모가 크기 때문에 대규모 그룹 환경에서 활발한 학습을 유도하기 위한 전략이 필요합니다. 

  • 이러한 전략 중 하나는 토론과 동료 학습을 유도하기 위해 생각-쌍-공유 접근 방식과 함께 청중 응답 시스템(ARS)을 사용하는 것입니다.27 28 그러나 교수진은 값비싼 기술 기반 ARS 대신 빠르게 확장하고 구현할 수 있는 색상으로 구분된 종이 기반 시스템을 개발했습니다
  • 두 번째 전략은 팀 기반 학습(TBL)을 사용하는 것입니다. TBL은 학습자 중심성을 높이고 동료와의 능동적인 학습을 촉진하며 기존의 문제 기반 학습 접근 방식과 달리 교수 대 학생 비율이 낮은 환경에서도 구현할 수 있습니다.29 베트남에서 이 접근 방식을 채택한 경우, 약 40명의 학생에게 5~8명의 학생으로 구성된 팀과 교류하는 교수 촉진자가 제공됩니다. 학생 팀은 (1) 대면 세션 전 준비 과제, (2) 준비 과제에 초점을 맞춘 객관식 문제로 구성된 개인 및 그룹 준비도 확인 시험, (3) 준비 과제의 자료를 '실제' 시나리오에 적용해야 하는 그룹 적용 활동의 세 단계에 걸쳐 함께 작업하고 서로에게 책임을 집니다.29 이 접근 방식은 학습에 대한 문제 기반 접근 방식을 허용하고, 동료 간 학습과 책임감을 부여하며, 팀워크를 촉진하고, 다른 소그룹 학습 방법보다 적은 교수진 자원으로 구현할 수 있기 때문에 베트남에서 성공적이었습니다.

The reformed curriculum is characterised by interactive teaching and learning activities constructively aligned to the achievement of the expected learning outcomes. In delivering the curriculum, reform efforts have focused on implementing active pedagogical approaches, reducing time spent in passive lectures. Passive learning approaches have been shown to negatively affect student understanding and discourage problem-solving, self-directed learning and other critical skills needed for healthcare professionals.24–26 This is a particular challenge in the Vietnam context due to large class sizes (400–600 medical students per year per UMP) and low faculty-to-student ratios. To overcome these challenges, faculty reviewed pedagogical approaches used in other settings and iterated and adapted them to fit the needs of Vietnamese classrooms. While small group learning is desired, the large class size necessitates strategies for bringing active learning to a large group setting.

  • One such strategy is the use of an audience response system (ARS) with think–pair–share approach to generate discussion and peer learning.27 28 However, in lieu of expensive technology-based ARSs, faculty developed a colour-coded paper-based system which could be quickly scaled up and implemented.
  • A second strategy is the use of team-based learning (TBL). TBL increases learner-centredness, promotes active learning with peers and, unlike more traditional problem-based learning approaches, can be implemented in settings with low faculty-to-student ratios.29 In the Vietnamese adaptation of this approach, approximately 40 students are provided a faculty facilitator who engages with teams of five to eight students. Student teams work together and are held accountable to one another in the three distinct phases of TBL: (1) a preparation assignment prior to the in-person session, (2) individual and group readiness assurance tests consisting of multiple choice questions focused on the preparation assignment, and (3) a group application activity that requires students to apply the material from the preparation assignment to a ‘real-world’ scenario.29 The approach has been successful in Vietnam because it allows for a problem-based approach to learning, enables peer-to-peer learning and accountability, promotes teamwork and can be implemented with fewer faculty resources than other small group learning methods.

베트남의 UME 역량에는 팀워크와 전문가 간 협업이 포함됩니다. 전문직 간 협업은 의학전문대학원 교육 인증위원회와 캐나다 왕립 의사 및 외과의사 대학을 비롯한 많은 국가의 의학교육 프레임워크에서 핵심 역량이지만,30 진정한 전문직 간 교육 모델은 자원이 풍부한 환경에서도 여전히 제한적입니다.31 32 CBME 개혁 이전 베트남의 의대생들은 시뮬레이션과 병동 실습을 통해 간호 기술을 배웠습니다. 학생들은 간호 교수진으로부터 교육을 받고, 병동에서 간호사를 관찰하고, 간호사의 감독 하에 환자 간호에 참여하여 실습 로테이션에 들어갑니다. 이 새로운 직종 간 교육 모델은 새로운 커리큘럼에서도 유지되었습니다

  • 전문직 간 교육을 더욱 촉진하기 위해 호치민 UMP는 2019년 9월 의학, 약학, 간호학 및 재활학 학생들을 위한 새로운 과정을 도입했습니다. 여러 분야의 교수진이 협력하여 학생들의 협업을 촉진하고 (1) 전문직 간 진료의 가치 이해, (2) 효과적인 의사소통, (3) 전문성, (4) 팀 리더십 기술 등 전문직 간 진료와 관련된 기술을 배양하는 과정을 설계했습니다. 이 과정은 베트남에서 전문직 간 교수진이 전문직 간 학생 그룹을 대상으로 가르치는 최초의 과정입니다. 

UME competencies in Vietnam include teamwork and interprofessional collaboration. Although interprofessional collaboration is a core competency in many countries’ medical education frameworks, including those from the Accreditation Council for Graduate Medical Education and the Royal College of Physicians and Surgeons of Canada,30 models for authentic interprofessional education are still limited, even in high-resourced settings.31 32 Prior to CBME reforms, medical students in Vietnam learnt nursing skills through both simulation and practice on the wards. Students are taught by nursing faculty, observe nurses on the wards and participate in patient care under the supervision of nurses, prior to entering clerkship rotations. This novel interprofessional education model was maintained in the new curriculum.

  • To further promote interprofessional education, UMP at HCMC introduced a new course in September 2019 for medicine, pharmacy, nursing and rehabilitation students. Faculty members from the different disciplines cooperated to design a course which promotes student collaboration and foster skills related to interprofessional practice including: (1) understanding the value of interprofessional care, (2) effective communication, (3) professionalism, and (4) team leadership skills. This is the first course in Vietnam taught to an interprofessional group of students by an interprofessional faculty.

테크놀로지 혁신
Technological innovation

테크놀로지를 의학 커리큘럼에 통합하면 성인 학습 이론을 의학 교육에 적용하고 학습 환경을 단순히 콘텐츠와 지식을 전달하는 공간에서 학습 과정과 학습자 평가를 촉진하는 공간으로 전환할 수 있습니다.33 이를 통해 학습 환경은 교실, 강의실, 도서관의 전통적인 영역을 넘어 가상 공간으로 확장될 수 있습니다. 베트남 의과대학 개혁의 핵심은 새로운 학습 테크놀로지를 도입하고 이를 적용할 수 있는 네트워크 인프라에 투자하는 것이었습니다.
Integration of technology into medical curricula can catalyse the application of adult learning theory in medical education and help to transform the learning environment from solely a distributor of content and knowledge into a space which facilitates the learning process and the assessment of the learner.33 In doing so, learning environments can be expanded beyond the traditional domains of classrooms, lecture halls and libraries into virtual spaces. A key pillar of the reform at medical universities in Vietnam has been to adapt new learning technologies and invest in network infrastructure to enable their application.

베트남의 의과 교육 프로그램은 공립 대학의 경우 Moodle과 같은 무료 오픈 소스 학습 관리 시스템(LMS)을 사용하고 맞춤화하여 가상 공간을 확장했으며, 사립 대학의 경우 Canvas 및 One45와 같은 구독 서비스 LMS를 사용했습니다. 일부 교육기관에서는 LMS를 수업 자료를 업로드하고 다운로드하는 플랫폼으로만 사용하기도 하지만, 학생과 교수진의 피드백 수집, 게시판 및 토론 포럼 생성, 평가 관리, 온라인 리소스 센터 또는 라이브러리 개발, 온라인 학습 과정 구현과 같은 고급 기능을 사용할 수 있도록 LMS를 커스터마이즈한 교육기관도 있습니다. 예를 들어, 

  • 미국 외과 대학원 프로그램을 위한 표준화된 역량 기반 온라인 커리큘럼을 만든 미국 컨소시엄인 외과 레지던트 교육 위원회(SCORE)와 협력하여 호치민 UMP의 일반외과 레지던트 프로그램에 사용할 수 있도록 SCORE 포털을 도입하고 맞춤화했으며, VinUni도 GME 프로그램의 일부로 SCORE 프레임워크를 사용하고 있습니다.
  • 아직 파일럿 단계에 있지만 역량 기반 온라인 교육 자료를 적용하면 모든 일반외과 레지던트가 공통 커리큘럼을 받고 공통 지식 기반을 개발할 수 있습니다. 이는 베트남 GME의 새로운 접근 방식으로, 일반적인 교사 중심 접근 방식에서 벗어나 학습자가 자신의 시간에 온라인 모듈에 액세스하여 자신의 속도에 맞춰 진행할 수 있는 학습자 중심 접근 방식으로 커리큘럼을 재조정하는 데 도움이 됩니다.

Medical education programmes in Vietnam have expanded their virtual footprint through the use and customisation of free, open-source learning management systems (LMS) such as Moodle for public universities, and subscription-service LMS such as Canvas and One45 for private universities such as VinUni. While some institutions have used the LMS solely as a platform to upload and download class materials, others have customised the LMS to allow for more advanced features, such as collecting feedback from students and faculty, creating message boards and discussion forums, administering assessments, developing online resource centres or libraries and implementing online learning courses. For example,

  • in collaboration with the Surgical Council on Resident Education (SCORE), a US consortium that created a standardised competency-based online curriculum for US surgical graduate programmes, the SCORE Portal was introduced and tailored for use in the general surgery residency programme at UMP at HCMC; VinUni also uses the SCORE framework as part of their GME programme.
  • While still in a pilot phase, the application of competency-based on-line training materials ensures that all general surgery residents can receive a common curriculum and develop a common knowledge base. This is a novel approach for GME in Vietnam, helping to reorient the curriculum away from the more typical teacher-centred approach to a learner-centred approach in which learners can access online modules on their own time and proceed at their own pace.

앞서 설명한 LMS 구현과 같은 테크놀로지에 대한 투자와 혁신 덕분에 베트남의 대학들은 2020년 2월 코로나19 팬데믹이 시작되었을 때 온라인 학습으로 빠르게 전환할 수 있었습니다. 교직원과 학생들은 이미 Zoom, Microsoft Teams, Google 행아웃과 같은 LMS 및 온라인 학습 플랫폼에 익숙해져 있었습니다. 그 결과 사회적 거리두기 조치가 시행되었을 때 프로그램 중단을 최소화할 수 있었습니다. 또한 온라인 교육 및 학습을 위한 역량을 갖추면 신종 SARS-CoV-2 바이러스에 대한 지식을 전파하는 데 도움이 되었습니다. UMP는 기존 인프라를 활용하여 의대생들을 위한 가상 코로나19 교육 프로그램을 신속하게 출시했습니다. 학생들은 혼합 접근 방식을 사용하여 접촉자 추적, 분류 및 검사를 포함한 SARS-CoV-2의 역학 및 예방에 대한 교육을 받았습니다. 현재까지 720명 이상의 의대생이 베트남의 코로나19 대응에 투입되었습니다.34 35 교수진과 학생들이 가상 교육 및 학습에 익숙해지면서 일부 교육기관은 코로나19 팬데믹 이후에도 커리큘럼의 20%를 가상으로 제공할 계획을 세웠습니다. 

Investments and innovations in technology, such as the implementation of LMS described previously, enabled universities in Vietnam to rapidly transition to online learning when the COVID-19 pandemic began in February 2020. Faculty and students were already familiar with LMS and online learning platforms, such as Zoom, Microsoft Teams and Google Hangout. As a result, disruption to the programme was minimised when social distancing measures were enforced. In addition, having capacity for online teaching and learning was beneficial for disseminating knowledge about the novel SARS-CoV-2 virus. The UMPs used their existing infrastructure to quickly roll out a virtual COVID-19 training programme for senior medical students. Students were trained, using a blended approach, on the epidemiology and prevention of SARS-CoV-2, including contact tracing, triage and testing. To date, over 720 medical students have been deployed in Vietnam’s COVID-19 response.34 35 As faculty and students become more familiar with virtual teaching and learning, some institutions have planned for 20% of the curriculum to be delivered virtually beyond the COVID-19 pandemic.

파트너십 활성화
Enabling partnerships

여러 새로운 파트너십이 베트남 의학교육의 혁신을 가능하게 하고 촉진하는 데 도움이 되었습니다. 산업계와 국제 기관 간의 파트너십은 혁신이 번창할 수 있는 생태계를 조성합니다.36-38 특히 국제 학술 파트너십은 혁신의 원천이 될 수 있습니다. 예를 들어,

  • 하버드 의과 대학(HMS)은 베트남 보건부의 보건 시스템 개혁을 위한 보건 전문가 교육 및 훈련 프로젝트 및 베트남 의과대학과 협력하여 의학교육 및 신흥 질병에 대한 접근성, 커리큘럼 및 교육 개선(IMPACT MED) 연합을 구성했습니다.39 미국 국제개발처의 자금 지원으로 IMPACT MED 연합은 커리큘럼 설계, 개발 및 실행에 대한 기술 지원 및 지식 이전을 제공하여 5개 의과대학의 UME 커리큘럼 개혁을 지원하고 있습니다.
  • 이와 유사하게, VinUni는 커리큘럼 개발 및 실행에 대한 기술 지원과 서비스 제공 및 환자 치료 개선을 통해 의료 전문가 교육을 혁신하는 것을 목표로 펜실베니아 대학교와 전략적 제휴 계약을 체결했습니다.40 이 두 가지 학술 파트너십에는 학생 및 교수진 교류와 연구 협력도 포함됩니다. 이러한 파트너십을 통해 새로운 개념과 접근 방식이 도입되면 지역 전문가와 이해관계자가 적응, 계획 및 실행 작업을 주도하여 혁신이 상황과 문화에 적합하도록 보장합니다. 

A number of novel partnerships have helped to enable and catalyse innovations in medical education in Vietnam. Partnerships between industry and international institutions create an enabling ecosystem for innovation to thrive.36–38 In particular, international academic partnerships can provide a source of innovation. For example,

  • Harvard Medical School (HMS) has partnered with the Vietnam MOH’s Health Professionals Education and Training for Health System Reforms Project and medical universities in Vietnam as part of the Improving Access, Curriculum and Teaching in Medical Education and Emerging Diseases (IMPACT MED) Alliance.39 With funding from the United States Agency for International Development, the IMPACT MED Alliance supports UME curricular reform at five medical universities through provision of technical assistance and knowledge transfer on curriculum design, development and implementation.
  • Similarly, VinUni has a strategic alliance agreement with the University of Pennsylvania with a goal of innovating health professional education through technical support on curriculum development and implementation, and by improving service delivery and patient care.40 Both of these academic partnerships also involve student and faculty exchanges, and promote research collaborations. As novel concepts and approaches are introduced through these partnerships, local experts and stakeholders lead the work of adapting, planning and implementing, ensuring that innovations are contextually and culturally appropriate.

두 번째 사례는 Microsoft와 공공 UMP 간의 공공-민간 파트너십으로, 처음에는 참여 UMP의 모든 대학생, 교수진 및 직원에게 Microsoft Office 365 제품군에 대한 무료 온라인 액세스를 제공한 IMPACT MED 얼라이언스가 촉진한 것입니다. Microsoft의 기술 지원을 통해 Microsoft Office 365 제품군의 전체 기능을 도입하고 대학 운영에 통합하여 할인된 전체 액세스 구독으로 전환했습니다. Microsoft Office 365 제품군 사용은 기관의 커뮤니케이션 및 관리 기능을 간소화하는 데 도움이 되었습니다. 대학 관리자와 경영진은 대학 커뮤니티의 모든 구성원과 쉽게 소통할 수 있고, 학생들은 가상 코스워크와 LMS 액세스를 위한 단일 진입 지점을 갖게 되었으며, 인사 관리와 같은 대학 행정 기능이 공통 시스템으로 통합되었습니다. 그 전에는 종이 기반 시스템을 사용했고, 교수진은 다양한 소셜 미디어 플랫폼을 통해 학생들과 소통했으며, 커뮤니케이션은 종이 기반 또는 개인 이메일 주소와 휴대전화를 통해 이루어졌습니다. 이 사례는 기술 솔루션 도입 및 유지를 위한 적절한 자금 조달 메커니즘과 함께 기술 및 기술 이전을 위한 민관 파트너십의 중요성을 보여줍니다. 
A second example is a public–private partnership between Microsoft and public UMPs, facilitated by the IMPACT MED Alliance, which initially provided free online access to the Microsoft Office 365 Suite for all university students, faculty and staff at the participating UMPs. With technical assistance from Microsoft, the full functionality of the Microsoft Office 365 Suite was introduced and incorporated into university operations, leading to transition to a discounted, full-access subscription. Using the Microsoft Office 365 Suite has helped to streamline the institutions’ communication and administrative functions. University administrators and leadership can easily communicate with all members of the university community; students have a single point of entry for virtual coursework and LMS access; and university administrative functions such as human resources management have been integrated into a common system. Prior to this, paper-based systems were employed; faculty communicated with students through various social media platforms; and communication was paper-based or via personal email addresses and mobile phones. This example demonstrates the importance of public–private partnerships with technology and technical skills transfer combined with appropriate financing mechanisms for introducing and sustaining technological solutions.

기술 및 기술 이전으로 이어진 민간 부문 파트너십의 또 다른 사례는 삼성과 UMP 간의 파트너십입니다. 삼성은 두 개의 UMP에 두 개의 시범 '스마트 교실'에 투자했습니다. 스마트 교실은 그룹 학습을 촉진하기 위해 설계되었으며 인터넷을 통해 쉽게 액세스하고 공유할 수 있는 기술이 적용되었습니다. 교실 인프라와 기술을 커리큘럼 설계 및 교육 방식에 맞추는 것은 새로운 커리큘럼의 성공과 지속 가능성을 위해 매우 중요합니다. 시범 교실을 시범 운영한 결과 학생, 교수진, 경영진 모두 이러한 연계의 가치를 인정하여 5개 UMP 모두에서 강의실 재설계에 대한 대학 투자로 이어졌습니다. 
Another example of a private sector partnership which resulted in technology and technical skills transfer is between Samsung and the UMPs. Samsung invested in two demonstration ‘smart classrooms’ at two UMPs. The smart- classrooms were designed to promote group learning and are technology-enabled to facilitate access and sharing across the internet. Aligning classroom infrastructure and technology with the curriculum design and pedagogical method is vital to the success and sustainability of the new curriculum. Following a trial of the demonstration classrooms, students, faculty and leadership all recognised the value of this alignment, leading to university investment in classroom redesigns at all five UMPs.

정책 활성화
Enabling policies

정책 변화는 의료 교육 기관이 혁신을 적응하고 실행하며 지속할 수 있도록 하기 때문에 개혁을 추진하고 유지하기 위해 종종 필요합니다. 베트남 정부는 2014년에 대학 자율성과 관련된 국가 정책을 시행하여 교육 혁신을 위한 환경을 조성했습니다.41-44 이러한 정책은 공립대학이 재정적으로 독립하도록 장려하고 자체 운영, 인적 자원 및 성장 전략을 관리하도록 장려합니다. 새로운 프레임워크는 공공기관이 민간 부문 및 국제 학술 기관과의 파트너십을 모색하여 테크놀로지에 대한 투자 우선순위를 정하고 새로운 커리큘럼, 교육학 및 교육 접근법을 촉진할 수 있도록 지원합니다. 또한 기관은 관리 및 행정 구조를 재설계할 수 있는 권한을 부여받습니다. 이러한 구조 개편은 UMP에서 보다 통합된 CBME 커리큘럼을 개발하는 데 중요한 역할을 했습니다.45 기존 모델에서는 학과장이 교육 내용에 대한 의사 결정 권한을 가졌기 때문에 특정 학과를 중심으로 커리큘럼이 구성되는 결과를 낳았습니다. 보다 통합적인 커리큘럼을 개발하기 위해서는 이러한 거버넌스 구조를 세분화하고 다학제 교수진으로 구성된 커리큘럼 위원회에 권한을 부여할 필요가 있었습니다(그림 2). 위원회 리더는 각 모듈에 무엇이 포함될지, 어떤 교수진이 가르칠지, 커리큘럼 품질 개선 프로그램의 일부로 어떤 조정을 해야 할지 결정합니다. 
Policy changes are often necessary to drive and sustain reform, as they can enable medical education institutions to adapt, implement and continue innovations. The Vietnamese government created an enabling environment for innovation in education through the implementation of national policies related to university autonomy in 2014.41–44 These policies promote public universities to become financially independent and encourage them to manage their own operations, human resources and strategies for growth. The new framework enables public institutions to seek partnerships with the private sector and international academic institutions, helping to prioritise investments in technology and to promote new curricula, pedagogy and educational approaches. In addition, institutions are empowered to redesign their management and administration structures. Such restructuring was important to the development of a more integrated CBME curriculum at the UMPs.45 In the traditional model, department chairs held decision-making authority over the teaching content, resulting in a curriculum organised around specific disciplines. To develop a more integrated curriculum, it was necessary to break down this governance structure and to provide authority to curriculum committees consisting of multidisciplinary teaching faculty (figure 2). Committee leaders decide what is included in each module, which faculty should teach and what adjustments should be made as part of the curriculum quality improvement programme.

대학 차원의 또 다른 정책적 지원은 커리큘럼의 지속적인 품질 개선(CQI)을 위한 시스템 개발과 그 실행입니다. 대학 내에 새로 구성된 부서는 학생의 요구에 맞게 커리큘럼과 교육을 지속적으로 개선하기 위해 학생 피드백, 교수진 및 학생 평가 데이터를 일상적으로 수집하고 사용하는 메커니즘을 만들어 CQI 프로세스를 주도하고 있습니다. 데이터는 커리큘럼 위원회와 대학 경영진에게 공유되며 향후 프로그램 조정에 관한 결정을 내리는 데 사용됩니다. 또한 이 시스템은 교수진과 관리자에게 새로운 접근 방식과 혁신의 채택에 대한 데이터를 제공하여 지속적으로 반복하고 개선하며 특정 상황에 맞게 조정할 수 있도록 합니다. 또한 이러한 단위는 향후 인증 프로세스를 위한 지원 데이터도 생성합니다. 
Another policy enabler at the university level has been the development of a system for continuous quality improvement (CQI) of the curriculum and its implementation. Newly formed units within the university lead the CQI process by creating mechanisms for routine collection and use of student feedback and faculty and student evaluation data with a goal of continually improving the curriculum and teaching to match student needs. Data are shared to curriculum committees and university leadership and are used to make decisions regarding future adjustments to the programme. This system also provides data to faculty and administrators on the adoption of novel approaches and innovations, allowing them to continually iterate, improve and tailor for specific contexts. These units also generate supporting data for future accreditation processes.

제한 사항
Limitations

이 리뷰는 최근 베트남 의학교육의 몇 가지 혁신을 강조합니다. 현재 진행 중인 의료 전문가 교육 시스템이나 커리큘럼 개혁에 대한 포괄적인 검토를 목적으로 하는 것은 아닙니다. 이 리뷰는 주로 공동 저자들의 지식과 경험, 그리고 영어 문헌에 발표된 내용을 바탕으로 작성되었습니다. 여기에 설명된 혁신은 베트남 내외의 다른 환경에도 적용될 수 있지만, 이러한 접근법을 성공적으로 도입하는 데 장벽이 존재할 수 있음을 인지하고 있습니다.

  • 첫째, 제도 개혁을 위해서는 국가 및 기관 차원에서 미래 지향적이고 강력한 리더십이 필요합니다. 이러한 리더십이 부족하면 혁신적인 접근법을 활성화하고 채택하는 데 한계가 있을 수 있습니다.
  • 둘째, 자원(인적, 재정적, 인프라적)은 특히 자원이 제한된 환경에서 변화를 가로막는 중요한 장벽이 될 수 있습니다. 예를 들어, 기술 기반 개혁에는 안정적인 인터넷 연결이 필요하며, 이는 자원이 제한된 많은 환경에서 중요한 장벽이 될 수 있습니다. 이는 필요한 자원을 동원하기 위한 강력한 리더십의 헌신과 새로운 파트너십의 중요성을 더욱 강조합니다.
  • 셋째, 기관의 문화는 혁신의 창출과 적응에 있어 중요한 요소입니다. 새로운 아이디어를 기꺼이 수용하고 위험을 감수하고 실패할 수 있는 의지가 없다면 혁신은 불가능할 것입니다.
  • 마지막으로, 모든 변화가 개선으로 이어지는 것은 아닙니다. 지금까지 성공적인 것으로 입증된 혁신에 대해 설명했지만, 그렇지 않은 혁신도 있을 수 있습니다. 성공하지 못한 개혁에서도 배울 점이 많지만, 이번 리뷰에서는 이에 초점을 맞추지 않았습니다.

This review highlights several recent innovations in medical education in Vietnam. It is not intended to be a comprehensive review of the health professional education system or the curriculum reforms currently under way. Our review is shaped primarily by the coauthors’ knowledge and experience and to what is published in the English language literature. While the innovations described here may apply to other settings within and outside Vietnam, we recognise that barriers may exist to the successful uptake of these approaches. First, institutional reform requires forward-thinking and robust leadership at both the national and institutional levels. A lack of such leadership can limit the ability to enable and adopt innovative approaches. Second, resources (human, financial and infrastructural) can be a significant barrier to change, particularly in the resource-limited setting. For example, technology-enabled reforms require reliable internet connectivity, which may be a significant barrier in many resource-limited settings. This further highlights the importance of strong leadership commitment and novel partnerships to mobilise necessary resources. Third, the culture of an institution is a crucial factor in the creation and adaptation of innovations. Without a willingness to embrace new ideas and to take risks and fail, innovation would not be possible. Lastly, not all change leads to improvement. We have described innovations that proved successful, but others may have been less so. While there is still much to learn from less successful reforms, that was not our focus in this review.

결론
Conclusion

플렉스너 보고서가 의학교육의 변화에 영감을 준 지 100년이 넘었습니다.46 오늘날 전 세계적으로 심각한 보건 인력 문제에 직면하여 전문 보건 교육에 대한 접근 방식을 다시 한 번 재고할 필요가 있습니다.4 특히 LMIC에서 의학교육의 혁신과 개혁이 필요합니다. 이 리뷰에서는 커리큘럼 설계, 교육학 및 기술 적용 분야를 포함하여 현재 베트남의 의학교육 개혁의 일환으로 개발된 혁신 사례를 공유했습니다. 또한 파트너십과 정책 변화가 어떻게 혁신을 가능하게 하고, 장려하며, 지속시킬 수 있는지에 대해서도 설명했습니다. 모든 의료 시스템에는 고유한 과제가 있지만, 베트남의 이러한 사례가 미래의 의학교육 혁신에 영감을 줄 수 있기를 바랍니다.
It has been more than 100 years since the Flexner report inspired a transformation of medical education.46 Today, in the face of significant global health workforce challenges, there is again a need to rethink our approach to professional health education.4 Innovation and reform in medical education are needed, particularly in LMICs. In this review, we have shared examples of innovations developed as part of Vietnam’s current medical education reform, including those in the areas of curriculum design, pedagogy and application of technology. We have also described how partnerships and policy changes can enable, encourage and sustain innovation. While every healthcare system has unique challenges, we hope these examples from Vietnam can inspire future innovations in medical education.

 


Medical education reforms are a crucial component to ensuring healthcare systems can meet current and future population needs. In 2010, a Lancet commission called for ‘a new century of transformative health professional education’, with a particular focus on the needs of low-income and-middle-income countries (LMICs), such as Vietnam. This requires policymakers and educational leaders to find and apply novel and innovative approaches to the design and delivery of medical education. This review describes the current state of physician training in Vietnam and how innovations in medical education curriculum, pedagogy and technology are helping to transform medical education at the undergraduate and graduate levels. It also examines enabling factors, including novel partnerships and new education policies which catalysed and sustained these innovations. Our review focused on the experience of five public universities of medicine and pharmacy currently undergoing medical education reform, along with a newly established private university. Research in the area of medical education innovation is needed. Future work should look at the outcomes of these innovations on medical education and the quality of medical graduates. Nonetheless, this review aims to inspire future innovations in medical education in Vietnam and in other LMICs.

동남아시아의 면허시험: 교육정책의 변화에서 찾아낸 교훈( Asia Pacific Sch, 2019)
Licensing examinations in Southeast Asia: Lesson learnt from exploring changes in education policy
Rachmadya Nur Hidayah1, Richard Fuller2 & Trudie E. Roberts3

 

I. 배경
I. BACKGROUND

동남아시아의 의료 서비스는 급속한 지역 개발과 함께 세계에서 가장 인구가 많은 지역 중 하나에 고품질의 안전한 의료 서비스를 제공해야 하는 중대한 과제에 직면해 있습니다. 질병 부담의 추세가 비전염성 질병으로 이동하고 있으며, 세계화로 인해 환자와 의료 전문가의 이동이 증가하고 있습니다. 저소득 및 중간 소득 국가 또는 개발도상국에서 선진국으로 이동하는 것이 그 반대보다 더 흔한 것으로 알려져 있습니다(Tangcharoensathien 외., 2018). 의료 인력의 이동은 의학교육의 국제적 이동(예: 해외 의과대학)으로 이어져 의학교육의 글로벌-지역적 긴장을 야기했습니다. 의료의 질과 환자 안전을 보장하기 위해 WHO는 보건 인력의 국제 채용에 관한 글로벌 실천 강령을 통해 규제 기관이 의무적으로 국가 면허 시험(NLE)을 시행할 것을 권고했으며, 이는 동남아시아 국가에서 면허 시험 시행이 증가하는 추세에 따라 이러한 높은 수준의 교육 정책을 지원하는 것으로 간주되었습니다(Tangcharoensathien 외., 2018). 
Healthcare in Southeast Asia faces a major challenge–that of delivering high quality, safe care to one of the most populated areas in the world, alongside rapid regional development. The trend in disease burden is shifting to non-communicable diseases, along with the challenge of globalisation which contributes to the migration of patients and health care professionals. It is known that the migration from low- and middle- income countries, or from developing to developed countries, is more common than the opposite (Tangcharoensathien et al., 2018). The migration of health care workforce has been followed by the international movement in medical education (e.g. offshore medical schools), which have created global-local tensions in medical education. In the interest of ensuring the quality of care and patient safety, the WHO through its Global Code of Practice on the International Recruitment of Health Personnel, recommended the implementation of mandatory national licensing examinations (NLEs) by regulatory bodies have been viewed as supporting this high-stakes education policy, which was in line with the increasing practice of licensing examinations in Southeast Asia countries (Tangcharoensathien et al., 2018).

II. 국가 면허 시험: 북미와 유럽에서 얻은 교훈
II. NATIONAL LICENSING EXAMINATIONS: LESSONS FROM NORTH AMERICA AND EUROPE

면허시험을 졸업생의 자질 보증(그리고 더 나은 의료 서비스 결과 지원)에 활용하는 정책은 다른 지역에서도 사용되어 왔습니다. 미국 의사 면허 시험(USMLE)과 캐나다 의사 위원회 자격 시험(MCCQE)이 가장 대표적인 예입니다. 처음에는 의사들 간의 편차를 줄이기 위한 노력의 일환으로 설립되었지만, USMLE는 미국이나 캐나다에서 면허를 취득하고자 하는 국내 졸업생뿐만 아니라 해외 의대 졸업생들을 위한 평가 시스템으로 발전해 왔습니다. USMLE와 MCCQE는 지난 20년 동안 에세이 및 구두 시험에서 보다 체계적인 MCQ 및 임상 술기 평가로 변경되는 등 시행 방식에 변화를 겪었습니다. 면허의 초점은 교육 성과, 지속적인 의학 교육 및 재검증에 대한 예측, 기술 및 심리 측정의 사용으로 옮겨졌습니다. 북아메리카의 NLE는 자국 의료 졸업생과 이주하는 의료 전문가 모두의 자질을 보장하기 위한 노력으로 간주됩니다.
The policy of using licensing examinations for quality assurance of graduates (and arguably supporting better health care outcomes) has been used in other parts of the world. The United States Medical Licensing Examination (USMLE) and the Medical Council of Canada Qualifying Examination (MCCQE) are the most notable examples. Although firstly established as an effort to reduce variation among practitioners, the USMLE has been developed to an assessment system for international medical graduates as well as home graduates who wish to be licensed in the US or Canada. Both the USMLE and MCCQE went through changes in the administration; from essay and oral examination into a more systematic MCQ and clinical skills assessment in the last two decades. Licensure’s focus shifted into educational outcomes, projecting of continuing medical education and revalidation, the use of technology and psychometrics. NLEs in the Northern America are viewed as effort on ensuring the quality of both home medical graduates and migrating health care professionals.

두 나라에서 NLE의 영향은 이 분야의 연구 주제였습니다. 특히 임상 술기 평가 시행 후 의과대학 커리큘럼의 변화가 보고되었습니다. NLE의 더 나은 성과는 일차 진료에서 더 나은 예방 및 관리, 환자 불만 감소, 국제 졸업생의 더 나은 환자 치료, 졸업 후 평가에서의 더 나은 성과와 관련이 있습니다. 그러나 NLE가 환자에게 직접적인 영향을 미친다는 증거는 없었기 때문에 NLE가 환자 안전을 보장하는지 여부에 대한 논란이 제기되고 있습니다. 이러한 논란에도 불구하고 두 나라의 NLE는 의과대학의 인증 시스템과 지속적인 전문성 개발에 의존하는 교육 품질 보증 시스템의 초석이 되었습니다
The impact of NLEs in both countries have been a topic for research in this area. Changes in medical schools’ curricula, especially after the implementation of clinical skills assessment, were reported. Better performance in NLEs have been associated with better preventive and management in primary care; fewer patient complaints; better patient care for international graduates and better performance in postgraduate assessment. However, there were no evidence showing direct impact of NLEs to patients, which raises debate on whether the NLEs ensure patient safety. Despite the debate, the NLEs in both countries have become cornerstones of educational quality assurance system, which also relies on the accreditation system of medical schools and continuing professional development.

유럽 국가들은 의료 전문가의 이동과 품질 보증 문제에 대해 서로 다른 접근 방식을 취하고 있습니다. 유럽연합(EU) 회원국은 다른 EU 국가에서 졸업한 의료 전문가의 자격을 인정하기 때문에, 특히 의사의 경우 EU 내에서 일할 수 있는 기회가 동등하게 주어집니다. 외국 졸업생을 위한 시험이 있기는 하지만 의학 지식과 기술보다는 언어 능력에 중점을 두고 있습니다. EU 국가 간 커리큘럼의 다양성에 대한 우려가 있지만, 많은 전문가들은 인증accreditation 시스템이 졸업생의 자질을 보장하기에 충분하다고 믿었습니다(Archer et al, 2017). 그럼에도 불구하고 스위스, 폴란드, 독일과 같은 국가에서는 자국 졸업생과 비유럽경제지역(EEA) 국가 졸업생에 대한 의료 면허 시험을 마련했습니다. 2015년에 영국의 일반 의학 협의회는 이 단계를 밟아 2023년에 시행할 의료 면허 평가(MLA)를 시작할 계획입니다(Archer et al., 2017). MLA는 의과대학 평가에 포함되도록 설계되었기 때문에 향후 교육에 영향을 미칠 것으로 예상할 수 있습니다. 
European countries have different approaches to the issue of health care professionals’ migration and quality assurance. As European Union (EU) member countries recognise the qualification of health care professionals graduating from other EU countries, there are equal opportunities for them to work within the EU, particularly for medical doctors. Although there are examinations for foreign graduates, the focus is more on language competency rather than medical knowledge and skills. Whilst there are concerns about the diversity of curricula among EU countries, many experts believed that the accreditation systems were sufficient to assure the quality of their graduates (Archer et. al, 2017). Despite this, countries such as Switzerland, Poland, and Germany, have established medical licensing examinations for their home graduates and non-European Economic Area (EEA) countries’ graduates. In 2015, the General Medical Council in the United Kingdom followed this step and plans to start their medical licensing assessment (MLA) (Archer et al., 2017) which the GMC will put into action in 2023. The MLA is designed to be embedded in medical schools’ assessment, which means that there will be consequences in education that could be anticipated in the future.

두 지역의 교육 및 의료 시스템이 어떻게 운영되는지에 따라 졸업생과 의료 행위의 질을 보장하는 접근 방식이 다를 수 있습니다. 그러나 이 정책은 인증 시스템과 함께 교육 관행과 의료 시스템을 변화시킬 수 있다는 점에 주목할 필요가 있습니다. 
The two regions might have different approach in quality assuring the graduates and medical practice depending on how the education and healthcare system work in the regions. However, it is worth to notice that the policy, along with accreditation systems, could change the practice of education and possibly the health care system.

III. 아세안 국가 시험: 더 나은 치료를 위한 지역적 접근의 가능성?
III. NATIONAL EXAMINATIONS IN ASEAN: THE POSSIBILITY OF REGIONAL APPROACH FOR BETTER CARE?

EU 국가와 아세안 국가가 취하는 접근 방식은 유사하지만, 두 환경의 환자 관리 정책의 맥락과 교육 시스템이 다르다는 점에 주목할 필요가 있습니다. 아세안 국가들은 '역동적 개발 파트너십에 관한 아세안 비전 2020'을 추진하면서 보건의료 전문가들의 자유로운 서비스 제공을 위한 여러 가지 프레임워크에 합의했습니다. 아세안 국가의 간호사, 의사, 치과의사를 위한 상호 인정 협정(MRA)은 2006~2009년에 이 파트너십에 따라 만들어졌습니다. 이 협정에 따라 각국은 의료 전문가 이주에 대한 규정을 마련해야 했습니다. 이 규정은 면허 시험의 형태로 이루어졌으며, 이후 여러 회원국의 의료 전문직 교육 정책을 형성했습니다.
Although the approaches taken by the EU countries and ASEAN countries are similar, it is worth noting that the context of patient management policies in both settings are different, as well as their education systems. As they work towards the ASEAN Vision 2020 on Partnership in Dynamic Development, ASEAN countries agreed a number of frameworks to enable the free flow of services for professionals in health care. The mutual recognition arrangements (MRA) for nurses, medical and dental practitioners in ASEAN countries were created in 2006-2009 in response to the partnership. This arrangement required countries to establish regulations for migration of health care professionals. The regulations came in the form of a licensing examination, which have subsequently shaped health care professions education policies in a number of member states.

이들 국가에서 국가 면허시험을 도입한 것은 단순히 외국인 의료 전문가를 규제하기 위해서만이 아니라, 졸업생의 자질을 향상시키기 위한 목적도 있었습니다. 아세안 회원국 대부분은 개발도상국으로 의사 수 부족, 의료 전문가의 불균등한 분포, 교육 및 의료 시스템 개발 등 비슷한 문제를 겪고 있습니다(Kittrakulrat, Jongjatuporn, Jurjai, Jarupanich, & Pongpirul, 2014; Sonoda 외, 2017; Tangcharoensathien 외, 2018). 인도네시아의 경우처럼 엄격한 품질 보증 시스템 없이 더 많은 의사를 배출하기 위해 의과대학 수가 증가하면서 졸업생의 자질 저하가 우려되는 문제도 종종 발생합니다. 이들 국가는 기존의 면허 및 재면허 정책을 통해 의료 전문가에 대한 규제가 부족했기 때문에 전 세계적으로 더 나은 의료 서비스를 달성하기 위해 교육과 평가를 활용하는 방법으로 NLE를 도입하기 시작했습니다.

For these countries, introducing national licensing examinations was not solely about the need to regulate foreign health care professionals, but also to improve the quality of graduates. Most of ASEAN member states are developing countries sharing similar challenges: inadequate numbers of physicians, uneven distribution of health care professionals, and a developing education and health care system (Kittrakulrat, Jongjatuporn, Jurjai, Jarupanich, & Pongpirul, 2014; Sonoda et al., 2017; Tangcharoensathien et al., 2018). The challenges are often followed by the increasing number of medical schools to produce more doctors –as in Indonesia’s case, without a rigid quality assurance system, raising concern of lower quality of graduates. As these countries lacked regulation of health care professionals through existing licensing and re-licensing policies, they started to introduce the NLE as a way of using education and assessment to achieve better health care globally.

현재까지 아세안 회원국 10개국 중 태국, 필리핀, 인도네시아, 말레이시아 등 4개국이 NLE를 시행하고 있습니다. 각 국가마다 정책을 도입한 이유는 다릅니다. 

  • 태국과 인도네시아는 유능한 의사를 확보하고 의과대학의 질을 향상시키기 위해 2006~2007년에 자국 및 해외 졸업생을 대상으로 NLE를 도입했습니다. 
  • 인도네시아에서는 지난 10년간 의과대학 수가 증가하면서 의대별로 교육의 질적 차이가 발생하여 보건 전문직 교육의 질을 개선하려는 정부 정책에 영향을 미칠 수 있다는 우려가 제기되었습니다. 
  • 필리핀과 말레이시아는 외국인 졸업생만을 대상으로 하는 NLE를 운영하고 있으며, 최근 몇 년 동안 그 수가 증가하고 있습니다. 

Four out of ten ASEAN member countries have implemented NLEs to date: Thailand, Philippines, Indonesia, and Malaysia. Each country has different reasons for introducing the policy.

  • Thailand and Indonesia introduced the NLE for home and international graduates in 2006-2007, to ensure the quality of competent doctors and to improve the quality of their medical schools. 
  • In Indonesia, there has been an increasing number of medical schools in the last decade, leading to concerns of differential quality of education delivered which impact on the Government’s policy to improve the quality of health professions education.
  • Philippines and Malaysia, both had NLEs for international graduates only, whose number has been increasing in the recent years.

동남아시아 지역의 다른 국가들에는 현재 NLE가 없지만 정책 입안자들 사이에서 이 주제가 논의되고 있습니다(Kittrakulrat 외., 2014; Sonoda 외., 2017). 베트남과 라오스는 NLE를 개발 중이며, 인도는 국제 졸업생들이 자국 의료 시스템에서 일할 준비를 갖추도록 보장하는 정책의 일환으로 NLE를 고려하고 있습니다. 의료 전문직의 이동이 증가하는 추세(Tangcharoensathien 외., 2018)는 NLE가 아세안 경제 공동체(AEC)의 이념에 부합하는 문제이며, 의료 전문직이 이 지역 내 다른 국가에서 의술을 펼칠 수 있는 자격을 갖출 수 있음을 의미합니다(Kittrakulrat 외., 2014). 비교를 위해 대만과 한국 등 아시아의 다른 국가에서는 의료/보건 전문가 면허시험을 시행하고 있으며, 이는 의학교육에 영향을 미치고 있는 것으로 나타났습니다.
Even though the other countries in the South East Asia region currently do not have NLEs, the topic is being discussed amongst policy makers (Kittrakulrat et al., 2014; Sonoda et al., 2017). Vietnam and Lao are in the process of developing NLEs, and India is considering the NLE as part of their policy in ensuring international graduates are prepared to work in its healthcare system. The increasing trend of health care professions’ migration (Tangcharoensathien et al., 2018) means that the NLE is a relevant issue aligned with the idea of ASEAN Economic Community (AEC), where medical professions can be qualified to practice medicine in another country in this region (Kittrakulrat et al., 2014). For a comparison, other countries in Asia such as Taiwan and South Korea have been establishing medical/ health care professionals licensing examinations, which showed impact on medical education.

아세안 국가들은 해외 졸업생 유입에 대응하기 위해 NLE를 시행하고 있지만, 시험이 대부분 현지 언어로 진행되기 때문에 AEC 시대에 면허 시험에 대한 지역적 접근이 복잡해졌습니다. 마찬가지로 문화, 경제, 교육 및 의료 시스템의 차이도 AEC에서 의료 전문가의 자유로운 이동에 또 다른 도전 과제가 되고 있습니다. NLE를 시행하는 국가마다 면허 시험의 방법과 유형은 다양하지만, 지식과 임상 기술을 평가하는 것이 가장 일반적인 결과입니다(Price et al., 2018). 그럼에도 불구하고 현재 시행되고 있는 NLE는 아세안에서 환자 치료와 의학교육에 대한 공통적이고 동등한 기준을 찾을 수 있는 기회를 열어줄 수 있습니다.
Although ASEAN countries have been implementing NLEs to address the influx of international graduates, the fact that the examinations are mostly delivered in local languange makes it complicated for a regional approach of licensing examinations in the era of AEC. Similarly, the difference in culture, economy, education and health care system, also poses another challenge for the free-flow of health care professionals in AEC. Variation of methods and types of licensing examinations exist in countries implementing NLEs; although the knowledge and clinical skills are the most common outcome to be assessed (Price et al., 2018). Even so, the current practice of NLEs might open opportunities for finding common and equal standard for patient care and medical education in ASEAN.

IV. 아세안의 의료 면허 시험에서 얻은 교훈: 의료 전문가 교육에 대한 투자
IV. LESSON LEARNT FROM MEDICAL LICENSING EXAMINATIONS IN ASEAN: INVESTING IN HEALTH CARE PROFESSIONALS’ EDUCATION

아세안 회원국과 같은 개발도상국에서 의사면허시험의 발전은 의료, 더 나아가 보건의료 전문가 교육 시스템을 개선할 수 있는 경로를 제공합니다. 이들 국가에서는 해외에서 자격을 갖춘 의사 수가 국내 졸업생에 비해 많지 않더라도 유능한 의료 전문가를 배출하기 위한 중요한 단계로 간주됩니다. 정책을 바꾸고, 교육 시스템을 개선하고, 더 나은 의료인을 배출하기 위해 한 국가가 어떻게 나아가는지에 대한 이야기입니다.
The development of the NLE in developing countries such as ASEAN members offers a pathway for improving medical and to further extent, health care professionals’ education systems. For these countries, even though the number of overseas qualified doctors may not be significant compared to domestic graduates, the policy was considered an important step towards producing competent health care professionals. It is about how a country moves forward by changing policies, improving education system, and aiming to produce better health care providers.

보건 전문가 자질 향상 정책은 보건 분야 인적 자원의 교육과 훈련을 혁신하는 데 투자하는 WHO 전략과 일치합니다. WHO가 강조하는 혁신 전략은 교육 표준과 자금 확보를 위한 국가 정책 수립을 통해 고등 교육과 보건 부문 간의 협력에 의존합니다. 따라서 NLE를 시행하는 아세안 국가들에게 NLE는 보건 교육 개선을 위한 투자이자 향후 보건의료 시스템을 지원할 수 있는 수단입니다. 
The policy to improve the quality of health professionals is in line with WHO strategy to invest in transforming the education and training of human resources in health. The transformational strategy emphasised by WHO relies on collaborative work between higher education and the health sectors through establishing national policies to secure the education standard and funding. Therefore, for ASEAN countries implementing the NLE, it represents an investment for health education improvement, and a means to support their health care systems in the future.

라오스의 NLE 개발 경험은 라오스가 어떻게 정책을 바꾸고 교육 시스템을 강화했는지 보여줍니다(Sonoda et al., 2017). 라오스는 NLE를 수립하는 과정에서 정책과 이해관계자들과의 협력을 통해 교육 시스템을 강화해야 했습니다. 라오스의 경우 이러한 노력은 더 나은 의료 서비스를 향한 중요한 움직임으로 간주되었습니다. 라오스 정부는 유능한 의료 전문가를 배출하기 위해서는 교육 시스템을 개선해야 한다는 점에 주목하여 교육 커리큘럼과 평가 관행을 수정했습니다.

  • 라오스의 경험 사례는 NLE 도입이 유능한 전문가를 배출하는 데 필요한 보건의료 전문직 교육의 개선을 이끌었음을 보여줍니다(Sonoda et al., 2017).
  • 태국에서는 면허 시험이 학부 교육(예: 위험에 처한 학생 인식)과 졸업후 교육의 변화를 이끌었습니다.
  • 싱가포르에서는 졸업후 과정에 국가시험을 도입하여 교육의 질을 보장하고 있습니다(Ruth et al., 2018). 

Lao’s experience in developing the NLE showed how the country changed its policy and strengthened their education system (Sonoda et al., 2017). In the process of establishing the NLE, Lao needed to strengthen their education system through policies and working with their stakeholders. In Lao’s case, these efforts were considered as significant moves toward better healthcare. The government took note that to produce competent health care professionals, they needed to improve the education system which made them revise educational curricula and assessment practices.

  • The example from Lao’s experience showed that the introduction of NLE drives improvement in health care professions education which is necessary to produce competent professionals (Sonoda et al., 2017).
  • In Thailand, the licensing examination drove changes in undergraduate education (e.g. recognising at-risk students) and postgraduate education.
  • While in Singapore, the national examinations have been introduced in postgraduate studies for ensuring quality assurance (Ruth et al., 2018).

아세안 국가들의 의료 서비스 개선에 있어 NLE의 역할은 아직 더 커질 가능성이 있습니다. 아세안에서 인구가 가장 많은 인도네시아의 경우, 의대 졸업생을 대상으로 한 NLE는 보건 인적 자원을 강화하고 의료 전문가의 자질을 향상시키기 위한 정부 전략의 일환이었습니다. 2013년 의사 역량 시험 공동위원회에서 보고한 바와 같이, 이 시험은 장기적으로 의과대학을 포함한 의학교육의 질을 높이기 위해 고안되었습니다. 인도네시아의 최근 연구(Hidayah, 2018)에 따르면 NLE는 교육 관행 개선, 학습 자원 및 시설 개선, 교육 기관과 의료 서비스 제공자 간의 협력 등 의학교육에 여러 가지 긍정적인 결과를 가져왔다고 강조합니다. 2007년 시행 이후 치의학, 간호학, 조산학 등 다른 보건의료 전문직도 인증제 강화와 함께 발전하고 있습니다. 이해관계자 간의 협력과 인도네시아의 문화는 교육을 확대하는 데 공동의 노력을 이끌어냈습니다. 히다야의 연구(2018)에 따르면 의과대학은 유능한 졸업생을 배출하기 위해 교육의 질을 업그레이드하고 있습니다. 이러한 노력은 커리큘럼 및 평가 개발을 위한 협력 강화, 임상 배치 확대, 교수진 개발을 위한 협력 등의 형태로 이루어졌습니다. 예를 들어, 인도네시아 국가위원회에서 보고한 것처럼 의과대학 간 평가 실습을 위한 지역별 협력이 있었습니다. 특히 신생 의과대학과 사립 의과대학의 경우 이러한 협력은 교육 실습을 활용하는 데 도움이 됩니다. 연구에 따르면 인도네시아와 같은 의학교육 개발의 맥락에서 NLE는 규제기관(예: 정부)과 주요 이해관계자(예: 의과대학협회)의 중요한 역할을 연결하여 개선을 촉진하는 촉매제 역할을 한다고 강조합니다(Hidayah, 2018). 장기적으로 이러한 교육의 질 개선은 의학교육과 의대 졸업생의 질을 보장함으로써 보건의료의 향상을 가져올 것으로 기대됩니다.
The extent of the NLEs’ roles in bringing health care improvement in ASEAN countries is potentially greater still. In Indonesia, which has the largest population in ASEAN, the NLE for medical graduates was part of government strategies to empower human resources for health and improve the quality of health care professionals. As reported by the Joint Committee on Medical Doctor Competence examination in 2013, the examination was designed to lever the quality of medical education, including medical schools, in the long run. Emergent work from Indonesia (Hidayah, 2018) highlights that the NLEs led to a number of positive consequences for medical education:  improvement in educational practices, improvement of learning resources and facilities, and collaboration between education institutions and health care providers. Since its implementation in 2007, other health care professionals such as dentistry, nursing, and midwifery have been developing, along with the strengthening of accreditation system. The collaboration between stake holders and the culture of Indonesia led to the collective work in scaling up the education. Hidayah’s work (2018) revealed that medical schools have been upgrading their education quality to produce competent graduates. The effort came in the form of increasing collaboration for curriculum and assessment development, expanding clinical placements, and collaboration for faculty development. For example, there have been regional collaboration for assessment practice between medical schools as reported by the National Committee in Indonesia. Especially for new and private schools, this collaboration helps them in leveraging their educational practice. Research highlights that in the context of developing medical education such as Indonesia, the NLE acts as a catalyst to drive improvement, linking the vital role of the regulator (i.e. the government) and major stakeholders (e.g. association of medical schools) (Hidayah, 2018). In the long term, this improvement in the quality of education is expected to bring improvement in health care by ensuring the quality of medical education and medical graduates.

광범위한 보건 전문직 교육 분야에서 국가 면허 시험은 여전히 뜨거운 논쟁의 대상이 되고 있는 정책입니다(Archer et al., 2017). 그러나 아세안 국가들의 국가면허시험 시행 경험은 보건의료의 질을 향상시키기 위해 보건의료 전문가의 질 보증 시스템에서 이 높은 수준의 평가가 더 큰 역할을 할 수 있는 기회를 보여줍니다. 앞으로 교육 정책이 개발도상국의 의료 서비스 질 향상을 지원하는 방법에서 얻은 교훈은 전 세계 의료 문제에 대한 접근 방식을 개선할 수 있는 잠재력을 가지고 있습니다.

In the broader field of health professions education, national licensing examinations remain a hotly debated policy (Archer et al., 2017). However, the experience of ASEAN countries in implementing NLEs reveals opportunities for a bigger role for this high-stakes assessment in the quality assurance system of health care professionals in order to enhance the quality of health care. In the future, lessons learnt from how an education policy can support developing countries in improving their health care quality, has the potential to enhance the approach to health care problems globally.

 


Abstract

Southeast Asian region has been experiencing significant challenges to their health care systems, driving some countries establishing policies to ensure the quality of the medical workforce through the introduction of national licensing examinations. This paper examines the experiences of Southeast Asian countries in implementing licensing examinations, highlighting the changes and development of the education policy and its impact on medical education. The lesson learnt from the high-stakes assessment policy offers a new perspective on how medical licensing examinations could represent a potential regional approach for a better health care.

말레이시아의 의학교육: 질 vs 양 (Perspect Med Educ, 2017)
Medical education in Malaysia: quality versus quantity
Rebecca S. Y. Wong1 · Samiah Yasmin Abdul Kadir1

 

말레이시아의 의학교육은 1963년 말라야 대학교 의과대학 설립으로 거슬러 올라가는 오랜 역사를 가지고 있습니다[1]. 현재 말레이시아에는 32개의 의과대학이 있습니다(공립 11개, 사립 21개). 2014년 말 기준 말레이시아 의과대학에 재학 중인 학생 수는 18,789명이며, 해외에서 의학을 공부하는 말레이시아인은 약 15,000명으로 추산됩니다[2]. 이는 향후 5년 내에 3만 명의 의사가 말레이시아 의료 시스템에 합류하고 2018년까지 의사 수가 두 배가 될 것임을 의미합니다[2]. 이러한 의사 대량 배출을 정당화하기 위해 세계보건기구(WHO)가 권고하는 선진국의 인구 400명당 의사 1명의 비율은 종종 사용되어 왔습니다. 
Medical education in Malaysia has a long history dating back to the establishment of the Faculty of Medicine at the University of Malaya in 1963 [1]. Currently, there are 32 medical schools in the country (11 public and 21 private). At the end of 2014, there were 18,789 students in all years in Malaysian medical schools and another estimated 15,000 Malaysians studying medicine abroad [2]. This implies that 30,000 doctors will join the Malaysian healthcare system within the next five years and by 2018 the number of doctors will double [2]. The World Health Organisation’s (WHO) recommendation of 1 doctor to 400 persons for a developed nation has frequently been used to justify this mass production of doctors.

의사가 너무 많은 것도 문제가 될 수 있지만, 질이 낮은 의사가 너무 많으면 문제가 더 악화될 수 있습니다. 2009년 이후 의학을 공부하기 위해 외국 대학에 입학하는 의대생의 20%가 최소 입학 자격을 갖추지 못했으며[3], 말레이시아 주류 언론에서는 의대 졸업생의 자질 부족에 대한 뉴스가 심심찮게 보도되고 있습니다. 특히 지난 10년간 의과대학의 확산과 관련된 또 다른 문제는 학교 자체의 질입니다. 최근 몇 년 동안 소규모 또는 덜 안정적인 사립 의과대학이 재정 문제에 직면하여 일부 의과대학이 문을 닫거나 국제 파트너가 협력을 철회했다는 보고가 있었습니다. 
While having too many doctors can be problematic, having too many poor-quality doctors can certainly worsen the problem. Since 2009, 20% of the medical students who enter foreign universities to study medicine lack the minimum entry qualifications [3] and news on the poor quality of medical graduates is not uncommon in the mainstream media of Malaysia. Another issue concerning the proliferation of medical schools, particularly in the past decade, is the quality of the schools themselves. In recent years there have been reports of smaller or less stable private medical schools facing financial problems, with some of them closing down or having their international partners withdraw their collaboration.

수량 문제를 해결하기 위해 정부는 2010년에 의대 입학 프로그램을 5년간 유예(2011년부터 2016년까지)한다고 발표했고[4], 2014년부터는 공립 의대에 입학하는 학생 수가 감소하고 있습니다. 또한, 소규모 의과대학이 문을 닫거나 규모를 축소하거나 소규모 캠퍼스를 대규모 캠퍼스로 합병하는 등 업계에서 이미 일부 통폐합이 이루어지고 있습니다[5].
To combat the issue of quantity, the government announced a five-year moratorium (from 2011 to 2016) on medical programmes in 2010 [4], whereas in 2014, the number of students being accepted into public medical schools has decreased. Besides, some consolidation has already taken place in the industry with the smaller players closing down, downsizing or the merging of the smaller campuses into bigger ones [5].

한편, 인턴십 기간이 1년에서 2년으로 연장된 것은 2008년부터 시작되었습니다. 이는 교육의 질 문제를 해결하기 위한 것이지만, 또 다른 문제를 야기했습니다. 의대 졸업생이 증가하고 인턴 기간이 연장되면서 국공립 병원에는 하우스 오피서가 넘쳐나고 있습니다. 최근에는 2015년에 보건부가 더 이상 인턴십 배치를 주선하지 않고 의대 졸업생이 직접 인턴십을 제공하는 국공립 병원에 지원해야 한다고 발표했습니다[6]. 이로 인해 공석이 한정되어 있고 의대 졸업생 수가 증가함에 따라 인턴십 대기 기간이 길어질 수 있습니다.
On the other hand, the extension of the duration of internship from one to two years started in 2008. While this is supposed to address the issue of quality, it has contributed to yet another problem. With the rising number of medical graduates and the extension of internship, the public hospitals are now flooded with house officers. More recently, in 2015, the government announced that the Ministry of Health will no longer arrange internship placements and that medical graduates are to apply to the government hospitals that offer internship themselves [6]. This may result in a longer wait for internship as there are limited vacancies and an increasing number of medical graduates.

말레이시아의 의료 시스템과 의료 교육이 직면한 몇 가지 시급한 문제를 해결하기 위해 현재 상황을 개선하기 위해 다음과 같은 조치를 취할 것을 제안합니다:
With a handful of burning issues facing the healthcare system and medical education in Malaysia, we propose that the following actions are taken to improve the current situation:

  • 전략적 장기 계획: 말레이시아의 의학교육을 위한 전략적 10년 이상의 장기 계획은 매우 중요합니다. 의료 및 의학교육 시스템에 변화를 부과하고 모니터링할 수 있는 보다 체계적인 방법이 있어야 합니다.
    A strategic long-term plan: A strategic ten-year or longer-term plan for medical education in Malaysia is crucial. There must be a more organized way of imposing and monitoring changes in the healthcare and medical education systems.
  • 글로벌 표준 준수: 말레이시아 의과대학은 세계의학교육연맹(WFME)에서 정한 표준을 참조해야 합니다. WFME는 1997년에 의학교육의 국제 표준에 관한 프로그램을 시작했으며, 2004년부터 의학교육을 개선하기 위해 WHO/WFME 전략적 파트너십의 지원을 받고 있습니다[7]. 말레이시아 상황에 맞게 수정할 수 있는 이 표준은 국가 규제 당국부터 커리큘럼 개발자, 프로그램 평가 및 인증을 담당하는 사람들에 이르기까지 의학교육의 여러 중요한 영역을 다루기 때문에 매우 유용합니다[8]. 
    Adhering to global standards: Malaysian medical schools should refer to the standards set by the World Federation for Medical Education (WFME). The WFME started a programme on the international standards in medical education in 1997, which has been supported by the WHO/WFME Strategic Partnership to improve medical education since 2004 [7]. These standards, which can be modified to suit the Malaysian context, are very useful as they cover many important areas of medical education [8], from the national regulators to the curriculum developers, and those responsible for the assessment and accreditation of the programmes.
  • WFME 프로그램을 사용하는 의과대학 인증: WFME와 WHO는 인증 기관은 아니지만 의과대학, 국가 기관 및 당국이 기본 의학 교육 인증을 수립하는 데 도움을 주는 프로그램을 제공합니다[9].
    Accreditation of medical schools using the WFME programme: Although the WFME and WHO are not accreditation bodies, they provide a programme for assisting medical schools, national agencies and authorities in establishing accreditation of basic medical education [9].
  • 기준 상향 조정: 입학 요건(특히 사립 의과대학의 경우)을 높여야 할 뿐만 아니라 졸업 요건도 높여야 합니다. 이는 우수한 학생들이 의과대학에 입학하고 우수한 학생들만이 의과대학을 졸업할 수 있도록 하기 위한 것입니다. WFME는 시험의 횟수와 성격이 지식 기반과 통합 학습의 습득을 장려해야 한다고 권고합니다. 평가 방법의 신뢰성과 타당성은 모두 중요하며, 의과대학은 이러한 평가가 외부 전문가에 의해 면밀히 검토될 수 있도록 보장해야 합니다[8]. 
    Raising the bar: Not only should the entry requirements be raised (especially for the private medical schools), the exit requirements should also be raised. This is to ensure high-quality students get selected into the course and only high-quality students are allowed to graduate from medical schools. The WFME recommends that the number and nature of examinations should encourage the acquisition of the knowledge base and integrated learning. Both the reliability and validity of the assessment methods are important, and medical schools should ensure that these assessments are open to scrutiny by external expertise [8].
  • 의대 과정 연장5년의 학부 의학 프로그램을 운영하는 학교의 경우, 이 과정을 6년으로 연장하면 학생들이 의료 행위를 시작하기 전에 더 적절하게 훈련받을 수 있습니다. 다시 말하지만, 프로그램 구조, 구성 및 기간과 관련하여 WFME 표준을 참조할 수 있습니다. 학생들이 적절한 교육을 받아야 할 뿐만 아니라, WFME는 기초 의학의 수평적 통합과 기초 의학 및 임상 과학 간의 수직적 통합을 강조합니다[8]. 
    Lengthening the medical course: For schools that run a five-year undergraduate medical programme, extending the course to six years may allow students to be more adequately trained before they start practising medicine. Again, the WFME standards may be referred to with regards to the programme structure, composition and duration. Not only should students be adequately trained, the WFME places emphasis on a horizontal integration of basic medical sciences and a vertical integration between basic medical sciences and clinical sciences [8].
  • 소규모 의과대학의 합병 또는 통폐합: 재정 위기에 처한 일부 의과대학의 현재 상황이 반복되지 않도록 소규모 의과대학의 추가 합병 또는 통폐합이 필요할 수 있습니다.
    Merging or consolidation of the smaller medical schools: It may be necessary to further merge or consolidate the smaller medical schools to avoid a repeat of the current situation with some medical schools in financial crisis.
  • 진로 상담: 진로 상담은 중등 교육이 끝날 무렵에 학생들에게 제공되어야 합니다. 학생과 학부모 모두에게 의대 외에 똑같이 좋은 다른 선택지에 대한 정보를 제공해야 합니다. 의대를 졸업한 학생의 경우 의학 연구원, 의학 강사, 제약회사의 의료 고문과 같은 다른 직업 옵션에 노출되어야 합니다. 
    Career counselling: This should be given to students towards the end of their secondary school education. Both students and parents need to be informed of other equally good options besides medicine. For those who have graduated from medical school, they should be exposed to other career options such as medical researcher, medical lecturer or medical advisor in pharmaceutical companies.
  • 인턴십과 2년간의 의무 복무를 신뢰할 수 있는 민간 병원에서 수행할 수 있도록 허용해야 합니다: 이는 공보의와 의무장교를 민간 병원으로 전환할 수 있습니다. 그러나 민간 병원의 영리 목적과 환자들은 이러한 개념을 환영하지 않을 수 있습니다.
    Allowing internship and the two-year compulsory government service to be carried out in credible private hospitals: This can divert house officers and medical officers to the private hospitals. However, the for-profit nature of private hospitals and the patients may not welcome this concept.
  • 도시에서 농촌 지역으로 하우스 오피서와 의료 오피서의 재배치: 말레이시아의 일부 지역에서는 의사 공급 과잉으로 인턴십 배치를 받기 위해 더 오래 기다려야 하므로, 정부는 인턴십의 일부(예: 2년 중 6개월)를 농촌 지역에서 의무적으로 이수하도록 하는 방안을 고려해야 합니다.
    Redistribution of house officers and medical officers from urban to rural areas: With an oversupply of doctors in some parts of Malaysia and a longer wait in obtaining an internship placement, the government should consider making part of the internship (e. g. six months out of the two years) compulsory in the rural areas.
  • 인력이 부족한 전문 분야로 의료 장교 유입 증가 : 정부는 인기있는 전문 분야의 자리를 놓고 경쟁하는 대신 인력이 부족한 분야로 의료 장교를 전환 할 수 있습니다.
    Increase in the intake of medical officers into specialization with shortage of manpower: Instead of competing for places in popular specialties, the government can divert medical officers to areas where there is limited manpower.

말레이시아의 의학교육은 이제 전략을 바꿔야 할 때입니다. 이를 위해서는 교육부, 보건부, 말레이시아 의학위원회, 말레이시아 자격청이 긴밀히 협력하여 상황이 통제 불능 상태가 되기 전에 말레이시아 의학교육을 위한 실현 가능한 장기 전략 계획을 수립해야 합니다.
It is time for medical education in Malaysia to change strategy. This requires the Ministry of Education, the Ministry of Health, the Malaysian Medical Council and the Malaysian Qualifications Agency to work closely on a feasible long-term strategic plan for medical education in Malaysia before things become out of control.


Perspect Med Educ. 2017 Feb;6(1):10-11. doi: 10.1007/s40037-016-0319-8. Epub 2017 Jan 3.

Medical education in Malaysia: quality versus quantity

Affiliations

1Faculty of Medicine, SEGi University, Kota Damansara, Selangor, Malaysia. rebecca@segi.edu.my.

2Faculty of Medicine, SEGi University, Kota Damansara, Selangor, Malaysia.

PMID: 28050881

PMCID: PMC5285279

DOI: 10.1007/s40037-016-0319-8

Free PMC article

말레이시아 의사면허시험: 이것이 나아갈 길인가?(Education in Medicine Journal. 2017)
Malaysian Medical License Examination (MMLE): Is This a Way Froward?
Amit Bhardwaj1, Kavitha Nagandla2, Sadoon Ibrahim3

 

 

소개
INTRODUCTION

말레이시아의 의학교육은 현재 이 시스템이 근본적인 환자 중심 치료를 위한 적절한 역량을 갖춘 의사를 배출하는지에 대한 도전에 직면해 있습니다(1). 현재 기존 교육 프로그램이 안전하고 유능한 의사를 배출하는 커리큘럼을 제공하는 데 충분히 견고한지 평가하는 프로세스가 없습니다. 보건부(MOH)는 모든 졸업생이 의사 면허를 취득하기 전에 반드시 응시해야 하는 공통 면허 시험으로 말레이시아 의사 면허 시험(MMLE)을 제안했습니다. 현재 이 시험은 외국 의과대학을 졸업한 말레이시아 학생들만 응시할 수 있습니다. 말레이시아 현지의 모든 공립 및 사립 대학은 학부생들이 향후 하우스맨십을 준비할 수 있도록 서로 다른 커리큘럼을 가지고 있습니다. 이 문제를 고려할 때 말레이시아의 모든 졸업생에게 의무적으로 의료 면허 시험을 실시하면 지식과 기술을 모니터링하는 데 도움이 될 수 있습니다. 말레이시아도 현지 대학 졸업생을 포함한 모든 의대 졸업생에게 의무 면허 시험을 고려할 때가 되었을까요? 
Medical education in Malaysia is currently facing challenges in trying to address whether the system produces doctors with adequate competencies for fundamental patient-centred care (1). Currently there is no process to evaluate whether the existing educational programs are robust enough in delivering curricula that produce safe and competent doctors. The Ministry of Health (MOH) proposed the Malaysian Medical Licensing Examination (MMLE) as a common licensing examination for which all graduates must sit prior to obtaining registration to practice. Currently, this exam is only taken by Malaysian students graduating from foreign medical colleges. All the local Malaysian universities, both public and private, have different curricula for preparing undergraduates for future housemanships. Considering this issue,
a mandated medical licensing exam for all graduates in Malaysia could help in monitoring their knowledge and skills. Is it time for Malaysia to consider a mandatory licensing exam for all medical graduates, including those of local colleges?

말레이시아 의료 면허 시험의 동인
THE DRIVERS FOR MALAYSIAN MEDICAL LICENSING EXAM

역사적 맥락
Historical Context

미국과 캐나다의 데이터는 졸업 시험이 유효하고 신뢰할 수 있으며, 졸업생의 임상 능력과 향후 성과 사이에 적절한 상관관계가 있음을 보여줍니다(2). 면허 시험에서 더 나은 성적을 거둔 학생이 더 나은 환자 진료를 수행한다는 증거도 있습니다(3, 4). 그러나 평판이 좋은 대학의 연구 결과에서 국가 면허 시험과 환자 치료 결과 개선 사이의 직접적인 연관성을 입증하지 못했다는 점은 우려스러운 부분입니다(5, 6).
Data from the US and Canada demonstrate that exit exams can be valid and reliable, showing adequate correlation between a graduate
s clinical skills and future performance (2). There is evidence suggesting that students who perform better in licensing exams practice better patient care (3, 4). However, it is concerning that findings from reputable universities have failed to establish a direct link between national licensing examinations and improved patient outcomes (5, 6).

정치적 맥락
Political Context

말레이시아의 세계화로 인해 의사에 대한 수요가 증가했으며, 이러한 수요는 사립 의과대학에서 충족되었습니다. 이러한 의과대학에 대한 인증은 국가 인증위원회와 말레이시아 의학위원회에서 수행합니다. 인증 시스템이 있음에도 불구하고 측정은 주로 결과보다는 과정에 초점을 맞추고 있습니다(7, 8). 따라서 기대되는 역량을 표준화하는 역할을 할 수 있는 국가 의료 면허 시험이 필요합니다. 이는 아마도 환자 안전이 주도하는 사회 문화적 변화로 인식될 수 있습니다.

  • 반면, 의과대학의 허점을 드러내는 시험이 의과대학의 권한에 어떤 영향을 미칠 것이며, 자율성을 유지하려는 의과대학의 기득권과 충돌하지는 않을까요?
  • 또한, 면허 시험이 모든 동남아시아국가연합(ASEAN) 국가에서 유효한 것으로 간주될까요? 국경을 넘어 의료 서비스를 제공하는 의료 전문가의 업무가 중요한 것으로 간주되는 아세안 경제 공동체(AEC) 이니셔티브를 지원할 수 있을까요(9)?

Globalisation in Malaysia has resulted in an increased demand for doctors, and this demand has been fulfilled by private medical schools. The accreditation of these medical schools is conducted by the National Accreditation Board and the Malaysian Medical Council. Despite an accreditation system, measurements have largely focused on processes rather than outcomes (7, 8). Therefore, the compulsion is for a national medical licensing examination that can serve to standardise the competencies expected. This would perhaps be perceived as a socio-cultural change driven by patient safety.

  • On the other hand, how will an exam that exposes the lacunae in medical schools affect the power of the colleges and will it conflict with their vested interest in retaining autonomy over their businesses?
  • Furthermore, will the licensing examination be considered valid in all the Association of Southeast Asian Nations (ASEAN) countries and will it support the ASEAN economic community (AEC) initiative, for which the practice of health professionals providing cross-border health services is considered crucial (9)?

세계의학위원회(GMC)는 기본 의학 학위를 어디서 공부했는지에 관계없이 모든 졸업생에게 의료 면허 평가(MLA)를 의무적으로 이수해야 한다고 발표했습니다. 2020년까지 GMC 등록을 위한 공통 기준을 설정하는 것이 목표입니다. 이러한 선례와 말레이시아 의료 정책 지침에 영국 프로토콜을 채택한 것을 통해 저자는 말레이시아 의료 위원회가 말레이시아의 미래 의료 교육을 선도하고 있다고 말합니다(10).
The General Medical Council (GMC) has announced that completion of the Medical Licensing Assessment (MLA) is mandatory for all graduates, irrespective of where they studied for their basic medical degree. The aim is to set a common threshold for entry into the GMC register by 2020. With this precedent, and with the adoption of UK protocols into Malaysian health care policy guidelines, the authors opine that the Malaysian Medical Council is leading medical education in Malaysia into the future (10).

교육적 맥락
Educational Context

말레이시아의 대부분의 의과대학은 커리큘럼 전달을 위해 장기 시스템 기반 모듈을 사용하는 통합 접근 방식을 채택하고 있습니다. Richard Hays는 잠재적, 계획된, 전달된, 평가된 커리큘럼과 숨겨진 커리큘럼 간의 차이점과 관계를 설명했습니다(11). 계획된 교육과정, 전달된 교육과정, 평가된 교육과정이 일치하지 않으면 계획된 교육 목표 달성에 방해가 되는 바람직하지 않은 교육과정(숨겨진 교육과정)이 드러날 수 있으며, 이를 교육과정 전쟁이라고 합니다(그림 1). 국가 면허 시험은 평가되는 커리큘럼을 명시함으로써 의과대학이 계획대로 커리큘럼을 제공하고 있는지 판단하는 데 도움이 될 것으로 기대됩니다. 그러나 면허 시험이 부과되면 새롭고 혁신적인 교수 학습 전략을 교육과정에 도입하려는 열정이 꺾일까요? (12). 또한, 면허 시험이 시행되면 MMLE를 통합하고 평가 간 중복을 피하는 통일된 자격 최종 시험을 개발하기 위해 말레이시아 의학위원회 및 의과대학 평가 전문가를 포함한 주요 이해 관계자의 조정된 계획이 필요합니다.
The majority of medical schools in Malaysia have adopted an integrated approach that uses organ systems-based modules for delivery of the curricula. Richard Hays has illustrated the differences and the relationship between potential, planned, delivered, assessed and hidden curricula (11). The planned, delivered and assessed curricula must match, as a mismatch might surface an undesirable curriculum
the hidden curriculumthat would hinder the achievement of the planned educational objectives, a concept referred to as a curriculum war (Figure 1). It is expected that a national licensing exam will make explicit the assessed curriculum; this will help to determine whether medical schools are delivering the curriculum as planned. However, if a licensing exam is imposed, would it deter enthusiasm for bringing new and innovative teaching and learning strategies into the curriculum? (12). Moreover, a licensing exam would mandate the need for coordinated planning by key stakeholders, including the Malaysian Medical Council and medical school assessment experts, in order to develop a unified, qualifying final exam that integrates the MMLE and avoids redundancy between assessments.

이론적 배경
Theoretical Context

국가 면허 시험의 근간이 되는 이론은 국가 시험을 통해 의사가 진료를 시작하기 전에 최소한의 역량 기준을 달성했는지 확인한다는 것입니다. 응시자가 국가 표준에 따라 유능하게 해결해야 하는 실제 임상 업무와 문제를 반영하는 임상 관련 청사진을 구축하는 것이 우선입니다. 스완슨 외(13) 및 반 데르 블뢰텐 외(14)에 따르면, 적절한 수준의 콘텐츠 유효성과 신뢰성을 달성하려면 임상 문제 전반에 걸쳐 광범위한 샘플링이 필요하다고 합니다. 효율적인 샘플링을 위해 객관식 구조화 임상시험(OSCE)과 객관식 문제를 결합하는 것이 해결책이 될 수 있습니다. 

  • 그러나 어떤 평가 형식을 사용하든 모든 의과대학 졸업생의 실력이 우수하고 환자 안전이 보장되는지 얼마나 정확하게 판단할 수 있을까요? 
  • 또한, 소송에 관한 문헌이 지식보다는 '소프트 스킬'에 더 초점을 맞추고 있다는 점을 고려할 때, 주로 지식 기반 또는 OSCE 스타일의 면허 시험이 이러한 역량을 적절하게 다룰 수 있을까요(14)? 

The theory underpinning the national licensing exam is that national examinations ensure that doctors have achieved minimum standards of competency before beginning to practice. The priority is the construction of a clinically relevant blueprint that reflects the actual clinical tasks and problems that the candidates will be required to competently address in accordance with national standards. According to Swanson et al. (13) and Van der Vleuten et al. (14), a wide sampling across clinical issues is required if an adequate level of content validity and reliability is to be achieved. The solution is perhaps to combine the objective structured clinical examination (OSCE) with multiple-choice questions for efficient sampling.

  • However, whatever assessment format is used, how precisely will one be able to determine whether every graduate of every medical school is good and that patient safety is assured?
  • Furthermore, given that the literature on litigation focuses more on ‘soft skills’ than on knowledge, will a licensing exam that is largely knowledge-based, or even OSCE-style, address this competency adequately (14)?

미국 의사 면허 시험(USMLE) 1단계 및 2단계 점수를 대학원 레지던트 선발 결정에 계속 사용하는 것에 대한 실망스러운 보고도 있습니다(4). 산부인과 핵심 역량 절차에 대한 말레이시아 전공의 수련(n = 407)에 대한 최근 전국적 연구에서, 의과대학 졸업생들마다 어느 의대를 졸업했느냐에 따라 전공의 수련에 시작할 때의 절차적 기술과 지식 적용에 눈에 띄게 차이가 있다는 우려가 확인되었습니다. 이 연구는 하우스맨십을 준비하는 대학원 교육의 적절성과 관련하여 의과대학에 제공하는 피드백에 시사점을 제공합니다. 이 연구 결과는 MMLE에 의한 통합 평가 또는 의과대학의 최종 시험에 MMLE를 통합해야 할 필요성을 강조합니다(15,16).
There are discouraging reports on the continued use of United States Medical Licensing Examination (USMLE) Step 1 and Step 2 scores for postgraduate medical residency selection decisions (4). In a recent nationwide study on the Malaysian house officers training (n = 407) needs in core competency procedures in Obstetrics & Gynaecology, concerns were identified that graduates from different medical schools exhibited noticeably variable procedural skills and knowledge application upon entering their housemanships. This study has implications for the feedback given to medical schools regarding the adequacy of graduate training in preparation for housemanships. This finding highlights the need for a unifying assessment by MMLE or for the integration of MMLE into medical schools
final exams (15,16).

사회경제적 맥락
Socioeconomic Context

시험의 이점이 시험 비용을 정당화하기에 충분한가? 시험의 이점에 대한 증거는 명확하지 않습니다(6). 국가가 이러한 시험의 결과를 측정하고 관리할 준비가 되어 있는가? 이 시험은 최근 졸업생들의 재정적 부담을 더욱 가중시킬 것입니다. 좋은 문제를 출제하는 방법과 표준 시험에 대한 여러 의과대학의 동의를 얻는 방법 등이 또 다른 과제입니다. 문제를 계획하고, 구성하고, 시험 결과를 분석하는 과정은 고된 작업입니다. 시험을 치를 수 있는 공간, 시험에 필요한 자금, 그리고 이 과정을 시작하고 유지할 수 있는 헌신적인 교수진이 상당히 필요합니다.
Are the advantages of the examination sufficient to justify its expense? Evidence of the exam
s benefits is far from clear-cut (6). Is the country ready to measure and manage the outcomes of such an exam? The exam will further increase the financial burden on recent graduates. Other challenges include how to write good questions and how to obtain agreement among different medical schools on the standard exam. The process of planning, constructing the questions and analysing the exams results is arduous. There is a significant need for space to accommodate the test, money to fund it, and committed faculty to initiate and maintain the process.

결론
CONCLUSIONS

사립 의과대학의 수를 늘림으로써 인력 수요 증가를 충족시킬 수 있다는 것은 의심의 여지가 없습니다. 그러나 자격을 갖춘 의사가 원하는 결과를 달성 할 수 있도록 의료 역량 표준화와 관련하여 계획, 자원 통합, 인프라 구축 및 품질 보증 시스템 개발에 대한 요구 사항이 있어야합니다. 이러한 요구가 해결된다면 의료 면허 시험은 의료 시스템을 발전시키고 말레이시아를 동남아시아 의학교육의 중심지로 만들 수 있는 길입니다. MMLE를 독립형 시험으로 유지하든, GMC의 계획대로 최종 시험에 통합하든, 이해관계자들은 면허 시험 시행을 위한 최선의 전략을 파악하기 위해 분석에 착수해야 합니다.
There is no doubt that an increase in work force demand is fulfilled by increasing the number of private medical schools. However, there should be requirements for the planning, consolidation of resources, infrastructure creation and development of a quality assurance system with regards to the standardisation of medical competencies so that qualified doctors are ensured to have achieved the desired outcomes. If this need is addressed, the way forward is a medical licensing exam that will advance the health care system and that can perhaps make Malaysia a nodal centre for medical education in South East Asia. Whether MMLE remains a standalone exam or is integrated into final exams, as planned by GMC mandates, stakeholders should undertake analysis to identify the best possible strategy for the implementation of the licensing exam.


Medical education in Malaysia is facing challenges related to the increasing number of local private and foreign medical universities. The key issue is whether the system is producing doctors who have adequate competencies for fundamental patient-centred care. The Malaysian Medical Licensing Examination (MMLE) was proposed by the Ministry of Health (MOH) as a common licensing examination for which all graduates must sit prior to obtaining registration to practice. Currently, this exam is only taken by Malaysian students graduating from foreign medical colleges. However, the local Malaysian universities, both public and private, have different curricula for preparing undergraduates for future housemanships. The question is whether the educational programs of these universities are robust enough in delivering curricula that produce safe and competent doctors. Moving forward, it must be determined whether there is a need for extending the Malaysian Medical Licensing Examination (MMLE) to all graduates of both local and foreign medical universities, thereby creating a marker of excellence by which to measure Malaysian education and practice.

보건의료전문직 국가면허시스템의 진화: 라오스의 질적기술사례연구 ( Hum Resour Health. 2017)
The evolution of the national licensing system of health care professionals: a qualitative descriptive case study in Lao People’s Democratic Republic 
Miwa Sonoda1,2,5* , Bounkong Syhavong3, Chanphomma Vongsamphanh3, Phisith Phoutsavath3, Phengdy Inthapanith3, Arie Rotem4 and Noriko Fujita2 

 

배경
Background

의료 전문가 인증은 현대 의료 인력 개발의 중요한 단계입니다. 인증은 평가 과정을 성공적으로 완료했음을 의미하며, 의료 전문가로 인정받을 수 있는 개인의 역량과 적합성을 검증합니다[1]. 이 인증을 받은 개인은 특정 활동을 법적으로 의무화하거나 금지하는 정의된 전문 표준 내에서 의료 서비스를 제공할 수 있는 자격을 얻게 됩니다. 대부분의 국가에서 의료인이 유효한 면허 없이 의료 행위를 하는 것은 불법입니다[2]. 전문가 면허에 대한 기준과 절차를 수립하고 시행하려면 일반적으로 지정된 보건 당국, 전문가 단체, 교육 기관 및 지역사회 전반과 같은 기타 이해관계자가 참여하는 법적 및 제도적 프레임워크가 필요합니다[3]. 전문가의 역량과 지식을 규제해야 할 뿐만 아니라 진료의 질과 안전도 보장해야 합니다. 
The certification of health care professionals is a crucial step in the development of a modern health care workforce. Certification marks the successful completion of a process of assessment and verifies the competence and suitability of an individual to be recognized as a health care professional [1]. This recognition entitles the individual to provide health care within defined professional standards that legally mandate or prohibit certain activities. In most countries, it is illegal for health care practitioners to conduct medical practice without a valid license [2]. The formulation and implementation of standards and procedures for professional licensing require a legal and institutional framework that commonly involves designated health authorities, professional organizations, and other stakeholders such as educational institutions and the community at large [3]. Not only should the professionals’ competences and knowledge be regulated but the quality and safety of care needs to also be ensured.

공공 및 민간 의료 시스템의 품질과 안전을 모두 보장해야 할 필요성에 대한 관심이 높아지면서 의료 전문가 면허를 위한 강력한 규제 및 제도적 프레임워크의 필요성이 강조되고 있습니다. 세계보건기구(WHO)는 보편적 건강보장(UHC) 달성을 위한 필수 조건으로 보건의료 인력의 가용성, 접근성, 수용성, 질을 강조하면서 보건의료 인적 자원(이하 HRH)을 더 나은 건강을 위한 핵심 구성 요소로 인식했습니다[4, 5]. 
Increasing attention is being paid to the need to ensure both the quality and safety of the public and private health care systems, thereby accentuating the need for a strong regulatory and institutional framework for the licensing of health care professionals. The World Health Organization (WHO) recognized human resources for health (hereafter referred to as HRH) as a key building block for better health, emphasizing the availability, accessibility, acceptability, and quality of the health care workforce as essential conditions in attaining universal health coverage (UHC) [4, 5].

WHO의 "보건을 위한 인적 자원에 관한 글로벌 전략 2030"에서도 다음 사항을 목표 중 하나로 인정하고 있습니다: "보건 인적 자원에 대한 증거 기반 정책을 통해 보건 인력의 성과, 품질 및 영향력을 최적화한다"[6]. 그러나 보건의료 인력의 질을 보장하기 위해 면허 시스템을 통해 정책 및 규제 프레임워크를 마련하는 것은 보건의료 시스템 개발에서 종종 무시되는 영역입니다[7]. 
The WHO’s “Global Strategy on Human Resources for Health 2030” also acknowledges the following as one of its objectives: “To optimize performance, quality and impact of the health workforce through evidence-informed policies on human resources for health” [6]. However, putting a policy and regulatory framework in place through a licensing system to ensure the quality of a health care workforce is often a neglected area in the development of HRH systems [7].

라오스 인민민주주의 공화국은 동남아시아의 개발도상국으로 인구는 약 620만 명이며, 이 중 32%가 도시 지역에 거주하고 있습니다. 라오스는 저소득 국가이지만 최근 몇 년 동안 경제가 꾸준히 성장하고 있습니다[8]. 필수적인 모자보건 서비스를 제공하기 위해서는 인구 1000명당 의사, 간호사, 조산사가 최소 2.3명이 필요하지만, 라오스는 인구 1000명당 의사, 간호사, 조산사가 1.3명에 불과해 양적, 질적 측면에서 여전히 심각한 인력 부족을 겪고 있습니다[9, 10]. 이러한 상황은 주변 국가에서도 관찰되는데, 베트남은 인구 1000명당 1.3명, 미얀마는 1.3명, 캄보디아는 1.0명으로, 이들 국가는 보건 인력 부족이 심각한 49개국으로 분류됩니다[10]. 따라서 충분한 수의 보건 인력을 확보하는 데 우선순위를 두고 있습니다[11]. 그러나 라오스 인민민주주의 공화국은 여전히 보건 인력이 제공하는 의료 서비스의 질을 보장할 수 있는 메커니즘이 부족합니다. 이는 의료 전문가가 역량 인증에 따라 면허를 취득하고 면허를 유지하기 위한 조건으로 지속적인 전문성 개발에 정기적으로 참여해야 한다는 조항과 규정이 없기 때문입니다. 
The Lao People’s Democratic Republic is a developing country in Southeast Asia with an estimated population of 6.2 million, 32% of whom live in urban areas. While Laos is a low-income country, its economy has been growing steadily in recent years [8]. The minimum threshold of 2.3 doctors, nurses, and midwives per 1000 population is considered necessary to deliver essential maternal and child health services; however, Lao People’s Democratic Republic still suffers from critical shortages in terms of the quantity and quality of HRH, having only 1.3 doctors, nurses, and midwives per 1000 population [9, 10]. This situation is also observed in neighboring countries, with thresholds of 1.3 in Vietnam, 1.3 in Myanmar, and 1.0 in Cambodia per 1000 population, countries categorized as being among the 49 countries with a critical shortage of health workers [10]. Therefore, priority has been placed on achieving a sufficient number of health workers [11]. However, Lao People’s Democratic Republic still lacks the mechanisms to ensure the quality of health care services provided by its health workforce. This is due to the absence of provisions and regulations that require health care professionals to be licensed based on competence certifications and requirements to regularly engage in continuing professional development as a condition for maintaining their license.

라오스 정부는 2005년 보건의료법이 제정된 이래 모든 국민에게 양질의 공평한 보건의료 서비스를 제공하는 것을 최우선 과제로 삼고 있습니다[12]. 지난 10년 동안 면허 시스템 개발을 위해 관련 법률과 정책이 제정되었지만, 이러한 조치가 효과적으로 활용되었는지 여부와 효과적인 이행을 촉진하기 위해 필요한 추가 단계를 결정하는 것은 여전히 어렵습니다. 이러한 정책을 개발하는 과정에서 어떤 노력과 제약이 있었는지 조사한 연구는 없으며, 소수의 공무원만이 이러한 노력에 대한 정확한 전반적인 관점을 가지고 있습니다. 
Providing quality and equitable health care services for all citizens has been a priority of the Lao government since the Law on Health Care was enacted in 2005 [12]. Although relevant legislation and policies have been formulated to enable the development of a licensing system over the last decade, determining whether such measures have been utilized effectively and what additional steps may be required to promote effective implementation remains difficult. No studies have examined the efforts made and constraints encountered in the development of these policies, and only a small number of officials have an accurate overall perspective of these efforts.

따라서 이 연구의 목적은 라오스 인민민주공화국의 인권 정책의 진화를 조사하는 것이며, 특히 라이선스 시스템을 포함한 규제 프레임워크의 개발에 주목합니다. 라오스 인민민주주의공화국이 면허 시스템 개발의 토대를 마련한 방법을 살펴봄으로써 라오스 보건의료 인력의 질과 보건의료 시스템에 대한 기여를 보장할 수 있는 건전한 규제 프레임워크의 개발 및 시행에 대한 귀중한 교훈과 유용한 통찰력을 제공하기 위해 이 검토가 설계되었습니다. 
Therefore, the aim of this study is to examine the evolution of HRH policies in Lao People’s Democratic Republic with particular attention on the development of a regulatory framework, including a licensing system. To examine how Lao People’s Democratic Republic laid the foundation for the development of a licensing system, this review is designed to impart valuable lessons and useful insights into the development and implementation of a sound regulatory framework capable of ensuring the quality of the Lao health care workforce and its contribution to the health care system.

사례 설명
Case description

질적, 서술적 사례 연구 방법론이 적용되어 현재 정책의 발전과 향후 시행을 개선하기 위한 의료 전문가 면허 시스템의 규제 프레임워크 개발을 검토했습니다. 이 질적, 서술적 사례 연구 설계는 라오스 보건부의 공식 승인을 받았습니다. 라오스의 의료 전문가와 관련된 법률, 규정, 정책 및 관련 보고서를 문서로 검토했습니다. 연구 정보는 1995년부터 2016년 사이에 발행된 209개의 라오스 법률, 규정, 정책, 보건 관련 지침 등 공식적으로 발표된 문서에서 얻었습니다. 이 중 75개는 "교육", 59개는 "의료 전문가의 역할 및 실무", 2개는 "자격", 73개는 "기타"로 분류되었습니다. 우리는 19개의 주요 문서가 면허 시스템에 중요하다는 것을 발견했으며, 이를 시간순으로 정리했습니다(표 1). 라이선스 시스템 전략 개발에 도움을 제공한 저자들은 라이선스 시스템의 진화 과정에서 몇 가지 주요 사건을 확인했습니다. 라이선싱 시스템 전략 개발에 참여했던 4명의 주요 이해관계자와의 인터뷰를 통해 문서 검토에서 관찰한 내용을 취합 및 검증하고 라이선싱 시스템 개발에서 중요한 요소가 무엇인지 물었습니다. 인터뷰 대상자는 보건부 관련 부서의 고위 공무원과 보건과학대학교 의과 및 치의학 학부 부학장으로 구성되었습니다. 인터뷰 대상자들은 익명을 보장하고 정보의 기밀을 엄격하게 유지한다는 조건에 동의했습니다. 모든 참가자는 구두로 사전 동의를 제공했습니다. 인터뷰는 발언자의 신원을 표기하지 않고 받아쓰기 방식으로 진행되었습니다. 또한 라이선스 시스템 전략을 개발하기 위한 일련의 위원회 회의에서 입수한 토론 노트와 회의 메모를 활용했습니다.  
A qualitative, descriptive case study methodology was applied to review the evolution of current policies and the development of a regulatory framework for a licensing system for health care professionals to improve its future implementation. This qualitative, descriptive case study design was officially approved by the Lao Ministry of Health. A document review was carried out of laws, regulations, policies, and relevant reports related to health care professionals in Lao People’s Democratic Republic. The study information was obtained from officially published documents including 209 Laotian laws, regulations, policies, and guidelines related to health that were issued between 1995 and 2016. Among these, 75 were categorized as “education,” 59 as “role and practice of health care professionals,” 2 as “qualifications,” and 73 as “others.” We found that 19 key documents were critical for the licensing system, and we ordered them chronologically (Table 1). Authors who had offered support in developing a licensing system strategy identified some key events in the evolution of the licensing system. Interviews were conducted with four key stakeholders who had also been involved in developing a licensing system strategy to collate and validate our observations from the document review and to ask what factors were important in the development of a licensing system. They consisted of senior officials from relevant departments in the Ministry of Health and vice deans from the faculty of medicine and dentistry at the University of Health Sciences. Interviewees agreed upon the conditions to remain anonymous and to have the confidentiality of their information rigorously maintained. All participants provided oral informed consent. Interviews were dictated without notation of the speaker’s identity. In addition, discussion notes and meeting memos that were obtained in a series of committee meetings to develop a licensing system strategy were utilized.


다음 섹션에서는 2005년부터 2015년까지 라오스의 라이선스 시스템 개발 연대표를 제시하고(표 1), 시스템 발전 과정에서 확인된 주요 사건에 대해 설명합니다. 
In the following section, we present the chronology of the development of a licensing system in Lao People’s Democratic Republic from 2005 to 2015 (Table 1), followed by a description of the key events identified in the evolution of the system.

라오스 인민민주공화국 라이선스 시스템의 연대기(2005년부터 2015년까지)
The chronology of a licensing system in Lao People’s Democratic Republic (from 2005 to 2015)

2005년 라오스 인민민주주의공화국은 보건의료 목표 달성을 위한 보건 거버넌스를 강화하기 위해 라오스 최초의 "보건의료에 관한 법률"을 제정했습니다[12]. 이 법은 16가지 범주의 의료 전문가를 규정하고 필수 책임 사항을 명시했습니다. 또한 보건의료전문가위원회의 역할은 제58조에 "보건부가 보건의료전문가의 활동을 관리하기 위한 사무국"으로 명시되어 있습니다. 이러한 이유로 법률 및 공식 문서 아카이브를 검토 할 때 시스템 개발의 시작 시점을 2005 년으로 간주했습니다. 
In 2005, Lao People’s Democratic Republic enacted the country’s first “Law on Healthcare” to strengthen health governance toward achieving its health care goals [12]. It regulated 16 categories of health care professionals with a statement of required responsibilities. The role of the Healthcare Professional Council was also specified in Article 58 as the “secretariat for the Ministry of Health in administering the activities of health care professionals.” For these reasons, when examining legal and official document archives, we considered 2005 as the starting point of the system development.

법적 프레임워크의 개발은 핵심 보건법에서 시작하여 하위 법령 및 기타 관련 법률로 이어졌습니다. 지난 10년 동안 보건 인적 자원을 개선하기 위한 주요 정책과 전략이 도입되면서 법률에 약간의 불일치와 중복이 발생했습니다. 
The development of a legal framework started with the core health law and was followed by sub-decrees and other relevant legislation. Over the past decade, key policies and strategies designed to improve human resources for health have been introduced, which have resulted in minor inconsistencies and overlap in legislation.

이 기간 동안 라오스 인민민주공화국의 보건의료 정책은 "농촌 보건의료 시설에 보건의료 종사자 배치 강화"에서 "보건의료 전문가의 질 향상"으로 전환되었습니다. 라오스 인민민주주의공화국은 지난 수십 년 동안 양적, 질적 측면에서 보건의료 강화를 위한 적극적인 조치를 취해왔지만 2006년부터 2012년까지 공공 부문에서 일하는 보건의료 종사자의 수는 거의 변하지 않았습니다. 같은 기간 인구가 23% 증가했음에도 불구하고 총 보건 종사자 수는 약 1만 2400명으로 정체되었습니다(그림 1)[13]. 이는 공공 부문에서 보건 인력을 흡수할 수 있는 능력이 제한되어 있었기 때문입니다. 특히 농촌 지역의 심각한 의료 인력 부족 문제를 해결하기 위해 라오스 보건부(MOH)는 의료 종사자 채용 및 유지를 적극적으로 장려하고 있습니다. 또한 라오스 정부는 최근 신규 졸업생을 특히 농촌 및 외딴 지역의 하위 의료 시설에 배치하는 법령을 공표했습니다. 그 결과 보건소 수준의 의료 종사자 비율이 증가하여 일반 인구가 기본적인 의료 서비스를 이용할 수 있게 되었습니다(그림 2)[14, 15]. "라오스 보건 부문 개혁 프레임워크"[16]에 따르면, 보건 서비스 제공을 개선하고 원하는 결과를 달성하기 위해 세 단계의 개혁이 적용되었습니다.

  • 1단계(2013~2015년)에서는 보건 관련 새천년개발목표(MDG)를 달성하기 위해 농촌 및 외딴 지역에 보건 인력의 규모와 배치를 늘리기 위한 조치가 우선적으로 이루어졌습니다.
  • 2단계(2015~2020년)와 3단계(2020~2025년)에서는 보건의료 서비스에 대한 지역사회의 접근성을 보장하고 서비스 제공의 질을 개선하기 위해 양적, 질적 측면에서 보건의료 인력 양산을 강화하는 데 중점을 두고 있습니다.

2025년까지 라오스 인민민주공화국은 보편적 의료 보장(UHC) 달성을 위한 의료 서비스 접근성을 보장하기 위해 숙련되고, 동기를 부여받고, 지원을 받고, 널리 분포된 충분하고 지속 가능한 인력을 확보할 것입니다. 
During this period, HRH policy in Lao People’s Democratic Republic shifted from “strengthening the deployment of health workers in rural health care facilities” to “improving the quality of health care professionals.” Although Lao People’s Democratic Republic has taken active steps to strengthen HRH in terms of quality and quantity over the past few decades, the number of health workers working in the public sector remained virtually unchanged between 2006 and 2012. The total number of health workers stagnated at approximately 12 400, despite a 23% increase in population during the same period (Fig. 1) [13]. This was because of the limited absorption capacity of HRH in the public sector. To address critical health care workforce shortages, especially in rural areas, the Ministry of Health (MOH) has actively promoted the recruitment and retention of health workers. In addition, the Lao government recently promulgated a decree on placing new graduates in lower-tier health care facilities, especially in rural and remote areas. As a result, the percentage of health care workers at the health center level has increased, making basic health care services available to the general population (Fig. 2) [14, 15]. According to “The Health Sector Reform Framework Laos” [16], three phases of reform were applied to improve the delivery of health care services and achieve the desired outcomes.

  • For reaching the health-related Millennium Development Goals (MDGs) in Phase I (2013–2015), actions to increase the size and deployment of the health workforce in rural and remote areas have been made a priority.
  • In Phases II (2015–2020) and III (2020–2025), increased emphasis has been placed on the enhancement of HRH production in terms of both quantity and quality to ensure community access to health care services and improve the quality of service provision.

By 2025, Lao People’s Democratic Republic will have a sufficient and sustainable workforce that is skilled, motivated, supported, and widely distributed to ensure access to health services in achieving universal health coverage (UHC).

 
 

 

연대기에서 이 라이선스 시스템의 발전과 관련된 몇 가지 중요한 사건을 확인할 수 있습니다: 

  • (a) 라오스 인민민주공화국이 표준 자격을 갖추기 위한 동남아시아국가연합(ASEAN) 상호인정협정(MRA)의 가속화,
  • (b) 초기 단계의 책임 조직 설립 및 리더 배정,
  • (c) 라오스 보건 시스템 개발에서 보건 인적 자원의 중요성 강조,
  • (d) 유능한 보건의료 전문가 양성을 위한 교육 시스템 강화,
  • (e) 신규 채용 공무원의 농촌 지역 3년 의무 근무,
  • (f) 민간 보건의료 시설에서 근무하기 위한 전문 자격증 취득 요건 강화.
  • 이러한 발전에 따라 (g) 라오스 인민민주주의 공화국은 마침내 면허 시스템에 대한 포괄적인 발전 전략을 수립했으며, 곧 시행될 예정입니다.

In the chronology, some critical events that are related to the evolution of this licensing system are observed:

  • (a) acceleration by the Association of Southeast Asian Nations (ASEAN) Mutual Recognition Arrangement (MRA) for Lao People’s Democratic Republic to have standard qualifications,
  • (b) establishment of responsible organizations and assignment of leaders in the early stages,
  • (c) emphasis on human resources for health in developing the health system in Lao People’s Democratic Republic,
  • (d) strengthened educational systems to foster competent health care professionals,
  • (e) 3 years of compulsory service in rural areas for newly recruited government servants, and
  • (f) requirements for obtaining a professional certificate to work at a private health care facility. Following these developments,
  • (g) Lao People’s Democratic Republic has finally formulated a comprehensive developmental strategy for its licensing system, which will be implemented soon.

의료 전문가 면허 시스템 발전의 주요 이벤트
Key events in the evolution of a licensing system for health care professionals

다음 이정표와 주요 이벤트는 라오스 인민민주공화국의 면허 시스템이 점진적으로 진화하는 과정을 설명합니다:
The following milestones and key events describe the gradual evolution of the licensing system in Lao People’s Democratic Republic:

라오스 인민민주공화국이 표준 자격을 갖추기 위한 아세안 MRA의 가속화
Acceleration by the ASEAN MRA for Lao People’s Democratic Republic to have standard qualifications

일반적으로 동남아시아 지역에서 의료 전문직의 이동은 기본 법률 체계, 교육 시스템 및 품질 표준의 발전과 밀접한 관련이 없는 것으로 보였습니다[17]. 오히려, 특히 라오스 인민민주공화국에서는 불충분한 자격 요건과 언어 장벽이 라오스 의료 전문가의 해외 이주에 큰 제약 요인으로 작용했습니다[18]. MRA는 특히 교육 시스템을 업그레이드함으로써 라오스 인민민주공화국의 지역 표준을 사용하여 의료 전문가 배출을 가속화했습니다. 
In general, the migration of health care professions in the region of Southeast Asia did not seem to correlate closely with the development of underlying legal schemes, educational systems, and quality standards [17]. Rather, especially in Lao People’s Democratic Republic, insufficient qualifications and language barriers had a major restraining effect on the migration of Laotian health care professionals abroad [18]. The MRA accelerated the production of health care professionals using a regional standard in Lao People’s Democratic Republic particularly by upgrading its educational system.

라오스 인민민주주의공화국에는 소수이긴 하지만 해외 의료 전문가가 증가하고 있지만, 최근까지 라오스에서 진료할 수 있는 외국인 의료 전문가의 자격을 규제하는 명확한 정책이 수립되지 않았고 이들의 진료를 모니터링할 수 있는 시스템도 마련되어 있지 않았습니다[13]. 그러나 2006년에 간호사를, 2009년에 의사와 치과의사를 포괄하는 아세안 MRA를 체결하면서[19,20,21] 회원국들은 외국인 의료 전문가를 규제하는 절차를 마련하여 규제 프레임워크를 개혁하도록 강제하고 있습니다. 라오스 인민민주주의 공화국은 외국인 의료인을 위해 필요한 조치를 시작했습니다. 예를 들어, 최근 민간 병원에서 일할 수 있는 외국인 의료 종사자 인증이 의회에서 요구되고 있습니다[22]. 아세안 경제 공동체(AEC)로의 통합이 증가함에 따라 외국인이 라오스 면허를 취득하는 데 필요한 조건을 자세히 설명하는 계획을 포함하여 새로 승인된 전략에 따른 국가 표준 수립이 가속화되고 있습니다[23]. 

Even though there is a small but increasing number of international health care professionals in Lao People’s Democratic Republic, no clear policies had been formulated to regulate the qualifications of foreign health care professionals to practice in Laos until recently and no systems had been in place to monitor their practices [13]. However, the ASEAN MRA which encompassed nurses in 2006 and medical practitioners and dentists in 2009 [19,20,21] enforces member states’ reform of their regulatory frameworks by establishing procedures to regulate foreign health care professionals. Lao People’s Democratic Republic has initiated the necessary arrangements for foreign practitioners. For example, certification of foreign health care workers to work in private hospitals has recently been required by the council [22]. As integration into the ASEAN economic community (AEC) increases, the establishment of national standards according to the newly endorsed strategy is being accelerated, including plans to detail the necessary conditions for foreigners to obtain a Lao license [23].

초기 단계의 책임 조직 설립 및 리더 배정
Establishment of responsible organizations and assignment of leaders in the early stages

보건의료법[12]이 채택된 지 2년 후인 2007년에 보건부는 두 개의 하위 법령을 발표했습니다.

  • 첫 번째는 등록 및 전문 자격증 발급 측면에서 보건의료전문가위원회의 역할을 명시했고[24],
  • 두 번째는 위원회 대표와 추가 공식 직책을 보건부의 지정된 리더에게 할당하도록 규정했습니다[25].

보건의료전문가협의회는 면허제도 발전 초기에 명확한 임무를 가지고 설립되었지만, 관리 역량이 부족하여 책임을 다할 수 없었습니다. 상근 기술 직원이 부재한 상황에서 지정된 고위 지도자들은 면허 및 등록에 대한 규정과 메커니즘을 수립하는 데 필요한 세부 작업을 수행 할 수 없었습니다. 2013년에 보건부 보건의료국에 "보건의료 전문직 사무국"이 설립되어 사무국 업무를 담당하고 보건의료 전문직 협의회 활동을 지원했습니다[26]. 비록 상근 전담 직원의 수는 제한적이었지만, 이들의 기여는 2015년에 최종적으로 제정된 면허제도 발전을 위한 세부 전략 개발에 큰 역할을 했다. 그러나 제도적 틀은 아직 제대로 작동하지 않고 있습니다. 
In 2007, 2 years after the adoption of the Law on Health Care [12], the MOH issued two sub-decrees.

  • The first specified the role of the Healthcare Professional Council in terms of registration and the issuance of professional certificates [24], while
  • the second stipulated that council representatives and additional official positions were to be allocated to designated leaders of the MOH [25].

Although the Healthcare Professional Council was established at an early stage of the evolution of the licensing system with a clear mandate, it was unable to fulfill its responsibilities due to its insufficient management capacity. In the absence of full-time technical staff, the designated senior leaders were unable to implement the detailed work necessary to establish the regulations and mechanisms for licensing and registration. In 2013, the “Office of Healthcare Professionals” was established in the Department of Health Care of the MOH to serve the secretariat and support the activities of the Healthcare Professional Council [26]. Even though the number of full-time dedicated staff remained limited, their contributions played a major role in the development of a detailed strategy for the development of the licensing system, which was finally enacted in 2015. However, the institutional framework is not functional yet.

라오스 보건 시스템 개발에서 보건 인적 자원의 중요성 강조
Emphasis on human resources for health in developing the health system in Lao People’s Democratic Republic

라오스 보건부는 보건의료 전문가의 질을 모니터링하고 규제하기 위한 프레임워크와 메커니즘을 구축해야 한다는 시급한 필요성을 인식하고 2011년 총리가 승인한 '2020년까지 보건 인력 개발 전략'에 우선 순위로 포함시켰습니다[11]. 이 전략은 보건의료 전문가 위원회의 기능이 시급히 필요함을 명시하고 그 기여를 보장하기 위한 우선순위를 확인했습니다. 이 전략은 다음 사항의 중요성을 강조했습니다: 
The urgent need to establish a framework and mechanisms for monitoring and regulating the quality of health care professionals has been recognized by the MOH and was included as a priority in “The Health Personnel Development Strategy by 2020” which was approved by the prime minister in 2011 [11]. This strategy stipulated the urgent need to functionalize the Healthcare Professional Council and identified priorities to ensure its contribution. The strategy stressed the importance of the following:

  • 보건의료전문가위원회와 그 이사회가 보건법에 명시된 책임을 성공적으로 수행할 수 있도록 충분한 자원과 전문성, 효과적인 관리 구조를 갖추도록 보장하는 조항을 검토하는 데 중점을 두었습니다:
    Reviewing provisions to ensure that the Healthcare Professional Council and its boards had sufficient resources and expertise as well as an effective management structure to enable the successful undertaking of its responsibilities as stipulated in the Health Law with an emphasis on:
  • 보건의료 졸업생 인증을 위한 역량 프레임워크 승인;
     Approving the competency framework for certifying health care graduates;
  • 면허의 근거가 되는 보건의료 졸업생에 대한 기존 시험의 방법과 내용 승인, 그리고
     Approving the methods and content of existing exams for health care graduates as a basis for licensing; and
  • 지속적인 전문성 개발 및 실무에 대한 증거를 기반으로 한 지속적인 정기 등록 조항을 포함하여 보건의료 전문가 면허에 대한 프레임워크 및 기준 개발.
     Developing a framework and criteria for the licensing of health care professionals, including provisions for continuing periodical registration based on evidence of continuing professional development and practice.

유능한 보건의료 전문가 양성을 위한 교육 시스템 강화
Strengthened educational systems for fostering competent health care professionals

전문가들의 국경 간 이동을 촉진하는 것 외에도 이러한 새로운 아세안 이니셔티브의 맥락에서 라오스 인민민주공화국의 의료 전문가 관련 교육 및 법률을 개선하는 것이 강조되고 있습니다. 의사, 치과의사, 간호사를 위한 전문 역량MRA에 의한 평가를 위한 구체적인 기준을 제공하기 위해 개발되었습니다[27,28,29]. 또한 국제 표준에 따라 이러한 역량을 기반으로 교육 커리큘럼을 개정하여 전문 과목에 더 많은 학습 단위를 할당했습니다 [30]. 또한, 기술자격만 갖춘 간호사가 아닌 전문간호사만이 MRA를 받을 수 있는 MRA 요건을 충족하기 위해 간호교육 프로그램을 2.5년에서 3년으로 연장했다.  
In addition to facilitating the cross-border movement of professionals, improving education and legislation related to health care professionals in Lao People’s Democratic Republic in the context of these new ASEAN initiatives has been emphasized. Professional competencies for medical doctors, dentists, and nurses were developed to provide specific criteria for their assessment by the MRA [27,28,29]. In addition, in line with international standards, the educational curriculum was revised based on these competencies, allocating more study units for professional subjects [30]. Moreover, to meet MRA requirements, in which only a professional nurse can be eligible for the MRA as opposed to a nurse with only technical qualifications, nursing education programs were extended from 2.5 to 3 years.

졸업생들이 라오스에서 실무에 종사할 수 있는 역량을 갖추도록 국가 시험이 계획되어 있습니다. 현재 라오스 보건부 관할 하에 10개의 보건의료 교육기관이 있지만, 의사, 치과의사, 간호사, 약사, 조산사 등 모든 수준과 범주의 보건의료 전문가에 대한 제도적 품질 보증 시스템은 아직 공식적으로 적용되지 않고 있습니다[11, 13]. 의사, 치과의사, 간호사는 2020년까지, 그 외 직종은 2025년까지 국가시험이 시행될 것으로 예상됩니다[23]. 예외적으로 조산사는 2010년에 국가시험이 도입된 이후 조산사 자격증을 발급받아 진료를 할 수 있게 되었습니다[13]. 국가 시험에는 보건복지부가 발행한 국가 조산사 교육 표준과 국제 조산사 연맹(ICM)에 따른 객관적이고 체계적인 임상 시험(OSCE)이 포함됩니다.  
National examinations are planned to ensure the competency of graduates to practice in Lao People’s Democratic Republic. Although 10 health care educational institutions are currently under the jurisdiction of the MOH, an institutional quality assurance system has yet to be formally applied across all levels and categories of health care professionals including medical doctors, dentists, nurses, pharmacists, and midwives [11, 13]. It is anticipated that a national examination for medical doctors, dentists, and nurses will be implemented by 2020 and for others by 2025 [23]. As a sole exception, since the introduction of a national examination in 2010, midwives have been issued a certificate to allow them to practice [13]. The national examination includes an objective and structured clinical examination (OSCE) based on national midwifery education standards issued by the MOH and following the International Confederation of Midwives (ICM).

신규 채용된 보건 분야 공무원의 농촌 지역 3년 의무 복무
Three-year compulsory service in rural areas for newly recruited government servants in the health sector

라오스 인민민주공화국은 심각한 보건 인력 부족을 겪고 있으며, 특히 외딴 지역과 농촌 지역에서 보건 인력의 불공평한 분배가 두드러집니다[9]. 라오스 인민민주공화국과 같이 자원이 부족한 국가에서의 인력 부족은 주로 생산 및 고용 능력이 낮기 때문에 발생합니다. 라오스 보건부는 거의 모든 병원에 적용되는 공공 부문의 허가된 직책 수가 제한되어 있기 때문에 모든 간호 졸업생(연간 약 500~600명)이나 다른 보건 전문직 졸업생을 모두 고용할 수 없습니다[13]. 한정된 정부 예산으로 지역 사회 보건의 이 중요한 문제를 해결하기 위해 보건부는 "농촌 지역 신규 졸업생 배치에 관한 장관급 합의서(제103/MOH/23호, 2012년 1월)"를 제정했습니다. [31]을 제정하여 모든 종류의 보건 분야 신규 채용 공무원에 대해 농촌 지역에서 3년의 의무 복무를 의무화했습니다. 농촌 지역에서 3 년의 의무 복무를 마친 사람은 면허 시험에 응시할 수 있습니다. 이는 해당 지역에 직원을 배치하는 데 대한 인센티브입니다. 그러나 공식적인 감독 체계를 갖춘 교육 프로그램을 개발하는 것은 앞으로의 과제가 될 것입니다.  
Lao People’s Democratic Republic has suffered a critical shortage of HRH, and the inequitable distribution of HRH is particularly evident in remote and rural areas [9]. A shortage of HRH in a resource-limited country such as Lao People’s Democratic Republic is primarily caused by low production and employment capacity. The MOH cannot employ all nursing graduates (approximately 500–600 per year) or all graduates in other health professions due to the limited number of sanctioned posts in the public sector, which applies to almost all hospitals [13]. To address this critical issue in community health with a limited government budget, the MOH enacted the “Ministerial Agreement on the Stationing of New Graduates in Rural Areas (No. 103/MOH/23, Jan. 2012)” [31], which mandates 3 years of compulsory service in rural areas for all kinds of newly employed government servants in the health sector. Those who complete the 3-year compulsory service in rural areas are eligible to sit for the licensing examination. It is an incentive for placing staff in such areas. However, the development of a training program with a formal supervision scheme will be a challenge in the future. 

개인 의료 시설에서 일하기 위해 전문 자격증을 취득하기위한 요건
Requirements for obtaining a professional certificate to work at a private health care facility

거의 모든 병원이 공공 부문에 속해 있지만 개인 클리닉의 수가 증가하고 있습니다. 민간 병원에 관한 라오스 법령(라오스 정부 법령 제 151호, 2014년 4월 28일)[23]은 2014년에 공포되었으며, 민간 병원에서 일하고자 하는 의료 전문가는 반드시 보건부에서 발급한 자격증을 소지해야 한다고 규정하고 있습니다. 이는 실무 능력을 증명하는 전문 자격증과는 다르며, 공무원 명부에 등록되지 않은 의료인이 민간 의료 시설에서 일할 수 있도록 공식적으로 인정하는 것입니다. 라오스 보건부는 민간 부문의 의료 서비스 품질을 관리하기 위해 2014년부터 공식 자격증을 발급하기 시작했습니다. 그러나 라오스 정부는 잠재적인 이중 제도의 복잡성을 피하기 위해 의료 전문가를 위한 고유한 면허 시스템으로 통합할 계획입니다.
Almost all hospitals belong to the public sector, but the number of private clinics is increasing. The Lao decree on private hospitals (no. 151/Gov, 28 April 2014) [23] was promulgated in 2014 and stipulates that a health care professional who intends to work at a private hospital must hold a certificate issued by the MOH. This is not the same as a professional license to prove competency of practice; in contrast, it provides official recognition for a health care practitioner who is not registered in the government servant registry to work in a private health care facility. Needing to control the quality of health care in the private sector, the MOH began issuing official certifications in 2014. However, the government of Lao People’s Democratic Republic intends for this stream to be integrated as a unique system of licensure for health care professionals to avoid the complexity of a potential dual system.

의료 전문가 면허 및 등록을 위한 10개년 전략
Ten-year strategy for the licensing and registration of health care professionals

2015년에 보건부는 "라오스 인민민주공화국의 의료 전문가 면허 및 등록 시스템 전략 2016-2025"[23]를 승인했습니다. 이 전략은 궁극적으로 면허 및 등록 시스템의 틀을 정의하고 향후 10년간의 목표, 목적, 대상, 우선순위 조치로 구성되었으며, 6개의 기둥에 중점을 두었습니다(표 2).
In 2015, the MOH endorsed the “Strategy on Healthcare Professional Licensing and Registration System in Lao People’s Democratic Republic 2016-2025” [23]. This strategy ultimately defined the framework for the licensing and registration system and consisted of goals, objectives, targets, and prioritized actions over the next 10 years, with a focus on six pillars (Table 2).

면허 및 등록을 향한 경로는 교육 기관에서 시행하는 졸업 시험으로 시작하여 학위 수여로 이어집니다. 학위를 수여받은 졸업생은 의료 전문가 위원회의 감독하에 시행되는 국가 역량 시험에 응시할 자격이 주어집니다. 정식 면허를 취득하려면 응시자는 공공 또는 민간 환경에서 3년간의 감독 실습을 완료해야 합니다. 전문 실무 지식과 기술을 최신 상태로 유지하기 위해 등록된 의료 전문가는 의료 전문가 위원회에서 승인한 지속적인 전문성 개발(CPD) 활동 참여 평가를 바탕으로 5년마다 등록을 갱신해야 합니다(그림 3).
The pathway toward licensing and registration will commence with an exit exam administered by the training institution, leading to the awarding of an academic degree. Graduates who are awarded the degree are entitled to sit for a national competency examination that is administered under the supervision of the Healthcare Professional Council. To obtain a full license, candidates must complete 3 years of supervised practice in either public or private settings. To ensure that professional practice knowledge and skills are current, registered health care professionals are required to renew their registration every 5 years based on an assessment of their participation in continuing professional development (CPD) activities approved by the Healthcare Professional Council (Fig. 3).

또한 최근 승인된 보건 부문 개발 계획(제8차, 2016~2020년)[33]에는 면허 및 등록 시스템 구축을 위한 국가 프로젝트가 포함되어 있습니다. 이러한 전략을 바탕으로 현재 보건부는 WHO와 협력하여 이 새로운 시스템을 지원하기 위한 보건의료전문가협의회를 설립하는 작업을 진행 중입니다.
In addition, the recently endorsed health sector development plan (the 8th, 2016–2020) [33] includes a national project for establishing a licensing and registration system. Based on these strategies, progress is now underway by the MOH in collaboration with the WHO to establish the Health Professionals Council to support this new system.

토론
Discussion

이 질적, 서술적 사례 연구는 라오스 인민민주공화국의 보건의료 정책의 진화 과정을 면허 시스템 규제 프레임워크의 개발 과정을 통해 살펴봅니다. 이러한 프레임워크의 도입은 라오스 보건의료 시스템 개발에 중요한 이정표가 될 것이며, 향후에도 중요한 영향을 미칠 것입니다. 라오스 보건의료의 질을 보장하는 궁극적인 목표는 안전하고 효과적인 보건의료 서비스를 제공하고 의료 과실로부터 국민을 보호하는 것입니다. 예를 들어, 면허 취득은 미국 의료 시스템에서 4가지 주요 성과로 이어졌습니다: 

  • 비숙련 개입으로 인한 돌팔이 제거 또는 개인 수준에서 적격 치료 지연, 
  • 치료 개선으로 인한 감염 감소, 
  • 의료인의 지식과 기술 향상을 통한 과학 발전, 
  • 비숙련 의료인에게 지불되는 비용 낭비로 인한 비용 절감[34]. 

라오스 인민민주주의 공화국에서는 국가 시험과 지속적인 전문성 개발을 통해 의료 전문가의 역량을 보장할 것입니다. 또한 의료 과실로 인해 면허를 취소할 수 있는 제도를 도입함으로써 보건의료인의 윤리적 측면과 직업적 태도의 중요성이 강조될 것입니다[23].
This qualitative, descriptive case study examined the evolutionary process of HRH policies in Lao People’s Democratic Republic by depicting the development of a regulatory framework for a licensing system. Introducing such a framework will be a significant milestone in the development of a Lao health care system and one that will have significant implications for the future. The ultimate goal of ensuring the quality of HRH is to provide safe and effective health care services and protect the population against malpractice. For example, licensure led to four major achievements in the US health care system:

  • the elimination of quackery from unskilled interventions or delays in qualified care at the individual level,
  • the reduction of infection by improved treatment,
  • the advancement of science through improved knowledge and skills of medical practitioners, and
  • cost savings from wasted payment to unskilled practitioners [34].

In Lao People’s Democratic Republic, the competency of health care professionals will be guaranteed by national examinations and continuing professional development. Moreover, by introducing a system in which licenses can be revoked because of malpractice, the ethical aspects of HRH and the importance of professional attitudes will be highlighted [23].

최근 몇 년 동안 라오스뿐만 아니라 제도가 마련되어 있지 않은 주변 국가에서도 면허 시스템 개발의 필요성이 가속화되고 있습니다. 이러한 발전의 주요 요인 중 하나는 2015년 말에 시작된 세 가지 범주의 의료 전문가에 대한 아세안 MRA 의무화입니다. 또 다른 요인은 아세안 경제 공동체가 주도하는 의료 서비스의 국제적 자유 흐름으로 인해 행정적 감독이 덜한 민간 의료 서비스에 대한 통제에 대한 요구가 증가하고 있다는 점입니다[35].
The need to develop a licensing system has accelerated in recent years not only in Lao People’s Democratic Republic but also in neighboring countries where no systems are in place. One of the primary factors in this development is the ASEAN MRA mandate for three categories of health care professionals, which was initiated at the end of 2015. Another factor is the increasing demand for control over private health care services, which have less administrative oversight [35] due to the international free flow of health services led by the ASEAN economic community.

또 다른 원동력은 "농촌 보건 의료 시설에 보건 종사자 배치 강화"에서 "보건 의료 전문가의 질 향상"으로의 HRH 정책 전환입니다. MDGs 달성을 위해 농촌 보건의료 시설에 보건의료 인력 배치를 늘리는 것과 동시에 보건의료 인력의 질을 보장하기 위해서는 면허 취득이 필수적입니다. 접근성이 뛰어나고 필요한 역량을 갖춘 보건의료 종사자가 있어야 지역사회의 기대에 부응하는 양질의 의료 서비스를 제공할 수 있고, UHC 달성을 향한 진전에 기여할 수 있습니다[6]. 일본의 경우 UHC 도입에 앞서 보건의료인력의 양적 측면을 강화하고, 나아가 교육 과정과 면허시험을 고도화하여 질적 측면을 개선하였다[36]. 이러한 과정은 아직 면허 제도를 개발 중인 다른 아세안 국가에도 적용될 수 있을 것으로 기대된다. 
Another driving force is the HRH policy shift from “strengthening the deployment of health workers in rural health care facilities” to “improving the quality of health care professionals.” Parallel with the increase of HRH deployment to rural health care facilities to achieve MDGs, licensing is essential to ensure the quality of HRH. The mere availability of health care workers is not sufficient; health care workers who are accessible and equipped with required competencies can deliver quality care that meets the expectations of the community and contributes to progress toward achieving UHC [6]. In the case of Japan, prior to introducing UHC, the quantitative aspects of health care workers were strengthened; furthermore, quality was improved by upgrading educational courses and licensing examinations [36]. This process is expected to be applied to other ASEAN countries that are still developing their licensing systems.

효과적인 정책과 계획을 개발하기 위해서는 HRH 데이터를 수집, 분석, 활용할 수 있는 강력한 기술적 역량과 해외 모범 사례를 활용할 수 있는 능력이 필요합니다[37]. 주목할 만한 점은 수집된 법적 제도와 정책의 중복이 비교적 적은 것으로 보인다는 점입니다. 라오스 인민민주주의공화국에서는 보건의료전문가위원회의 효과적인 거버넌스가 부재한 상황에서 유일한 규제기관으로서 강력한 하향식 관리를 통해 보건부의 장기적인 관점과 의지가 보건의료 전문가를 위한 정책의 체계적인 개발과 실행에 상당한 역할을 하고 있다. 다른 나라에서 볼 수 있듯이, 보건부 및 국가위원회를 포함한 여러 규제 기관의 존재는 시스템을 복잡하게 만들고 보건의료 전문가 규제에 필요한 작업을 부분적으로 만 수행합니다 [38]. 의료 전문가를 규제하고 허가하는 책임은 일반적으로 많은 국가에서 국가위원회에 귀속되지만, 보건부 또는 기타 전문 협회와 같은 다른 당국이이 역할을 수행하거나 일부 국가에서는 MOH 직원이 위원회에 겸직 할 수 있습니다 [36]. 향후 라오스 위원회의 제도적 역량을 강화하기 위해서는 여러 이해관계자 간의 명확한 경계와 효과적인 협력이 필요할 것입니다.
The development of effective policies and plans requires strong technical capacity to compile, analyze, and utilize HRH data, as well as the ability to draw upon best practices from abroad [37]. It is notable that there seem to be relatively few overlaps in the collected legal instruments and policies. In Lao People’s Democratic Republic, through strong top-down management as a sole regulator, both the long-term perspective and commitment of the MOH have played a substantial role in the systematic development and implementation of policies for health care professionals, as effective governance by the Healthcare Professional Council remains absent. As seen in other countries, the existence of multiple regulatory bodies, including the MOH and National Council, makes the system complex and only partially fulfills the tasks required for regulating health care professionals [38]. Although the responsibility to regulate and authorize health care professionals is usually attributed to the National Council in many countries, other authorities such as the Ministry of Health or other professional associations can take this role, or in some countries, the MOH staff concurrently belong to the Council [36]. To strengthen the Lao Council’s institutional capacity in the future, clear demarcations and effective collaboration between multiple stakeholders will be needed.

라오스 인민민주주의공화국이 면허 시스템을 개발하기 시작했지만 일부 문제는 여전히 다양한 이해관계자 간의 합의에 도달하기 위해 신중하게 논의해야 합니다. 예를 들어, 소수 민족에 대한 국가 시험을 위한 특별 조치가 필요할 수 있습니다.

  • 최근 인구조사에 따르면 전체 인구의 48.5%를 차지하는 47개의 소수 민족이 주로 고지대에 거주하는 것으로 확인되었습니다.
  • 이러한 인종적 다양성은 문화적, 언어적 장벽으로 인해 의료 서비스 제공과 교육에 여전히 큰 어려움을 겪고 있습니다[13].
  • 또한 외국 의과대학 졸업자에 대한 면허 제도는 라오스 내 의과대학 졸업자와 동등한 조건을 적용할지 여부에 대해 논의 중입니다.
  • 또한, '간호사'와 '전문간호사'와 같이 동일한 전문직종에 대해 서로 다른 면허 등급을 부여하는 방안도 향후 논의 및 규제될 수 있습니다.

면허와 관련된 한 가지 단점은 의료 전문가가 되기 위한 엄격한 장벽을 만든다는 것입니다. 지역사회에 봉사할 수 있는 충분한 수의 의료 전문가를 공급하려면 향후 면허는 라오스 인민민주공화국의 실제 수요를 해결하기 위한 기존 정책에 부합해야 합니다. 
Even though Lao People’s Democratic Republic has started to develop a licensing system, some issues still need to be discussed carefully to reach consensus between different stakeholders. For example, special arrangements might be made for the national examinations of ethnic minorities. The latest census identified 47 distinct ethnic groups that constitute 48.5% of the total population, predominately in the highlands. This ethnic diversity still presents major challenges to health care delivery and education due to cultural and linguistic barriers [13]. Additionally, the licensing system for those who have graduated from foreign medical schools is under discussion as to whether to apply equivalent conditions to those who have graduated from medical schools in Lao People’s Democratic Republic. Additionally, the establishment of different license levels in the same professional categories, such as “Registered Nurse” and “Licensed Practical Nurse,” might be discussed and regulated in the future. One disadvantage regarding licensure is that it creates stringent barriers to becoming a health care professional. To supply sufficient numbers of health care professionals who serve the community, future licensure should conform to existing policies to address the actual demands in Lao People’s Democratic Republic.

라오스 인민민주공화국과 같은 국가에서는 보건의료 전문가 위원회역량 개발을 강화하기 위한 정부 예산모든 면허 취득 경로에 대한 법안을 포함한 전략과 실행 계획에 따라 보건 정책을 실행에 옮길 필요가 있습니다. 특히 외부 지원에 대한 실질적, 기술적 의존도가 높은 자원이 제한된 국가에서는 개발 파트너와 협력하고 조율하는 것도 중요합니다. 따라서 보건부의 헌신은 필수 불가결합니다.
When moving forward in countries such as Lao People’s Democratic Republic, it is necessary to transform health policy into practice by following strategies and action plans, including governmental budgets for strengthening the capacity development of the Healthcare Professional Council, as well as legislation for every licensure pathway. It is also important to coordinate and collaborate with development partners, especially in countries with limited resources where substantial and technical reliance on external support is common. Therefore, the commitment of the MOH is indispensable.

결론
Conclusion

라오스 인민민주공화국의 라이선싱 시스템 정책 발전을 위한 지속적인 개발 과정에 대해 설명했습니다. 이 정보는 특히 면허에 초점을 맞춘 보건의료 전문가 개발의 정책 전환에 대한 광범위한 개요를 가능하게 할 것으로 기대됩니다. 라오스 인민민주공화국에서는 최근 몇 년 동안 지역 경제 통합과 보편적 의료 보장 달성을 향한 정책 변화로 인해 면허 제도의 필요성이 증가했습니다. 국가 면허 제도는 국가 시험, 지속적인 전문성 개발, 적절한 경우 면허 취소를 통해 의료 전문가의 역량을 보장하는 데 도움이 되는 보건 시스템 개발의 중요한 이정표가 될 것입니다. 모든 국민에게 보편적 의료 접근성과 사회적 보호를 보장할 수 있는 보건 시스템을 달성하기 위해서는 보건 인적 자원의 지속적인 개발을 위한 정책적 개입과 거버넌스 강화가 요구됩니다. 
The process of ongoing development toward policy evolution for a licensing system in Lao People’s Democratic Republic has been described. This information is expected to enable a broad overview of policy transitions in developing health care professionals, especially those focusing on licensing. In Lao People’s Democratic Republic, the need for licensing systems has increased in recent years due to regional economic integration and a shift in policy toward achieving universal health coverage. A national licensing system would be a significant milestone in health system development, helping to ensure the competency of health care professionals by means of a national examination, continuing professional development, and the revoking of licenses when appropriate. A policy intervention and strengthened governance for continuous development of human resources for health are demanded in order to achieve a health system that can guarantee universal access to health care and social protection to all citizens.

 


Hum Resour Health. 2017 Aug 7;15(1):51. doi: 10.1186/s12960-017-0215-2.

The evolution of the national licensing system of health care professionals: a qualitative descriptive case study in Lao People's Democratic Republic

Affiliations collapse

1JICA Project for Sustainable Development of Human Resources for Health to Improve MNCH Services, 2nd Floor, Sacombank Building, 044 Haengboun Rd, Ban Haisok, Chanthabouly District, Vientiane, Lao People's Democratic Republic. sonomiwa@gmail.com.

2National Center for Global Health and Medicine, 1-21-1 Toyama Shinjyuku, Tokyo, 162-8655, Japan. sonomiwa@gmail.com.

3Japan Agency for Medical Research and Development, 1-7-1 Otemachi, Chiyoda, Tokyo, 100-0004, Japan. sonomiwa@gmail.com.

4Ministry of Health, Ban thatkhao, Sisattanack District, Rue Simeuang, Vientiane, Lao People's Democratic Republic.

5University of New South Wales, 11/17 Sutherland Crs, Darling Point, Sydney, NSW, 2027, Australia.

6National Center for Global Health and Medicine, 1-21-1 Toyama Shinjyuku, Tokyo, 162-8655, Japan.

PMID: 28784154

PMCID: PMC5547512

DOI: 10.1186/s12960-017-0215-2

Free PMC article

 

Abstract

Background: The introduction of a systematic framework for the licensing of health care professions, which is a crucial step in ensuring the quality of human resources for health (HRH), is still evolving in Lao People's Democraic Republic. The aim of this study was to review and document the evolution of Lao HRH policies and the development of its national licensing system.

Case presentation: A qualitative descriptive case study methodology was applied to document and describe how Lao People's Democratic Republic laid the foundation for the development of a licensing system. The results demonstrate that Lao People's Democratic Republic is currently in the process of transitioning the focus of its HRH policies from the quantity and deployment of services to remote areas to improvements in the quality of services. The key events in the process of developing the licensing system are as follows: (1) the systematic development of relevant policies and legislation, (2) the establishment of responsible organizations and the assignment of responsible leaders, (3) the acceleration of development efforts in response to the Association of Southeast Asian Nations Mutual Recognition Arrangement for standard qualifications, (4) the strengthening of educational systems for fostering competent health care professionals, (5) the introduction of a 3-year compulsory service component in rural areas for newly recruited government servants, and (6) the introduction of a requirement to obtain a professional health care certificate to work in a private hospital. The Lao Ministry of Health (MOH) has endorsed a specific strategy for licensing to realize this system.

Conclusion: The need for licensing systems has increased in recent years due to regional economic integration and a shift in policy toward achieving universal health coverage. A national licensing system would be a significant milestone in health system development, helping to ensure the competency of health care professionals by means of a national examination, continuing professional development, and the revoking of licenses when appropriate.

Keywords: Health policy; Health workforce; Lao People’s Democratic Republic; Licensing.

라오스의 의학교육(Med Teach, 2019) 
Medical education in Laos  
Timothy Alan Witticka, Ketsomsouk Bouphavanhb, Vannyda Namvongsac, Amphay Khounthepd and Amy Graye,f,g  

 

소개
Introduction

배경
Background

라오스는 동남아시아의 내륙 산악 국가입니다(그림 1). 인구는 약 700만 명이며 인구의 63%가 농촌 지역에 거주하는 것으로 추정됩니다(세계보건기구 2015). 라오스에는 총 49개의 민족이 공식적으로 인정되고 있으며, 각 민족은 고유한 방언, 관습, 건강 신념을 가지고 있습니다. 라오스의 경제는 노동력의 약 80%를 고용하는 농업에 크게 의존하고 있습니다(라오스 보건부 외. 2014). 
Laos is a landlocked, mountainous country in South-East Asia (Figure 1). It has a population of approximately seven million people and it is estimated that 63% of the population live in rural areas (World Health Organisation 2015). A total of 49 ethnic groups are officially recognized in Laos, each having their own dialect, customs, and health beliefs. The economy is heavily based on agriculture, which employs an estimated 80% of the labor force (Ministry of Health Lao PDR et al. 2014).

 

20세기 이전 라오스가 포함된 지역은 수 세기에 걸쳐 여러 왕국의 통치를 받았습니다(Evans 2002). 1893년부터 1953년 라오스가 독립할 때까지 60년 동안 프랑스는 라오스에 강력한 영향력을 행사했습니다(Sweet 2015). 제2차 인도차이나 전쟁 이후 1975년 신라오학삿(NLHX) 또는 파텟 라오가 라오스의 통치기구로 취임했습니다. 이 시점에 라오스 인민민주주의 공화국(라오스)이 수립되었습니다(Evans 2002). 이후 라오스는 일당 국가로 남아있습니다(아카봉 외. 2014). 
Prior to the 20th century, the area encompassed by Laos was governed by multiple kingdoms over a period of many centuries (Evans 2002). From 1893, the French had a strong presence in the country for a period of 60 years until Laos gained independence in 1953 (Sweet 2015). Following the Second Indochina War, in 1975, the Neo Lao Hak Xat (NLHX) or Pathet Lao took over as the country’s governing body. It was at this point that the Lao People’s Democratic Republic (Lao PDR) was established (Evans 2002). Laos has since remained a one-party state (Akkhavong et al. 2014).

의료 시스템 개요
Overview of the healthcare system

사회경제 및 보건 지표가 개선되고 있지만 라오스는 이 지역 최빈국 중 하나이며 인간개발지수에서 188개 국가 및 지역 중 141위에 랭크되어 있습니다(유엔개발계획 2015). 세계보건기구(WHO)에 따르면 라오스인의 평균 기대 수명은 66세이며, 5세 미만 사망률은 1000명당 71명, 모성 사망률은 10만 명당 220명입니다(세계보건기구 2015). 비전염성 질병의 비율이 증가하고 있지만 감염으로 인한 질병 부담은 상당합니다(Akkhavong 외. 2014; 세계보건기구 2015). 
Although socioeconomic and health indicators are improving, Laos is one of the least developed countries in the region and ranks as 141 out of 188 countries and territories on the Human Development Index (United Nations Development Programme 2015). The World Health Organization (WHO) reports the average life expectancy for a Lao person to be 66 years, with an under-5 mortality rate of 71 deaths per 1000 live births and maternal mortality ratio of 220 deaths per 100000 live births (World Health Organisation 2015). Infections cause a significant burden of disease although rates of non-communicable diseases are rising (Akkhavong et al. 2014; World Health Organisation 2015).

라오스의 의료 시스템에는 정부 소유의 사용자 부담 시스템성장하는 민간 의료 부문이 포함됩니다. 정부 시스템은 중앙, 지방 및 지역 수준에서 운영되며 라오스 의료 종사자 및 자원봉사자들이 근무하고 있습니다(Akkhavong 외. 2014; Yoon 외. 2016). 2011년 라오스에는 인구 1만 명당 약 2.4명의 의사와 7.5명의 간호사가 있었습니다(라오스 보건부 및 세계보건기구 2012). 이는 세계보건기구(WHO)가 권장하는 인구 1만 명당 의사 및 간호사 최소 기준인 23명(세계보건기구 2010)과 비교됩니다.  
The healthcare system in Laos includes a government-owned, user pays system along with a growing private healthcare sector. The government system operates at central, provincial and district levels and is staffed by Lao health practitioners and volunteers (Akkhavong et al. 2014; Yoon et al. 2016). In 2011 there were approximately 2.4 doctors and 7.5 nurses per 10000 people in Laos (Ministry of Health Lao PDR and World Health Organisation 2012). This compares with the WHO recommended a minimum threshold of 23 doctors and nurses per 10000 people (World Health Organisation 2010).

기사 목적
Article objectives

이 글은 라오스의 의료 교육 시스템을 설명하는 것을 목표로 합니다. 중요한 역사적 발전, 관리 기관, 이해관계자, 현재 학부 및 대학원 연구 프로그램에 대해 간략히 설명하고 앞으로의 잠재적 방법과 함께 중요한 과제에 대해 논의합니다.
This article aims to describe the medical education system in Laos. We outline important historical developments, the governing bodies, stakeholders, current undergraduate, and postgraduate study programs and we discuss important challenges along with potential ways forward.

국가별 상황
Country in context

라오스 의학교육의 역사
History of medical education in Laos

라오스 의학교육의 역사는 크게 네 시기로 나눌 수 있습니다.

  • 첫 번째 시기인 1893년부터 1950년까지는 라오스의 프랑스 식민지 시대와 관련이 있습니다. 이 기간 동안 프랑스는 외국 의사들로 구성된 의료 네트워크를 구축했습니다(Sweet 2015). 외국인 의사를 유지하는 데 여러 가지 문제가 발생하자 1900년대 초에 현지 의료 인력 교육이 지원되었습니다. 소수의 라오스 학생들이 하노이로, 이후에는 사이공과 프놈펜으로 보내져 의료 보조원으로 훈련받았습니다. 1940년대 후반부터 라오스 학생들은 프랑스에서 의학 공부를 시작했습니다(Sweet 2015)
  • The history of medical education in Laos can be divided into four time periods. The first period, from 1893 to 1950, correlates with the French colonization of Laos. During this time the French established a health network staffed by foreign doctors (Sweet 2015). Having encountered multiple issues retaining foreign practitioners, in the early 1900s local health workforce training was supported. Small cohorts of Lao students were sent to Hanoi then later Saigon and Phnom Penh to train as medical assistants. From the late 1940s, Lao students undertook medical studies in France (Sweet 2015).
  • 두 번째 시기인 1950년부터 1975년까지는 라오스가 프랑스로부터 독립한 시기와 관련이 있습니다. 의과 대학 (그림 2)은 1957 년 비엔티안에 설립되었습니다. 세계보건기구와 프랑스 정부가 기술 및 재정 지원을 제공했습니다. 처음에는 프랑스어로 진행되는 4년제 의료 보조 과정을 제공했으며, 1961년 7명의 첫 졸업생이 배출되었습니다(토비수크). 이후 7년 과정의 의학 박사 과정이 개설되어 1969년 25명의 첫 번째 코호트가 이 과정을 시작했습니다. 이 현지 교육 외에도 라오스 학생들은 이 기간 동안 다른 나라에서 의학을 공부할 수 있는 후원을 계속 받았습니다(Sweet 2015).
  • The second period, from 1950 to 1975, correlates with Lao independence from France. The School of Medicine (Figure 2) was established in Vientiane in 1957. Technical and financial assistance was provided by WHO and the French Government. It initially offered a four-year medical assistant course taught in French and the first class of seven students graduated in 1961 (Thovisouk). A seven-year Doctor of Medicine course was later established with the first cohort of 25 students commencing this course in 1969. In addition to this local training, Lao students continued to receive sponsorship to study medicine in other countries throughout this period (Sweet 2015).
  • 제2차 인도차이나 전쟁의 끝은 1975년부터 1990년까지 지속된 제3차 인도차이나 전쟁의 시작을 알렸습니다. 이 기간 동안 상당수의 교육받은 라오스 전문가들이 라오스를 떠났습니다 (나이팅게일 2011). 프랑스의 존재는 감소한 반면 사회주의 국가의 지원은 상당히 증가했습니다1960년대 초부터 1991년까지 17000명 이상의 라오스 학생들이 소련의 대학에서 교육을 받은 것으로 추정되지만, 이들 중 몇 퍼센트가 의학을 공부했는지는 확실하지 않습니다(Sweet 2015). 해외 연수 제도로 인해 교육의 질이 다양하고 라오스 의사들이 귀국 후 공통된 전문 언어가 없는 등 여러 가지 문제가 발생했습니다.
  • The end of The Second Indochina War marked the beginning of the third period, which lasted from 1975 to 1990. A significant number of educated Lao professionals left the country during this time (Nightingale 2011). The French presence decreased while support from socialist countries rose considerably. It is estimated that over 17000 Lao students received training at universities in the Soviet Union from the early 1960s until 1991, although it is unclear what proportion of these students studied medicine (Sweet 2015). Multiple issues arose as a result of overseas training schemes including varied quality of education and Lao doctors lacking a common professional language upon their return home.
  • 현지 의사 교육도 증가하여 1978년에는 의대생 코호트가 연간 20명 수준에서 100명 이상으로 증가했습니다(Sweet 2015). 이에 따라 교사 수, 교사 자질, 시설, 교육 자료도 함께 증가했습니다. 소련, 베트남, 쿠바 직원들은 1960년대에 설립되어 1970년대 라오스 정부가 지방 간호 교육을 위해 부활시킨 의과대학과 지방 보건대학에 현장 직원과 기술 지원을 제공하면서 이 공백을 메우기 위해 노력했습니다(Sweet 2015).
  • Training of doctors locally also increased, with medical student cohorts rising from 20 to over 100 students per year level in 1978 (Sweet 2015). A corresponding increase in teacher numbers, teacher quality, facilities, and educational materials proved challenging. Soviet, Vietnamese, and Cuban staff sought to fill this void, providing both on the ground staff and technical support to the School of Medicine and to provincial health colleges which had been established in the 1960s, and revived by the Lao government in the 1970s to deliver nursing training in the provinces (Sweet 2015).
  • 마지막 시기인 1990년부터 현재까지 외국 지원의 성격이 변화하고 의료 교육에 대한 현지 통제가 증가했습니다(Sweet 2015). 문화적, 언어적 유사성뿐만 아니라 근접성과 공동체적 유대감으로 인해 태국의 대학들은 라오스 보건부와 협력 협약을 맺고 대학원 교육 및 지속적인 의학교육 기회를 제공했습니다. 현지에 기반을 둔 대학원 의학 교육도 설립되었습니다. 1990년대 초, 미국의 비정부기구(NGO)인 Health Frontiers는 보건과학대학(UHS)과 협력하여 소아과 대학원 교육 프로그램을 개발하기 시작했습니다. 비슷한 시기에 프랑스 NGO인 라오스 협력 위원회(CCL)는 UHS와 협력하여 수술, 마취 및 공중보건 분야의 현지 기반 대학원 교육을 개발했습니다(Phongsavath and Soukaloun 2014; Sweet 2015).
  • The final period, from 1990 to the present day, has seen a change in the nature of foreign support and an increase in local control over medical education (Sweet 2015). As a result of cultural and language similarities, as well as proximity and collegial ties, universities in Thailand formed cooperation arrangements with the Lao Ministry of Health to provide postgraduate training and continuing medical education opportunities. Locally based postgraduate medical training was also established. In the early 1990s, an American non-government organization (NGO), Health Frontiers, began working with the University of Health Sciences (UHS) to develop a pediatric postgraduate training program. At a similar time, the French NGO Comité pour la Coopération avec le Laos’ (CCL) partnered with the UHS to develop locally based postgraduate training in surgery, anesthetics and public health (Phongsavath and Soukaloun 2014; Sweet 2015).
  • 1990년대 초, 의과 대학(이전에는 "의과 대학"으로 불림)은 의학, 치과, 약학 등 세 개의 학부로 구성되었으며 보건부가 관리했습니다. 1995년에 대학 관리가 교육부로 이관되면서 큰 변화가 일어났습니다. 그 후 라오스 의과 대학은 라오스 국립 대학교에 합병되어 의과학 학부가 설립되었습니다. 2007년에 보건부가 의료 교육 관리를 다시 맡으면서 다시 변화가 일어났고, 이 시점에 UHS가 설립되었습니다.
  • In the early 1990s, the University of Medical Science (previously termed “School of Medicine”) consisted of three faculties: medicine, dentistry, and pharmacy and was governed by the Ministry of Health. Significant changes took place in 1995 when administration of the university was transferred to the Ministry of Education. The University of Medical Science was then merged into the National University of Laos, leading to the establishment of the Faculty of Medical Science. Change took place again in 2007 when the Ministry of Health regained administration of medical training and at this point in time the UHS was established.

교육 기관
Training institutions

UHS는 라오스에서 의사가 되기 위한 교육을 제공하는 유일한 기관입니다. 의료 보조원을 양성하는 4개의 주립 보건대학(루앙프라방, 타케크, 사반나켓, 팍세)과 다른 여러 주에 지역 보건 기초 과정을 제공하는 추가 보건대학이 있습니다. UHS, 주립 보건 대학, 보건 학교는 모두 별도의 조직이지만 보건부에서 관리합니다(그림 3).
The UHS is the only institution in Laos providing training to become a medical doctor. There are four provincial health colleges which train medical assistants (in Luang Prabang, Thakhek, Savannakhet, and Pakse) and additional health schools in a number of other provinces, offering basic courses in community health. Whilst the UHS, provincial health colleges, and health schools are all separate organizations they are administered by the Ministry of Health (Figure 3).

최근까지 UHS는 비엔티안의 4개 캠퍼스에 7개 학부가 있었습니다. 여기에는 기초과학부, 의학부, 간호학부, 치의학부, 약학부, 의료기술부, 대학원 학부, 그리고 행정 부서와 교육 개발 센터(EDC)가 포함됩니다. 교육 개발 센터는 2010년에 WHO의 지원으로 설립되었습니다. 이 센터는 WHO 서태평양 지역 대학 교육 개발 센터 네트워크의 일부입니다. 라오스의 EDC는 UHS에 전략적 계획을 지원하고, 학부와 협력하여 교육의 질을 향상시키며, 의학교육 활동에 대한 UHS와 외부 기관 간의 협력을 위한 구심점 역할을 합니다. 2018년에 UHS의 구조가 개정되었습니다. 여러 가지 변화 중에서도 6년간의 학부 의학 교육과 대학원 의학 교육이 모두 의학부 산하로 통합되어 의학 커리큘럼에 대한 보다 통합적인 접근이 가능해졌습니다. 가장 최근의 변화에서 EDC는 연구 및 교육 개발 연구소로서의 역할이 확대되었습니다.
Until recently the UHS had seven faculties located across four campuses in Vientiane. These include the Faculty of Basic Sciences, Medicine, Nursing, Dentistry, Pharmacy, Medical Technology, and the Postgraduate Faculty, in addition to an Administrative Affairs department and the Education Development Centre (EDC). The EDC was established in 2010 with support from WHO. It forms part of a network of education development centers at universities in the WHO Western Pacific Region. The EDC in Laos provides support to the UHS with strategic planning, works with faculties to improve the quality of their education and is a focal point for collaboration between the UHS and external organizations on medical education activities. In 2018 the structure of UHS has been revised. Among a number of changes, all six years of undergraduate medical training, as well as post-graduate medical training have been brought together under the Faculty of Medicine, enabling a more integrated approach to the medical curriculum. In the most recent changes, the EDC has an expanded role as an Institute for Research and Education Development.

학부 과정
Undergraduate studies

입학
Admission

UHS는 의대생을 위한 6년제 학부 커리큘럼을 제공하며, 학생들은 입학 시험을 통해 입학합니다(의학부). 시험은 화학, 생물학, 수학 과목에 대한 객관식 시험 3문항으로 구성되며, 각 시험은 2시간 동안 진행됩니다. 이 과정은 UHS와 보건부(MoH) 직원으로 구성된 위원회에서 감독합니다. 
The UHS offers a six-year undergraduate curriculum for medical students to which students are admitted based on entrance examinations (Faculty of Medicine). The examinations consist of three multiple choice question examinations, each two hours in length, covering subject areas of chemistry, biology, and mathematics. The process is overseen by a committee comprising UHS and Ministry of Health (MoH) staff.

학생들은 이 시험의 결과에 따라 순위를 매긴 다음 세 가지 그룹에 따라 선발됩니다.

  • 지원자의 70%로 구성된 그룹 1에는 그룹 2 또는 그룹 3 지원 자격이 없는 가장 높은 순위를 받은 학생이 포함됩니다.
  • 두 번째 그룹은 군대, 경찰 또는 지방, 주 또는 중앙 병원에서 근무한 경험이 있는 지원자(예: 의료 보조원)로 구성되며, 고용주가 의과 과정 입학 시 특별 배려를 받도록 추천한 지원자들로 구성됩니다.
  • 세 번째 그룹은 라오스 내 미리 정해진 취약 지역 출신 지원자로 구성됩니다.
  • 코스 입학생의 약 15%는 2그룹과 3그룹 지원자 각각에게서 나옵니다. 총 입학 학생 수는 해마다 90명에서 150명 이상까지 다양합니다. 

Students are ranked based on their results in these examinations and then selected according to three groupings.

  • Group one, which consists of 70% of entrants, includes the students with the highest ranks who are not eligible as group two or three applicants.
  • Group two consists of applicants who have worked for the military, police or in district, provincial or central hospitals (for example as a medical assistant) and who have been nominated by their employer to receive special consideration for entry into the medical course.
  • The third group consists of applicants who are from a pre-determined list of disadvantaged districts in Laos.
  • Approximately 15% of the course intake comes from each of group two and group three applicants. Total student admission numbers have varied from year to year, ranging from 90 to over 150 students.

코스 개요 및 교육 방법
Course outline and teaching methods

1969년 라오스에 의사 커리큘럼이 개설된 이래로 이 과정은 6~7년 동안 다양하게 진행되었습니다. 가장 최근의 변화는 2003년에 6년 풀타임 과정으로 축소된 것입니다(일본국제협력기구 2010). 2018년까지 1~3학년은 기초과학부에서, 4~6학년은 의학부에서 관리해 왔습니다. 표 1은 각 학년별 수업 내용을 개괄적으로 보여줍니다.
Since the establishment of a medical doctor curriculum in Laos in 1969, the course has varied between six and seven years in length. The most recent change occurred in 2003 when it was reduced to what remains a six-year, full-time course (Japan International Cooperation Agency 2010). Up until 2018, years one to three have been managed by the Faculty of Basic Sciences and years four to six have been managed by the Faculty of Medicine. Table 1 outlines the course content for each year level.

과목은 블록 단위로 진행되며 대부분의 수업은 라오스어로 진행됩니다. 4, 5, 6학년의 임상 로테이션 기간 동안 학생들은 오전에 배치된 병원에 출근하고 오후에는 대학 캠퍼스로 돌아와 교육을 받습니다. 제한된 인원으로 인해 학생들은 주로 대규모 그룹 강의 형식으로 교육을 받습니다. 임상 로테이션에서는 소규모 그룹 학습이 가능하지만 내용과 형식은 개별 현장에 따라 다릅니다. 제한된 교직원 수와 제한된 교육 시설 등 여러 가지 현실적인 문제로 인해 의과대학에서는 학생들이 받는 공식적인 교육과 임상 로테이션에서의 경험을 일치시키는 데 어려움을 겪고 있습니다. 이는 개별 과목 콘텐츠 간의 제한된 통합과 함께 지속적인 과제를 제기합니다.
Subjects are taught in blocks and the majority of teaching is undertaken in the Lao language. During the clinical rotations in years four, five, and six, students attend their placement in the morning and return to the university campus each afternoon for teaching. Limited staff numbers mean that students are predominantly taught in large group, lecture format. Smaller group learning is possible on clinical rotations however the content and format is dependent on individual sites. A number of practical issues, including limited staff numbers and limited teaching facilities, make it challenging for the Faculty of Medicine to align the formal teaching students receive with their experiences on clinical rotations. This, along with the limited integration between individual subject content poses ongoing challenges.

평가
Assessment

1~3학년 학생들은 각 블록 과목을 이수한 후 시험을 치릅니다. 각 시험의 형식과 내용은 과목 코디네이터가 결정합니다. 각 학년 수료 시 학생의 최종 점수는 개별 과목 시험 결과와 각 과목의 학점 가중치를 합산하여 계산됩니다.
In years one to three, students undertake examinations following the completion of each block subject. The format and content of each exam are decided by the subject coordinator. A student’s final mark at the completion of each year is calculated based on a sum of the individual subject examination results and the credit weighting for each subject.

4~6학년에는 블록 과목과 시험이 이어지며, 임상 로테이션에 참여하는 학생에게는 로그북과 병원 보고서라는 두 가지 추가 평가가 실시됩니다. 

  • 로그북은 허들 요건이며, 사례 발표, 진행 기록 작성 및 특정 절차를 포함한 역량을 다루며 담당자가 서명합니다.
  • 병원 보고서는 각 병원의 의료 교사가 제출하는 평가서로, 교사가 로테이션 기간 동안 학생의 성과를 관찰한 내용을 바탕으로 지식, 태도, 기술 영역을 평가합니다.

In years four to six, students continue to undertake block subjects followed by exams, with two additional assessments for students on clinical rotations – a logbook and a hospital report.

  • The logbook is a hurdle requirement and covers competencies including case presentations, progress note writing and specific procedures, on which staff signs off.
  • The hospital report is an assessment submitted by medical teachers at each hospital, assessing domains of knowledge, attitudes, and skills, based on the teachers’ observations of student performance throughout the rotation.

최종 시험은 6학년 말에 완료됩니다. 여기에는 객관식(MCQ) 시험, 객관적 구조화 임상 시험(OSCE), 그룹 연구 논문 제출이 포함됩니다. MCQ 시험은 4~6학년의 교과 내용을 다루며, OSCE는 내과, 외과, 소아과, 산부인과 등 각 핵심 전문과목에서 2개씩 총 8개 스테이션으로 구성됩니다. 학위를 받으려면 학생들은 이 두 가지 최종 시험과 연구 논문을 통과하고, 각 과목을 통과해야 하며, 재학 기간 동안 전문성을 입증해야 합니다(의학부, 토비수크). 
Final examinations are completed at the end of year six. These include a Multiple Choice Question (MCQ) examination, an Objective Structured Clinical Examination (OSCE) and submission of a group research thesis. The MCQ exam covers course content from years four to six, whilst the OSCE includes a total of eight stations, with two stations from each of the core specialties; internal medicine, surgery, pediatrics and obstetrics, and gynecology. To receive their degree students must pass these two final examinations and their research thesis, have passed each of their individual subjects and have demonstrated professionalism during the period of their studies (Faculty of Medicine; Thovisouk).

의학 학위를 마친 후 학생들의 경력 경로는 매우 다양합니다.

  • 정부는 일부 졸업생에게 제한된 감독을 받을 수 있는 지역 병원에서 일하도록 배정하는 반면,
  • 다른 졸업생은 중앙 또는 지방 병원이나 UHS에서 자원봉사를 하며 유급 취업의 기회를 기다립니다.
  • 소수의 졸업생은 비정부기구 또는 민간 부문에 취업합니다(일본국제협력기구 2010). 

Following completion of the medical degree, student career paths vary greatly.

  • The government assigns some graduates to work in district hospitals, where they may have limited supervision,
  • while others volunteer at central or provincial hospitals or at the UHS, waiting for an opportunity for paid employment.
  • A small proportion of graduates take up employment with a non-government organization or in the private sector (Japan International Cooperation Agency 2010).

대학원 과정
Postgraduate studies

라오스에서 졸업 후 수련 과정은 비교적 새로운 것입니다. 1997년에 소아과 레지던트가 최초로 개설된 대학원 교육 과정입니다(나이팅게일 2011). 현재 내과, 소아과, 산부인과, 외과, 안과, 방사선과, 마취과, 가정의학과, 응급의학과 등 총 9개의 대학원 레지던트 과정과 2017년에 시작된 공중보건 및 가정의학 석사 과정을 포함한 2개의 의학 석사 프로그램이 운영되고 있습니다. 모든 프로그램의 교육은 주로 라오스에서 2~3년에 걸쳐 이루어지며, 일부 프로그램의 학생들은 태국에서 단기 병원 로테이션을 이수합니다. 교육은 병원과 1차 진료 환경에서 모두 이루어지는 경우가 많으며, 이는 라오스의 인구가 주로 농촌에 기반을 두고 있기 때문에 특히 중요합니다(세계보건기구 2015). 현지에 기반을 둔 대학원 교육의 강점은 현지 보건 전문가를 유지하여 '두뇌 유출'을 최소화하고 현지 보건 시스템에 익숙한 졸업생을 확보할 수 있다는 점입니다. 문제는 다양한 방식으로 이러한 프로그램을 형성한 여러 국제기구의 지원을 받으면서 현지 교육 역량을 장기적으로 개발하는 데 의존해야 한다는 점입니다. 많은 레지던시에서 지원 국제기구는 교육 및 행정 제공을 지원하기 위해 국제 자원 봉사자를 지속적으로 현지에 상주시킵니다(나이팅게일 2011).
Post-graduate training pathways are relatively new in Laos. In 1997 the pediatric residency was the first postgraduate training pathway to be established (Nightingale 2011). There are now a total of nine postgraduate residencies and two medical masters programs including internal medicine, pediatrics, obstetrics and gynecology, surgery, ophthalmology, radiology, anesthetics, family medicine, and emergency medicine which commenced in 2017, along with a Masters in Public Health and Family Medicine. Training in all programs is predominantly completed in Laos over a period of two to three years, with students in some programs completing short hospital rotations in Thailand. Training often takes place in both hospital and primary care settings, which has been particularly important for Laos given its population is predominantly rurally based (World Health Organisation 2015). The strength of the locally based postgraduate education has been to retain local health professionals thereby minimize the “brain drain” and having graduates who are familiar with the local health system. The challenges are that they rely on the long-term development of local teaching capacity whilst receiving support from different international organizations which have shaped these programs in varying ways. In a number of residencies, the supporting international organizations have a continuous presence of international volunteers in-country to support the delivery of teaching and administration (Nightingale 2011).

레지던트 프로그램마다 입학 절차는 다르지만, 일반적으로 예비 학생들은 입학 시험을 치르고 선배 동료의 공식적인 추천을 받아 프로그램에 입학해야 합니다. 레지던트는 임상 책임을 맡게 되며 병상 튜토리얼, 강의, 저널 클럽, 케이스 토론, 임상 회의 참여 등을 통해 교육을 진행합니다. 일지, 연구 논문을 제출하고 최종 MCQ 및 OSCE 시험을 통과해야 하는 등 총체적이고 형성적인 평가가 사용됩니다. 
There are differing admission processes for each residency program, however, in general, prospective students are required to sit an entrance examination and have a senior colleague formally recommend them for admission into the program. Residents assume clinical responsibilities and teaching is delivered through bedside tutorials, lectures, journal clubs, case discussions, and participation in clinical meetings. Summative and formative assessments are used, including the requirement to submit a logbook, research thesis and pass final MCQ and OSCE examinations.

레지던트 수련 프로그램을 마친 후 졸업생은 대부분 중앙, 주 또는 지역 단위의 공중보건 시스템에서 근무하며, 대부분의 경우 채용된 병원으로 복귀해야 합니다. 일부 졸업생에게는 국제 펠로우십의 형태로 라오스에서 제공되지 않는 추가 하위 전문 교육을 이수할 수 있는 기회가 제공됩니다(나이팅게일 2011). 이 펠로우십은 국제 기부자들의 재정적 지원을 받으며 일반적으로 태국에서 수료합니다. 
Following completion of a residency training program, graduates most commonly work in the public health system, at a central, provincial or district level, and in most instances are required to return to the hospital from which they were recruited. A select number of graduates are offered the opportunity to complete further sub-specialty training which is not available in Laos, in the form of an international fellowship (Nightingale 2011). These fellowships are supported financially by international donors and are commonly completed in Thailand.

지속적인 전문성 개발
Continuing professional development

라오스에서 일하는 의사 및 기타 의료 전문가를 위한 면허 시스템이 현재 개발 중입니다(Sonoda 외. 2017). 이 단계에서는 지속적인 전문성 개발(CPD)에 대한 의무 요건은 없습니다. 그러나 CPD를 취득할 수 있는 방법은 여러 가지가 있습니다. 

  • 레지던트 프로그램 졸업생의 요구를 충족하기 위해 현지에서 운영되고 조직된 의학 컨퍼런스가 성장하고 있습니다. 
  • 보건부와 기부자를 통해 지원되는 프로그램을 통해 지역 및 지방 병원의 의사들이 단기 또는 중기적으로 상급 시설에 배치되어 특정 분야에 대한 전문성을 향상시킬 수 있습니다. 
  • 또한 아동 질병 통합 관리(IMCI), WHO 아동 병원 진료 포켓북, 조기 필수 신생아 치료 및 응급 산과 치료와 같은 국가 보건 프로그램과 기타 기부자 프로그램을 통해서도 CPD가 제공됩니다.

A licensing system for medical doctors and other health professionals working in Laos is currently being developed (Sonoda et al. 2017). At this stage, there is no mandatory requirement for continuing professional development (CPD). There are, however, a number of avenues through which CPD is obtained.

  • Locally run and organized medical conferences have grown to meet the needs of graduates of residency programs.
  • Programs supported through the Ministry of Health and donors facilitate short to medium term placement of doctors from district and provincial hospitals in higher level facilities to upskill these staff in particular areas.
  • CPD is also provided through national health programs such as the Integrated Management of Childhood Illness (IMCI), the WHO Pocketbook of Hospital Care for Children, Early Essential Newborn Care and Emergency Obstetric Care, as well as other donor programs.

CPD 참여는 출석을 지원하고 의료 전문가의 상대적으로 낮은 임금을 보충하기 위해 일당제 문화에 크게 의존합니다(Vian 외. 2013). 이는 지속 가능성에 영향을 미치며, 때때로 교육의 초점이 현지의 필요보다는 기부자의 의제에 의해 주도되고 보수에 의해 동기가 부여될 수 있음을 의미합니다. 그러나 일부 지역 컨퍼런스가 참가비를 부과하고 일당을 제공하지 않는 다른 프로그램이 제공하는 가치에 비해 참석률이 저조한 것으로 나타나면서 기대치가 변화하고 있습니다.  Participation in CPD relies largely on a culture of per diems to support attendance and to supplement the relatively low wages of health care professionals (Vian et al. 2013). This impacts on sustainability and means the focus of education can at times be driven by donor agendas and be motivated by remuneration, rather than the local need. Expectations are however shifting with some local conferences charging attendance fees and other programs not offering per diems being well attended because of the value they offer.

강점과 과제
Strengths and challenges

현재 UHS의 강점은 의학교육의 격차를 해소해야 한다는 명확한 인식이 있다는 점입니다. 세계의학교육연맹의 세계 의학교육 질 향상 표준(2015)에 따라 의학 커리큘럼의 개혁을 평가하고, 질 표준을 시행하는 면허 시험을 개발하는 프로세스가 마련되어 있습니다. 또한 국제 파트너와의 장기적인 관계를 통해 교육 개혁을 위한 기술 지원과 함께 일하는 현지 직원의 역량을 강화할 수 있는 기반을 마련했습니다. UHS에 교육 개발 센터를 설립한 것은 이러한 환경에서 교육 역량 강화가 절실히 필요했기 때문이며, 외부 파트너가 노력을 조정할 수 있는 구심점을 제공합니다. 마지막으로, 많은 개발도상국이 두뇌 유출로 어려움을 겪고 있지만(Lofters 2012), 지난 20년 동안 라오스는 다른 나라에서 라오스 의학 학위를 인정하지 않았고, 졸업생이 필요한 특정 분야에서 일하도록 요구하는 현지에서 운영하는 레지던트 프로그램의 존재로 인해 상대적으로 두뇌 유출에서 벗어날 수 있었습니다.
A strength the current situation for UHS is that there is clear recognition of the gaps in medical education which must be bridged. Processes are in place to assess the reform of the medical curriculum against World Federation for Medical Education Global Standards for Quality Improvement (World Federation for Medical Education 2015) and develop a licensing examination which enforces quality standards. Furthermore, long-term relationships with international partners have provided a foundation for technical support for educational reform and to build the capacity of local staff with whom these partners work. The creation of the Education Development Centre at UHS was born out of the critical need for education capacity building in this setting and provides a focal point through which external partners can coordinate efforts. Finally, while many developing countries suffer brain drain (Lofters 2012), in the last two decades Laos has been relatively spared, as a result of the lack of recognition of the Lao medical degree in other countries and the presence of locally run residency programs which require graduates to work in specific areas of need.

하지만 여전히 많은 과제가 남아 있습니다. 가장 큰 문제 중 하나는 아마도 영어(또는 기타 외국어) 능력일 것입니다. 라오스는 의료와 문화 전반에서 독서 문화가 제한되어 있습니다(Duerden 2017). 이러한 상황은 변화하고 있지만 라오스어로 된 의학 텍스트와 지침은 거의 존재하지 않으며, 존재한다고 하더라도 통화, 수, 배포 및 개인 접근성에 제한이 있습니다(Gray 외. 2017). 라오스어와 태국어의 유사성으로 인해 태국어 교과서가 사용되지만 이 솔루션은 불완전합니다. 수도와 주요 도시 외곽에서는 인터넷 접속이 어려울 수 있으며, 접속이 가능하더라도 속도가 느리고 비용이 많이 들 수 있습니다. 라오스 언어 리소스가 없는 경우 온라인 학습은 검색, 필터링 및 검색된 내용을 이해하기 위한 충분한 언어 능력에 의존하며, 많은 경우 불가능하지는 않더라도 매우 어려운 일입니다. 이러한 기술을 가진 사람들은 종종 젊은 세대의 학생과 의사이며, 이러한 젊은 학습자와 나이든 교사 사이에 지식의 불균형이 발생하고 있으며, 학습자들은 점점 더 이를 인식하고 있습니다. 
Many challenges do however remain. One of the greatest is perhaps English (or other foreign) language capacity. Both in medicine and the broader culture, Laos has had a limited reading culture (Duerden 2017). While this is changing, few Lao language medical texts and guidelines exist and many which do are limited in their currency, number, distribution, and accessibility to individuals (Gray et al. 2017). Thai textbooks are used due to the similarity between Lao and Thai but this solution is imperfect. Internet access can be challenging outside of the capital and major cities, but even when available it can be slow and costly. In the absence of Lao language resources, online learning is reliant on sufficient language skills to search, filter and understand what is found, and for many, this is highly challenging if not impossible. Those with these skills are often the younger generation of students and doctors, creating an imbalance in the currency of knowledge between these younger learners and old teachers, of which learners are increasingly aware.

또 다른 큰 문제는 교육 역량입니다. 교사 수가 부족하기 때문에 UHS는 이전에 문제 기반 학습 커리큘럼을 구현하기 위해 분류했지만 실현 불가능한 교사 대 학습자 비율로 인해 실현되지 못했습니다. 같은 문제가 실질적인 임상 술기 교육 제공을 방해하고 있습니다. 기존 직원의 경우 열악한 보수, 경쟁적인 요구, 불명확한 책임, 자신의 한계에 대한 인식이 교육에 대한 몰입도에 영향을 미칩니다. 초등학교에서 이루어지는 기초 교육을 포함한 교육은 전통적으로 의학에서 중요한 비판적 사고와 개념 이해보다는 지식 습득, 교훈적 방법, 권위자로서의 교사에 대한 존경에 중점을 두어 왔습니다. 새로운 방식으로 가르칠 수 있는 역량을 개발하려면 시간과 노력, 의지가 필요하며 무엇보다도 문화의 변화가 필요합니다.
The other great challenge is the teaching capacity. Teacher numbers are insufficient, so whilst UHS previously sort to implement a problem-based learning curriculum, it was not realized due to infeasible teacher-to-learner ratios. The same issue impedes delivery of practical clinical skills training. For existing staff, poor remuneration, competing demands, unclear responsibilities and the awareness of their own limitations impact on the engagement with teaching. Education, including the foundation laid in primary schools, has traditionally focused on knowledge acquisition, didactic methods and respect for the teacher as an authority, rather than on critical thinking and conceptual understanding which are critical in medicine. Developing the capacity to teach in new ways requires time, effort, willingness, and above all, culture change.

앞으로 나아갈 길
The way forward

라오스의 의학교육 상황은 여러 가지 도전 과제에도 불구하고 매우 낙관적인 이야기입니다. 라오스의 교육 개발 리더들은 극복해야 할 장애물을 인식하고 변화와 혁신을 기꺼이 시도하고 있습니다. 최근 라오스에서는 콘텐츠, 교육 및 평가 방법을 조정하고 커리큘럼을 통합하기 위해 의과대학의 6년 과정을 모두 의과대학 산하로 통합하는 변화가 있었습니다. 다른 의과대학의 발전 경로를 따라가는 것도 쉽지만, UHS는 이전에 해왔던 것에서 '도약'해야 할 필요성을 인식했습니다. 10년 이상 걸릴 수 있는 자체 커리큘럼으로 파트너 기관이 지금 하고 있는 것을 모방하려고 하는 대신, UHS는 5년 후 이들 기관에서 의학교육이 어떤 모습일지 생각하고 이를 목표로 삼아야 합니다. 예를 들어, 팀 기반 학습은 현재의 교직원 대 학생 비율로 실현 가능한 교육 방법으로서 잠재력이 있습니다(Punja 외. 2014). 
For all of its challenges, the situation for medical education in Laos is a story of great optimism. The leaders of education development in the country recognize the hurdles to be overcome and are willing to change and innovate. There has been a recent change to bring all six years of the medical course under the Faculty of Medicine in order to align content, teaching and assessment methods and to enable curriculum integration. While it would be easy to follow the path of development of other medical schools, UHS have recognized the need to “leapfrog” over what has been done before. Instead of trying to emulate what it’s partner institutions are doing now with their own curricula which could take a decade or more to achieve, UHS must think instead about what medical education may look like in these institutions five years from now and aim for that instead. For instance, potential exists in team-based learning as an educational method which is feasible with current staff to student ratios (Punja et al. 2014).

교육과 학습을 위한 라오스어 의료 자료는 매우 중요합니다. 교사와 학습자의 요구를 충족하기 위해 이러한 자료를 만드는 데는 엄청난 잠재력과 함께 큰 어려움이 있습니다. 의학 텍스트를 완전히 번역하려면 귀중한 시간과 자원이 소모되며 금방 구식이 될 것입니다. 또한 이 작업을 완료하기에 충분한 외국어 및 라오스어 능력을 갖춘 사람의 수도 제한되어 있습니다. 그러나 커리큘럼 콘텐츠는 시간이 지남에 따라 전략적으로 구축할 수 있습니다. 교육 역량이 부족하고 온라인 리소스를 찾는 학습자가 증가하는 상황에서 멀티미디어 리소스와 블로그와 같은 기타 정보 공유 수단을 집중적으로 개발하면 시간이 지남에 따라 조정할 수 있는 콘텐츠를 적시에 제공할 수 있는 잠재력을 제공할 수 있습니다. 이러한 리소스는 콘텐츠의 표준화를 가능하게 하고, 교사와 학습자 모두에게 성과에 대한 표준을 설정하며, 액세스되는 학습 자료의 품질을 모니터링하는 수단을 제공할 수 있습니다. 
Lao language medical resources for teaching and learning are crucial. There is both enormous potential and great challenges in creating these to meet the needs of teachers and learners. Complete translation of medical texts consumes both valuable time and resources and will quickly become outdated. Furthermore, the number of individuals with sufficient foreign and Lao language skills to complete this task is limited. Curriculum content could, however, be strategically built over time. With a lack of teaching capacity and a growing number of learners seeking online resources, targeted development of multimedia resources and other means of sharing information such as blogs offer potential to make content available in a timely way which is able to be adapted over time. These resources could enable standardization of content, set a standard for performance for both teachers and learners and provide a means of monitoring the quality of learning material which is being accessed.

라오스의 의료 교육에는 오랜 기간 동안 외국 이해관계자들이 참여해 왔으며, 이들은 각자의 관점, 의료 전통 및 관심사를 가지고 있습니다. 이들은 다양한 도전을 통해 라오스의 의학교육을 지원해 왔지만, 그 자체로 상충되는 메시지와 접근 방식을 가지고 있습니다. UHS와 라오스는 신뢰할 수 있는 파트너의 장기적인 지원을 계속 필요로 할 것입니다. 이러한 노력은 기부자 주기나 우선순위에 의존하지 않고 명확한 현지 전략에 따라 추진되는 안전하고 일관된 자금 기반에 의해 지원될 때 가장 효과적일 것입니다.

Medical Education in Laos has a long history of involvement from foreign stakeholders, who bring with them their own perspectives on practice, medical traditions, and interests. While they have supported medical education in Laos through various challenges, they themselves bring conflicting messages and approaches. The UHS and Laos will continue to require long-term support from trusted partners. These efforts would be most effective if supported by a secure and consistent funding base, which is not dependent on donor cycles or priorities but driven by a clear local strategy.

 


Med Teach. 2019 Aug;41(8):877-882. doi: 10.1080/0142159X.2018.1552780. Epub 2019 Feb 1.

Medical education in Laos

Affiliations

1Centre for International Child Health, University of Melbourne , Parkville , Australia.

2Education Development Centre, University of Health Sciences , Vientiane, Lao PDR.

3Department of Paediatrics, Setthathirath Hospital , Vientiane, Lao PDR.

4Faculty of Medicine, University of Health Sciences , Vientiane, Lao PDR.

5Royal Children's Hospital , Parkville , Australia.

6Murdoch Children's Research Institute , Parkville , Australia.

PMID: 30707856

DOI: 10.1080/0142159X.2018.1552780

Abstract

Medical education in Laos has undergone significant developments over the last century. A transition from a foreign to locally trained medical workforce has taken place, with international partners having an ongoing presence. Undergraduate and postgraduate medical education in Laos is now delivered by a single, government administered university. The transition to locally based training has had many flow-on benefits, including the retention of Lao doctors in the country and having graduates who are familiar with the local health system. A number of challenges do however exist. Medical resources in the Lao language are limited, teacher numbers and capacity are lacking and complex factors have led to a lack of uniformity in graduate competencies. Despite these challenges, the situation for medical education in Laos is a story of great optimism. Local staff has recognized the need for simple yet innovative solutions and processes are in place for the establishment of a licensing system for medical doctors and reforming existing curricula. Sustained, long-term relationships with partner organizations along with constructive use of technology are likely to be important factors affecting the future direction of medical education in Laos.

사례 기반 다지선다형 문항 작성을 위한 ChatGPT 프롬프트(Spanish Journal of Medical Education, 2023)
A ChatGPT Prompt for Writing Case-Based Multiple-Choice Questions 
Yavuz Selim Kıyak1 *

 

소개
Introduction

의과대학에서는 의대생의 고차원적인 능력, 특히 임상적 추론 능력을 평가하기 위해 다양한 필기 평가 방법을 사용합니다. 이러한 방법에는 사례 기반 객관식 문제(MCQ), 핵심 기능 문제, 스크립트 일치도 테스트 등이 포함됩니다(1). 이 중 사례 기반 MCQ는 매우 효과적이고 일반적으로 활용되는 것으로 알려져 있습니다(2). 그러나 의과대학의 한정된 자원으로 양질의 문항(문항)을 작성하는 과정은 상당한 도전 과제입니다.  
Medical schools use various written assessment methods to evaluate higher-order skills of medical students, particularly clinical reasoning. These methods encompass case-based multiple-choice questions (MCQs), key-feature questions, script concordance tests, and others (1). Among these, case-based MCQ stands out as both highly effective and commonly utilized (2). However, the process of writing high-quality items (questions) presents a significant challenge on the limited resources of medical schools. 

특히, 고차원적인 기술을 평가하기 위해 고안된, 맥락이 풍부한 단일 문항을 작성하려면 의대 교수의 상당한 시간 투자가 필요하며, 이는 종종 몇 시간으로 연장되기도 합니다(3). 많은 노력이 필요한 과정이지만, 의학교육에서 상당한 양의 MCQ에 대한 수요는 도전의 수준을 크게 높입니다. 진도 테스트만을 위한 문제은행을 개발하는 경우에도 수천 개의 문제를 통합해야 합니다(4). 따라서 의과대학이 직면한 중요한 과제는 고품질의 MCQ를 대량으로 작성하는 것입니다. 
More specifically, writing a single context-rich MCQ designed to assess higher-order skills demands a significant investment of a medical teacher's time, often extending into hours (3). While it requires an effortful process, the demand for a substantial quantity of MCQs within medical education significantly increases the level of challenge. Even developing a question bank solely for progress testing necessitates incorporating thousands of questions (4). Hence, the significant challenge faced by medical schools lies in writing a high quantity of high-quality MCQs. 

기존 솔루션
Existing Solutions

교육 분야에서 대량의 MCQ가 요구됨에 따라 연구자들은 소프트웨어를 사용하여 한 번에 많은 수의 문항을 생성하는 새로운 방법을 개발하게 되었습니다. 자동 문항 생성(AIG)은 "컴퓨터 기술을 사용하여 모델을 사용하여 문항을 생성하는 프로세스"(5)로서 이러한 목적으로 개발되었습니다. 템플릿 기반 AIG는 임상 문제에 직면했을 때 내용 전문가의 사고 과정을 드러내는 인지 모델을 만드는 것을 기반으로 합니다. 그런 다음 주제별 전문가는 질병/문제에 대한 인지 모델을 기반으로 항목 모델을 만듭니다. 문항 모델에는 템플릿 MCQ가 구축되고, 주제별 전문가가 제공한 사양을 고려하여 문항 내 적절한 부분에 적절한 단어를 할당하는 단어가 만들어집니다(6). 문항 모델은 수백 개의 문항을 만들 수 있기 때문에 템플릿 기반 AIG는 기존의 문항 작성 방식보다 실현 가능성이 높습니다.  
High volume of MCQ need in education led researchers to develop new methods to generate high number of items at once by using software. Automatic Item Generation (AIG) has been developed for this purpose as "a process of using models to generate items using computer technology" (5). Template-based AIG is based on creating a cognitive model, which reveals subject matter experts’ thinking process when they faced with a clinical problem. Then, subject matter experts create an item model based on the cognitive model on a disease/problem. In the item model, a template MCQ is built, and the words assigning the appropriate words to the appropriate parts in the items considering the specifications provided by subject matter experts (6). An item model is able to result in hundreds of MCQs, therefore template-based AIG is more feasible than traditional way of item writing. 

템플릿 기반 AIG는 지난 10년 동안 큰 성공을 거두었습니다. 영어, 프랑스어, 중국어, 스페인어, 한국어, 터키어 등 다양한 언어로 성공적으로 구현된 사례가 문헌에 나와 있습니다(6,7). 또한 캐나다의 국가 의사 면허 시험에서도 자동 생성된 문제가 사용되었습니다(8).  
Template-based AIG has shown a great success in the last ten years. Successful implementations have been shown in the literature in various languages, such as English, French, Chinese, Spanish, Korean, and Turkish (6,7). Furthermore, automatically generated questions have been used in national medical licensure exam in Canada (8). 

템플릿 기반 인공지능이 기존 문항 작성에 비해 우월함에도 불구하고, 인지 모델과 문항 모델을 개발하기 위해서는 분야별 전문가가 필요하기 때문에 여전히 인간의 노력에 의존하고 있습니다. 인공지능(AI)을 사용하면 시험 개발자가 문항을 생성하는 데 드는 노력이 줄어들지만, 연구자들은 여전히 템플릿 기반 방식에 비해 만족스럽지 못한 문항을 제공했기 때문에 이러한 목적으로 AI를 사용하는 것보다 템플릿 기반 AIG를 선호했습니다(6).  
Despite the superiority of template-based AIG over traditional item writing, it still depends on human efforts as it requires subject matter experts to develop cognitive models and item models. Although artificial intelligence (AI) is less effortful for test developers to generate items, researchers still preferred template-based AIG over the use of AI for this purpose because AI provided unsatisfying MCQs compared to template-based methods (6). 

MCQ 생성을 위한 AI
AI for Generating MCQs

AIG의 비템플릿 기반 방법은 자연어 처리(NLP)에 의존하는데, 과거에 사용되긴 했지만 만족스러운 MCQ를 생성하지 못했습니다(6). 그러나 대규모 언어 모델(LLM)인 생성적 사전 학습 트랜스포머(GPT)와 같은 NLP 기반 AI 솔루션이 이를 변화시킬 것으로 보입니다. GPT-3.5와 같은 LLM이 '우리말 해킹'에 성공했기 때문입니다(9).  
Non-template based methods in AIG relies on Natural Language Processing (NLP), which has been used but did not produce satisfying MCQs in the past (6). However, it seems NLP-based AI solutions like generative pretrained transformer (GPT), which is a large language model (LLM), is about to change this. It is because LLMs, such as GPT-3.5, achieved a success in “hacking our languages” (9). 

GPT-3.5의 변형인 ChatGPT는 2022년 11월에 공개적으로 도입되었습니다. 단 5일 만에 사용자 100만 명을 돌파했습니다(10). 이는 의료 전문직 교육에도 영향을 미쳤습니다(11-14). 그 결과 일부 의학교육 연구자들은 ChatGPT의 출시일을 ChatGPT 이전 시대와 이후 시대를 구분하는 중요한 기준으로 제시하기도 했습니다(15). 이러한 성공은 아마도 GPT-3가 이전 모델의 10배에 달하는 훨씬 더 큰 데이터 말뭉치를 보유하고 있다는 사실(16)과 ChatGPT가 무료로 공개되었기 때문일 것입니다(15).  
ChatGPT, a variant of GPT-3.5, was publicly introduced in November 2022. It passed one million user mark in only five days (10). It affected health professions education as well (11–14). As a result, some medical education researchers suggested the release date of ChatGPT as a crucial delineation between the pre-ChatGPT era and the post-ChatGPT era (15). This success was probably due to the fact that GPT-3 possesses a data corpus that is significantly larger, being ten times the size of prior models (16), and due to ChatGPT’s free and public release (15). 

저희의 개인적인 경험에 따르면 ChatGPT를 사용하여 고품질의 MCQ를 생성하는 것은 프롬프트의 품질에 달려 있습니다. 그래서 저희는 의대생들을 위해 Esh Tatla가 개발한 프롬프트를 수정하여 프롬프트를 개발했습니다(17). 원래 프롬프트가 유용하더라도 의학교육에서 객관식 문항을 구성하는 원칙에 더 가깝게 부합하는 MCQ를 생성하기 위해 수정이 필요했습니다. 지속적인 테스트와 개선을 통해 완성된 프롬프트는 표 1에 제시되어 있습니다. 또한 시험 개발자와 의대생의 이해를 돕기 위해 정답에 대한 설명을 제공하는 부분도 포함되어 있습니다.  
Our personal experiences show that generating high quality MCQs using ChatGPT depends on the quality of prompts. Therefore, we developed a prompt by revising the prompt developed by Esh Tatla for medical students (17). Even if the original prompt was useful, it required us to make revisions to generate MCQs that align more closely with the principles of constructing multiple-choice items in medical education. The prompt, achieved through continuous testing and refinement, is presented in Table 1. It also includes a part to provide explanations of the answers that may help test developers and medical students to better understand. 

이 프롬프트를 사용하려면 사용자는 대문자로 표시된 두 부분만 채우면 됩니다:
To use this prompt, the user needs only to fill two parts presented using capital letters:

  1. [주제를 입력하세요]
    [PLEASE INSERT A TOPIC]
  2. [난이도(예: 쉬움, 어려움)를 입력하세요]
    [PLEASE INSERT A DIFFICULTY LEVEL (E.G. EASY, DIFFICULT]

첫 번째 부분에서는 사용자가 주제를 삽입합니다. 이 주제는 의과 대학 교육의 학습 목표로 간주할 수도 있습니다. 두 번째 부분은 MCQ의 난이도를 결정하는 부분입니다. 
In the first part, the user inserts a topic. The topic can also be considered as a learning objective of a training in a medical school. The second part is to determine the difficulty of the MCQ.

표 2는 "일차 진료 환경에서 본태성 고혈압 관리"를 주제로, 난이도를 "쉬움"으로 설정한 프롬프트를 사용하여 생성된 MCQ를 보여 줍니다https://chat.openai.com/share/4342df6f-7cbe-43df-98dc-b110b53983e3(무료 연구 미리보기, 9월 25일 버전)에서 액세스할 수 있습니다. 
Table 2 shows an MCQ generated by using the prompt with “managing essential hypertension in a primary care setting” as the topic, and “Easy” as the difficulty level, which is accessible from https://chat.openai.com/share/4342df6f-7cbe-43df-98dc-b110b53983e3. (Free Research Preview, September 25 Version). 

의학교육학 박사 학위를 가진 의사로서 위의 MCQ는 그럴듯하고 잘 구성되어 있는 것 같습니다. 문항의 질에 대한 철저한 분석도 가능하겠지만, 프롬프트를 입력하고 버튼을 누르는 것만으로 몇 초 만에 이러한 MCQ를 생성할 수 있다는 점이 우선 흥미롭습니다. 과거에는 이러한 MCQ를 작성하려면 해당 분야 전문가들의 오랜 시간이 필요했습니다. 하지만 오늘날에는 인터넷 연결과 일반 컴퓨터, 그리고 상세한 프롬프트만 있으면 몇 초 만에 효율적으로 MCQ를 생성할 수 있습니다. 이 때문에 일부 연구자들은 2022년 11월부터 시작되는 이 시기를 포스트 ChatGPT 시대라고 명명하기도 합니다. 
As a physician with a PhD in medical education, it seems the MCQ above is both plausible and well-constructed. While a thorough analysis of the item's quality could be carried out, it is initially intriguing to generate such an MCQ within seconds by simply inputting a prompt and pressing a button, which is an extraordinary success. In the past, writing such MCQs required extensive time of subject matter experts. However, in today, with an internet connection, an average computer, and the detailed prompt, we can efficiently generate MCQs in a few seconds. This is why some researchers name this period began in November 2022 as post-ChatGPT era. 

향후 연구를 위한 권장 사항
Recommendations for Future Research

ChatGPT를 이용한 자동 MCQ 생성은 많은 연구 질문에 대한 해답을 제시합니다. 그 중 일부는 다음과 같습니다:
Automatic MCQ generation using ChatGPT comes up with many research questions that need to be answered. Some of them are:
 MCQ가 과학 및 임상 지식 측면에서 주제별 전문가의 평가를 통해 수용 가능한 것으로 간주됩니까? 
 프롬프트가 심리측정 표준을 충족하는 MCQ를 생성할 수 있나요? 
 ChatGPT가 생성한 MCQ의 품질은 인간 전문가가 생성한 것과 비교했을 때 어떤 차이가 있나요? 
 ChatGPT가 생성한 질문의 인지 수준이 의도한 학습 목표 및 블룸의 분류 체계와 어떻게 일치하나요? 
 MCQ의 의도된 난이도가 실제 평가 환경에서 실제 난이도와 일치합니까? 
 다양한 보건 전문직 교육 환경에서 MCQ를 생성하는 데 프롬프트를 효과적으로 사용할 수 있습니까? 
 MCQ를 시험에 포함하기 전에 사람이 편집해야 합니까? 
 설명이 형성 평가에서 의대생에게 효과적인 피드백으로 작용하나요? 
 Are the MCQs considered acceptable through evaluation by subject matter experts in terms of scientific and clinical knowledge? 
 Is the prompt capable of generating MCQs that meet psychometric standards? 
 How does the quality MCQs generated by ChatGPT compared to those created by human experts? 
 How do the cognitive levels of questions generated by ChatGPT align with the intended learning objectives and Bloom's taxonomy? 
 Do the intended difficulty levels of the MCQs align with their actual difficulty in real assessment settings? 
 Can the prompt be effectively used to generate MCQs in various health professions education settings? 
 Do the MCQs need editing by humans before incorporating them into exams? 
 Do the explanations serve as effective feedback for medical students in formative assessments? 

결론
Conclusions

 ChatGPT는 몇 초 만에 사례 기반 MCQ를 생성할 수 있는 흥미로운 기회를 제공합니다. 
 연구자들은 ChatGPT를 사용하여 생성된 MCQ의 품질을 평가하기 위한 연구를 수행해야 합니다.
 ChatGPT provides an exciting opportunity for generating case-based MCQs in a few seconds. 
 Researchers should conduct studies to evaluate the quality of MCQs generated by using ChatGPT. 


Table 1. A prompt for generating MCQs in medical education. 

You are developing a question bank for medical exams focusing on the topic of [PLEASE INSERT A TOPIC]. Please generate a high-quality single best answer multiple-choice question. Follow the principles of constructing multiple-choice items in medical education. Generate the questions using the following framework:

Case (write as a single narrative paragraph without providing each part separately): 
Patient details (gender/age)
Presenting complaint
Relevant clinical history
Physical examination findings
Diagnostic test results (optional)

Question stem: [Insert relevant information from the above sections without compromising the answer]
Acceptable question style: Ask for the BEST answer, NOT one that is TRUE/FALSE.

Answer options:
[Insert plausible answer option]
[Insert plausible answer option]
[Insert plausible answer option]
[Insert plausible answer option]
[Insert plausible answer option]

Explanation:
• Identify and explain the correct answer.
• Explain why this is the most appropriate answer based on evidence-based guidelines or expert consensus.
• Briefly explain why the other answer options are less correct or incorrect.

Difficulty level: [PLEASE INSERT A DIFFICULTY LEVEL (E.G. EASY, DIFFICULT]


Table 2. An MCQ generated by using the prompt in ChatGPT.

Case:

Mr. Johnson, a 55-year-old male, presents to the primary care clinic with a chief complaint of recurrent headaches. He reports a sedentary lifestyle and a family history of hypertension. His blood pressure readings during previous routine check-ups have consistently been elevated. Mr. Johnson denies any chest pain, shortness of breath, or visual disturbances. His past medical history is significant for obesity and a sedentary occupation. On physical examination, his blood pressure is 150/92 mmHg, and his BMI is 31. Laboratory results show normal renal function and lipid profile.  

Question stem:

Given Mr. Johnson's presentation with recurrent headaches, a family history of hypertension, and consistently elevated blood pressure readings, what is the BEST initial management strategy in a primary care setting? 

Answer options:

A. Initiate lifestyle modifications, including dietary changes and increased physical activity.
B. Start antihypertensive medication immediately to lower blood pressure.
C. Order a 24-hour ambulatory blood pressure monitoring for further assessment.
D. Recommend immediate admission to the hospital for comprehensive evaluation.
E. Suggest over-the-counter analgesics for headache relief and reassess in six months.

Explanation:

The correct answer is A. Initiate lifestyle modifications, including dietary changes and increased physical activity.

Explanation:

Hypertension management guidelines emphasize lifestyle modifications as the initial approach in patients with uncomplicated essential hypertension. For individuals like Mr. Johnson, who presents with elevated blood pressure, obesity, and a sedentary lifestyle, initiating lifestyle changes is crucial. These changes should include adopting a heart-healthy diet (such as the DASH diet), increasing physical activity, and weight reduction. Lifestyle modifications have been shown to significantly contribute to blood pressure control and may obviate the need for immediate pharmacologic intervention.

Option B (Start antihypertensive medication immediately) is not the best initial approach unless there are compelling indications for immediate treatment, which are not evident in Mr. Johnson's case. Option C (Order a 24-hour ambulatory blood pressure monitoring) may be considered in certain situations but is not the primary step in the management of essential hypertension. Option D (Recommend immediate admission to the hospital) is not warranted in the absence of hypertensive emergencies. Option E (Suggest over-the-counter analgesics) does not address the underlying hypertension and is not the primary management strategy for this patient. 


 


Abstract

The significant challenge faced by medical schools is the effortful process of writing a high quantity of high-quality case-based multiple-choice questions (MCQs) to assess the higher-order skills of medical students. The demand for a high volume of MCQs in education has led to the development of Automatic Item Generation (AIG), specifically template-based AIG, which involves creating cognitive and item models by subject matter experts to generate hundreds of MCQs at once using software. It demonstrated significant success in various languages and even being incorporated into national medical licensure exams. However, this method still heavily depends on the efforts of subject matter experts. This paper introduces a detailed ChatGPT prompt for quickly generating case-based MCQs and provides important research questions for future exploration into ChatGPT's potential in generating items, signaling the beginning of the artificial intelligence era in medical education, encouraging health professions education researchers to delve deeper into its potential.

+ Recent posts